Download as pdf or txt
Download as pdf or txt
You are on page 1of 121

403 CASE DIGESTS | CONSTITUTIONAL LAW REVIEW | JUDGE ESTELA ALMA SINGCO | FIRST SEMESTER | 2020-2021

CONSTITUTIONAL LAW I REVIEW INCORPORATION CLAUSE


Razon vs Tagitis G.R. No. 182498 (Dec. 3, 2009)
1 ST
BATCH OF CASE DIGESTS
PRINCIPLE OF EFFECTIVE NATIONALITY
Nottebohm Case (Liechtenstein v. Guatemala)
POLITICAL LAW
People v. Perfecto, 43 phil. 887
STATE IMMUNITY
Department of Agriculture v. NLRC, 227 SCRA 693, Nov. 11, 1993
WHEN TOOK EFFECT
[Vitug]
De Leon vs. Esguerra 153 SCRA 602, August 31, 1987
Santos v. Santos, 92 Phil. 281
Lambino v. Comelec, GR no. 174153, October 25, 2006
Kawananakoa v. Polybank, 205 U.S. 349
Providence Washington Insurance Co. v. Republic of the Philippines,
THE AMENDATORY PROCESS
29 SCRA 598
Gonzales v. COMELEC, 21 SCRA 774
Imbong v. COMELEC, 35 SCRA 28
SUITS AGAINST GOVERNMENT AGENCIES
Planas v. COMELEC, 59 SCRA 105
Republic v. Domingo, 657 SCRA 621
Sanidad v. COMELEC, 73 SCRA 333
Sanders v. Veridiano G.R. No. L-46930 June 10, 1988
Defensor-Santiago v. COMELEC, 270 SCRA 106
Bureau of Printing v. Bureau of Printing Employees Association, 1
Tolentino v. COMELEC, 41 SCRA 702
SCRA 340
Occena v. COMELEC, 104 SCRA 1
Mobil Philippines Exploration, Inc. v. Customs Arrastre Service,
Almario v. Alba, 127 SCRA 69
18SCRA 1120
Javellana v. Executive secretary, 50 SCRA 50
National Airports Corporation v. Teodoro, 91 Phil. 203
Air Transport Office v. Ramos, 664 SCRA 36
CONSTRUCTION & INTERPRETATION OF THE 1987
PNB v. CIR, 81 SCRA 314
CONSTITUTION
Rayo v.CFI Bulacan, 110 SCRA 460
Manila Prince Hotel vs. Government Service Insurance System, 267
SSS v. Court Appeals, 120 SCRA 707
SCRA 408
Municipality of San Fernando, La Union v. Firme, 195 SCRA 692
Magallona v. Ermita, 655 SCRA 476
Marbury v. Madison (1803)
SUITS AGAINST PUBLIC OFFICERS
Shauf v. Court of Appeals, 191 SCRA 713
THE MARITIME BASELINES LAW (R.A. NO. 9522)
Sanders v. Veridiano G.R. No. L-46930 June 10, 1988
Professor Merlin M. Magallona, et al. v. Hon. Eduardo Ermita, et al.,
Wylie v. Rarang, 209 SCRA 357
G.R. No. 187167, 655 SCRA 476, August 16, 2011, En Banc [Carpio]
Lansang v. Court of Appeals, G.R. 102667, February 23, 2000
Chavez v. Sandiganbayan, 193 SCRA 282 [1991]
BANGSAMORO JURIDICAL ENTITY CONCEPT
Republic v. Sandoval, 220 SCRA 124, March 19, 1993, En Banc
PROVINCE OF NORTH COTABATO VS. GRP, G.R. NO. 183591
[Campos, Jr.]
OCTOBER 14 2008
IS THE RULE THAT THE STATE MAY NOT BE SUED AT ALL,
CITIZENSHIP – 1935 CONSTITUTION
ABSOLUTE?
Fermin Caram Provision; Chiongbian vs. De Leon (G.R. No. L-2007,
Merit v. Gov’t. of P.I., 34 Phil. 311
January 31, 1949
Froilan v. Pan Oriental Shipping, G.R. No. L‑6060, September 30,
Rep. vs. Chule Lim, GR No. 153883, January 13, 2004
1950
Co vs. HRET (G.R. No. 92191-92, July 30, 1991)
Department of Agriculture v. NLRC, 227 SCRA 693, Nov. 11, 1993
In re: Vicente Ching, Bar Matter 914, October 1, 1999
[Vitug]
Dy Cuenco vs. Sec of Justice, May 26, 1962
EPG Construction Co. vs. Vigilar, 354 SCRA 566, March 16, 2001
Ma vs. Fernandez, July 26, 2010
Ministerio v. CFI, G.R. No. L-31635 August 31, 1971
Rep. vs. Sagun, Feb. 15. 2011
RESTRICTIVE DOCTRINE OF STATE IMMUNITY FROM SUIT
JONES LAW OF 1916
Department of Agriculture v. NLRC, 227 SCRA 693, Nov. 11, 1993
Tecson vs. COMELEC (G.R. No.161434, March 3, 2004)
[Vitug]
Valles vs. COMELEC, GR No. 137000, August 9, 2000
United States of America v. Ruiz (136 SCRA 487)
U.S. vs. Guinto, 182 SCRA 644
CITIZENSHIP - RES JUDICATA
Roa vs. Collector of Customs, 23 Phil.,315
SCOPE OF PROHIBITION AGAINST SUING THE STATE
Alvarez v. Commonwealth of the Philippines, 65 Phil. 302, 313
NATURALIZATION
(1938)
Moy Ya Lim Yao vs. Comsr. of Immigration, 41 SCRA 292
Municipality of San Fernando, La Union v. Firme, 195 SCRA 692
So vs. Rep., GR No. 170603, January 29, 2007
UP v. Dizon, G.R. No. 171182, 679 SCRA 54, 23 August 2012, 1st
Limkaichong vs. COMELEC, GR No. 179120, April 1, 2009
Div. [Bersamin]
Burca vs. Republic (GR No. L-24252-Jan. 30, 1967)
Meritt vs. Government, 31 SCRA 311, 318; Republic vs. Villasor, 54
SCRA 84
LOSS OF PHILIPPINE CITIZENSHIP
Board of Immigration Commissioners vs. Callano, 25 SCRA 890
SUABILITY V. LIABILITY
Labo vs. COMELEC, 176 SCRA 1
Republic v. NLRC G.R. No. 120385 October 17, 1996
Valles vs. COMELEC, G.R. No. 137000, August 9, 2000
Municipality of Hagonoy Bulacan v. Dumdum G.R. No. 168289 March
Yu vs. Santiago, 169 SCRA 364
22, 2010
UP v. Dizon GR No. 171782, August 23, 2012
PD 725
Frivaldo vs. COMELEC G.R. No. 120295 June 28, 1996
DIPLOMATIC IMMUNITY
Liang v. People, G.R. No. 125865, January 28, 2000
REPUBLIC ACT NO. 8171
Holy See v. Rosario, 238 SCRA 524
Tabasa vs, CA, GR No. 125793, August 29, 2006
China National Machinery Equipment Corporation v. Sta. Maria, 665
Altarejos vs. COMELEC, Nov. 10, 2004
SCRA 189
JUSMAG v. NLRC, 239 SCRA 224
NATURALIZED CITIZENS CANNOT HAVE DUAL CITIZENSHIP
World Health Organization v. Aquino, 48 SCRA 242
Lewis vs. COMELEC, August 4, 2006
SEAFDEC v. NLRC, 241 SCRA 580
Lopez vs. COMELEC, GR No. 182701, July 23, 2008
Jacot vs. COMELEC, GR No. 179848, November 27, 2008
EXEMPTION FROM LEGAL REQUIREMENTS
Maquiling vs. COMELEC, GR No. 195649, April 16, 2013
Araneta v. Gatmaitan, 101 Phil. 323
BM No. 1678, Petition for Leave to Resume the Practice of Law,
Banahaw Broadcasting Corporation v. Pacana, 649 SCRA 196
Benjamin M. Dacanay, December 17, 2007
Arasola v. Trinidad, 40 Phil. 252
Government of the Philippine Islands v. Monte de Pieded,G.R. No. L-
EFFECT OF REPATRIATION
9959, December 13, 1916
Bengzon III vs. HRET, GR No. 142840, May 7, 2001
Republic v. Garcia, 679 SCRA 54
Arigo v. Swift, 735 SCRA 102 (2014)
ISSUES ON CITIZENSHIP
Mercado vs. Manzano, 307 SCRA 630
DECLARATION OF PRINCIPLES AND STATE POLICIES
Lopez vs. COMELEC, GR No. 182701, July 23, 2008
Espina v. Zamora, G.R. No. 143855, September 21, 2010
Llamanzares vs. COMELEC, G.R. No. 221697, March 8, 2016
Oposa v. Factoran, G.R. No. 101083, July 30, 1993
Marcos vs Comelec (Sept. 18, 1995)
Imbong v. Executive Secretary, G.R. No. 204819, April 8, 2014
Cordora vs Comelec (Feb. 19, 2009)
Legaspi v. CSC, G.R. No. L-72119, May 29, 1987
Japson vs Comelec (Jan. 19, 2009)
Manila Prince Hotel v. GSIS, G.R. No. 122156, February 3, 1997.
Jalosjos vs Comelec (Oct. 19, 2010)
Maquiling vs Comelec (April 16, 2013)
INCORPORATION CLAUSE
Ugdoracion, Jr. vs. COMELEC, 552 SCRA 231
Tanada vs. Angara, May 2, 1997
Pimentel vs. Ermita, 462 SCRA 622, July 6, 2005
Page 1 of 121
403 CASE DIGESTS | CONSTITUTIONAL LAW REVIEW | JUDGE ESTELA ALMA SINGCO | FIRST SEMESTER | 2020-2021

Mejoff vs. Director of Prisons G.R. No. L-4254 September 26, 1951 Osmeña vs Pendatun, G.R. No. L-17144
Kuroda vs. Jalandoni G.R. No. L-2662 March 26, 1949 People vs. Jalosjos, 324 SCRA 689
Mijares, et al. vs. Javier, et al., April 12, 2005 Paredes vs. Sandiganbayan, G.R. No. 89989
Llamanzares vs. COMELEC, G.R. No. 221697, March 8, 2016
The Province of North Cotabato v. The Government of the Republic ELECTORAL TRIBUNALS
of the Philippines Peace Panel, GR No. 183591, October 14, 2008 Vera vs. Avelino, G.R. No. L-543
Ang Ladlad LGBT Party v. COMELEC, GR No.190582, April 8, 2010 LIMKAICHONG vs. COMELEC; Biraogo vs. Nograles; Paras vs.
Lim vs. Exec. Sec., April 11, 2002 Nograles & Villando vs. COMELEC, April 1, 2009
Ichong vs. Hernandez, 101 Phil. 1155 [1957] RONALD F. VILLANDO vs. HRET, Limkaichong, et al.
Sec. of Justice vs. Lantion G.R. No. 139465 January 18, 2000

CIVILIAN SUPREMACY OVER THE MILITARY


People v. Lagman (G.R. No. L-45893, July 13, 1938) POLITICAL LAW
Aquino v. Esperon (G.R. No. 174994, August 31, 2007)
Kapunan v. Quisumbing (G.R. Nos. 148213–17A, March 13, 2009)
1 People v. Perfecto, 43 phil. 887
Trillanes v. Pimentel (G.R. No. 179817, June 27, 2008)
Gudani and Balutan v. Senga (G.R. No. 170165, August 15, 2006) FACTS:
IBP v. Zamora (G.R. No. 141284, August 15, 2000)

SEPARATION OF THE CHURCH AND STATE The facts here are that the editor of a newspaper
Estrada vs. Escritor, June 22, 2006 published an article, naturally in writing, which
Islamic Da’Wah Council of the Philippines vs. Office of the Executive may have had the tendency to impeach the
Secretary, July 9, 2003
honesty, virtue, or reputation of members of the
THE FILIPINO FIRST POLICY Philippine Senate, thereby possibly exposing
Manila Prince Hotel v. GSIS, 267 SCRA 408 (1997) (Bellosillo) them to public hatred, contempt, or ridicule,
Wilson P. Gamboa v. Finance Secretary Margarito B. Teves, et al., which is exactly libel, as defined by the Libel
G.R. No. 176579, June 28, 2011, En Banc (Carpio) Law. Sir J. F. Stephen is authority for the
Board of Medicine v. Ota, G.R. No. 166097, July 14
statement that a libel is indictable when
THE RIGHT TO LIFE OF THE UNBORN FROM CONCEPTION defaming a "body of persons definite and small
James M. Imbong, et al. v. Hon. Paquito N. Ochoa, Jr., et al., GR No. enough for individual members to be recognized
204819, April 8, 2014, En Banc [Mendoza] as such, in or by means of anything capable of
being a libel." (Digest of Criminal Law, art. 267.)
RIGHTS OF PARENTS IN THE REARING OF THEIR CHILDREN
SPARK vs. Quezon City, et al
But in the United States, while it may be proper
to prosecute criminally the author of a libel
BALANCED & HEALTHFUL ECOLOGY charging a legislator with corruption, criticisms,
Prov. of Rizal vs. Exec. Sec., December 13, 2005 no matter how severe, on a legislature, are
International Service for the Acquisition of Agri-Biotech Applications.
within the range of the liberty of the press,
Inc. v. Greenpeace Southeast Asia (Philippines), et al., December 8,
2015 unless the intention and effect be seditious. (3
Mosqueda, et al. v. Pilipino Banana Growers & Exporters Association, Wharton's Criminal Law, p. 2131.) With these
Inc., et al., GR No. 189185, August 16, 2016 facts and legal principles in mind, recall that
Resident Marine Mammals of the Protected Seascape Tanon straits, article 256 begins: Any person who, by . . .
et al. v. Secretary Angelo Reyes, et al., GR No.180771, April 21,
2016
writing, shall defame, abuse, or insult any
Minister of the Crown or other person in
LOCAL AUTONOMY authority," etc.
Basco vs. Pagcor G.R. No. 91649 May 14, 1991
Province of North Cotabato vs. GRP Peace Panel, (GR No. 183591,
Oct. 14, 2008)
ISSUE: Is Art. 256 a political law that is
Abas Kida vs. Senate of the Philippines, 659 SCRA 270 abrogated by the Treaty of Paris?

LEGISLATIVE DEPARTMENT RULING: Yes. It is a general principle of the


Santiago vs. COMELEC, 3/19/97
Jose Jesus M. Disini, Jr. et al. vs. The Secretary of Justice, GR No. public law that on acquisition of territory the
203335, February 11, 2014 previous political relations of the ceded region
Garcia vs, Drilon, GR No. 179267, June 25, 2013 are totally abrogated. "Political" is here used to
Pimentel III vs. COMELEC, G. R. No. 178413, March 13, 2008 denominate the laws regulating the relations
sustained by the inhabitants to the sovereign.
NON-DELEGATION OF LEGISLATIVE POWER
Abakada Guro Party List vs. Executive Secretary, September 1, 2005 (American Insurance Co. vs. Canter [1828], 1
Gerochi vs. DOE, GR. No. 159796, July 17, 2007 Pet., 511; Chicago, Rock Island and Pacific
Echegaray vs. Secretary of Justice G.R. No. 132601 January 19, 1999 Railway Co. vs. McGlinn [1885], 114 U.S., 542;
EASTERN SHIPPING LINES V. POEA, 166 SCRA 533 Roa vs. Collector of Customs [1912], 23 Phil.,
Secretary of Finance, et al. vs. La Suerte Cigar, GR No. 166498, June
11, 2009
315.) Mr. Justice Field of the United States
Review Center Assos. of the Philippines vs. Ermita, GR No. 180046, Supreme Court stated the obvious when in the
April 2, 2009 course of his opinion in the case of Chicago,
SEMA VS. COMELEC, G. R. No. 177597, July 16, 2008 Rock Island and Pacific Railway Co. vs.
McGlinn, supra, he said: "As a matter of course,
PARTY-LIST SYSTEM
BA-RA 7941 vs. COMELEC, G.R. No. 1777271, May 4, 2007 all laws, ordinances and regulations in conflict
Veterans Federation Party vs. COMELEC, 342 SCRA 244, October 6, with the political character, institutions and
2000; Constitution of the new government are at once
Partido Ng Manggagawa vs. COMELEC, March 15, 2006 displaced. Thus, upon a cession of political
BANAT vs. COMELEC, G.R. No. 179271, April 21, 2009
jurisdiction and legislative power — and the
ATONG PAGLAUM, INC. vs. COMELEC, GR No. 203646, April 2, 2013
Aquino vs. COMELEC, GR No. 189793, April 7, 2010 latter is involved in the former — to the United
Aldaba, et al. vs. COMELEC, GR No. 188078, January 25, 2010 States, the laws of the country in support of an
Ang Ladlad LGBT Party v. COMELEC, GR No. 190582, April 8, 2010 established religion or abridging the freedom of
the press, or authorizing cruel and unusual
LEGISLATIVE PERKS
PP vs. Jalosjos, 324 SCRA 689
punishments, and he like, would at once cease
Trillanes IV vs. Pimentel, June 27, 2008 to be of obligatory force without any
Trillanes vs. Marigomen, G.R. No. 179817, March 14, 2018 declaration to that effect." To quote again from
Liban v. Gordon, G.R. No. 175352, July 15, 2009 the United States Supreme Court: "It cannot be
Avelino vs. Cruz admitted that the King of Spain could, by treaty
Santiago vs. Guingona (298 SCRA 756)
or otherwise, impart to the United States any
RULES OF PROCEEDINGS of his royal prerogatives; and much less can it
Arroyo vs. De Venecia, 277 SCRA 268 be admitted that they have capacity to receive
Baguilat vs. Alvarez, G.R. No. 227757, July 25, 2017 or power to exercise them. Every nation
acquiring territory, by treaty or otherwise,
DISCIPLINING MEMBERS
Page 2 of 121
403 CASE DIGESTS | CONSTITUTIONAL LAW REVIEW | JUDGE ESTELA ALMA SINGCO | FIRST SEMESTER | 2020-2021

must hold it subject to the Constitution and become inoperative, respondent OIC Governor
laws of its own government, and not according could no longer rely on Section 2, Article III,
to those of the government ceding it." thereof to designate respondents to the
elective positions occupied by petitioners.
According to our view, article 256 of the
Spanish Penal Code was enacted by the Petitioners must now be held to have acquired
Government of Spain to protect Spanish security of tenure specially considering that
officials who were the representatives of the the Barangay Election Act of 1982 declares it
King. With the change of sovereignty, a new "a policy of the State to guarantee and
government, and a new theory of government, promote the autonomy of the barangays to
as set up in the Philippines. It was in no sense ensure their fullest development as self-reliant
a continuation of the old, although merely for communities.2 Similarly, the 1987
convenience certain of the existing institutions Constitution ensures the autonomy of local
and laws were continued. The demands which governments and of political subdivisions of
the new government made, and makes, on the which the barangays form a part, 3 and limits
individual citizen are likewise different. No the President's power to "general supervision"
longer is there a Minister of the Crown or a over local governments.
person in authority of such exalted position that
the citizen must speak of him only with bated
breath. "In the eye of our Constitution and
laws, every man is a sovereign, a ruler and a REVISION VS AMENDMENT /
freeman, and has equal rights with every other QUANTITATIVE & QUALITATIVE TEST
man. We have no rank or station, except that
of respectability and intelligence as opposed to 3 Lambino v. Comelec, GR no. 174153,
indecency and ignorance, and the door to this October 25, 2006
rank stands open to every man to freely enter FACTS:
and abide therein, if he is qualified, and
whether he is qualified or not depends upon the Lambino (an ally of President Arroyo) made a
life and character and attainments and conduct proposal to change the 1987 constitution.He
of each person for himself. Every man may proposed changing the form of government
lawfully do what he will, so long as it is not from presidential to parliamentary and
malum in se or malum prohibitum or does not bicameral legislature to unicameral legislature
infringe upon the qually sacred rights of and affecting 105 provisions through People’s
others." Initiative.

ISSUE:Can revisions be made with People’s


Initiative? (No) What is the difference between
WHEN TOOK EFFECT revision and amendment? Is Lambino’s
2 De Leon vs. Esguerra 153 SCRA 602, proposal amendment or revision? (Revision)
August 31, 1987
RULING:
FACTS:
Article XVII of the Constitution speaks of three
Petitioners are barangay officials elected for a modes of amending the Constitution. The first
period of 6 years. The provisional constitution mode is through Congress upon three-fourths
of the Aquino gave the power of the OIC vote of all its Members. The second mode is
governor to replace barangay officials within 1 through a constitutional convention. The third
year from promulgation. On Feb 8, 1987, the mode is through a people's initiative.
governor issued a memo replacing petitioners.
Petitioner contends that by Feb 2, 1987, the There can be no mistake about it. The framers
1987 constitution became effective and that of the Constitution intended, and wrote, a clear
the OIC governor has lost the power to replace distinction between "amendment" and
them. In the Comment, respondents argue "revision" of the Constitution. The framers
that their power to replace is 1 year from their intended, and wrote, that only Congress or a
appointment under Section 2, Article III of the constitutional convention may propose
provisional constitution. revisions to the Constitution. The framers
intended, and wrote, that a people's initiative
ISSUE: When did the 1987 constitution took may propose only amendments to the
effect? Constitution. Where the intent and language of
the Constitution clearly withhold from the
RULING: Feb. 2, 1987. But while February 8, people the power to propose revisions to the
1987 is ostensibly still within the one-year Constitution, the people cannot propose
deadline, the aforequoted provision in the revisions even as they are empowered to
Provisional Constitution must be deemed to propose amendments.
have been overtaken by Section 27, Article
XVIII of the 1987 Constitution reading. The qualitative test inquires into the
qualitative effects of the proposed change in
SECTION 27. This Constitution shall take effect the constitution. The main inquiry is whether
immediately upon its ratification by a majority the change will "accomplish such far reaching
of the votes cast in a plebiscite held for the changes in the nature of our basic
purpose and shall supersede all previous governmental plan as to amount to a
Constitutions. revision."37 Whether there is an alteration in
the structure of government is a proper
subject of inquiry. Thus, "a change in the
The 1987 Constitution was ratified in a
nature of [the] basic governmental plan"
plebiscite on February 2, 1987. By that date,
includes "change in its fundamental framework
therefore, the Provisional Constitution must be
or the fundamental powers of its Branches."38
deemed to have been superseded. Having
A change in the nature of the basic
Page 3 of 121
403 CASE DIGESTS | CONSTITUTIONAL LAW REVIEW | JUDGE ESTELA ALMA SINGCO | FIRST SEMESTER | 2020-2021

governmental plan also includes changes that 2. R. B. H. No. 2, calling a convention to


"jeopardize the traditional form of government propose amendments to said Constitution, the
and the system of check and balances."39 convention to be composed of two (2) elective
delegates from each representative district, to
Under both the quantitative and qualitative be "elected in the general elections to be held
tests, the Lambino Group's initiative is a on the second Tuesday of November, 1971;"
revision and not merely an amendment. and
Quantitatively, the Lambino Group's proposed
changes overhaul two articles - Article VI on 3. R. B. H. No. 3, proposing that Section 16,
the Legislature and Article VII on the Executive Article VI, of the same Constitution, be
- affecting a total of 105 provisions in the amended so as to authorize Senators and
entire Constitution.40 Qualitatively, the members of the House of Representatives to
proposed changes alter substantially the basic become delegates to the aforementioned
plan of government, from presidential to constitutional convention, without forfeiting
parliamentary, and from a bicameral to a their respective seats in Congress.
unicameral legislature.

A change in the structure of government is a


revision of the Constitution, as when the three Subsequently, Congress passed a bill, which,
great co-equal branches of government in the upon approval by the President, on June 17,
present Constitution are reduced into two. This 1967, became Republic Act No. 4913,
alters the separation of powers in the providing that the amendments to the
Constitution. A shift from the present Constitution proposed in the aforementioned
Bicameral-Presidential system to a Resolutions No. 1 and 3 be submitted, for
Unicameral-Parliamentary system is a revision approval by the people, at the general
of the Constitution. Merging the legislative and elections which shall be held on November 14,
executive branches is a radical change in the 1967.
structure of government.
ISSUE: Whether or not they are constitutional
The abolition alone of the Office of the insofar as they implement the amendatory
President as the locus of Executive Power process?
alters the separation of powers and thus
constitutes a revision of the Constitution. RULING: Yes. Referring particularly to the
Likewise, the abolition alone of one chamber of contested proposals for amendment, the
Congress alters the system of checks-and- sufficiency or insufficiency, from a
balances within the legislature and constitutes constitutional angle, of the submission thereof
a revision of the Constitution. for ratification to the people on November 14,
1967, depends — in the view of those who
By any legal test and under any jurisdiction, a concur in this opinion, and who, insofar as this
shift from a Bicameral-Presidential to a phase of the case, constitute the minority —
Unicameral-Parliamentary system, involving upon whether the provisions of Republic Act
the abolition of the Office of the President and No. 4913 are such as to fairly apprise the
the abolition of one chamber of Congress, is people of the gist, the main idea or the
beyond doubt a revision, not a mere substance of said proposals, which is — under
amendment. On the face alone of the Lambino R. B. H. No. 1 — the increase of the maximum
Group's proposed changes, it is readily number of seats in the House of
apparent that the changes will radically alter Representatives, from 120 to 180, and —
the framework of government as set forth in under R. B. H. No. 3 — the authority given to
the Constitution the members of Congress to run for delegates
to the Constitutional Convention and, if elected
thereto, to discharge the duties of such
delegates, without forfeiting their seats in
THE AMENDATORY PROCESS
Congress. We — who constitute the minority
4 Gonzales v. COMELEC, 21 SCRA 774 — believe that Republic Act No. 4913 satisfies
such requirement and that said Act is,
FACTS: accordingly, constitutional.

The year is 1967 and the several amendments It should be noted that the contested
to the 1935 constitution were proposed. There Resolutions were approved on March 16, 1967,
are 3 resolutions and 1 law for a constitutional so that, by November 14, 1967, our citizenry
convention - the chosen way of amending the shall have had practically eight (8) months to
constitution. The constitutionality of these 3 be informed on the amendments in question.
resolutions and 1 law is questioned. Then again, Section 2 of Republic Act No. 4913
provides:
They are:
1. R. B. H. (Resolution of Both Houses) No. 1, (1) that "the amendments shall be published
proposing that Section 5, Article VI, of the in three consecutive issues of the Official
Constitution of the Philippines, be amended so Gazette, at least twenty days prior to the
as to increase the membership of the House of election;"
Representatives from a maximum of 120, as
provided in the present Constitution, to a
maximum of 180, to be apportioned among
the several provinces as nearly as may be (2) that "a printed copy of the proposed
according to the number of their respective amendments shall be posted in a conspicuous
inhabitants, although each province shall place in every municipality, city and provincial
have, at least, one (1) member; office building and in every polling place not
later than October 14, 1967," and that said
Page 4 of 121
403 CASE DIGESTS | CONSTITUTIONAL LAW REVIEW | JUDGE ESTELA ALMA SINGCO | FIRST SEMESTER | 2020-2021

copy "shall remain posted therein until after Paragraph 1, Sec. 8(a) of R.A. No. 6132 is
the election;" impugned by both petitioners as violative of
the constitutional guarantees of due process,
(3) that "at least five copies of said equal protection of the laws, freedom of
amendment shall be kept in each polling place, expressions, freedom of assembly and
to be made available for examination by the freedom of association. This is a provision that
qualified electors during election day;" bans delegates representing political parties
and other organizations.
(4) that "when practicable, copies in the
principal native languages, as may be ISSUE: Are these disqualifications for
determined by the Commission on Elections, candidates for delegates of the constitutional
shall be kept in each polling place;" convention constitutional?.

(5) that "the Commission on Elections shall RULING: Yes. Unlike in the apportionment of
make available copies of said amendments in representative districts, the Constitution does
English, Spanish and, whenever practicable, in not expressly or impliedly require such
the principal native languages, for free apportionment of delegates to the convention
distributing:" and on the basis of population in each
congressional district. Congress, sitting as a
(6) that the contested Resolutions "shall be Constituent Assembly, may constitutionally
printed in full" on the back of the ballots which allocate one delegate for, each congressional
shall be used on November 14, 1967. district or for each province, for reasons of
economy and to avoid having an unwieldy
convention. If the framers of the present
We are not prepared to say that the foregoing
Constitution wanted the apportionment of
measures are palpably inadequate to comply
delegates to the convention to be based on the
with the constitutional requirement that
number of inhabitants in each representative
proposals for amendment be "submitted to the
district, they would have done so in so many
people for their ratification," and that said
words as they did in relation to the
measures are manifestly insufficient, from a
apportionment of the representative districts.
constitutional viewpoint, to inform the people
of the amendment sought to be made.
As observed by the Solicitor General in his
Answer, the overriding objective of the
challenged disqualification, temporary in
nature, is to compel the elected delegates to
These were substantially the same means serve in full their term as such and to devote
availed of to inform the people of the all their time to the convention, pursuant to
subject submitted to them for ratification, their representation and commitment to the
from the original Constitution down to the people; otherwise, his seat in the convention
Parity Amendment. will be vacant and his constituents will be
deprived of a voice in the convention. The
inhibition is likewise "designed to prevent
popular political figures from controlling
5 Imbong v. COMELEC, 35 SCRA 28 elections or positions. Also it is a brake on the
appointing power, to curtail the latter's desire
FACTS: to 'raid' the convention of "talents" or attempt
to control the convention." (p. 10, Answer in
The year is 1967. Petitioners are interested in L-32443.)
becoming delegates for the planned
constitutional convention to amend the 1987 The ban against all political parties or
constitution. Congress has passed two organized groups of whatever nature
resolutions and 1 law regarding the contained in par. 1 of Sec. 8(a), is confined to
appointment of delegates. party or organization support or assistance,
whether material, moral, emotional or
The following are the contested provision otherwise. The very Sec. 8(a) in its provisos
which the petitioner claim is unconstitutional: permits the candidate to utilize in his campaign
the help of the members of his family within
the fourth civil degree of consanguinity or
affinity, and a campaign staff composed of not
Petitioner asserts that Sec. 2 on the
more than one for every ten precincts in his
apportionment of delegates is not in
district. It allows the full exercise of his
accordance with proportional representation
freedom of expression and his right to peaceful
and therefore violates the Constitution and the
assembly, because he cannot be denied any
intent of the law itself, without pinpointing any
permit to hold a public meeting on the pretext
specific provision of the Constitution with
that the provision of said section may or will
which it collides.
be violated. The right of a member of any
political party or association to support him or
Sec. 5 of R.A. 6132 is attacked on the ground oppose his opponent is preserved as long as
that it is an undue deprivation of liberty such member acts individually. The very party
without due process of law and denies the or organization to which he may belong or
equal protection of the laws. Said Sec. 5 which may be in sympathy with his cause or
disqualifies any elected delegate from running program of reforms, is guaranteed the right to
"for any public office in any election" or from disseminate information about, or to arouse
assuming "any appointive office or position in public interest in, or to advocate for
any branch of the government government constitutional reforms, programs, policies or
until after the final adjournment of the constitutional proposals for amendments.
Constitutional Convention."

Page 5 of 121
403 CASE DIGESTS | CONSTITUTIONAL LAW REVIEW | JUDGE ESTELA ALMA SINGCO | FIRST SEMESTER | 2020-2021

It is therefore patent that the restriction RULING: Yes. The first question for Our
contained in Sec. 8(a) is so narrow that the determination is whether We have authority to
basic constitutional rights themselves remain pass upon the validity of Presidential Decree
substantially intact and inviolate. And it is No. 73, in view of the Solicitor General's
therefore a valid infringement of the aforesaid allegation to the effect that said question is a
constitutional guarantees invoked by political one. I am of the opinion — on which
petitioners. the Members of the Court are unanimous —
that the contention of the Solicitor General is
The questioned par. 1 of Sec. 8 (a) likewise untenable and that the issue aforementioned
can easily pass the balancing-of-interest test. is a justiciable one. Indeed, the contested
19 decree purports to have the force and effect of
a legislation, so that the issue on the validity
thereof is manifestly a justiciable one, on the
authority, not only of a long list of cases in
which the Court has passed upon the
In the apt words of the Solicitor General:
constitutionality of statutes and/or acts of the
Executive,1 but, also, of no less than that of
Subdivision (1) of Section 2, Article VIII of the
1935 Constitution,2 which expressly provides
It is to be noted that right now the nation is on for the authority of this Court to review cases
the threshold of rewriting its Constitution in a involving said issue.
hopeful endeavor to find a solution to the
grave economic, social and political problems As regards the authority of the President to
besetting the country. Instead of directly issue Presidential Decree No. 73, "submitting
proposing the amendments Congress has to the Filipino people (on January 15, 1973) for
chosen to call a Constitutional Convention ratification or rejection the Constitution of the
which shall have the task of fashioning a Republic of the Philippines proposed by the
document that shall embody the aspirations 1971 Constitutional Convention and
and ideals of the people. Because what is to be appropriating funds therefor," I find it
amended is the fundamental law of the land, it unnecessary, for the time being, to pass upon
is indispensable that the Constitutional such question, because the plebiscite ordained
Convention be composed of delegates truly in said Decree has been postponed. In any
representative of the people's will. Public event, should the plebiscite be scheduled to be
welfare demands that the delegates should held at any time later, the proper parties may
speak for the entire nation, and their voices be then file such action as the circumstances may
not those of a particular segment of the justify.
citizenry, or of a particular class or group of
people, be they religious, political, civic or
With respect to the question whether or not
professional in character. Senator Pelaez,
martial law per se affects the validity of a
Chairman of the Senate Committee on Codes
submission to the people for ratification of
and Constitutional Amendments, eloquently
specific proposals for amendment of the
stated that "the function of a constitution is not
Constitution, I consider this matter as one
to represent anyone in interest or set of
intimately and necessarily related to the
interests, not to favor one group at the
validity of Proclamation No. 1102 of the
expense or disadvantage of the candidates —
President of the Philippines. This question has
but to encompass all the interests that exist
not been explicitly raised, however, in any of
within our society and to blend them into one
the cases under consideration
harmonious and balanced whole. For the
constitutional system means, not the
predominance of interests, but the harmonious
balancing thereof."
7 Sanidad v. COMELEC, 73 SCRA 333

FACTS:
So that the purpose for calling the
Constitutional Convention will not be deflated The year is 1976. The 1973 constitution
or frustrated, it is necessary that the delegatee already took effect. Art. XVII on transitory
thereto be independent, beholden to no one provisions provides that an interim National
but to God, country and conscience. Assembly (a legislative body) would be called
but it would be on the discretion of the
6 Planas v. COMELEC, 59 SCRA 105 president based on peace and order when he
shall call that body. Until he calls the body, the
president exercises both executive and
FACTS:
legislative powers.
In the middle of the 1971 Constitutional
On that backdrop, the President made several
Convention, President Marcos declared Martial
proposals to amend the constitution and called
Law. Thereafter he issued Presidential Decree
citizen’s assemblies(barangays) to ratify them.
No. 73 which calls for the holding of the
plebiscite on January 15, 1973. Petitioners
question this decree as according to them, the Hence, this is a petition to stop him from
president does not have the power to schedule amending the constitution. Petitioners argue
the plebiscite and allocate funds therefor. that the power to amend the constitution is a
While the case is pending, President Marcos normal legislative power and just because the
delayed the date of the plebiscite. president has legislative power during martial
law does not mean that he has the power to
propose amendments to the constitution as
ISSUE: is PD 73 constitutional?
well.

Page 6 of 121
403 CASE DIGESTS | CONSTITUTIONAL LAW REVIEW | JUDGE ESTELA ALMA SINGCO | FIRST SEMESTER | 2020-2021

In particular, the amendments are in PD 991 crisis government today are more or less
and 1033 regarding the voting age and the concentrated in the President. 20
length of the effectivity of Martial Law. According to Rossiter, "(t)he concentration of
government power in a democracy faced by an
emergency is a corrective to the crisis
inefficiencies inherent in the doctrine of the
ISSUE: What is the nature of the amendatory separation of powers. In most free states it has
process? Can President Marcos exercise the generally been regarded as imperative that the
amendatory process during Martial Law? total power of the government be parceled out
among three mutually independent branches
executive, legislature, and judiciary. It is
RULING: Yes. There are, therefore, two
believed to be destructive of constitutionalism
periods contemplated in the constitutional life
if any one branch should exercise any two or
of the nation, i.e., period of normalcy and
more types of power, and certainly a total
period of transition. In times of normally, the
disregard of the separation of powers is, as
amending process may be initiated by the
Madison wrote in the Federalist, No. 47, 'the
proposals of the (1) regular National Assembly
very definition of tyranny.' In normal times the
upon a vote of three-fourths of all its
separation of powers forms a distinct
members; or (2) by a Constitutional
obstruction to arbitrary governmental action.
Convention called by a vote of two-thirds of all
By this same token, in abnormal times it may
the Members of the National Assembly.
form an insurmountable barrier to a decisive
However the calling of a Constitutional
emergency action in behalf of the state and its
Convention may be submitted to the electorate
independent existence.
in an election voted upon by a majority vote of
all the members of the National Assembly. In
times of transition, amendments may be 8 Defensor-Santiago v. COMELEC, 270 SCRA
proposed by a majority vote of all the Members 106
of the National Assembly upon special call by
the interim Prime Minister,. FACTS:

The year is 1997. The current constitution is


the 1987 constitution which allows People’s
This Court in Aquino v. COMELEC," had already Initiative as a method of proposing
settled that the incumbent President is vested amendments to the constitution. However,
with that prerogative of discretion as to when there must be an enabling law. Hence, RA
he shall initially convene the interim National 6735 was passed. Santiago challenges this law
Assembly. Speaking for the majority opinion in as insufficient as an enabling law.
that case, Justice Makasiar said: "The
Constitutional Convention intended to leave to The actual controversy is that Atty. Delfin is
the President the determination of the time gathering signatures to lift term limits of
when he shall initially convene the interim elective officials by amending the constitution.
National Assembly, consistent with the
prevailing conditions of peace and order in the ISSUE: Whether or not RA 6735 is a sufficient
country." Concurring, Justice Fernandez, enabling law?
himself a member of that Constitutional
Convention, revealed: "(W)hen the Delegates RULING: No. There is undue delegation of
to the Constitutional Convention voted on the legislative powers.Although Congress may
Transitory Provisions, they were aware of the delegate to another branch of the Government
fact that under the same, the incumbent the power to fill details in the execution,
President was given the discretion as to when enforcement or administration of a law, it is
he could convene the interim National essential, to forestall a violation of the
Assembly; it was so stated plainly by the principle of separation of powers, that said
sponsor, Delegate Yaneza; as a matter of fact, law: (a) be complete in itself — it must set
the proposal that it be convened 'immediately', forth therein the policy to be executed, carried
made by Delegate Pimentel (V) was rejected. out or implemented by the delegate — and (b)
The President's decision to defer the convening to fix standard — the limits of which are
of the interim National Assembly soon found sufficiently determinate or determinable — to
support from the people themselves. In the which the delegate must conform in the
plebiscite of January 10-15, 1973, at which the performance of his functions. Indeed, without
ratification of the 1973 Constitution was a statutory declaration of policy, which is the
submitted, the people voted against the essence of every law, and, without the
convening of the interim National Assembly. In aforementioned standard, there would be no
the referendum of July 24, 1973, the Citizens means to determine, with reasonable
Assemblies ("bagangays") reiterated their certainty, whether the delegate has acted
sovereign will to withhold the convening of the within or beyond the scope of his authority.
interim National Assembly. Again, in the Hence, he could thereby arrogate upon himself
referendum of February 27, 1975, the the power, not only to make the law, but, also
proposed question of whether the interim — and this is worse — to unmake it, by
National Assembly shall be initially convened adopting measures inconsistent with the end
was eliminated, because some of the members sought to be attained by the Act of Congress,
of Congress and delegates of the thus nullifying the principle of separation of
Constitutional Convention, who were deemed powers and the system of checks and
automatically members of the I interim balances, and, consequently, undermining the
National Assembly, were against its inclusion very foundation of our republican system.
since in that referendum of January, 1973, the
people had already resolved against it. The rule is that what has been delegated,
cannot be delegated or as expressed in a Latin
In general, the governmental powers in maxim: potestas delegata non delegari potest.
crisis government the Philippines is a
Page 7 of 121
403 CASE DIGESTS | CONSTITUTIONAL LAW REVIEW | JUDGE ESTELA ALMA SINGCO | FIRST SEMESTER | 2020-2021

59 The recognized exceptions to the rule are indisputable that the amendment now
as follows: proposed to be submitted to a plebiscite is only
the first amendment the Convention propose
(1) Delegation of tariff powers to the President We hold that the plebiscite being called for the
under Section 28(2) of Article VI of the purpose of submitting the same for ratification
Constitution; of the people on November 8, 1971 is not
authorized by Section 1 of Article XV of the
(2) Delegation of emergency powers to the Constitution, hence all acts of the Convention
President under Section 23(2) of Article VI of and the respondent Comelec in that direction
the Constitution; are null and void.

(3) Delegation to the people at large;

(4) Delegation to local governments; and We have arrived at this conclusion for the
following reasons:
(5) Delegation to administrative bodies. 60

1. The language of the constitutional provision


aforequoted is sufficiently clear. lt says
Empowering the COMELEC, an administrative
distinctly that either Congress sitting as a
body exercising quasi-judicial functions, to
constituent assembly or a convention called for
promulgate rules and regulations is a form of
the purpose "may propose amendments to this
delegation of legislative authority under no. 5
Constitution," thus placing no limit as to the
above. However, in every case of permissible
number of amendments that Congress or the
delegation, there must be a showing that the
Convention may propose. The same provision
delegation itself is valid. It is valid only if the
also as definitely provides that "such
law (a) is complete in itself, setting forth
amendments shall be valid as part of this
therein the policy to be executed, carried out,
Constitution when approved by a majority of
or implemented by the delegate; and (b) fixes
the votes cast at an election at which the
a standard — the limits of which are
amendments are submitted to the people for
sufficiently determinate and determinable — to
their ratification," thus leaving no room for
which the delegate must conform in the
doubt as to how many "elections" or plebiscites
performance of his functions. 61 A sufficient
may be held to ratify any amendment or
standard is one which defines legislative
amendments proposed by the same
policy, marks its limits, maps out its
constituent assembly of Congress or
boundaries and specifies the public agency to
convention, and the provision unequivocably
apply it. It indicates the circumstances under
says "an election" which means only one.
which the legislative command is to be
effected. 62

(2) Very little reflection is needed for anyone


to realize the wisdom and appropriateness of
Insofar as initiative to propose amendments to
this provision. As already stated, amending the
the Constitution is concerned, R.A. No. 6735
Constitution is as serious and important an
miserably failed to satisfy both requirements
undertaking as constitution making itself.
in subordinate legislation. The delegation of
Indeed, any amendment of the Constitution is
the power to the COMELEC is then invalid.
as important as the whole of it if only because
the Constitution has to be an integrated and
harmonious instrument, if it is to be viable as
the framework of the government it
9 Tolentino v. COMELEC, 41 SCRA 702 establishes, on the one hand, and adequately
formidable and reliable as the succinct but
FACTS: comprehensive articulation of the rights,
liberties, ideology, social ideals, and national
The year is 1971. The 1971 Constitutional and nationalistic policies and aspirations of the
Convention was inaugurated and they passed people, on the other. lt is inconceivable how a
Organic Resolution no. 1 - amending the 1935 constitution worthy of any country or people
constitution to set the voting to 18. COMELEC can have any part which is out of tune with its
set the plebiscite of the resolution on the same other parts..
date as the senatorial elections. Hence,
petitioners challenge the constitutionality of
Organic Resolution 1.
A constitution is the work of the people thru its
ISSUE: Is OR 1 constitutional? drafters assembled by them for the purpose.
Once the original constitution is approved, the
RULING: No. The ultimate question, therefore part that the people play in its amendment
boils down to this: Is there any limitation or becomes harder, for when a whole constitution
condition in Section 1 of Article XV of the is submitted to them, more or less they can
Constitution which is violated by the act of the assumed its harmony as an integrated whole,
Convention of calling for a plebiscite on the and they can either accept or reject it in its
sole amendment contained in Organic entirety. At the very least, they can examine it
Resolution No. 1? The Court holds that there before casting their vote and determine for
is, and it is the condition and limitation that all themselves from a study of the whole
the amendments to be proposed by the same document the merits and demerits of all or any
Convention must be submitted to the people in of its parts and of the document as a whole.
a single "election" or plebiscite. It being And so also, when an amendment is submitted

Page 8 of 121
403 CASE DIGESTS | CONSTITUTIONAL LAW REVIEW | JUDGE ESTELA ALMA SINGCO | FIRST SEMESTER | 2020-2021

to them that is to form part of the existing did propose the amendments embodied in the
constitution, in like fashion they can study with resolutions now being assailed.
deliberation the proposed amendment in
relation to the whole existing constitution and Moreover, as ruled in Del Rosario v.
or any of its parts and thereby arrive at an Commission on Elections, whether the
intelligent judgment as to its acceptability. Constitutional Convention will only propose
amendments to the Constitution or entirely
overhaul the present Constitution and propose
an entirely new Constitution, is of no moment;
This cannot happen in the case of the because the same will be submitted to the
amendment in question. Prescinding already people for ratification. The fact that the
from the fact that under Section 3 of the present Constitution may be revised and
questioned resolution, it is evident that no replaced with a new one is no argument
fixed frame of reference is provided the voter, against the validity of the law because
as to what finally will be concomitant ‘amendment’ includes the ‘revision’ or total
qualifications that will be required by the final overhaul of the entire Constitution. At any
draft of the constitution to be formulated by rate, whether the Constitution is merely
the Convention of a voter to be able to enjoy amended in part or revised or totally changed
the right of suffrage, there are other would become immaterial the moment the
considerations which make it impossible to same is ratified by the sovereign people.
vote intelligently on the proposed amendment,
although it may already be observed that Now, as to how this power to amend can be
under Section 3, if a voter would favor the exercised, the Constitution is clear. The
reduction of the voting age to eighteen under Interim Batasang Pambansa, sitting as a
conditions he feels are needed under the constituent body, can propose amendments.
circumstances, and he does not see those In that capacity, only a majority vote is needed
conditions in the ballot nor is there any and not three-fourth votes for that is not a
possible indication whether they will ever be or requirement as far as a constitutional
not, because Congress has reserved those for convention is concerned. It is not a
future action, what kind of judgment can he requirement either when, as in this case, the
render on the proposal? Interim Batasang Pambansa exercises its
constituent power to propose amendments.

As to the requisite standard for a proper


10 Occena v. COMELEC, 104 SCRA 1 submission, the question may be viewed not
only from the standpoint of the period that
FACTS: must elapse before the holding of the
plebiscite but also from the standpoint of such
amendments having been called to the
Petitioners Samuel Occena and Ramon A.
attention of the people so that it could not
Gonzales, suing as taxpayers, challenged the
plausibly be maintained that they were
validity of three Batasang Pambansa
properly informed as to the proposed changes.
Resolutions proposing constitutional
As to the period, the Constitution indicates the
amendments and argued that the
way the matter should be resolved. There is no
amendments proposed are so extensive in
ambiguity to the applicable provision: “Any
character that they go far beyond the limits of
amendment to, or revision of, this Constitution
the authority conferred on the Interim
shall be valid when ratified by a majority of the
Batasang Pambansa as Successor of the
votes cast in a plebiscite which shall be held
Interim National Assembly. For them, what
not later than three months after the approval
was done was to revise and not to amend.
of such amendment or revision.” Here, the
three resolutions were approved by the
ISSUE: Whether or not the Interim Batasang Interim Batasang Pambansa sitting as a
Pambansa has the power to propose constituent assembly on February 5 and 27,
amendments and how may it be exercised. 1981. In the Batasang Pambansa Blg. 22, the
date of the plebiscite is set for April 7, 1981. It
RULING: Yes. The applicable provision in the is thus within the 90-day period provided by
1976 Amendments is quite explicit. Insofar as the Constitution.
pertinent it reads thus: “The Interim Batasang
Pambansa shall have the same powers and its 11 Almario v. Alba, 127 SCRA 69
Members shall have the same functions,
responsibilities, rights, privileges, and
FACTS:
disqualifications as the interim National
Assembly and the regular National Assembly
and the Members thereof.” One of such powers As provided for in Batas Pambansa Blg. 643,
is precisely that of proposing amendments. the Filipino electorate will go to the polls on
January 27, 1984 to either approve or reject
amendments to the Constitution proposed by
The 1973 Constitution in its Transitory
Resolution Nos. 104, 105, 110, 111, 112, and
Provisions vested the Interim National
113 of the Batasang Pambansa. The proposed
Assembly with the power to propose
amendments are embodied in four (4)
amendments upon special call by the Prime
separate questions to be answered by simple
Minister by a vote of the majority of its
YES or NO answers. Petitioners herein seek to
members to be ratified in accordance with the
enjoin the submission on January 27, 1984 of
Article on Amendments. When, therefore, the
Question Nos. 3 (“grant” as an additional mode
Interim Batasang Pambansa, upon the call of
of acquiring lands belonging to the public
the President and Prime Minister Ferdinand E.
domain) and 4 (the undertaking by the
Marcos, met as a constituent body its authority
government of a land reform program and a
to do so is clearly beyond doubt. It could and
social reform program), which cover
Resolution Nos. 105 and 113, to the people for
Page 9 of 121
403 CASE DIGESTS | CONSTITUTIONAL LAW REVIEW | JUDGE ESTELA ALMA SINGCO | FIRST SEMESTER | 2020-2021

ratification or rejection on the ground that Constitution was not a free election, hence null
there has been no fair and proper submission and void.
following the doctrine laid down in Tolentino v.
COMELEC. The petitioners do not seek to ISSUE: Whether or not the proposed new or
prohibit the holding of the plebiscite but only revised Constitution has been ratified
ask for more time for the people to study the conformably with Art. XV of the 1935
meaning and implications of Resolution Nos. Constitution.
105 and 113 until the nature and effect of the
proposals are fairly and properly submitted to RULING: No. Under Section 1 of Art. XV of
the electorate. said Constitution, three (3) steps are essential,
namely: (1) That the amendments to the
ISSUE: Whether or not Questions 3 and 4 can Constitution be proposed either by Congress or
be presented to the people on a later date. by a convention called for that purpose, “by a
vote of three-fourths of all the Members of the
RULING: No. The necessity, expediency, and Senate and the House of Representatives
wisdom of the proposed amendments are voting separately,” but “in joint session
beyond the power of the courts to adjudicate. assembled”; (2) That such amendments be
Precisely, whether or not "grant" of public land “submitted to the people for their ratification”
and "urban land reform" are unwise or at an “election”; and (3) That such
improvident or whether or not the proposed amendments be “approved by a majority of
amendments are unnecessary is a matter the votes cast” in said election.
which only the people can decide. The
questions are presented for their Petitioner argued that the last two
determination. Assuming that a member or requirements were not complied with.
some members of this Court may find
undesirable any additional mode of disposing As to the ratification, other provisions of the
of public land or an urban land reform 1935 Constitution concerning “elections”
program, the remedy is to vote "NO" in the must, also, be taken into account, namely,
plebiscite but not to substitute his or their Section I of Art. V and Art. X of said
aversion to the proposed amendments by Constitution.
denying to the millions of voters an
opportunity to express their own likes or
For the age requirement, Section I of Art. V of
dislikes. The issue before us has nothing to do
the 1935 Constitution provides that “Suffrage
with the wisdom of the proposed amendments,
may be exercised by male citizens of the
their desirability, or the danger of the power
Philippines not otherwise disqualified by law,
being abused. The issue is whether or not the
who are twenty-one years of age or over…”
voters are aware of the wisdom, the
Here, it is admitted that persons 15 years of
desirability, or the dangers of abuse. The
age or over, but below 21 years have allegedly
petitioners have failed to make out a case that
ratified the new or revised Constitution drafted
the average voter does not know the meaning
by the 1971 Constitutional Convention. In fact,
of "grant" of public land or of "urban land
the number of people who allegedly voted at
reform.
the Citizens’ Assemblies exceeded the number
of registered voters under the Election Code in
12 Javellana v. Executive secretary, 50 SCRA force in January 1973. Moreover, Art. XV
50 envisages — with the term “votes cast” —
choices made on ballots — not orally or by
FACTS: raising — by the persons taking part in
plebiscites. Hence, the viva voce voting in the
While the Convention was in session, President Citizens’ Assemblies was and is null and void
Marcos issued Proclamation No. 1081 placing ab initio.
the entire Philippines under Martial Law.
Thereafter, the Convention approved its As to the procedure of ratification, Section 2 of
“Proposed Constitution of the Republic of the said Art. X ordains that “the Commission on
Philippines”. And the next day, the President Elections shall have exclusive charge of the
Marcos issued Presidential Decree No. 73, enforcement and administration of all laws
“submitting to the Filipino people for relative to the conduct of elections”. Moreover,
ratification or rejection the Constitution of the it shall decide, save those involving the right
Republic of the Philippines proposed by the to vote, all administrative questions affecting
1971 Constitutional Convention, and election…”; and “all law enforcement agencies
appropriating funds therefor,” as well as and instrumentalities of the Government,
setting the plebiscite for said ratification or when so required by the Commission, shall act
rejection of the Proposed Constitution on as its deputies for the purpose of ensuring
January 15, 1973. free, orderly, and honest elections.” Lastly,
“the decisions, orders, and ruling of the
Javellana through this petition asked the Court Commission” shall not be subject to review,
to restrain petitioner from implementing any of except by the Supreme Court. Here, none of
the provisions of the proposed Constitution not the foregoing provisions was followed by the
found in the present 1935 Constitution. It so-called Barangays or Citizens’ Assemblies.
alleged that the President acted without or in The election was held viva voce, thus depriving
excess of jurisdiction in calling for the the electorate of the right to vote secretly.
immediate implementation of the proposed Moreover, such Barrio Assemblies took place
Constitution on the ground that he is without without the intervention of the Commission on
power to approve the proposed Constitution Elections, and without complying with the
and without power to proclaim the ratification provisions of the Election Code of 197. They
by the Filipino people of the proposed were held under the supervision of the very
Constitution. Moreover, it alleged that that the officers and agencies of the Executive
election held to ratify the proposed Department sought to be excluded therefrom
by Art. X of the 1935 Constitution and the ones
Page 10 of 121
403 CASE DIGESTS | CONSTITUTIONAL LAW REVIEW | JUDGE ESTELA ALMA SINGCO | FIRST SEMESTER | 2020-2021

who would mostly benefit from the provisions may be consummated with Renong Berhad,
in the proposed Constitution. Also, there was petitioner filed a petition before the Court.
no reasonable means of checking the accuracy
of the returns files by the officers who ISSUE: Whether or not Sec. 10, second par.,
conducted said plebiscites. With the Art. XII, of the 1987 Constitution is a self-
aforementioned violations, it renders null and executing provision.
void the contested proceedings or alleged
plebiscite in the Citizens’ Assemblies. RULING: Yes. It is a self-executing provision.
Under the doctrine of constitutional
Lastly, as to the question on the authority of supremacy, if a law or contract violates any
the President to proclaim the ratification of the norm of the constitution, that law or contract
proposed Constitution, the Court held that he whether promulgated by the legislative or by
has no authority to do so. Art. X of the 1935 the executive branch or entered into by private
Constitution was precisely inserted to place persons for private purposes is null and void
beyond the Executive the power to supervise and without any force and effect. Thus, since
or even exercise any authority whatsoever the Constitution is the fundamental,
over “all laws relative to the conduct of paramount and supreme law of the nation, it
elections”. Here, the alleged total results is deemed written in every statute and
tabulated by the Department of Local contract. A provision that lays down a general
Governments and Community Development principle, such as those found in Art. II of the
was turned over to the acting President of the 1987 Constitution, is usually not self-
National Association or Federation who in turn executing. But a provision which is complete in
reported such results to the President who itself and becomes operative without the aid of
proclaimed the said ratification. However, the supplementary or enabling legislation, or that
Solicitor General has submitted no copy of any which supplies sufficient rule by means of
report of the alleged results. Moreover, there which the right it grants may be enjoyed or
is even no certification by the COMELEC in protected, is self-executing.
support of the alleged results of the citizens’
and no certification from the alleged Barangay A constitutional provision is self-executing if
Assemblies were presented. The questions in the nature and extent of the right conferred
the plebiscite were also not proper. In sum, and the liability imposed are fixed by the
the new or revised Constitution proposed by constitution itself, so that they can be
the 1971 Constitutional Convention was not determined by an examination and
ratified in accordance with the provisions of construction of its terms, and there is no
the 1935 Constitution. language indicating that the subject is referred
to the legislature for action. Unless it is
expressly provided that a legislative act is
CONSTRUCTION & INTERPRETATION OF necessary to enforce a constitutional mandate,
THE 1987 CONSTITUTION the presumption now is that all provisions of
13 Manila Prince Hotel vs. Government the constitution are self-executing.
Service Insurance System, 267 SCRA 408
Section 10, second par., Art. XII of the 1987
FACTS: Constitution is a mandatory positive
command, which is complete in itself and
which needs no further guidelines or
A petition for prohibition and mandamus was
implementing laws or rules for its
filed by Manila Prince Hotel against GSIS. The
enforcement. From its very words, the
controversy arose when respondent
provision does not require any legislation to
Government Service Insurance System
put it in operation. It is per se judicially
(GSIS), pursuant to the privatization program
enforceable. When our Constitution mandates
of the Philippine Government, decided to sell
that in the grant of rights, privileges, and
through public bidding 30% to 51% of the
concessions covering national economy and
issued and outstanding shares of respondent
patrimony, the State shall give preference to
Manila Hotel Corporation (MHC). The winning
qualified Filipinos; it means just that –
bidder, or the eventual “strategic partner,” will
qualified Filipinos shall be preferred. In other
provide management expertise or an
words, when the Constitution speaks of
international marketing/reservation system,
“patrimony” it simply means “heritage.” Thus,
and financial support to strengthen the
when we speak of “national patrimony”, we
profitability and performance of the Manila
refer not only to the natural resources of the
Hotel.
Philippines but as well as the cultural heritage
of the Filipino people.
In a close only two (2) bidders participated:
petitioner Manila Prince Hotel Corporation, a
Filipino corporation, which offered to buy 51%
14 Magallona v. Ermita, 655 SCRA 476
of the MHC or 15,300,000 shares at P41.58 per
share, and Renong Berhad, a Malaysian firm,
with ITT-Sheraton as its hotel operator, which
bid for the same number of shares at P44.00 FACTS:
per share, or P2.42 more than the bid of
petitioner. Prior to the declaration of Renong RA 3046 was passed in 1961 which provides,
Berhard as the winning bidder, petitioner among others, the demarcation lines of the
Manila Prince Hotel matched the bid price and baselines of the Philippines as an archipelago.
sent a manager’s check as bid security, which This is in consonance with UNCLOS I. RA 5446
GSIS refused to accept. amended RA 3046 in terms of typographical
errors and included Section 2 in which the
Apprehensive that GSIS has disregarded the government reserved the drawing of baselines
tender of the matching bid and that the sale in Sabah in North Borneo.

Page 11 of 121
403 CASE DIGESTS | CONSTITUTIONAL LAW REVIEW | JUDGE ESTELA ALMA SINGCO | FIRST SEMESTER | 2020-2021

RA 9522 took effect in March 2009 amending William Marbury (Marbury), an end-of-term
RA 5446. The amendments, which are in appointee of President John Adams (President
compliance with UNCLOS III in which the Adams) to a justice of the peace position in the
Philippines is one of the signatory, shortening District of Columbia, brought suit against
one baseline while optimizing the other and President Thomas Jefferson’s (President
classifying Kalayaan Group of Island and Jefferson) Secretary of State, James Madison,
Scarborough Shoal as Regimes of Island. seeking delivery of his commission.

Petitioners in their capacity as taxpayer, ISSUE:


citizen and legislator assailed the
constitutionality of RA 9522. It argued among Whether or not the applicant has a vested right
others, that RA 9522, "converts "internal in the commission he demands?
waters into archipelagic waters, hence
subjecting these waters to the right of RULING: No, the Court has no jurisdiction to
innocent and sea lanes passage under UNCLOS do so. Although the Court determined that the
III, including overflight. Petitioners applicant had a vested legal right in his
extrapolate that these passage rights appointment because his commission had
indubitably expose Philippine internal waters been signed by the President, sealed by the
to nuclear and maritime pollution hazards, in Secretary of State, and the appointment was
violation of Article II, Section 7, Section 8, and not revocable and that Section 13 of the
Section 16 of the Constitution. Judiciary Act of 1789 provided that such writs
might be issued by the Supreme Court to cases
ISSUE: Whether or not Article II, Section 7, including this present case. Still, SC held that
Section 8, and Section 16 of the Constitution Article III of the Constitution provides that
can be invoked to assail the constitutionality of such power to issue the said writ applies only
RA 9522. to cases "affecting ambassadors, other public
ministers and consuls" and to cases "in which
RULING: No. The Court emphasized that the the state shall be party. In all other cases, the
Philippines exercises sovereignty over the supreme court shall have appellate
body of water lying landward of the baselines, jurisdiction."
including the air space over it and the
submarine areas underneath, regardless Thus, § 13 of the Act of 1789, giving the Court
whether internal or archipelagic waters. authority to issue writs of mandamus to an
However, sovereignty will not bar the officer, was contrary to the Constitution as an
Philippines from complying with its obligation act of original jurisdiction, and therefore void.
in maintaining freedom of navigation and the It is emphatically the province and duty of the
generally accepted principles of international judicial department to say what the law is.
law. It can be either passed by legislature as a Those who apply the rule to particular cases
municipal law or in the absence thereof, it is must necessarily expound and interpret that
deemed incorporated in the Philippines law rule. If two laws conflict with each other, the
since the right of innocent passage is a courts must decide on the operation of each.
customary international law, thus The constitution is superior to any ordinary act
automatically incorporated thereto. of the legislature and here by extending the
Court's original jurisdiction to include cases
Petitioners’ invocation of non-executory like Marbury's, Congress had exceeded its
constitutional provisions in Article II authority. And when an act of Congress is in
(Declaration of Principles and State Policies) conflict with the Constitution, it is the
must also fail. Our present state of obligation of the Court to uphold the
jurisprudence considers the provisions in Constitution because, by Article VI, it is the
Article II as mere legislative guides, which, "supreme law of the land.
absent enabling legislation, "do not embody
judicially enforceable constitutional rights x x
x." Article II provisions serve as guides in
formulating and interpreting implementing 15 THE MARITIME BASELINES LAW (R.A. NO.
legislation, as well as in interpreting executory 9522)
provisions of the Constitution. Although Oposa 16 Professor Merlin M. Magallona, et al. v.
v. Factoran treated the right to a healthful and Hon. Eduardo Ermita, et al., G.R. No.
balanced ecology under Section 16 of Article II 187167, 655 SCRA 476, August 16, 2011,
as an exception, the present petition lacks En Banc [Carpio]
factual basis to substantiate the claimed
constitutional violation. The other provisions
petitioners cite, relating to the protection of FACTS:
marine wealth (Article XII, Section 2,
paragraph 2) and subsistence fishermen RA 3046 was passed in 1961 which provides,
(Article XIII, Section 7), are not violated by among others, the demarcation lines of the
RA9522. baselines of the Philippines as an archipelago.
This is in consonance with UNCLOS I. RA 5446
15 Marbury v. Madison (1803) amended RA 3046 in terms of typographical
errors and included Section 2 in which the
FACTS: government reserved the drawing of baselines
in Sabah in North Borneo.
Before the inauguration of President Jefferson,
outgoing President Adams nominated 42 new RA 9522 took effect in March 2009 amending
justices of the peace for the District, which RA 5446. The amendments, which are in
were confirmed by the Senate the day before compliance with UNCLOS III in which the
President Jefferson’s inauguration. Philippines is one of the signatory, shortening
one baseline while optimizing the other and

Page 12 of 121
403 CASE DIGESTS | CONSTITUTIONAL LAW REVIEW | JUDGE ESTELA ALMA SINGCO | FIRST SEMESTER | 2020-2021

classifying Kalayaan Group of Island and emphasized that the Philippines exercises
Scarborough Shoal as Regimes of Island. sovereignty over the body of water lying
landward of the baselines, including the air
Petitioners in their capacity as taxpayer, space over it and the submarine areas
citizen and legislator assailed the underneath, regardless whether internal or
constitutionality of RA 9522:- it reduces the archipelagic waters. However, sovereignty will
territory of the Philippines in violation to the not bar the Philippines from complying with its
Constitution and it opens the country to obligation in maintaining freedom of
maritime passage of vessels and aircrafts of navigation and the generally accepted
other states to the detriment of the economy, principles of international law. It can be either
sovereignty, national security and of the passed by legislature as a municipal law or in
Constitution as well. They added that the the absence thereof, it is deemed incorporated
classification of Regime of Islands would be in the Philippines law since the right of
prejudicial to the lives of the fishermen. innocent passage is a customary international
law, thus automatically incorporated thereto.
ISSUE: Whether or not RA 9522 is
unconstitutional. This does not mean that the states are placed
in a lesser footing; it just signifies concession
RULING: Yes. First, RA 9522 did not delineate of archipelagic states in exchange for their
the territory of the Philippines but is merely a right to claim all waters inside the baseline. In
statutory tool to demarcate the country’s fact, the demarcation of the baselines enables
maritime zone and continental shelf under the Philippines to make maritime delineation
UNCLOS III. SC emphasized that UNCLOS III which binds the international community.
is not a mode of acquiring or losing a territory
as provided under the laws of nations. UNCLOS
III is a multilateral treaty that is a result of a 17 BANGSAMORO JURIDICAL ENTITY
long-time negotiation to establish a uniform CONCEPT
sea-use rights over maritime zones (i.e., the 18 PROVINCE OF NORTH COTABATO VS.
territorial waters [12 nautical miles from the GRP, G.R. NO. 183591 OCTOBER 14 2008
baselines], contiguous zone [24 nautical miles
from the baselines], exclusive economic zone FACTS:
[200 nautical miles from the baselines]), and This is a consolidation of a number of cases
continental shelves. In order to measure said regarding the issues, mostly in relation to its
distances, it is a must for the state parties to constitutionality, surrounding the
have their archipelagic doctrines measured in Memorandum of Agreement on the Ancestral
accordance to the treaty—the role played by Domain (MOA-AD) between the Government
RA 9522. The contention of the petitioner that of the Philippines (GPR) and the Moro Islamic
RA 9522 resulted in the loss of 15,000 square Liberation Front (MILF) and the issue
nautical miles is devoid of merit. The truth is, regarding the extent of the powers of the
RA 9522, by optimizing the location of base President in pursuing the peace process.
points, increased the Philippines total maritime
space of 145,216 square nautical miles. The MO-AD provides, among others, ¬-- (1)
the “Bangsamoro Homeland” – the ownership
Second, the classification of KGI and of which is vested exclusively in the
Scarborough Shoal as Regime of Islands is Bangsamoro people by virtue of their prior
consistent with the Philippines’ sovereignty. rights of occupation; (2) Bangsamoro people
Had RA 9522 enclosed the islands as part of have a right to self-governance; (3) its
the archipelago, the country would be violating territory includes the land mass, maritime,
UNCLOS III since it categorically stated that terrestrial, fluvial, alluvial including aerial
the length of the baseline shall not exceed 125 domain and atmospheric space embracing
nautical miles. So what the legislators did is to Mindanao- Sulu-Palawan geographic region;
carefully analyze the situation: the country, for (4) that the Bangsamoro Judicial Entity (BJE)
decades, had been claiming sovereignty over shall have jurisdiction over all natural
KGI and Scarborough Shoal on one hand and resources within its internal waters and
on the other hand they had to consider that territorial waters from beyond this up to
these are located at non-appreciable distance baselines of RP – South East and South West
from the nearest shoreline of the Philippine of mainland Mindanao; and within territorial
archipelago. So, the classification is in waters, there is joint jurisdiction with the
accordance with the Philippines sovereignty Philippine government and sharing of minerals
and State’s responsible observance of its pacta therein. Moreover, (5) BJE is free to enter into
sunt servanda obligation under UNCLOS III. any economic cooperation/ trade relations with
foreign countries and can establish foreign
Third, the new base line introduced by RA 9522 trade missions to other countries although the
is without prejudice with delineation of the external defense remains the duty and
baselines of the territorial sea around the obligation of the Philippine government. It also
territory of Sabah, situated in North Borneo, (6) defines the relationship of the Central
over which the Republic of the Philippines has Government and BJE as “associative”
acquired dominion and sovereignty. characterized by shared authority and
responsibility.
Lastly, the UNCLOS III and RA 9522 are not
incompatible with the Constitution’s ISSUE: Whether or not MOA-AD is
delineation of internal waters. Petitioners constitutional.
contend that RA 9522 transformed the internal
waters of the Philippines to archipelagic waters RULING: No. The concept of association is not
hence subjecting these waters to the right of recognized under the present constitution. No
innocent and sea lanes passages, exposing the province, city, or municipality, not even the
Philippine internal waters to nuclear and ARMM, is recognized under our laws as having
maritime pollution hazards. The Court an “associative” relationship with the national
Page 13 of 121
403 CASE DIGESTS | CONSTITUTIONAL LAW REVIEW | JUDGE ESTELA ALMA SINGCO | FIRST SEMESTER | 2020-2021

government. Indeed, the concept implies RULING: Yes. The Supreme Court ruled that
powers that go beyond anything ever granted he is a Filipino Citizen. By virtue of Article IV
by the Constitution to any local or regional of the Constitution which provides: SECTION
government. Association implies that the 1. The following are citizens of the Philippines:
associative entity is a state, and that it is in (2) Those born in the Philippine Islands of
the position to become independent. The foreign parents who, before the adoption of
Constitution, however, does not contemplate this Constitution, had been elected to public
any state in this jurisdiction other than the office in the Philippine Islands.
Philippine State, much less does it provide for
a transitory status that aims to prepare any In the case at bar, Victoriano Chiongbian,
part of Philippine territory for independence. father of herein petitioner, having been elected
Indeed, BJE is a state in all but name as it to a public office in the Philippines before the
meets the criteria of a state laid down in the adoption of the Constitution, became a Filipino
Montevideo Convention, namely, a permanent citizen by virtue of Article IV, section 1,
population, a defined territory, a government, subsection 2 of the Constitution. William
and a capacity to enter into relations with Chiongbian, the herein petitioner, who was
other states. The defining concept underlying then a minor, also became a Filipino citizen by
the relationship between the national reason of subsection 3 (Article IV) of the
government and the BJE being itself contrary Constitution, his father having become a
to the present Constitution, the specific Filipino citizen upon the adoption of said
provisions of the MOA-AD on the formation and Constitution. This is also in conformity with the
powers of the BJE are in conflict with the settled rule of our jurisprudence that a
Constitution and the laws. legitimate minor child follows the citizenship of
his father.
MOA-AD is inconsistent with the Constitution
and laws for it exceed those granted to any As to the first argument of the respondent:
Local Government Unit (LGU). Even if BJE is subsection was adopted by the Constitutional
considered as an autonomous region, the Convention merely to grant Filipino citizenship
MOA-AD would require a constitutional to Delegate Caram is that members of the
amendment in order to vest BJE with treaty Constitutional Convention could not have
making power. While Article X Section 20 dedicated a provision of our Constitution
provides that the organic acts of the merely for the benefit of one person without
autonomous regions shall provide legislative considering that it could also affect others.
powers over such other matters authorized by Also, they adopted subsection 2, they
law for the promotion of the general welfare of permitted, if not willed, that said provision
the people, the same must be subject to the should function to the full extent of its
provisions of the Constitution. And since under substance and its terms, not by itself alone,
our constitutional system, only the President but in conjunction with all other provisions of
has the sole authority to negotiate with other that great document.
states as regards treaty-making, then a
constitutional amendment is needed to grant a
Secondly, respondents argued that the original
similar power to BJE. Lastly, it is not in
draft of said subsection 2 contained the phrase
consonance with a number of statutes. Thus,
— "and their descendants," — which was
it is unconstitutional.
deleted from the final draft, thus showing that
this privilege of citizenship was intended to be
CITIZENSHIP – 1935 CONSTITUTION strictly personal to the one who had been
elected to public office and did not extend to
19 Fermin Caram Provision; Chiongbian vs. his descendants. However SC ruled that mere
De Leon (G.R. No. L-2007, January 31, deletion of the phrase — "and their
1949 descendants," — is not determinative of any
conclusion. It could have been done because
the learned framers of our Constitution
FACTS: considered it superfluous, knowing full well
that the meaning of such a phrase was
This is a petition seeking to permanently adequately covered by subsection 3.
prohibit respondent Customs Officials from
cancelling the registration certificates of Lastly, Respondents' allegation that the
petitioner's vessels, and respondent Philippine petitioner violated the contract of sale with the
Shipping Administration from rescinding the Philippine Shipping Administration on the
sale of three vessels to petitioner. ground of misrepresentation is without merit
and that such was not a deliberate
The primary basis for respondents' and misrepresentation but an error and in which
intervenor's acts is the allegation that any person not versed in the law is prone to
petitioner is not a Filipino citizen and therefore commit. It is clear that petitioner merely
not qualified by law to operate and own vessels meant that his father was a Filipino citizen by
of Philippine registry. The Philippine Shipping operation of law and not by birth.
Administration also alleges that petitioner
violated the contract of sale of three vessels Petition for the issuance of the writ of
executed between them, on the ground of prohibition is hereby granted.
misrepresentation, petitioner having alleged in
said contract that his father was a naturalized 20 Rep. vs. Chule Lim, GR No. 153883,
Filipino citizen. The Philippine Shipowners' January 13, 2004
Association was later allowed to intervene and
it filed its answer against the petitioner. FACTS:
Chule Y. Lim filed a petition for correction of
ISSUE: Whether or not Petitioner William entries under Rule 108 of the Rules of Court
Chiongbian is a Filipino citizen. with the Regional Trial Court of Lanao del
Norte. She claimed that she was born on 29
Page 14 of 121
403 CASE DIGESTS | CONSTITUTIONAL LAW REVIEW | JUDGE ESTELA ALMA SINGCO | FIRST SEMESTER | 2020-2021

October 1954 in Buru-an, Iligan City. Her birth Northern Samar.


was registered in Kauswagan, Lanao del Norte
but the Municipal Civil Registrar of Kauswagan The petitioners filed election protests against
transferred her record of birth to Iligan City. the private respondent premised on the
following grounds:
During the hearing, Lim claims that her
nationality was entered as Chinese when it 1) Jose Ong, Jr. is not a natural-born citizen of
should have been Filipino considering that her the Philippines; and
father and mother never got married. Only her 2) Jose Ong Jr. is not a resident of the second
deceased father was Chinese, while her district of the Northern Samar.
mother is Filipina. She claims that her being a
registered voter attests to the fact that she is The HRET in its decision dated November 6,
a Filipino citizen. 1989, found for the private respondent.
Hence, these petitions for certiorari.
The Republic opposes, claiming that
respondent did not comply with the ISSUE:
constitutional requirement of electing Filipino Whether or not Jose Ong, Jr. is a natural-born
citizenship when she reached the age of citizen of the Philippines
majority. It cites Article IV, Section 1(3) of the
1935 Constitution, which provides that the RULING:
citizenship of a legitimate child born of a YES, Jose Ong Jr. is a natural-born citizen of
Filipino mother and an alien father followed the the Philippines. The records show that in the
citizenship of the father, unless, upon reaching year 1895, Ong Te (respondent’s grandfather),
the age of majority, the child elected Philippine arrived in the Philippines from China. Ong Te
citizenship. Likewise, the Republic invokes the established his residence in the municipality of
provision in Section 1 of Commonwealth Act Laoang, Samar on land which he bought from
No. 625, that legitimate children born of the fruits of hard work. Ong Te was able to
Filipino mothers may elect Philippine obtain a certificate of residence from then
citizenship by expressing such intention “in a Spanish colonial administration.
statement to be signed and sworn to by the
party concerned before any officer authorized The father of the private respondent, Jose Ong
to administer oaths, and shall be filed with the Chuan was born in China in 1905 and was
nearest civil registry. The said party shall brought by Ong Te to Samar in the year 1915.
accompany the aforesaid statement with the Jose Ong Chuan spent his childhood in the
oath of allegiance to the Constitution and the province of Samar. As Jose Ong Chuan grew
Government of the Philippines.” older in the rural and seaside community of
Laoang, he absorbed Filipino cultural values
ISSUES: and practices. He was baptized into
Christianity.
Whether or not Lim is a Filipino citizen.
As the years passed, Jose Ong Chuan met a
RULING: natural-born Filipino, Agripina Lao and the two
fell in love. They got married in 1932 according
Plainly, the above constitutional and statutory to Catholic faith and practice. The couple bore
requirements of electing Filipino citizenship eight children, one of whom is the private
apply only to legitimate children. These do not respondent who was born in 1948. In the
apply in the case of respondent who was meantime, the father of the private
concededly an illegitimate child, considering respondent, unsure of his legal status and in
that her Chinese father and Filipino mother an unequivocal affirmation of where he cast his
were never married. As such, she was not life and family, filed with the Court of First
required to comply with said constitutional and Instance of Samar an application for
statutory requirements to become a Filipino naturalization on February 15, 1954. On April
citizen. By being an illegitimate child of a 28, 1955, the CFI of Samar, after trial,
Filipino mother, respondent automatically declared Jose Ong Chuan a Filipino citizen.
became a Filipino upon birth. Stated
differently, she is a Filipino since birth without On May 15, 1957, the Court of First Instance
having to elect Filipino citizenship when she of Samar issued an order declaring the
reached the age of majority. decision of April 28, 1955 as final and
executory and that Jose Ong Chuan may
This notwithstanding, the records show that already take his Oath of Allegiance.
respondent elected Filipino citizenship when
she reached the age of majority. She The pertinent portions of the Constitution
registered as a voter in Misamis Oriental when found in Article IV read:
she was 18 years old. The exercise of the right
of suffrage and the participation in election
exercises constitute a positive act of election SECTION 1, the following are citizens of the
of Philippine citizenship. Philippines:

21 Co vs. HRET (G.R. No. 92191-92, July 30,


1991)
1. Those who are citizens of the
FACTS: Philippines at the time of the adoption
Among the candidates who vied for the of the Constitution;
position of representative in the second 2. Those whose fathers or mothers are
legislative district of Northern Samar are the citizens of the Philippines;
petitioners, Sixto Balinquit and Antonio Co and 3. Those born before January 17, 1973,
the private respondent, Jose Ong, Jr. of Filipino mothers, who elect
Respondent was proclaimed the duly elected Philippine citizenship upon reaching
representative of the second district of the age of majority; and
Page 15 of 121
403 CASE DIGESTS | CONSTITUTIONAL LAW REVIEW | JUDGE ESTELA ALMA SINGCO | FIRST SEMESTER | 2020-2021

4. Those who are naturalized in Philippine citizenship pursuant to Article IV,


accordance with law. section I(4) of the Constitution and
Commonwealth Act No. 625. Said election
appears in an affidavit dated May 15, 1951,
The Court interprets Section 1, Paragraph 3 stating that petitioner was born in Dapa,
above as applying not only to those who elect Surigao, on February 16, 1923; that his
Philippine citizenship after February 2, 1987 parents are "Benito Dy Cuenco, Chinese (now
but also to those who, having been born of deceased)" and "Julita Duyapit, Filipina, a
Filipino mothers, elected citizenship before native of Surigao, Philippines"; that he is
that date. The provision in question was "married to Rosalinda Villanueva, a Filipino,"
enacted to correct the anomalous situation by whom he has four (4) legitimate children;
where one born of a Filipino father and an alien that he renounces all allegiance to the Republic
mother was automatically granted the status of China; that he recognizes and accepts the
of a natural-born citizen while one born of a supreme authority of the Republic of the
Filipino mother and an alien father would still Philippines and will maintain true faith and
have to elect Philippine citizenship. allegiance thereto; and that he will obey,
support and defend the Constitution and laws
of the Philippines. On the same date,
petitioner, likewise, took the corresponding
22 In re: Vicente Ching, Bar Matter 914, oath of allegiance to the Republic of the
October 1, 1999 Philippines.
FACTS: The Commissioner of Immigration referred the
Vicente D. Ching, a legitimate child of a Filipino matter to the Secretary of Justice who,
mother and an alien Chinese father, was born rendered an opinion (No. 129) holding that the
on April 11, 1964 in Tubao La Union, under the alleged Philippine citizenship of petitioner's
1935 Constitution. mother had not been sufficiently established,
that said election of Philippine citizenship by
He completed his Bachelor of Laws at SLU in petitioner herein was legally ineffectual and
Baguio on July 1998, filed an application to that he did not thereby become a Filipino
take the 1998 Bar citizen.
Examination. On April 5, 1999, Ching was one
of the bar passers. The oath taking ceremony ISSUE:
was scheduled on May 5, 1999. Because of his Whether or not the election of Philippine
questionable status of Ching's citizenship, he Citizenship of the petitioner is valid.
was not allowed to take oath. He was required
to submit further proof of his citizenship. RULING:
No. Election must be made within a reasonable
The Office of the Solicitor General commented period after reaching the age of majority.
that Ching, under the 1935 Constitution, was Three years is a reasonable period, however,
a Chinese citizen and continues to be so, this may be extended under certain
unless upon reaching the age of majority he circumstances as when the person concerned
elected Philippine citizenship, under the has always considered himself a Filipino
compliance with the provisions of citizen. In the case at bar, the petitioner
Commonwealth Act No. 265. He pointed out reached the age of majority in 1944, but he
the Ching has not formally elected Philippine made his election of Filipino citizenship 1951
citizenship, and if ever he does, it would when he was 28 years old, 7 years after he
already be beyond the "reasonable time" reached the age of majority.
allowed by the present jurisprudence.
24 Ma vs. Fernandez, July 26, 2010
ISSUE:
Whether or not he has elected Philippine FACTS:
citizenship within a reasonable time.
Petitioners Felix, Jr., Balgamelo and Valeriano
RULING: were all born under aegis of the 1935
Ching, despite the special circumstances, Philippine Constitution in the years 1948,
failed to elect Philippine citizenship within a 1951, and 1957, respectively.
reasonable time. The reasonable time means
that the election should be made within 3 They were all raised in the Philippines and have
years from "upon reaching the age of resided in this country for almost sixty (60)
majority", which is 21 years old. Instead, he years; they spent their whole lives, studied
elected Philippine citizenship 14 years after and received their primary and secondary
reaching the age of majority which the court education in the country; they do not speak
considered not within the reasonable time. nor understand the Chinese language, have
Ching offered no reason why he delayed his not set foot in Taiwan, and do not know any
election of Philippine citizenship, as procedure relative of their father; they have not even
in electing Philippine citizenship is not a traveled abroad; and they have already raised
tedious and painstaking process. All that is their respective families in the Philippines.
required is an affidavit of election of Philippine
citizenship filed with the nearest civil registry. During their age of minority, they secured
from the Bureau of Immigration their Alien
23 Dy Cuenco vs. Sec of Justice, May 26, Certificates of Registration (ACRs).
1962
Immediately upon reaching the age of twenty-
one, they claimed Philippine citizenship in
FACTS:
accordance with Section 1(4), Article IV, of the
Alfonso Dy Cuenco wrote to the Commissioner
1935 Constitution. All three took their oath of
of Immigration requesting the cancellation of
allegiance to the Republic of the Philippines.
his alien certificate of registration, upon the
ground that he had exercised the right to elect Later on, a certain Mat G. Catral (Mr. Catral),
Page 16 of 121
403 CASE DIGESTS | CONSTITUTIONAL LAW REVIEW | JUDGE ESTELA ALMA SINGCO | FIRST SEMESTER | 2020-2021

alleged that Felix (Yao Kong) Ma and his seven elected Philippine citizenship. Consequently,
(7) children are undesirable and overstaying she sought a judicial declaration of her election
aliens. Hence, the Legal Department of the of Philippine citizenship averring that she was
Bureau of Immigration charged them for raised as a Filipino and she is a registered
violation of the Philippine Immigration Act of voter in Baguio City and had voted in local and
1940. national elections as shown in the Voter
Certification. She asserted that by virtue of her
ISSUE: positive acts, she has effectively elected
Philippine citizenship and such fact should be
Whether or not petitioners are Filipino citizens.
annotated on her record of birth so as to entitle
Ruling: her to the issuance of a Philippine passport.

The 1935 Constitution declares as citizens of The OSG opposed, pointing out that while
the Philippines those whose mothers are Sagun executed an oath of allegiance before a
citizens of the Philippines and elect Philippine notary public, there was no affidavit of her
citizenship upon reaching the age of majority. election of Philippine citizenship. Additionally,
The mandate states: her oath of allegiance which was not registered
with the nearest local civil registry was
Section 1. The following are citizens of the executed when she was already 33 years old
Philippines: or 12 years after she reached the age of
majority.
(1) xxx;... x x x x
ISSUE:
(4) Those whose mothers are citizens of the
Whether or not Norma has complied with the
Philippines and, upon reaching the age of
majority, elect Philippine citizenship. procedural requirements in the election of
Philippine citizenship.
In 1941, Commonwealth Act No. 625 was
enacted. It laid down the manner of electing RULING:
Philippine citizenship, to wit: When respondent was born on August 8, 1959,
the governing charter was the 1935
Section 1. The option to elect Philippine Constitution, which declares as citizens of the
citizenship in accordance with subsection (4), Philippines those whose mothers are citizens
Section 1, Article IV, of the Constitution shall of the Philippines and elect Philippine
be expressed in a statement to be signed and citizenship upon reaching the age of majority.
sworn to by the party concerned before any Sec. 1, Art. IV of the 1935 Constitution reads:
officer authorized to administer oaths, and
shall be filed with the nearest civil registry. The Section 1. The following are citizens of the
said party shall accompany the aforesaid Philippines:
statement with the oath of allegiance to the xxxx
Constitution and the Government of the (4) Those whose mothers are citizens of the
Philippines Philippines and, upon reaching the age of
majority, elect Philippine citizenship.
Notably, the petitioners timely took their oath
of allegiance to the Philippines. This was a Under Article IV, Section 1(4) of the 1935
serious undertaking. It was commitment and Constitution, the citizenship of a legitimate
fidelity to the state coupled with a pledge "to child born of a Filipino mother and an alien
renounce absolutely and forever all allegiance" father followed the citizenship of the father,
to any other state. This was unqualified unless, upon reaching the age of majority, the
acceptance of their identity as a Filipino and child elected Philippine citizenship.
the complete disavowal of any other
nationality. Commonwealth Act (C.A.) No. 625, enacted
pursuant to Section 1(4), Article IV of the 1935
Petitioners have passed decades of their lives
Constitution, prescribes the procedure that
in the Philippines as Filipinos. Their present
should be followed in order to make a valid
status having been formed by their past,
election of Philippine citizenship, to wit:
petitioners can no longer have any national
identity except that which they chose upon
Section 1. The option to elect Philippine
reaching the age of reason.
citizenship in accordance with subsection (4),
Section 1, Article IV, of the Constitution shall
25 Rep. vs. Sagun, Feb. 15. 2011 be expressed in a statement to be signed and
sworn to by the party concerned before any
FACTS: officer authorized to administer oaths, and
Nora Fe Sagun is the legitimate child of Albert shall be filed with the nearest civil registry. The
S. Chan, a Chinese national, and Marta said party shall accompany the aforesaid
Borromeo, a Filipino citizen. She was born on statement with the oath of allegiance to the
August 8, 1959 in Baguio City and did not elect Constitution and the Government of the
Philippine citizenship upon reaching the age of Philippines.
majority. In 1992, at the age of 33 and after
getting married to Alex Sagun, she executed Based on the foregoing, the statutory
an Oath of Allegiance to the Republic of the formalities of electing Philippine citizenship
Philippines. Said document was notarized but are: (1) a statement of election under oath;
was not recorded and registered with the Local (2) an oath of allegiance to the Constitution
Civil Registrar of Baguio City. and Government of the Philippines; and (3)
registration of the statement of election and of
In 2005, Sagun applied for a Philippine the oath with the nearest civil registry.
passport. Her application was denied due to
the citizenship of her father and there being no Furthermore, no election of Philippine
annotation on her birth certificate that she has citizenship shall be accepted for registration
under C.A. No. 625 unless the party exercising
Page 17 of 121
403 CASE DIGESTS | CONSTITUTIONAL LAW REVIEW | JUDGE ESTELA ALMA SINGCO | FIRST SEMESTER | 2020-2021

the right of election has complied with the Allan’s son to Bessie, though an American
requirements of the Alien Registration Act of citizen, was a Filipino citizen by virtue of
1950. In other words, he should first be paternal filiation as evidenced by the
required to register as an alien. Pertinently, respondent’s birth certificate. The 1935
the person electing Philippine citizenship is Constitution on citizenship did not make a
required to file a petition with the Commission distinction on the legitimacy or illegitimacy of
of Immigration and Deportation (now Bureau the child, thus, the allegation of bigamous
of Immigration) for the cancellation of his alien marriage and the allegation that respondent
certificate of registration based on his was born only before the assailed marriage
aforesaid election of Philippine citizenship and had no bearing on respondent’s citizenship in
said Office will initially decide, based on the view of the established paternal filiation
evidence presented the validity or invalidity of evidenced by the public documents presented.
said election. Afterwards, the same is elevated
to the Ministry (now Department) of Justice for But while the totality of the evidence may not
final determination and review. establish conclusively that respondent FPJ is a
natural-born citizen of the Philippines, the
The trial court’s conclusion that respondent evidence on hand still would preponderate in
duly elected Philippine citizenship is erroneous his favor enough to hold that he cannot be held
since the records indisputably show that guilty of having made a material
respondent failed to comply with the legal misrepresentation in his certificate of
requirements for a valid election. Specifically, candidacy in violation of Section 78, in relation
respondent had not executed a sworn to Section 74 of the Omnibus Election Code.
statement of her election of Philippine
citizenship. The only documentary evidence 27 Valles vs. COMELEC, GR No. 137000,
submitted by respondent in support of her August 9, 2000
claim of alleged election was her oath of
allegiance, executed 12 years after she FACTS:
reached the age of majority, which was Rosalind Ybasco Lopez was born on May 16,
unregistered. 1934 in Australia to an Australian mother and
a Filipino father. On June 27 1952 she married
Leopoldo Lopez a Filipino citizen and has been
actively participating in the electoral process.
JONES LAW OF 1916
26 Tecson vs. COMELEC (G.R. No.161434, In 1992 she ran for governor of Davao oriental,
March 3, 2004) which was contested by her opponent on the
ground of her Australian citizenship. The case
FACTS: was dismissed because she was able to
The petitions at bar seek the disqualification of produce enough documentary evidence to
respondent Fernando Poe, Jr.’s COC filed last support the Filipino citizenship of her late
December 31, 2003. father therefore proving her own citizenship
through Jus Sanguinis.
On January 9, 2004, petitioner in G.R. No.
161824, Victorino X. Fornier, filed with the In 1998 the issue was raised again when she
COMELEC a Petition for Disqualification. ran for re-election as governor and the
Petitioner argues that marriage of FPJ’s Comelec again dismissed the case stating that
parents [Allan Fernando(Spanish) Poe & no new evidence was presented. Petitioner
Bessie Kelley(American)] was void, due to his filed a motion for reconsideration which was
father’s first marriage with one Paulita Gomez. denied. on same date
This means that FPJ is an illegitimate child and
should follow his ISSUE:
mother’s nationality (American). Whether or not the respondent is a Filipino
Citizen and is allowed to run for public office.
ISSUES:
Whether or not FPJ is a Filipino citizen. RULING:
By virtue of the principle of Jus Sanguinis, the
RULING: private respondent is a Filipino Citizen because
The 1935 Constitution on Citizenship, the her father was a Filipino Citizen by virtue of the
prevailing fundamental law on respondent’s Jones Law and the Philippine Bill of 1902. The
birth, provided that among the citizens of the fact that she is born in Australia is not
Philippines are "those whose fathers are tantamount to her losing her Philippine
citizens of the Philippines." Citizenship.

Tracing respondent’s paternal lineage, his


grandfather Lorenzo, as evidenced by the
CITIZENSHIP - RES JUDICATA
latter’s death certificate was identified as a
Filipino Citizen. His citizenship was also drawn 28 Roa vs. Collector of Customs, 23 Phil.,315
from the presumption that having died in 1954
at the age of 84, Lorenzo would have been FACTS:
born in 1870. In the absence of any other Tranquilino Roa, was born in the town of
evidence, Lorenzo’s place of residence upon Luculan, Mindanao, Philippine Islands, on July
his death in 1954 was presumed to be the 6, 1889. His father was Basilio Roa Uy Tiong
place of residence prior his death, such that Co, a native of China, and his mother was
Lorenzo Pou would have benefited from the Basilia Rodriguez, a native of this country. His
"en masse Filipinization" that the Philippine Bill parents were legally married in the Philippine
had effected in 1902. Being so, Lorenzo’s Islands at the time of his birth.
citizenship would have extended to his son,
Allan---respondent’s father. The father of the appellant went to China about
the year 1895, and died there about 1900.
Respondent, having been acknowledged as
Page 18 of 121
403 CASE DIGESTS | CONSTITUTIONAL LAW REVIEW | JUDGE ESTELA ALMA SINGCO | FIRST SEMESTER | 2020-2021

Subsequent to the death of his father, in May, preliminary injunction. Moya Lim Yao and Lau
1901, the appellant was sent to China by his Yuen Yeung appealed.
mother for the sole purpose of studying (and
always with the intention of returning) and Issue:
returned to the Philippine Islands on the
steamship Kaifong, arriving at the port of Cebu Whether or not Lau Yuen Yeung ipso facto
October 1, 1910, from Amoy, China, and became a Filipino citizen upon her marriage to
sought admission to the Philippine Islands. At a Filipino citizen.
this time the appellant was a few days under
21 years and 3 months of age.
Ruling:

The Board of Special Inquiry found that the Under Section 15 of Commonwealth Act
appellant was a Chinese person and a subject 473, an alien woman marrying a Filipino,
of the Emperor of China and not entitled to native born or naturalized, becomes ipso
land. facto a Filipina provided she is not
disqualified to be a citizen of the
ISSUE: Philippines under Section 4 of the same
Whether or not Roa is a citizen of the law. Likewise, an alien woman married to an
Philippines. alien who is subsequently naturalized here
follows the Philippine citizenship of her
RULING: husband the moment he takes his oath as
YES. By section 4 of the Philippine Bill of 1902 Filipino citizen, provided that she does not
the doctrine or principle of citizenship by place suffer from any of the disqualifications under
of birth which prevails in the United States was said Section 4. Whether the alien woman
extended to the Philippine Islands, but with requires to undergo the naturalization
limitations. In the United States every person, proceedings, Section 15 is a parallel provision
with certain specific exceptions, born in the to Section 16. Thus, if the widow of an
United States is a citizen of that country. applicant for naturalization as Filipino, who
Under section 4 every person born after the dies during the proceedings, is not required to
11th of April, 1899, of parents who were go through a naturalization proceedings, in
Spanish subjects on that date and who order to be considered as a Filipino citizen
continued to reside in this country are at the hereof, it should follow that the wife of a living
moment of their birth ipso facto citizens of the Filipino cannot be denied the same privilege.
Philippine Islands. From the reading of section This is plain common sense and there is
4 and taking into consideration the Act of absolutely no evidence that the Legislature
March 23, 1912, it is clear that Congress intended to treat them differently. As the laws
realized that there were inhabitants in the of our country, both substantive and
Philippine Islands who did not come within the procedural, stand today, there is no such
provisions of said section, and also that procedure (a substitute for naturalization
Congress did not then by express legislation proceeding to enable the alien wife of a
determine the political status of such persons. Philippine citizen to have the matter of her own
citizenship settled and established so that she
may not have to be called upon to prove it
NATURALIZATION everytime she has to perform an act or enter
29 Moy Ya Lim Yao vs. Comsr. of into a transaction or business or exercise a
Immigration, 41 SCRA 292 right reserved only to Filipinos), but such is no
proof that the citizenship is not vested as of
Facts: the date of marriage or the husband's
acquisition of citizenship, as the case may be,
for the truth is that the situation obtains even
Lau Yuen Yeung applied for a passport visa to
as to native-born Filipinos.
enter the Philippines as a non-immigrant on 8
February 1961. In the interrogation made in
connection with her application for a Everytime the citizenship of a person is
temporary visitor's visa to enter the material or indispensible in a judicial or
Philippines, she stated that she was a Chinese administrative case. Whatever the
residing at Kowloon, Hongkong, and that she corresponding court or administrative
would visit the Philippines to visit her uncle. authority decides therein as to such citizenship
She was permitted to come into the Philippines is generally not considered as res judicata,
on 13 March 1961 for a period of one month. hence it has to be threshed out again and
After repeated extensions, Lau Yuen Yeung again as the occasion may demand. Lau Yuen
was allowed to stay in the Philippines up to 13 Yeung, was declared to have become a Filipino
February 1962. On 25 January 1962, she citizen from and by virtue of her marriage to
contracted marriage with Moy Ya Lim Yao alias Moy Ya Lim Yao al as Edilberto Aguinaldo Lim,
Edilberto Aguinaldo Lim an alleged Filipino a Filipino citizen of 25 January 1962.
citizen. Because of the contemplated action of
the Commissioner of Immigration to confiscate 30 So vs. Rep., GR No. 170603, January 29,
her bond and order her arrest and immediate 2007
deportation, after the expiration of her
authorized stay, she brought an action for Facts:
injunction. At the hearing, it was admitted that
Lau Yuen Yeung could not write and speak So filed a petition for Naturalization under
either English or Tagalog, except for a few Commonwealth Act 473. In the proceedings,
words. She could not name any Filipino he contends that he was born in the
neighbor, with a Filipino name except one, Philippines, a man of good moral character and
Rosa. She did not know the names of her had been studying in one of the schools in the
brothers-in-law, or sisters-in-law. The Court of country namely UST where he graduated Cum
First Instance of Manila denied the prayer for Laude in Pharmacy and was then taking

Page 19 of 121
403 CASE DIGESTS | CONSTITUTIONAL LAW REVIEW | JUDGE ESTELA ALMA SINGCO | FIRST SEMESTER | 2020-2021

Medicine. He also complied with the his traits. Their testimonies do not convince
requirements of publication with no contest. the Court that they personally know petitioner
He also presented 2 witnesses: well and are therefore in a position to vouch
Atty Artemio Adasa JR and Mark Salcedo. RTC for his qualifications. Petitioner’s witnesses
granted the petition in 2003. However, it was clearly did not personally know him well
contested by the Solicitor General claiming enough; their testimonies do not satisfactorily
that the witnesses presented are not credible establish that petitioner has all the
as is required by the naturalization process. qualifications and none of the disqualifications
prescribed by law.
Issue:
31 Limkaichong vs. COMELEC, GR No.
Whether or not Edison So did meet all the 179120, April 1, 2009
qualification needed to be a naturalized Filipino
citizen. Facts:

Ruling: Two petitions were consolidated on the issue


about the qualifications of Jocelyn Limkaichong
No. Naturalization signifies the act of formally to run for, be elected to, and assume and
adopting a foreigner into the political body of discharge the position as Representative of the
a nation by clothing him or her with the 1st District of Negros Oriental. The contention
privileges of a citizen. Under current and of the parties who sought her disqualification
existing laws, there are three ways by which is that she is not a natural-born citizen, hence,
an alien may become a citizen by she lacks the citizenship requirement in
naturalization: (a) administrative Section 6, Article VI of the 1987 Constitution.
naturalization pursuant to R.A. No. 9139; (b) In the election that ensued, she was voted for
judicial naturalization pursuant to C.A. No. by the constituents of Negros Oriental and
473, as amended; and (c) legislative garnered the highest votes. She was
naturalization in the form of a law enacted by eventually proclaimed as the winner and has
Congress bestowing Philippine citizenship to since performed her duties and responsibilities
an alien. as Member of the House of Representatives.

First. C.A. No. 473 and R.A. No. 9139 are The proponents against Limkaichong's
separate and distinct laws – the former covers qualification stated that she is not a natural-
all aliens regardless of class while the latter born citizen because her parents were Chinese
covers native-born aliens who lived here in the citizens at the time of her birth. They went on
Philippines all their lives, who never saw any to claim that the proceedings for the
other country and all along thought that they naturalization of Julio Ong Sy, her father,
were Filipinos; who have demonstrated love never attained finality due to procedural and
and loyalty to the Philippines and affinity to the substantial defects.
customs and traditions. To reiterate, the
intention of the legislature in enacting R.A. No. Issue:
9139 was to make the process of acquiring
Philippine citizenship less tedious, less Whether or not the citizenship of
technical and more encouraging which is Limkaichong's parents may be questioned in
administrative rather than judicial in nature. an election case.
Thus, although the legislature believes that
there is a need to liberalize the naturalization Ruling:
law of the Philippines, there is nothing from
which it can be inferred that C.A. No. 473 was
No. The proper proceeding in cancelling the
intended to be amended or repealed by R.A.
naturalization certificate of one person should
No. 9139. What the legislature had in mind
be in accordance with Section 18 of CA No.
was merely to prescribe another mode of
473. Clearly under the law and jurisprudence,
acquiring Philippine citizenship which may be
it is the State, through the Solicitor General or
availed of by native born aliens. The only
the representative designated by statute, that
implication is that a native-born alien has
may question in the appropriate
the choice to apply for judicial or
denaturalization proceeding.
administrative naturalization, subject to
the prescribed qualifications and
disqualifications. 32 Burca vs. Republic (GR No. L-24252-Jan.
30, 1967)
In this case, petitioner applied for
naturalization by judicial act, though at the Facts:
time of the filing of his petition, administrative
naturalization under R.A. No. 9139 was On petition to declare Zita Ngo — also known
already available. Consequently, his as Zita Ngo Burca — "as possessing all
application should be governed by C.A. No. qualifications and none of the
473. Thus, absent a specific provision [dis]qualifications for naturalization under
expressly amending C.A. No. 473, the law Commonwealth Act 473 for the purpose of
stands and the qualifications and cancelling her Alien Registry with the Bureau
disqualifications set forth therein are of Immigration". She avers that she is of legal
maintained. age, married to Florencio Burca, a Filipino
citizen, and a resident of Real St., Ormoc City;
In any event, petitioner failed to prove that the that before her marriage, she was a Chinese
witnesses he presented were competent to citizen, subject of Nationalist China, with ACR
vouch for his good moral character and are No. A-148054; that she was born on March 30,
themselves possessed of good moral 1933 in Gigaquit, Surigao, and holder of Native
character. Petitioner’s witnesses did not testify Born Certificate of Residence No. 46333. The
on his specific acts; they did not elaborate on Solicitor General opposed and moved to
Page 20 of 121
403 CASE DIGESTS | CONSTITUTIONAL LAW REVIEW | JUDGE ESTELA ALMA SINGCO | FIRST SEMESTER | 2020-2021

dismiss the petition on two main grounds, viz: they are citizens of the Philippines and
(1) that "there is no proceeding established by personally know the petitioner to be a resident
law, or the rules for the judicial declaration of of the Philippines for the period of time
the citizenship of an individual"; and (2) that required by this Act and a person of good
as an application for Philippine citizenship, "the repute and morally irreproachable, and that
petition is fatally defective for failure to contain said petitioner has in their opinion all the
or mention the essential allegations required qualifications necessary to become a citizen of
under Section 7 of the Naturalization Law", the Philippines and is not in any way
such as, among others, petitioner's former disqualified under the provisions of this Act".
places of residence, and the absence of the Petitioner likewise failed to "set forth the
affidavits of at least two supporting witnesses. names and post-office addresses of such
witnesses as the petitioner may desire to
Issue: introduce at the hearing of the case". The
necessity for the affidavit of two witnesses
Whether or not Burca is deemed as a Filipino cannot be overlooked. It is important to know
Citizen in accordance with the provisions of the who those witnesses are. The State should not
Revised Naturalization Law vis-à-vis the be denied the opportunity to check on their
Philippine Constitution background to ascertain whether they are of
good standing in the community, whose word
may be taken on its face value, and who could
Ruling:
serve as "good warranty of the worthiness of
the petitioner".
No. Petitioner did not meet the requirements
specified in the Revised Naturalization Law.
The petition is fatally defective for failure to
contain or mention the essential allegations LOSS OF PHILIPPINE CITIZENSHIP
required Under Sec. 7 of the Naturalization
33 Board of Immigration Commissioners vs.
Law, such as, among others, petitioner’s
Callano, 25 SCRA 890
former places of residence, and the absence of
the affidavits of at least 2 supporting
witnesses. Facts:

The court ruled that: (1) An alien woman On July 13, 1962, the Department of Foreign
married to a Filipino who desires to be a citizen Affairs informed the Commissioner of
of this country must apply therefor by filing a Immigration that, on the basis of the findings
petition for citizenship reciting that she made by the National Bureau of Investigation,
possesses all the qualifications set forth in the signatures of former Secretary of Foreign
Section 2, and none of the disqualifications Affairs, Felixberto M. Serrano, on certain
under Sec. 4, both of the Revised documents, amongst them authorizing the
Naturalization law; (2) Said petition must be documentation of Beato Go Callano and
filed in the CFI where petitioner had resided at others, were not authentic. The Commissioner
least one year immediately preceding the filing of Immigration issued a warrant of exclusion
of the petition; and (3) Any action by any other commanding the deportation officer "to carry
office, agency, board or official, administrative out the exclusion of the above-named
or otherwise – other than the judgement of a applicants (the Go Callano brothers) on the
competent court of justice – certifying or first available transportation and on the same
declaring that an alien wife of the Filipino class of accommodation in which they arrived
citizen is also a Filipino citizen, is hereby to the port whence they came or to the country
declared null and void. of which they are nationals."

The petition avers that petitioner was born in Months later, the Court of First Instance issued
Gigaquit, Surigao that her former residence a writ of preliminary injunction restraining the
was Surigao, Surigao, and that presently she respondents in the case from deporting the
is residing at Regal St., Ormoc City. In court, petitioners. After trial, the Court rendered
however, she testified that she also resided in judgment finding that, according to
Junquera St., Cebu, where she took up a petitioners' undisputed evidence, "the
course in home economics, for one year. petitioners herein are the illegitimate children
Section 7 of the Naturalization Law of Emilia Callano, a Filipino citizen, with her
requires that a petition for naturalization common-law husband — a Chinese citizen,"
should state petitioner's "present and and concluding that "until the petitioners left
former places of residence". Residence for China in 1947, they must be considered as
encompasses all places where petitioner citizens of the Philippines as they were born of
actually and physically resided. Cebu, a Filipino mother and an alien father who,
where she studied for one year, perforce however, was not married to their mother."
comes within the term residence. The reason Notwithstanding the above finding and
for exacting recital in the petition of present conclusion, however, the Court dismissed the
and former places of residence is that case holding that the petitioners are citizens
"information regarding petitioner and of the Republic of China and not being properly
objection to his application are apt to be documented for entry into the Philippines as
provided by people in his actual, physical found by the Immigration Commissioner.
surrounding". 14 And the State is deprived of
full opportunity to make inquiries as to The grounds upon which the Court based its
petitioner's fitness to become a citizen, if all decision were:
the places of residence do not appear in the
petition. So it is, that failure to allege a former (1) because petitioners stayed in
place of residence is fatal. China for a period of fifteen years
before returning to the Philippines,
Said petition is not supported by the affidavit
of at least two credible persons, "stating that
Page 21 of 121
403 CASE DIGESTS | CONSTITUTIONAL LAW REVIEW | JUDGE ESTELA ALMA SINGCO | FIRST SEMESTER | 2020-2021

they must be considered as citizens of distinguished from that which is inferred from
the Chinese Republic; conduct.

(2) as petitioners were recognized by Moreover, petitioners were all minors


their alien father as his children, they when they where brought to China in
became Chinese citizens under the 1446. They were without legal capacity to
Chinese law of nationality. renounce their status. Upon their return to the
Philippines only Beato Go Callano had attained
While the Court also found that the cable the age of majority, but even as to him there
authorization mentioned heretofore was a could not have been renunciation because he
forgery, it held that, for the purpose of the did not manifest by direct and appropriate
petition before it, "it was immaterial to language that he was disclaiming Philippine
determine the genuineness or falsity of the citizenship. On the contrary, after he has
cable authorization. For if the petitioners are attained the age of majority, he applied for
Filipino citizens, they are entitled to remain registration as a Philippine citizen and sought
within the territorial jurisdiction of the Republic entry into this country, which are clear indicia
in whatever way they might have entered." of his intent to continue his former status. The
Like the court of origin, the Court of Appeals foregoing shows that the petitioners have not
found that herein respondents were the lost their Philippine citizenship.
illegitimate children of Go Chiao Lin, a Chinese
citizen, and Emilia Callano, a Filipino citizen. 34 Labo vs. COMELEC, 176 SCRA 1
Like the court of origin, the Court of Appeals
found that herein respondents were the Facts:
illegitimate children of Go Chiao Lin, a Chinese
citizen, and Emilia Callano, a Filipino citizen. Petitioner Labo was proclaimed mayor-elect of
Baguio City. Private respondent Lardizabal, the
Issue: losing candidate, filed a petition for quo
warranto questioning petitioner’s citizenship.
Whether or not petitioners lose their Philippine The latter claims that petitioner is a
citizenship upon the performance of certain naturalized Australian citizen, having married
acts or the happening of certain events in an Australian citizen. Records also showed
China petitioner’s oath and affirmation of allegiance
to the Queen of Australia. These were not
Ruling: denied; petitioner however claimed that his
naturalization in Australia made him at worst
No. The petitioners are admittedly Filipino only a dual national and did not divest him of
citizens at birth, and their status must be his Philippine citizenship and that his
governed by Philippine law wherever they may naturalization in Australia was annulled after it
be, in conformity with Article 15 of the Civil was found that his marriage to the Australian
Code. Under Article IV, Section 2, of the citizen was bigamous.
Philippine Constitution, "Philippine citizenship
may be lost or reacquired in the manner Issue:
provided by law," which implies that the
question of whether a Filipino has lost his Whether or not petitioner is a Filipino Citizen
Philippine citizenship shall be determined by
no other than the Philippine law. Ruling:

Section 1 of Commonwealth Act No. 63, as NO. CA No. 63 enumerates the modes by
amended by Republic Act No. 106, provides which Philippine citizenship may be lost.
that a Filipino citizen may lose his citizenship Among these are: (1) naturalization in a
by naturalization in a foreign country; express foreign country; (2) express renunciation of
renunciation of citizenship; subscribing to an citizenship; and (3) subscribing to an oath of
oath of allegiance to support the constitution allegiance to support the Constitution or laws
or laws of a foreign country; rendering service of a foreign country. All of which are applicable
to, or accepting a commission in, the armed to the petitioner. In connection with this,
forces of a foreign country; cancellation of the Article IV, Section 5, of the present
certificate of naturalization; declaration by Constitution provides that, “Dual allegiance of
competent authority that he is a deserter of citizens is inimical to the national interest and
the Philippine armed forces in time of war; in shall be dealt with by law.”
the case of a woman by marriage to a foreigner
if, by virtue of laws in force in her husband's Even if it be assumed that, as the petitioner
country, she acquires his nationality. asserts, his naturalization in Australia was
Recognition of the petitioners by their annulled after it was found that his marriage
alien father is not among the ground for to the Australian citizen was bigamous, that
losing Philippine citizenship under circumstance alone did not automatically
Philippine law, and it cannot be said that the restore his Philippine citizenship. His
petitioners lost their former status by reason divestiture of Australian citizenship does not
of such recognition. About the only mode of concern us here. That is a matter between him
losing Philippine citizenship which closely and his adopted country. What we must
bears on the petitioners is renunciation. But consider is the fact that he voluntarily and
even renunciation cannot be cited in support freely rejected Philippine citizenship and
of the conclusion that petition lost their willingly and knowingly embraced the
Philippine citizenship because the law requires citizenship of a foreign country. The possibility
an express renunciation which means a that he may have been subsequently rejected
renunciation that is made known distinctly and by Australia, as he claims, does not mean that
explicitly and not left to inference or
implication; a renunciation manifested by
direct and appropriate language, as
Page 22 of 121
403 CASE DIGESTS | CONSTITUTIONAL LAW REVIEW | JUDGE ESTELA ALMA SINGCO | FIRST SEMESTER | 2020-2021

he has been automatically reinstated as a 1, 1902 and the Philippine Autonomy Act of
citizen of the Philippines. Aug. 29, 1916, also known as the Jones Law.

Under CA No. 63 as amended by PD No. 725, Under both organic acts, all inhabitants of the
Philippine citizenship may be reacquired by Philippines who were Spanish subjects on April
direct act of Congress, by naturalization, or by 11, 1899 and resided therein including their
repatriation. It does not appear in the record, children are deemed to be Philippine citizens.
nor does the petitioner claim, that he has Private respondents father, Telesforo Ybasco,
reacquired Philippine citizenship by any of was born on Jan. 5, 1879 in Daet, Camarines
these methods. He does not point to any Norte.... Thus, under the Philippine Bill of 1902
judicial decree of naturalization as to any and the Jones Law, Telesforo Ybasco was
statute directly conferring Philippine deemed to be a Philippine citizen. By virtue of
citizenship upon him. Neither has he shown the same laws, which were the laws in force at
that he has complied with PD No. 725, the time of her birth, Telesforo’s daughter,
providing that: herein private respondent Rosalind Ybasco
Lopez, is likewise a citizen of the Philippines.
… (2) natural-born Filipinos who have lost their
Philippine citizenship may reacquire Philippine The signing into law of the 1935 Philippine
citizenship through repatriation by applying Constitution has established the principle of
with the Special Committee on Naturalization jus sanguinis as basis for the acquisition of
created by Letter of Instruction No. 270, and, Philippine citizenship, xxx So also, the
if their applications are approved, taking the principle of jus sanguinis, which confers
necessary oath of allegiance to the Republic of citizenship by virtue of blood relationship, was
the Philippines, after which they shall be subsequently retained under the 1973 and
deemed to have reacquired Philippine 1987 Constitutions. Thus, the herein private
citizenship. The Commission on Immigration respondent, Rosalind Ybasco Lopez, is a
and Deportation shall thereupon cancel their Filipino citizen, having been born to a Filipino
certificate of registration. father. The fact of her being born in Australia
is not tantamount to her losing her Philippine
Philippine citizenship is not a cheap commodity citizenship. If Australia follows the principle of
that can be easily recovered after its jus soli, then at most, private respondent can
renunciation. It may be restored only after the also claim Australian citizenship resulting to
returning renegade makes a formal act of re- her possession of dual citizenship.
dedication to the country he has abjured and
he solemnly affirms once again his total and 36 Yu vs. Santiago, 169 SCRA 364
exclusive loyalty to the Republic of the
Philippines. This may not be accomplished by Facts:
election to public office.
Petitioner Yu was originally issued a
35 Valles vs. COMELEC, G.R. No. 137000, August Portuguese passport in 1971. On February 10,
9, 2000 1978, he was naturalized as a Philippine
citizen. Despite his naturalization, he applied
Facts: for and was issued Portuguese Passport by the
Consular Section of the Portuguese Embassy
Rosalind Ybasco Lopez was born on May 16, in Tokyo on July 21, 1981. Said Consular Office
1934 in Australia to a Filipino father and an certifies that his Portuguese passport expired
Australian mother. In 1949, at the age of on 20 July 1986. He also declared his
fifteen, she left Australia and came to settle in nationality as Portuguese in commercial
the Philippines, where she later married a documents he signed, specifically, the
Filipino and has since then participated in the Companies registry of Tai Shun Estate Ltd.
electoral process not only as a voter but as a filed in Hongkong sometime in April 1980. The
candidate, as well. In the May 1998 elections, CID detained Yu pending his deportation case.
she ran for governor but Valles filed a petition Yu, in turn, filed a petition for habeas corpus.
for her disqualification as candidate on the An internal resolution of 7 November 1988
ground that she is an Australian. referred the case to the Court en banc. The
Court en banc denied the petition. When his
Issue: Motion for Reconsideration was denied,
petitioner filed a Motion for Clarification.
Whether or not Rosalind is an Australian or a
Filipino Issue:

Ruling Whether or not petitioner’s acts constitute


renunciation of his Philippine citizenship
The Philippine law on citizenship adheres to
the principle of jus sanguinis. Thereunder, a Ruling:
child follows the nationality or citizenship of
the parents regardless of the place of his/her Express renunciation was held to mean a
birth, as opposed to the doctrine of jus soli renunciation that is made known distinctly and
which determines nationality or citizenship on explicitly and not left to inference or
the basis of place of birth. implication. Petitioner, with full knowledge,
and legal capacity, after having renounced
Rosalind Ybasco Lopez was born a year before Portuguese citizenship upon naturalization as
the 1935 Constitution took into effect and at a Philippine citizen resumed or reacquired his
that time, what served as the Constitution of prior status as a Portuguese citizen, applied for
the Philippines were the principal organic acts a renewal of his Portuguese passport and
by which the United States governed the represented himself as such in official
country. These were the Philippine Bill of July documents even after he had become a
Page 23 of 121
403 CASE DIGESTS | CONSTITUTIONAL LAW REVIEW | JUDGE ESTELA ALMA SINGCO | FIRST SEMESTER | 2020-2021

naturalized Philippine citizen. Such resumption for local elective office must be inter alia a
or reacquisition of Portuguese citizenship is citizen of the Philippines and a qualified voter
grossly inconsistent with his maintenance of of the constituency where he is running.
Philippine citizenship. Section 117 of the Omnibus Election Code
provides that a qualified voter must be, among
While normally the question of whether or not other qualifications, a citizen of the Philippines,
a person has renounced his Philippine this being an indispensable requirement for
citizenship should be heard before a trial court suffrage under Article V, Section 1, of the
of law in adversary proceedings, this has Constitution.
become unnecessary as this Court, no less,
upon the insistence of petitioner, had to look In the certificate of candidacy he filed on
into the facts and satisfy itself on whether or November 19, 1987, Frivaldo described
not petitioner's claim to continued Philippine himself as a "natural-born" citizen of the
citizenship is meritorious. Philippine Philippines, omitting mention of any
citizenship, it must be stressed, is not a subsequent loss of such status. The evidence
commodity or were to be displayed when shows, however, that he was naturalized as a
required and suppressed when convenient. citizen of the United States in 1983 per the
following certification from the United States
District Court, Northern District of California,
as duly authenticated by Vice Consul Amado P.
PD 725 Cortez of the Philippine Consulate General in
37 Frivaldo vs. COMELEC G.R. No. 120295 San Francisco, California, U.S.A.
June 28, 1996
The reason for this inquiry is the provision in
Facts: Article XI, Section 9, of the Constitution that
all public officials and employees owe the State
Petitioner Juan G. Frivaldo was proclaimed and the Constitution "allegiance at all times"
governor-elect of the province of Sorsogon on and the specific requirement in Section 42 of
January 22, 1988, and assumed office in due the Local Government Code that a candidate
time. On October 27, 1988. the League of for local elective office must be inter alia a
Municipalities, Sorsogon Chapter (hereafter, citizen of the Philippines and a qualified voter
League), represented by its President, of the constituency where he is running.
Salvador Estuye, who was also suing in his Section 117 of the Omnibus Election Code
personal capacity, filed with the Commission provides that a qualified voter must be, among
on Elections a petition for the annulment of other qualifications, a citizen of the Philippines,
Frivaldo this being an indispensable requirement for
suffrage under Article V, Section 1, of the
Constitution.
In his answer dated May 22, 1988, Frivaldo
admitted that he was naturalized in the United
States as alleged but pleaded the special and In the certificate of candidacy he filed on
affirmative defenses that he had sought November 19, 1987, Frivaldo described
American citizenship only to protect himself himself as a "natural-born" citizen of the
against President Marcos. Philippines, omitting mention of any
subsequent loss of such status. The evidence
shows, however, that he was naturalized as a
Frivaldo moved for a preliminary hearing on
citizen of the United States in 1983 per the
his affirmative defenses but the respondent
following certification from the United States
Commission on Elections decided instead by its
District Court, Northern District of California,
Order of January 20, 1988, to set the case for
as duly authenticated by Vice Consul Amado P.
hearing on the merits. His motion for
Cortez of the Philippine Consulate General in
reconsideration was denied in another Order
San Francisco, California, U.S.A.
dated February 21, 1988. He then came to this
Court in a petition for certiorari and prohibition
to ask that the said orders be set aside on the If he really wanted to disavow his American
ground that they had been rendered with citizenship and reacquire Philippine
grave abuse of discretion. Pending resolution citizenship, the petitioner should have done so
of the petition, we issued a temporary order in accordance with the laws of our country.
against the hearing on the merits scheduled by Under CA No. 63 as amended by CA No. 473
the COMELEC and at the same time required and PD No. 725, Philippine citizenship may be
comments from the respondents. reacquired by direct act of Congress, by
naturalization, or by repatriation.
Issue:
It does not appear that Frivaldo has taken
these categorical acts. He contends that by
Whether or not Juan G. Frivaldo was a citizen
simply filing his certificate of candidacy he had,
of the Philippines at the time of his election on
without more, already effectively recovered
January 18, 1988, as provincial governor of
Philippine citizenship. But that is hardly the
Sorsogon. All the other issues raised in this
formal declaration the law envisions — surely,
petition are merely secondary to this basic
Philippine citizenship previously disowned is
question.
not that cheaply recovered. If the Special
Committee had not yet been convened, what
Ruling: that meant simply was that the petitioner had
to wait until this was done, or seek
The reason for this inquiry is the provision in naturalization by legislative or judicial
Article XI, Section 9, of the Constitution that proceedings.
all public officials and employees owe the State
and the Constitution "allegiance at all times" The argument that the petition filed with the
and the specific requirement in Section 42 of Commission on Elections should be dismissed
the Local Government Code that a candidate for tardiness is not well-taken. The herein
Page 24 of 121
403 CASE DIGESTS | CONSTITUTIONAL LAW REVIEW | JUDGE ESTELA ALMA SINGCO | FIRST SEMESTER | 2020-2021

private respondents are seeking to prevent


Frivaldo from continuing to discharge his office Petitioner was investigated by Special
of governor because he is disqualified from Prosecutor Atty. Donato and on the same day,
doing so as a foreigner. Qualifications for petitioner was accused of violating Section 8,
public office are continuing requirements and Chapter 3, Title 1, Book 3 of the 1987
must be possessed not only at the time of Administrative Code. Petitioner’s passport was
appointment or election or assumption of revoked by the US Department of State.
office but during the officer's entire tenure. Hence, respondent (petitioner Tabasa) is now
Once any of the required qualifications is lost, an undocumented and undesirable alien and
his title may be seasonably challenged. If, say, may be summarily deported pursuant to Law
a female legislator were to marry a foreigner and Intelligence Instructions No. 53.
during her term and by her act or omission
acquires his nationality, would she have a right Petitioner’s passport was revoked due to
to remain in office simply because the federal charges filed against him. Federal
challenge to her title may no longer be made charges are as follows:
within ten days from her proclamation? It has
been established, and not even denied, that
In possession of a firearm and one count of
the evidence of Frivaldo's naturalization was
sexual battery (all in violation of the California
discovered only eight months after his
Penal Code)
proclamation and his title was challenged
shortly thereafter.
Upon revocation of petitioner’s passport, he
loses the privilege to remain in the country.
This Court will not permit the anomaly of a
person sitting as provincial governor in this
country while owing exclusive allegiance to Petitioner filed before the CA a petition for
another country. The fact that he was elected habeas corpus. That he was not afforded due
by the people of Sorsogon does not excuse this process; that no warrant of arrest for
patent violation of the salutary rule limiting deportation may be issued by immigration
public office and employment only to the authorities before a final order of deportation
citizens of this country. The qualifications is made; that no notice of the cancellation of
prescribed for elective office cannot be erased his passport was made by the U.S. Embassy;
by the electorate alone. The will of the people that he is entitled to admission or to a change
as expressed through the ballot cannot cure of his immigration status as a non-quota
the vice of ineligibility, especially if they immigrant because he is married to a Filipino
mistakenly believed, as in this case, that the citizen as provided in Section 13, paragraph
candidate was qualified. Obviously, this rule (a) of the Philippine Immigration Act of 1940;
requires strict application when the deficiency and that he was a natural-born citizen of the
is lack of citizenship. If a person seeks to serve Philippines prior to his derivative naturalization
in the Republic of the Philippines, he must owe when he was seven years old due to the
his total loyalty to this country only, abjuring naturalization of his father, Rodolfo Tabasa, in
and renouncing all fealty and fidelity to any 1968. At the time Tabasa filed the petition, he
other state. was 35 years old.

It is true as the petitioner points out that the Tabasa then filed a supplementary petition
status of the natural-born citizen is favored by alleging that he has acquired Filipino
the Constitution and our laws, which is all the citizenship by repatriation in accordance with
more reason why it should be treasured like a RA 8171 and because he is now a Filipino
pearl of great price. But once it is surrendered citizen, he can no longer be deported.
and renounced, the gift is gone and cannot be
lightly restored. This country of ours, for all its CA denied the petition on the grounds that he
difficulties and limitations, is like a jealous and had only acquired citizenship to escape from
possessive mother. Once rejected, it is not the federal charges filed against him by the US
quick to welcome back with eager arms its and that he has not successfully acquired
prodigal if repentant children. The returning citizenship because he does not fall under any
renegade must show, by an express and of the requirements under RA 8171.
unequivocal act, the renewal of his loyalty and
love.

ISSUE:
38 Tabasa vs, CA, GR No. 125793, August 29,
2006 WON petitioner has the privilege reacquire
Filipino citizenship through RA 8171.
FACTS:

Tabasa was a natural-born citizen of the


Philippines. In 1968, when petitioner was 7 RULING:
years old, his father Rodolfo Tabasa, became
a naturalized citizen of the United States. By Petition is not meritorious.
derivative naturalization, Joevani Tabasa
acquired the citizenship of his father. RA 8171 provides repatriation to two classes
Petitioner arrived in the Philippines in August of persons:
1995 and was admitted as “balikbayan” for
one year. Petitioner was arrested and detained Filipino women who have lost their Philippine
by agent Wilson Soluren of BID on May 23, citizenship by marriage to aliens and natural-
1996 in Aklan and was brought to BID born Filipinos who have lost their Philippine
Detention Center, Manila.

Page 25 of 121
403 CASE DIGESTS | CONSTITUTIONAL LAW REVIEW | JUDGE ESTELA ALMA SINGCO | FIRST SEMESTER | 2020-2021

citizenship, including their minor children, on Committee on Naturalization, after he filed a


account of political or economic necessity. petition for repatriation pursuant to Republic
Act No. 8171.
Petitioner does not fall under RA 8171 because
what is referred in the second class of persons On the date of the hearing, the parties were
privileged with repatriation refers to his father. required to submit their Memoranda within
three days. Private respondents filed their
Even if the Court would concede that petitioner Memorandum, while petitioner did not file one
can avail the benefit of RA 8171, he failed to within the required period. Petitioner,
follow procedure for reacquisition. however, filed a Reply Memorandum
subsequently.
Procedure for reacquisition through
repatriation is as follows: Atty. Zaragoza, Jr hearing officer of this case
recommended that petitioner Altarejos be
SECTION 1. Composition. — The composition disqualified from being a candidate for the
of the Special Committee on position of mayor.

Naturalization, with the Solicitor General as Petitioner points out that he took his Oath of
Chairman, the Undersecretary of Foreign Allegiance to the Republic of the Philippines on
Affairs and the Director-General of the December 17, 1997. In view thereof, he ran
National Intelligence Coordinating Agency, as and was even elected as Mayor of San Jacinto,
members, shall remain as constituted. Masbate during the 1998 elections. He argues
that if there was delay in the registration of his
Certificate of Repatriation with the Bureau of
SECTION 2. Procedure. — Any person desirous
Immigration and with the proper civil registry,
of repatriating or reacquiring
the same was brought about by the inaction on
the part of said offices since the records of the
Filipino citizenship pursuant to R.A. No. 8171 Special Committee on Naturalization show that
shall file a petition with the his Certificate of Repatriation and Oath of
Allegiance have long been transmitted to said
Special Committee on Naturalization which offices.
shall process the same. If their applications are
approved[,] they shall take the necessary oath ISSUE:
of allegiance to the Republic of the Philippines,
after which they shall be deemed to have
When does the citizenship qualification of a
reacquired Philippine citizenship. The
candidate for an elective office apply?
Commission on Immigration and Deportation
shall thereupon cancel their certificate of
registration (emphasis supplied). RULING:

SECTION 3. Implementing Rules. — The In Frivaldo v. Commission on Elections, the


Special Committee is hereby authorized to Court ruled that the citizenship qualification
promulgate rules and regulations and must be construed as applying to the time of
prescribe the appropriate forms and the proclamation of the elected official and at the
required fees for the processing of petitions. start of his term. The Court, through Justice
Artemio V. Panganiban, discussed, thus:
SECTION 4. Effectivity . — This Administrative
Order shall take effect immediately. Under Sec. 39 of the Local Government Code,
(a)n elective local official must be:
Petitioner only took the oath of allegiance to
the Republic of the Philippines then executed a citizen of the Philippines;
an affidavit of repatriation which he registered
together with his birth certificate with the a registered voter in the barangay,
office of the Local Civil Registrar of Manila. The municipality, city, or province x x x where he
said office issued him a certificate of such intends to be elected;
registration.
a resident therein for at least one (1) year
immediately preceding the day of the election;
39 Altarejos vs. COMELEC, Nov. 10, 2004
able to read and write Filipino or any other
local language or dialect.
FACTS:
In addition, candidates for the position of
Private respondents Jose Almie Altiche and governor x x x must be at least twenty-three
Vernon Versoza filed to the COMELEC a (23) years of age on election day.
petition to disqualify and to deny due course
or cancel the certificate of candidacy of
petitioner on the ground that he is not a
Filipino citizen and that he made a false From the above, it will be noted that the law
representation in his certificate of candidacy does not specify any particular date or time
that he was not a permanent resident of or when the candidate must possess citizenship,
immigrant to a foreign country. unlike that for residence (which must consist
of at least one year’s residency immediately
Petitioner object that he did not commit false preceding the day of election) and age (at least
representation in his application for candidacy twenty-three years of age on election day).
as mayor because he was already issued a
Certificate of Repatriation by the Special
Page 26 of 121
403 CASE DIGESTS | CONSTITUTIONAL LAW REVIEW | JUDGE ESTELA ALMA SINGCO | FIRST SEMESTER | 2020-2021

Moreover, in the case of Frivaldo v.


Commission on Elections, the Court ruled that
the repatriation of Frivaldo RETROACTED to
the date of the filing of his application. In said ISSUE:
case, the repatriation of Frivaldo was by virtue
of Presidential Decree No. 725, which took
Whether or not petitioners and others who
effect on June 5, 1975. The Court therein
retained and / or reacquired Philippine
declared that Presidential Decree No. 725 was
citizenship pursuant to R.A. 9225 may vote as
a curative statute, which is retroactive in
absentee voter under the Overseas Absentee
nature. The retroactivity of Frivaldos
Voting Act.
repatriation to the date of filing of his
application was justified by the Court, thus:

The reason for this is simply that if, as in this


case, it was the intent of the legislative RULING:
authority that the law should apply to past
events i.e., situations and transactions The Court granted the petition that those who
existing even before the law came into being retain or re‑acquire Philippine citizenship under
in order to benefit the greatest number of the Citizenship Retention and Re‑Acquisition
former Filipinos possible thereby enabling Act of 2003 (Republic Act No. 9225), may
them to enjoy and exercise the constitutionally exercise the right to vote under the system of
guaranteed right of citizenship, and such absentee voting according to the Overseas
legislative intention is to be given the fullest Absentee Voting Act of 2003 (Republic Act No.
effect and expression, then there is all the 9189).
more reason to have the law apply in a
retroactive or retrospective manner to Section 5, Paragraph 1 of the R.A. 9225 states
situations, events and transactions that “Those intending to exercise their right of
subsequent to the passage of such law. That suffrage must meet the requirements under
is, the repatriation granted to Frivaldo x x x Section 1, Article V of the Constitution,
can and should be made to take effect as of Republic Act No. 9189, otherwise known as
date of his application. As earlier mentioned, "The Overseas Absentee Voting Act of 2003"
there is nothing in the law that would bar this and other existing laws.
or would show a contrary intention on the part
of the legislative authority; and there is no There is no provision in the dual citizenship law
showing that damage or prejudice to anyone, requiring dual citizens to actually establish
or anything unjust or injurious would result residence and physically stay in the Philippines
from giving retroactivity to his repatriation. first before they can exercise their right to
Neither has Lee shown that there will result the vote. On the contrary, the law acknowledged
impairment of any contractual obligation, that dual citizens are most likely non-residents
disturbance of any vested right or breach of and granted them the same right of suffrage
some constitutional guaranty. as that granted an absentee voter under the
Overseas Absentee Voting Act. The Overseas
Petitioner’s repatriation retroacted to the date Absentee Voting Act, in essence, aims to
he filed his application in 1997. Petitioner was, enfranchise as much as possible all overseas
therefore, qualified to run for a mayoralty Filipinos who, save for the residency
position in the government in the May 10, requirements, are qualified to vote as an
2004 elections. ordinary voter under ordinary conditions.

41 Lopez vs. COMELEC, GR No. 182701, July


23, 2008
NATURALIZED CITIZENS CANNOT HAVE
DUAL CITIZENSHIP
FACTS:
40 Lewis vs. COMELEC, August 4, 2006
Petitioner Lopez, a dual citizen, was a
candidate for the position of Chairman of
FACTS: Barangay Bagacay, San Dionisio, Iloilo City
held on October 29, 2007. He was eventually
Petitioners are successful applicants for declared the winner.
recognition of Philippine citizenship under On October 25, 2007, respondent Villanueva
Citizenship Retention and Re‑Acquisition Act of filed a petition before the Provincial Election
2003 (R.A. 9225). The Citizenship Retention Supervisor of the Province of Iloilo, praying for
and Re‑Acquisition Act accords to such the disqualification of Lopez because he was
applicants the right of suffrage, among others. ineligible from running for any public office.
Long before the May 10, 2004 national and Lopez argued that he is a Filipino-American, by
local elections, petitioners sought registration virtue of the Citizenship Retention and Re-
and certification as "overseas absentee voter" acquisition Act of 2003. He said, he possessed
under the Overseas Absentee Voting Act of all the qualifications to run for Barangay
2003 (R.A. 9189). The COMELEC advised that Chairman.
the applicants do not have the right to vote in On February 6, 2008, COMELEC issued the
such elections because they lack the one-year Resolution granting the petition for
residence requirement prescribed by the disqualification of Lopez from running as
Constitution for regular voters. The COMELEC Barangay Chairman. COMELEC said, to be able
argued that dual citizens under the Citizenship to qualify as a candidate in the elections, Lopez
Retention and Re‑Acquisition Act must first should have made a personal and sworn
establish their domicile in the Philippines renunciation of any and all foreign citizenship.
through positive acts before they can avail of His motion for reconsideration having been
themselves of the Overseas Absentee Voting denied, Lopez resorted to petition for
Act. certiorari, imputing grave abuse of discretion
Page 27 of 121
403 CASE DIGESTS | CONSTITUTIONAL LAW REVIEW | JUDGE ESTELA ALMA SINGCO | FIRST SEMESTER | 2020-2021

on the part of the COMELEC for disqualifying General (PCG) of Los Angeles, California. The
him from running and assuming the office of Los Angeles PCG issued on 19 June 2006 an
Barangay Chairman. Order of Approval of petitioner’s request, and
on the same day, petitioner took his Oath of
ISSUE: Allegiance to the Republic of the Philippines
Whether or not there was grave abuse of before Vice Consul Edward C. Yulo. On 27
discretion on the part of the COMELEC for September 2006, the Bureau of Immigration
disqualifying petitioner. issued and Identification Certificate,
recognizing petitioner as a citizen of the
RULING: Philippines.
No. The Supreme Court dismissed the
petition. The COMELEC committed no grave Six months after, on 26 March 2007, petitioner
abuse of discretion in disqualifying petitioner filed his Certificate of Candidacy for the
as candidate for Chairman in the Barangay Position of Vice-Mayor of the Municipality of
elections of 2007. Catarman, Camiguin.
Lopez was born a Filipino but he deliberately
sought American citizenship and renounced his In the meantime, the 14 May 2007 National
Filipino citizenship. He later on became a dual and Local Elections were held. Petitioner
citizen by re-acquiring Filipino citizenship. garnered the highest number of votes for the
R.A. No. 9225 expressly provides for the position of Vice Mayor.
conditions before those who re-acquired
Filipino citizenship may run for a public office
On 12 June 2007, the COMELEC Second
in the Philippines.
Division finally issued its Resolution11
Section 5 of the said law states:
disqualifying the petitioner from running for
Section 5. Civil and Political Rights and
the position of Vice-Mayor of Catarman,
Liabilities. – Those who retain or re-acquire
Camiguin, for failure to make the requisite
Philippine citizenship under this Act shall enjoy
renunciation of his US citizenship
full civil and political rights and be subject to
all attendant liabilities and responsibilities
under existing laws of the Philippines and the
following conditions:
(2) Those seeking elective public office in the ISSUE:
Philippines shall meet the qualification for
holding such public office as required by the Whether or not petitioner has validly complied
Constitution and existing laws and, at the time the citizenship requirement as required by law
of the filing of the certificate of candidacy, for persons seeking public office.
make a personal and sworn renunciation of
any and all foreign citizenship before any
public officer authorized to administer an oath.
Lopez was able to regain his Filipino RULING:
Citizenship by virtue of the Dual Citizenship
Law when he took his oath of allegiance before
Contrary to the assertions made by petitioner,
the Vice Consul of the Philippine Consulate
his oath of allegiance to the Republic of the
General’s Office in Los Angeles, California; the
Philippines made before the Los Angeles PCG
same is not enough to allow him to run for a
and his Certificate of Candidacy do not
public office.
substantially comply with the requirement
Lopez’s failure to renounce his American
of a personal and sworn renunciation of
citizenship as proven by the absence of an
foreign citizenship, because these are distinct
affidavit that will prove the contrary leads this
requirements to be complied with for different
Commission to believe that he failed to comply
purposes.
with the positive mandate of law.

42 Jacot vs. COMELEC, GR No. 179848, Section 3 of Republic Act No. 9225 requires
November 27, 2008 that natural-born citizens of the Philippines,
who are already naturalized citizens of a
foreign country, must take the following oath
FACTS:
of allegiance to the Republic of the Philippines
to reacquire or retain their Philippine
Petitioner Nestor A. Jacot assails the citizenship.
Resolution dated 28 September 2007 of the ,
affirming the Resolution dated 12 June 2007 of
By the oath dictated in the afore-quoted
the COMELEC Second Division, disqualifying
provision, the Filipino swears allegiance to the
him from running for the position of Vice-
Philippines, but there is nothing therein on his
Mayor of Catarman, Camiguin in the 14 May
renunciation of foreign citizenship.
2007 National and Local Elections, on the
ground that he failed to make a personal
renouncement of his US citizenship. The law categorically requires persons seeking
elective public office, who either retained their
Philippine citizenship or those who reacquired
Petitioner was a natural born citizen of the
it, to make a personal and sworn renunciation
Philippines, who became a naturalized citizen
of any and all foreign citizenship before a
of the US on 13 December 1989. Petitioner
public officer authorized to administer an oath
sought to reacquire his Philippine citizenship
simultaneous with or before the filing of the
under Republic Act No. 9225, otherwise known
certificate of candidacy.
as the Citizenship Retention and Re-
Acquisition Act.
Hence, Section 5(2) of Republic Act No. 9225
compels natural-born Filipinos, who have been
He filed a request for the administration of his
naturalized as citizens of a foreign country, but
Oath of Allegiance to the Republic of the
Philippines with the Philippine Consulate
Page 28 of 121
403 CASE DIGESTS | CONSTITUTIONAL LAW REVIEW | JUDGE ESTELA ALMA SINGCO | FIRST SEMESTER | 2020-2021

who reacquired or retained their Philippine It must be emphasized that COMELEC First
citizenship Division found that Arnado used his U.S.
Passport at least six times after he renounced
(1) to take the oath of allegiance under Section his American citizenship. This was debunked
3 of Republic Act No. 9225, and by the COMELEC En Banc, which found that
Arnado only used his U.S. passport four times,
(2) for those seeking elective public offices in and which agreed with Arnados claim that he
the Philippines, only used his U.S. passport on those occasions
because his Philippine passport was not yet
issued. The COMELEC En Banc argued that
to additionally execute a personal and sworn
Arnado was able to prove that he used his
renunciation of any and all foreign citizenship
Philippine passport for his travels on the
before an authorized public officer prior or
following dates: 12 January 2010, 31 January
simultaneous to the filing of their certificates
2010, 31 March 2010, 16 April 2010, 20 May
of candidacy, to qualify as candidates in
2010, and 4 June 2010.
Philippine elections.

The COMELEC En Banc concluded that "the use


of the US passport was because to his
knowledge, his Philippine passport was not yet
issued to him for his use."This conclusion,
43 Maquiling vs. COMELEC, GR No. 195649, however, is not supported by the facts. Arnado
April 16, 2013 claims that his Philippine passport was issued
on 18 June 2009. The records show that he
FACTS: continued to use his U.S. passport even after
he already received his Philippine passport.
This Resolution resolves the Motion for Arnados travel records show that he presented
Reconsideration filed by respondent on May his U.S. passport on 24 November 2009, on 21
10, 2013 and the Supplemental Motion for January 2010, and on 23 March 2010. These
Reconsideration filed on May 20, 2013. facts were never refuted by Arnado.

We are not unaware that the term of office of Thus, the ruling of the COMELEC En Banc is
the local officials elected in the May 2010 based on a misapprehension of the facts that
elections has already ended on June 30, 2010. the use of the U.S. passport was discontinued
Arnado, therefore, has successfully finished his when Arnado obtained his Philippine passport.
term of office. While the relief sought can no Arnados continued use of his U.S. passport
longer be granted, ruling on the motion for cannot be considered as isolated acts contrary
reconsideration is important as it will either to what the dissent wants us to believe.
affirm the validity of Arnados election or affirm
that Arnado never qualified to run for public It must be stressed that what is at stake here
office. is the principle that only those who are
exclusively Filipinos are qualified to run for
Respondent failed to advance any argument to public office. If we allow dual citizens who wish
support his plea for the reversal of this Courts to run for public office to renounce their
Decision dated April 16, 2013. Instead, he foreign citizenship and afterwards continue
presented his accomplishments as the Mayor using their foreign passports, we are creating
of Kauswagan, Lanao del Norte and reiterated a special privilege for these dual citizens,
that he has taken the Oath of Allegiance not thereby effectively junking the prohibition in
only twice but six times. It must be stressed, Section 40(d) of the Local Government Code.
however, that the relevant question is the
efficacy of his renunciation of his foreign
citizenship and not the taking of the Oath of
Allegiance to the Republic of the Philippines.
Neither do his accomplishments as mayor 44 BM No. 1678, Petition for Leave to
affect the question before this Court. Resume the Practice of Law, Benjamin M.
Dacanay, December 17, 2007
ISSUE:
FACTS:
Whether or not a dual citizen can run for a
local elective position? Benjamin M. Dacanay was admitted to the
Philippine Bar in 1960. He migrated to Canada
RULING: to seek medical attention for his ailments. He
renounced his Philippine citizenship and
The Motion for Reconsideration was denied, applied for Canadian citizenship to avail its free
dual citizens are ineligible for local public office medical program. Two years after, pursuant to
Republic Act (RA) 9225 (Citizenship Retention
Indeed, there is no doubt that Section 40(d) of and Re-Acquisition Act of 2003) petitioner
the Local Government Code disqualifies those reacquired his Philippine citizenship and
with dual citizenship from running for local intended to resume his law practice. Hence
elective positions. this petition.

There is likewise no doubt that the use of a


passport is a positive declaration that one is a
citizen of the country which issued the ISSUE:
passport, or that a passport proves that the
country which issued it recognizes the person Whether or not the petitioner Benjamin M.
named therein as its national. Dacanay lost his membership in the Philippine

Page 29 of 121
403 CASE DIGESTS | CONSTITUTIONAL LAW REVIEW | JUDGE ESTELA ALMA SINGCO | FIRST SEMESTER | 2020-2021

bar when he gave up his Philippine citizenship EFFECT OF REPATRIATION


in May 2004.
45 Bengzon III vs. HRET, GR No. 142840,
May 7, 2001

RULING: FACTS:

No. Section 2, Rule 138 (Attorneys and The citizenship of respondent Cruz is at issue
Admission to Bar) of the Rules of Court: in this case, in view of the constitutional
SECTION 2. Requirements for all applicants for requirement that “no person shall be a
admission to the bar. – Every applicant for Member of the House of Representatives
admission as a member of the bar must be a unless he is a natural-born citizen.”
citizen of the Philippines, at least twenty-one
years of age, of good moral character, and a
Cruz was a natural-born citizen of the
resident of the Philippines; and must produce
Philippines. He was born in Tarlac in 1960 of
before the Supreme Court satisfactory
Filipino parents. In 1985, however, Cruz
evidence of good moral character, and that no
enlisted in the US Marine Corps and without
charges against him, involving moral
the consent of the Republic of the Philippines,
turpitude, have been filed or are pending in
took an oath of allegiance to the USA. As a
any court in the Philippines.
Consequence, he lost his Filipino citizenship for
under CA No. 63 [(An Act Providing for the
Applying the provision, the Office of the Bar Ways in Which Philippine Citizenship May Be
Confidant opines that, by virtue of his Lost or Reacquired (1936)] section 1(4), a
reacquisition of Philippine citizenship, in 2006, Filipino citizen may lose his citizenship by,
petitioner has again met all the qualifications among other, “rendering service to or
and has none of the disqualifications for accepting commission in the armed forces of a
membership in the bar. It recommends that he foreign country.”
be allowed to resume the practice of law in the
Philippines, conditioned on his retaking the
Whatever doubt that remained regarding his
lawyer’s oath to remind him of his duties and
loss of Philippine citizenship was erased by his
responsibilities as a member of the Philippine
naturalization as a U.S. citizen in 1990, in
bar.
connection with his service in the U.S. Marine
Corps.
The Constitution provides that the practice of
all professions in the Philippines shall be
In 1994, Cruz reacquired his Philippine
limited to Filipino citizens save in cases
citizenship through repatriation under RA 2630
prescribed by law. The practice of law is a
[(An Act Providing for Reacquisition of
privilege denied to foreigners. The exception is
Philippine Citizenship by Persons Who Lost
when Filipino citizenship is lost by reason of
Such Citizenship by Rendering Service To, or
naturalization as a citizen of another country
Accepting Commission In, the Armed Forces of
but subsequently reacquired pursuant to RA
the United States (1960)]. He ran for and was
9225. Under RA 9225, if a person intends to
elected as the Representative of the 2nd
practice the legal profession in the Philippines
District of Pangasinan in the 1998 elections. He
and he reacquires his Filipino citizenship
won over petitioner Bengson who was then
pursuant to its provisions "(he) shall apply with
running for reelection.
the proper authority for a license or permit to
engage in such practice." Stated otherwise,
before a lawyer who reacquires Filipino Subsequently, petitioner filed a case for Quo
citizenship pursuant to RA 9225 can resume Warranto Ad Cautelam with respondent HRET
his law practice, he must first secure from this claiming that Cruz was not qualified to become
Court the authority to do so, conditioned on: a member of the HOR since he is not a natural-
born citizen as required under Article VI,
section 6 of the Constitution.
(a) the updating and payment in full of the
annual membership dues in the IBP;
HRET rendered its decision dismissing the
petition for quo warranto and declaring Cruz
(b) the payment of professional tax;
the duly elected Representative in the said
election.
(c) the completion of at least 36 credit hours
of mandatory continuing legal education; this
is specially significant to refresh the
applicant/petitioner’s knowledge of Philippine
laws and update him of legal developments ISSUE:
and
WON Cruz, a natural-born Filipino who became
(d) the retaking of the lawyer’s oath which will an American citizen, can still be considered a
not only remind him of his duties and natural-born Filipino upon his reacquisition of
responsibilities as a lawyer and as an officer of Philippine citizenship.
the Court, but also renew his pledge to
maintain allegiance to the Republic of the
Philippines.
RULING:
Compliance with these conditions will restore
his good standing as a member of the Filipino citizens who have lost their citizenship
Philippine bar. may however reacquire the same in the
manner provided by law. C.A. No. 63
enumerates the 3 modes by which Philippine

Page 30 of 121
403 CASE DIGESTS | CONSTITUTIONAL LAW REVIEW | JUDGE ESTELA ALMA SINGCO | FIRST SEMESTER | 2020-2021

citizenship may be reacquired by a former


citizen: FACTS:

1. by naturalization, Petitioner Ernesto Mercado and Private


Respondent Eduardo Manzano were
2. by repatriation, and candidates for Vice Mayor in Makati on the May
11 1998 Elections. Manzano won on the count
3. by direct act of Congress. of 103,853 votes while Ernesto Mercado placed
second on the vote count of 100,894.
** However, the proclamation of winner was
suspended in lieu of a pending petition who
alleged the Private Respondent to be a US
Repatriation may be had under various
Citizen and not of the Philippines.
statutes by those who lost their citizenship due
to:

1. desertion of the armed forces;


Division of Comelec granted the petition and
ordered the cancellation of Private
2. services in the armed forces of the allied
Respondent’s candidacy on the grounds that
forces in World War II;
he has dual citizenship, and that under section
40(d) of the Local Government Code, those
3. service in the Armed Forces of the United holding dual citizenship are disqualified from
States at any other time, running for any elective local position.

4. marriage of a Filipino woman to an alien;


and
Private respondent was a son of both Filipino
5. political economic necessity citizens who were residing in San Francisco,
California, on which the United States follow
Repatriation results in the recovery of the Jus Soli or Citizenship on the basis of the birth
original nationality This means that a place. On filing his candidacy on April 27 1998,
naturalized Filipino who lost his citizenship will Private Respondent admitted that he is
be restored to his prior status as a naturalized registered as a foreigner with the Bureau of
Filipino citizen. On the other hand, if he was Immigration under Alien Certificate of
originally a natural-born citizen before he lost Registration no. B-31632 and alleged that he
his Philippine citizenship, he will be restored to is a Filipino Citizen because he was born on
his former status as a natural-born Filipino. 1955 of a Fil Father and Fil Mother on San
Francisco, California.

R.A. No. 2630 provides:


ISSUE:
Sec 1. Any person who had lost his Philippine
citizenship by rendering service to, or Is the Private Respondent a dual citizen and
accepting commission in, the Armed Forces of thus, makes him incapable to run for public
the United States, or after separation from the office?
Armed Forces of the United States, acquired
United States citizenship, may reacquire
Philippine citizenship by taking an oath of
allegiance to the Republic of the Philippines
RULING:
and registering the same with Local Civil
Registry in the place where he resides or last
resided in the Philippines. The said oath of He was a Natural Born Filipino citizen by
allegiance shall contain a renunciation of any operation of the 1935 Constitution as his
other citizenship. mother and father were Filipinos at time of his
birth. At the age of 6 Private Respondent, was
brought to the PH by his parents using a US
passport as traveling document and registered
him an alien with the PH Bureau of
Having thus taken the required oath of Immigration. However, this did not cause him
allegiance to the Republic and having the loss of his PH citizenship because he did
registered the same in the Civil Registry of not sworn allegiance to the US nor did he
Magantarem, Pangasinan in accordance with renounce his Filipino Citizenship.
the aforecited provision, Cruz is deemed to
have recovered his original status as a natural-
born citizen, a status which he acquired at
birth as the son of a Filipino father. It bears
stressing that the act of repatriation allows him Private Respondent registered himself as a
to recover, or return to, his original status voter on his age of majority and voted on the
before he lost his Philippine citizenship. 1992, 1995, and 1998 elections. Which
effectively renounced his US citizenship under
American law. Under PH law, he was no longer
an American citizen.

ISSUES ON CITIZENSHIP
46 Mercado vs. Manzano, 307 SCRA 630 The question was not of dual citizenship but of
DUAL ALLEGIANCE, wherein dual allegiance
Page 31 of 121
403 CASE DIGESTS | CONSTITUTIONAL LAW REVIEW | JUDGE ESTELA ALMA SINGCO | FIRST SEMESTER | 2020-2021

refers to the situation in which a person Filipino citizenship may run for a public office
simultaneously owes, by some positive act, in the Philippines.
loyalty to two or more states. While Dual
Citizenship is involuntary, dual allegiance is Section 5 of the said law states:
the result of an individual volition.
Section 5. Civil and Political Rights and
Liabilities. – Those who retain or re-acquire
Philippine citizenship under this Act shall enjoy
Thus, by registering himself to vote and by full civil and political rights and be subject to
filing and signing a certificate of candidacy, all attendant liabilities and responsibilities
wherein every certificate contains an oath of under existing laws of the Philippines and the
allegiance Private Respondent renounced his following conditions:
US Citizenship.
(2) Those seeking elective public office in the
Philippines shall meet the qualification for
47 Lopez vs. COMELEC, GR No. 182701, July holding such public office as required by the
23, 2008 Constitution and existing laws and, at the time
of the filing of the certificate of candidacy,
Petitioner Lopez, a dual citizen, was a make a personal and sworn renunciation of
candidate for the position of Barangay any and all foreign citizenship before any
Chairman of Brgy. Bagacay, San Dionisio, public officer authorized to administer an oath.
Iloilo City in the synchronized Barangay and
Sangguniang Kabataan Elections held on Lopez was able to regain his Filipino
October 29, 2007 without first making a Citizenship by virtue of the Dual Citizenship
personal and sworn renunciation of his foreign Law when he took his oath of allegiance before
citizenship. He was eventually declared the the Vice Consul of the Philippine Consulate
winner. General’s Office in Los Angeles, California; the
same is not enough to allow him to run for a
On October 25, 2007, respondent Villanueva public office.
filed a petition before the Provincial Election
Supervisor of the Province of Iloilo, praying for Lopez’s failure to renounce his American
the disqualification of Lopez because he was citizenship as proven by the absence of an
ineligible from running for any public office. affidavit that will prove the contrary leads this
Commission to believe that he failed to comply
Lopez argued that he is a Filipino-American, by with the positive mandate of law.
virtue of the Citizenship Retention and Re-
acquisition Act of 2003. He said he possessed While it is true that the petitioner won the
all the qualifications to run for Barangay elections, took his oath and began to discharge
Chairman. the functions of Barangay Chairman, his
victory cannot cure the defect of his candidacy.
On February 6, 2008, COMELEC issued the Garnering the most number of votes does not
Resolution granting the petition for validate the election of a disqualified candidate
disqualification of Lopez from running as because the application of the constitutional
Barangay Chairman. COMELEC said, to be able and statutory provisions on disqualification is
to qualify as a candidate in the elections, Lopez not a matter of popularity.
should have made a personal and sworn
renunciation of any and all foreign citizenship.
48 Llamanzares vs. COMELEC, G.R. No.
His motion for reconsideration having been 221697, March 8, 2016
denied, Lopez resorted to petition for
certiorari, imputing grave abuse of discretion
on the part of the COMELEC for disqualifying Facts:
him from running and assuming the office of
Barangay Chairman. In her COC for Presidency on the May 2016
elections, Grace Poe declared that she is a
ISSUE: natural-born citizen of the Philippines and that
her residence up to day before May 9, 2016
Whether or not there was grave abuse of would be 10 years and 11 months counted
discretion on the part of the COMELEC for from May 24, 2005.
disqualifying petitioner.

RULING:
Grace Poe was born in 1968., found as
No. The Supreme Court dismissed the petition. newborn infant in Jaro,Iloilo and was legally
The COMELEC committed no grave abuse of adopted by RONALD ALLAN KELLY POE (FPJ)
discretion in disqualifying petitioner as and JESUS SONORA POE (SUSAN ROCES) in
candidate for Chairman in the Barangay 1974. She immigrated to the US in 1991 after
elections of 2007. Lopez was born a Filipino but her marriage to Theodore Llamanzares who
he deliberately sought American citizenship was then based at the US. Grace Poe then
and renounced his Filipino citizenship. He later became a naturalized American citizen in
on became a dual citizen by re-acquiring 2001.
Filipino citizenship.

R.A. No. 9225 expressly provides for the


conditions before those who re-acquired On December 2004, he returned to the
Philippines due to his father’s deteriorating
medical condition, who then eventually demice
Page 32 of 121
403 CASE DIGESTS | CONSTITUTIONAL LAW REVIEW | JUDGE ESTELA ALMA SINGCO | FIRST SEMESTER | 2020-2021

on February 3,2005. She then quitted her job person and guarantees full respect for human
in the US to be with her grieving mother and rights," Article XIII, Section 1 which mandates
finally went home for good to the Philippines Congress to "give highest priority to the
on MAY 24, 2005. enactment of measures that protect and
enhance the right of all the people to human
dignity, reduce social, economic, and political
inequalities x x x" and Article XV, Section 3
On JULY 18, 2006, the BI granted her petition which requires the State to defend the "right
declaring that she had reacquired her Filipino of children to assistance, including proper care
citizenship under RA 9225. She registered as and nutrition, and special protection from all
a voter and obtained a new Philippine forms of neglect, abuse, cruelty, exploitation,
Passport. and other conditions prejudicial to their
development." Certainly, these provisions
contradict an intent to discriminate against
foundlings on account of their unfortunate
status.
In 2010, before assuming her post as
appointes Chairperson of the MTRCB , she
Recent legislation is more direct. R.A. No. 8043
renounced her American citizenship to satisfy
entitled "An Act Establishing the Rules to
the RA 9225 requirements as to Reacquistion
Govern the Inter-Country Adoption of Filipino
of Filipino Citizenship. From then on, she
Children and For Other Purposes" (otherwise
stopped using her American passport.
known as the "Inter-Country Adoption Act of
1995"), R.A. No. 8552, entitled "An Act
Establishing the Rules and Policies on the
Adoption of Filipino Children and For Other
Petitions were filed before the COMELEC to Purposes" (otherwise known as the Domestic
deny or cancel her candidacy on the ground Adoption Act of 1998) and this Court's A.M. No.
particularly among others, that she cannot be 02-6-02-SC or the "Rule on Adoption," all
considered a natural born Filipino citizen since expressly refer to "Filipino children" and
she was a FOUNDLING and that her bioligical include foundlings as among Filipino children
parents cannot be proved as Filipinos. The who may be adopted.
Comelec en banc cancelled her candidacy on
the ground that she is in want of citizenship Foundlings are likewise citizens under
and residence requirements and that she international law. Under the 1987
committed misrepresentation in her COC. Constitution, an international law can become
part of the sphere of domestic law either by
ISSUE: 1) WON GRACE POE IS A NATURAL transformation or incorporation. The
BORN CITIZEN OF THE PHILIPPINES transformation method requires that an
international law be transformed into a
HELD : ) domestic law through a constitutional
mechanism such as local legislation
1) Presumptions regarding paternity is neither
unknown nor unaccepted in Philippine Law. 1) Universal Declaration of Human Rights
The Family Code of the Philippines has a whole ("UDHR") has been interpreted by this Court
chapter on Paternity and Filiation. That said, as part of the generally accepted principles of
there is more than sufficient evidence that international law and binding on the State.
petitioner has Filipino parents and is therefore
a natural-born Filipino. The Solicitor General 2) The Philippines has also ratified the UN
offered official statistics from the Philippine Convention on the Rights of the Child (UNCRC)
Statistics Authority (PSA that from 1965 to
1975, the total number of foreigners born in 3) In 1986, the country also ratified the
the Philippines was 15,986 while the total 1966 International Covenant on Civil and
number of Filipinos born in the country was Political Rights (ICCPR).
10,558,278. The statistical probability that any
child born in the Philippines in that decade is
The common thread of the UDHR, UNCRC and
natural-born Filipino was 99.83%. Other
ICCPR is to obligate the Philippines to grant
circumstantial evidence of the nationality of
nationality from birth and ensure that no child
the petitioner's parents are the fact that she
is stateless. This grant of nationality must be
was abandoned as an infant in a Roman
at the time of birth, and it cannot be
Catholic Church in Iloilo City. She also has
accomplished by the application of our present
typical Filipino features: height, flat nasal
naturalization laws, Commonwealth Act No.
bridge, straight black hair, almond shaped
473, as amended, and R.A. No. 9139, both of
eyes and an oval face.
which require the applicant to be at least
eighteen (18) years old.
1) As a matter of law, foundlings are as a class,
natural-born citizens. While the 1935
That the Philippines is not a party to the 1930
Constitution's enumeration is silent as to
Hague Convention nor to the 1961 Convention
foundlings, there is no restrictive language
on the Reduction of Statelessness does not
which would definitely exclude foundlings
mean that their principles are not binding.
either. Because of silence and ambiguity in the
While the Philippines is not a party to the 1930
enumeration with respect to foundlings, there
Hague Convention, it is a signatory to the
is a need to examine the intent of the framers.
Universal Declaration on Human Rights. This
All exhort the State to render social justice. Of
Court noted that the Philippines had not signed
special consideration are several provisions in
or ratified the "International Convention for
the present charter:
the Protection of All Persons from Enforced
Disappearance." Yet, we ruled that the
Article II, Section 11 which provides that the proscription against enforced disappearances
"State values the dignity of every human in the said convention was nonetheless binding
Page 33 of 121
403 CASE DIGESTS | CONSTITUTIONAL LAW REVIEW | JUDGE ESTELA ALMA SINGCO | FIRST SEMESTER | 2020-2021

as a "generally accepted principle of Was Imelda a resident, for election purposes,


international law." Another case where the of the First District of Leyte for a period of one
number of ratifying countries was not year at the time of the May 9, 1995 elections.
determinative is Mijares v. Ranada, where only
four countries had "either ratified or acceded
to" the 1966 "Convention on the Recognition
and Enforcement of Foreign Judgments in Civil Held:
and Commercial Matters" when the case was
decided in 2005. In sum, all of the
international law conventions and instruments
on the matter of nationality of foundlings were
designed to address the plight of a defenseless 1. Imelda was a resident of the First District of
class which suffers from a misfortune not of Leyte for election purposes, and therefore
their own making. We cannot be restrictive as possessed the necessary residence
to their application if we are a country which qualifications to run in Leyte as a candidate for
calls itself civilized and a member of the a seat in the House of Representatives for the
community of nations following reasons:

49 Marcos vs Comelec (Sept. 18, 1995)


a. Minor follows the domicile of his parents. As
domicile, once acquired is retained until a new
Facts: one is gained, it follows that in spite of the fact
of petitioner's being born in Manila, Tacloban,
Leyte was her domicile of origin by operation
of law. This domicile was established when her
father brought his family back to Leyte.
Petitioner Imelda Romualdez-Marcos filed her
Certificate of Candidacy for the position of
b. Domicile of origin is not easily lost. To
Representative of the First District of Leyte in
successfully effect a change of domicile, one
1995, providing that her residence in the place
must demonstrate:
was seven (7) months.
1. An actual removal or an actual change of
domicile;

On March 23, 1995, Cirilo Roy Montejo, the


2. A bona fide intention of abandoning the
incumbent Representative of the First District
former place of residence and establishing a
of Leyte and also a candidate for the same
new one; and
position filed a petition for cancellation and
disqualification with the COMELEC charging
Marcos as she did not comply with the 3. Acts which correspond with the purpose.
constitutional requirement for residency as she
lacked the Constitution’s one-year residency In the absence of clear and positive proof
requirement for candidates for the House of based on these criteria, the residence of origin
Representative. should be deemed to continue. Only with
evidence showing concurrence of all three
In her Amended Corrected Certificate of requirements can the presumption of
Candidacy, the petitioner changed seven continuity or residence be rebutted, for a
months to since childhood under residency. change of residence requires an actual and
Thus, the petitioner’s motion for deliberate abandonment, and one cannot have
reconsideration was denied. two legal residences at the same time.
Petitioner held various residences for different
purposes during the last four decades. None of
these purposes unequivocally point to an
intention to abandon her domicile of origin in
On May 11, 1995, the COMELEC issued a Tacloban, Leyte.
Resolution allowing petitioner’s proclamation
showing that she obtained the highest number
c. It cannot be correctly argued that petitioner
of votes in the congressional elections in the
lost her domicile of origin by operation of law
First District of Leyte. The COMELEC reversed
as a result of her marriage to the late President
itself and issued a second Resolution directing
Ferdinand E. Marcos in 1952. A wife does not
that the proclamation of petitioner be
automatically gain the husband’s domicile.
suspended in the event that she obtains the
What petitioner gained upon marriage was
highest number of votes.
actual residence. She did not lose her domicile
of origin. The term residence may mean one
thing in civil law (or under the Civil Code) and
quite another thing in political law. What
In a Supplemental Petition dated 25 May 1995, stands clear is that insofar as the Civil Code is
Marcos claimed that she was the concerned-affecting the rights and obligations
overwhelming winner of the elections based on of husband and wife — the term residence
the canvass completed by the Provincial Board should only be interpreted to mean "actual
of Canvassers. residence." The inescapable conclusion
derived from this unambiguous civil law
Issues: delineation therefore, is that when petitioner
married the former President in 1954, she kept
her domicile of origin and merely gained a new
home, not a domicilium necessarium.

Page 34 of 121
403 CASE DIGESTS | CONSTITUTIONAL LAW REVIEW | JUDGE ESTELA ALMA SINGCO | FIRST SEMESTER | 2020-2021

Cordora’s motion for reconsideration for


lack of merit.
d. Even assuming for the sake of argument
that petitioner gained a new "domicile" after ISSUE : WON there is Probable Cause to
her marriage and only acquired a right to Hold Tambunting for Trial for Having
choose a new one after her husband died, Committed an Election Offense
petitioner's acts following her return to the
country clearly indicate that she not only HELD : Probable cause constitutes those
impliedly but expressly chose her domicile of facts and circumstances which would lead
origin (assuming this was lost by operation of a reasonably discreet and prudent man to
law) as her domicile. This "choice" was believe that an offense has been
unequivocally expressed in her letters to the committed. Determining probable cause
Chairman of the PCGG when petitioner sought is an intellectual activity premised on the
the PCGG's permission to "rehabilitate (our) prior physical presentation or submission
ancestral house in Tacloban and Farm in Olot, of documentary or testimonial proofs
Leyte ... to make them livable for the Marcos either confirming, negating or qualifying
family to have a home in our homeland." the allegations in the complain
Furthermore, petitioner obtained her residence
certificate in 1992 in Tacloban, Leyte, while Tambunting does not deny that he is born
living in her brother's house, an act which of a Filipino mother and an American
supports the domiciliary intention clearly father. Neither does he deny that he
manifested in her letters to underwent the process involved in INS
Form I-130 (Petition for Relative)
50 Cordora vs Comelec (Feb. 19, 2009) because of his father’s citizenship.
Tambunting claims that because of his
parents’ differing citizenships, he is both
FACTS: In his complaint affidavit filed Filipino and American by birth. Cordora,
before the COMELEC Law Department, on the other hand, insists that
Cordora asserted that Tambunting made Tambunting is a naturalized American
false assertions. Cordora stated that citizen.
Tambunting was not eligible to run for
local public office because Tambunting We agree with Commissioner Sarmiento’s
lacked the required citizenship and observation that Tambunting possesses
residency requirements. dual citizenship. Because of the
circumstances of his birth, it was no
To disprove Tambunting’s claim of being longer necessary for Tambunting to
a natural-born Filipino citizen, Cordora undergo the naturalization process to
presented a certification from the Bureau acquire American citizenship. The process
of Immigration which stated that, in two involved in INS Form I-130 only served to
instances, Tambunting claimed that he is confirm the American citizenship which
an American: upon arrival in the Tambunting acquired at birth. The
Philippines on 16 December 2000 and certification from the Bureau of
upon departure from the Philippines on Immigration which Cordora presented
17 June 2001. According to Cordora, contained two trips where Tambunting
these travel dates confirmed that claimed that he is an American. However,
Tambunting acquired American the same certification showed nine other
citizenship through naturalization in trips where Tambunting claimed that he
Honolulu, Hawaii on 2 December 2000. is Filipino. Clearly, Tambunting possessed
Cordora concluded: dual citizenship prior to the filing of his
certificate of candidacy before the 2001
The COMELEC Law Department elections. The fact that Tambunting had
recommended the dismissal of Cordora’s dual citizenship did not disqualify him
complaint against Tambunting because from running for public office.
Cordora failed to substantiate his charges
against Tambunting. Cordora’s reliance To begin with, dual citizenship is different
on the certification of the Bureau of from dual allegiance. The former arises
Immigration that Tambunting traveled on when, as a result of the concurrent
an American passport is not sufficient to application of the different laws of two or
prove that Tambunting is an American more states, a person is simultaneously
citizen. considered a national by the said states.
For instance, such a situation may arise
when a person whose parents are citizens
of a state which adheres to the principle
The COMELEC En Banc affirmed the of jus sanguinis is born in a state which
findings and the resolution of the follows the doctrine of jus soli. Such a
COMELEC Law Department. The COMELEC person, ipso facto and without any
En Banc was convinced that Cordora voluntary act on his part, is concurrently
failed to support his accusation against considered a citizen of both states
Tambunting by sufficient and convincing
evidence. Dual allegiance, on the other hand, refers
to the situation in which a person
Cordora filed a motion for reconsideration simultaneously owes, by some positive
which raised the same grounds and the act, loyalty to two or more states. While
same arguments in his complaint. In its dual citizenship is involuntary, dual
Resolution promulgated on 20 February allegiance is the result of an individual’s
2007, the COMELEC En Banc dismissed volition.

Page 35 of 121
403 CASE DIGESTS | CONSTITUTIONAL LAW REVIEW | JUDGE ESTELA ALMA SINGCO | FIRST SEMESTER | 2020-2021

In Sections 2 and 3 of R.A. No. 9225, the Ty had been residing in the USA for the last 25
framers were not concerned with dual years. When Ty filed his Certificate of
citizenship per se, but with the status of Candidacy on 28 March 2007, he falsely
naturalized citizens who maintain their represented therein that he was a resident of
allegiance to their countries of origin Barangay6, Poblacion, General Macarthur,
even after their naturalization.12 Section Eastern Samar, for one year before 14 May
5(3) of R.A. No. 9225 states that 2007, and was not a permanent resident or
naturalized citizens who reacquire immigrant of any foreign country.
Filipino citizenship and desire to run for
elective public office in the Philippines
shall "meet the qualifications for holding
such public office as required by the While Ty may have applied for the
Constitution and existing laws and, at the reacquisition of his Philippine citizenship, he
time of filing the certificate of candidacy, never actually resided in Barangay 6,
make a personal and sworn renunciation Poblacion, General Macarthur, Eastern Samar,
of any and all foreign citizenship before for a period of one year immediately preceding
any public officer authorized to the date of election as required under Section
administer an oath" aside from the oath 39 of Republic Act No. 7160, otherwise known
of allegiance prescribed in Section 3 of as the Local Government Code of 1991
R.A. No. 9225. The twin requirements of
swearing to an Oath of Allegiance and
executing a Renunciation of Foreign
Citizenship served as the bases for our
recent rulings in Jacot v. Dal and Inspite of having reacquisition in his Philippine
COMELEC,13 Velasco v. COMELEC,14 and citizenship, Ty continued to make trips to the
Japzon v. COMELEC,15 all of which USA, the most recent of which was on 31
involve natural-born Filipinos who later October 2006 lasting until 20 January 2007.
became naturalized citizens of another
country and thereafter ran for elective
office in the Philippines. In the present
case, Tambunting, a natural-born Filipino, Ty already took his Oath of Allegiance to the
did not subsequently become a Republic of the Philippines, he continued to
naturalized citizen of another country. comport himself as an American citizen as
Hence, the twin requirements in R.A. No. proven by his travel records. He had also failed
9225 do not apply to him. to renounce his foreign citizenship as required
by Republic Act No. 9225, otherwise known as
Cordora concluded that Tambunting the Citizenship Retention and Reacquisition Act
failed to meet the residency requirement of 2003, or related laws.
because of Tambunting’s naturalization
as an American. Cordora’s reasoning fails
because Tambunting is not a naturalized
American. Moreover, residency, for the Japzon prayed for in his Petition that the
purpose of election laws, includes the COMELEC order the disqualification of Ty from
twin elements of the fact of residing in a running for public office and the cancellation of
fixed place and the intention to return the latter's Certificate of Candidacy.
there permanently,16 and is not
dependent upon citizenship.

In view of the above, we hold that Ty admitted that he was a natural-born Filipino
Cordora failed to establish that who went to the USA to work and subsequently
Tambunting indeed willfully made false became a naturalized American citizen. Ty
entries in his certificates of candidacy. On claimed, however, that prior to filing his
the contrary, Tambunting sufficiently Certificate of Candidacy for the Office of Mayor
proved his innocence of the charge filed of the Municipality of General Macarthur,
against him. Tambunting is eligible for Eastern Samar, on 28 March 2007, he already
the office which he sought to be elected performed the following acts:
and fulfilled the citizenship and residency
requirements prescribed by law.
(1) with the enactment of Republic Act No.
9225, granting dual citizenship to natural-born
51 Japson vs Comelec (Jan. 19, 2009) Filipinos, Ty filed with the Philippine Consulate
General in Los Angeles, California, USA, an
application for the reacquisition of his
Japzon instituted SPA No. 07-568 by filing Philippine citizenship;
before the COMELEC a Petition to disqualify
and/or cancel Ty's Certificate of Candidacy on (2) on 2 October 2005, Ty executed an Oath
the ground of material misrepresentation. of Allegiance to the Republic of the Philippines
Japzon averred in his Petition that Ty was a before Noemi T. Diaz, Vice Consul of the
former natural-born Filipino, having been born Philippine Consulate General in Los Angeles,
on 9 October 1943 in what was then Pambujan California, USA;
Sur, Hernani Eastern Samar (now the
Municipality of General Macarthur, Easter
(3) Ty applied for a Philippine passport
Samar) to spouses Ang Chim Ty (a Chinese)
indicating in his application that his residence
and Crisanta Aranas Sumiguin (a Filipino).
in the Philippines was at A. Mabini St.,
Barangay 6, Poblacion, General Macarthur,
Eastern Samar. Ty's application was approved

Ty eventually migrated to the United States of


America (USA) and became a citizen thereof.
Page 36 of 121
403 CASE DIGESTS | CONSTITUTIONAL LAW REVIEW | JUDGE ESTELA ALMA SINGCO | FIRST SEMESTER | 2020-2021

and he was issued on 26 October 2005 a General Macarthur, Eastern Samar for at least
Philippine passport; one (1) year before the elections held on 14
May 2007 as he represented in his certificate
(4) on 8 March 2006, Ty personally secured of candidacy.
and signed his Community Tax Certificate
(CTC) from the Municipality of General The petition was denied and COMELEC was in
Macarthur, in which he stated that his address favor of the defendant failing to obtain a
was at Barangay 6, Poblacion, General favorable resolution from the COMELEC,
Macarthur, Eastern Samar; Japzon proceeded to file the instant Petition for
Certiorari, that the COMELEC had committed
(5) thereafter, on 17 July 2006, Ty was grave abuse of discretion and lack of discretion
registered as a voter in Precinct 0013A, for dismissing the petition.
Barangay 6, Poblacion, General Macarthur,
Eastern Samar; Japzon prays for the Court to annul and set
aside the Resolutions dated 31 July 2007 and
(6) Ty secured another CTC dated 4 January 28 September 2007 of the COMELEC First
2007 again stating therein his address as Division and en banc, respectively; to issue a
Barangay 6, Poblacion, General Macarthur, new resolution denying due course to or
Eastern Samar; and (7) finally, Ty executed on canceling Ty's Certificate of Candidacy; and to
19 March 2007 a duly notarized Renunciation declare Japzon as the duly elected Mayor of the
of Foreign Citizenship. Municipality of General Macarthur, Eastern
Samar.
He had reacquired his Philippine citizenship
and renounced his American citizenship, and Ty sought the dismissal of the present Petition.
he had been a resident of the Municipality of According to Ty, the COMELEC already found
General Macarthur, Eastern Samar, for more sufficient evidence to prove that Ty was a
than one year prior to the 14 May 2007 resident of the Municipality of General
elections. Therefore, Ty sought the dismissal Macarthur, Eastern Samar, one year prior to
of Japzon's Petition in SPA No. 07-568. the 14 May 2007 local elections. The Court
cannot evaluate again the very same pieces of
Ty acquired the highest number of votes and evidence without violating the well-entrenched
was declared Mayor of the Municipality of rule that findings of fact of the COMELEC are
General Macarthur, Eastern Samar, by the binding on the Court.
Municipal Board of Canvassers on 15 May
2007. The Office of the Solicitor General (OSG),
meanwhile, is of the position that Ty failed to
meet the one-year residency requirement set
by law to qualify him to run as a mayoralty
candidate in the 14 May 2007 local
The COMELEC First Division found that Ty
elections.The Court finds no merit in the
complied with the requirements of Sections 3
Petition at bar.
and 5 of Republic Act No. 9225 and reacquired
his Philippine citizenship, to wit:
On 19 March 2007, he personally executed a
Renunciation of Foreign Citizenship before a
Philippine citizenship is an indispensable
notary public. By the time he filed his
requirement for holding an elective public
Certificate of Candidacy for the Office of Mayor
office, and the purpose of the citizenship
of the Municipality of General Macarthur,
qualification is none other than to ensure that
Eastern Samar, on 28 March 2007, he had
no alien, i.e., no person owing allegiance to
already effectively renounced his American
another nation, shall govern our people and
citizenship, keeping solely his Philippine
our country or a unit of territory thereof.
citizenship.

The Court of Appeals set aside the appealed


orders of the COMELEC and the Court of
Pieces of evidence revealed that Ty executed Appeals and annulled the election of the
an Oath of Allegiance before Noemi T. Diaz, respondent as Municipal Mayor of Bolinao,
Vice Consul of the Philippine Consulate Pangasinan on the ground that respondent's
General, Los Angeles, California, U.S.A. on immigration to the United States in 1984
October 2, 2005 and executed a Renunciation constituted an abandonment of his domicile
of Foreign Citizenship on March 19, 2007 in and residence in the Philippines. Being a green
compliance with R.A. [No.] 9225. Moreover, card holder, which was proof that he was a
neither is Ty a candidate for or occupying permanent resident or immigrant of the United
public office nor is in active service as States, and in the absence of any waiver of his
commissioned or non-commissioned officer in status as such before he ran for election on
the armed forces in the country of which he January 18, 1988, respondent was held to be
was naturalized citizen disqualified under §68 of the Omnibus Election
Code of the Philippines (Batas Pambansa Blg.
Ty did not commit material misrepresentation 881).
in stating in his Certificate of Candidacy that
he was a resident of Barangay 6, Poblacion,
General Macarthur, Eastern Samar, for at least
one year before the elections on 14 May 2007.
ISSUE:
It reasoned that: Although Ty has lost his
domicile in [the] Philippines when he was
naturalized as U.S. citizen in 1969, the Whether or not the defedant has complied with
reacquisition of his Philippine citizenship and the residency requirement for elective
subsequent acts thereof proved that he has positions.
been a resident of Barangay 6, Poblacion,
Page 37 of 121
403 CASE DIGESTS | CONSTITUTIONAL LAW REVIEW | JUDGE ESTELA ALMA SINGCO | FIRST SEMESTER | 2020-2021

RULING: To successfully challenge Ty's disqualification,


Japzon must clearly demonstrate that Ty's
Yes, the defendant solely complied the ineligibility is so patently antagonistic to
residency requirements for elective position. constitutional and legal principles that
overriding such ineligibility and thereby giving
It bears to point out that Republic Act No. 9225 effect to the apparent will of the people would
governs the manner in which a natural-born ultimately create greater prejudice to the very
Filipino may reacquire or retain his Philippine democratic institutions and juristic traditions
citizenship despite acquiring a foreign that our Constitution and laws so zealously
citizenship, and provides for his rights and protect and promote. In this case, Japzon
liabilities under such circumstances. A close failed to substantiate his claim that Ty is
scrutiny of said statute would reveal that it ineligible to be Mayor of the Municipality, the
does not at all touch on the matter of residence instant Petition for Certiorari is dismiss.
of the natural-born Filipino taking advantage
of its provisions. Republic Act No. 9225 52 Jalosjos vs Comelec (Oct. 19, 2010)
imposes no residency requirement for the
reacquisition or retention of Philippine
citizenship; nor does it mention any effect of FACTS:
such reacquisition or retention of Philippine
citizenship on the current residence of the Petitioner Rommel Jalosjos was born in
concerned natural-born Filipino. Clearly, Quezon City. He migrated to Australia when
Republic Act No. 9225 treats citizenship he was eight years old and acquired Australian
independently of residence. This is only logical citizenship. In 2008, he returned to the
and consistent with the general intent of the Philippines and lived in Zamboanga, he took an
law to allow for dual citizenship. oath of allegiance to the Philippines and was
issued a certificate of reacquisition of
There is no basis for this Court to require Ty to citizenship by the Bureau of Immigration and
stay in and never leave at all the Municipality he renounced his Australian citizenship.
of General Macarthur, Eastern Samar, for the
full one-year period prior to the 14 May 2007
local elections so that he could be considered
a resident thereof. To the contrary, the Court Jalosjos applied for registration as a voter in
has previously ruled that absence from Ipil, Zamboanga Sibugay, but Private
residence to pursue studies or practice a Respondent Erasmo, the barangay captain,
profession or registration as a voter other than opposed the registration. COMELEC approved
in the place where one is elected, does not the application and included Jalosjos in the
constitute loss of residence. The Court also voter's list. This decision was affirmed at the
notes, that even with his trips to other MCTC and at the RTC.
countries, Ty was actually present in the
Municipality of General Macarthur, Eastern
Samar, Philippines, for at least nine of the 12
months preceding the 14 May 2007 local
elections. Even if length of actual stay in a Jalosjos then filed a certificate of candidacy
place is not necessarily determinative of the (COC) for Governor of Zamboanga Sibugay for
fact of residence therein, it does strongly the 2010 elections. Erasmo filed a petition to
support and is only consistent with Ty's cancel the COC on the ground of failure to
avowed intent in the instant case to establish comply with the one year residency
residence/domicile in the Municipality of requirement of the Local Government Code
General Macarthur, Eastern Samar. (LGC).

Japzon repeatedly brings to the attention of COMELEC held that Jalosjos failed to present
this Court that Ty arrived in the Municipality of ample proof of a bona fide intention to
General Macarthur, Eastern Samar, on 4 May establish a domicile in Ipil, Zamboanga
2006 only to comply with the one-year Sibugay. It held that when he first moved back
residency requirement, so Ty could run as a to the Philippines, he was merely a guest or
mayoralty candidate in the 14 May 2007 transient at his brother's house in Ipil, and for
elections. In Aquino v. COMELEC, the Court did this reason, he cannot claim Ipil as his
not find anything wrong in an individual domicile. Meanwhile, Jalosjos won the
changing residences so he could run for an elections.
elective post, for as long as he is able to prove
with reasonable certainty that he has effected
a change of residence for election law
purposes for the period required by law. As this ISSUE:
Court already found in the present case, Ty has
proven by substantial evidence that he had Whether or not the COMELEC is correct in
established residence/domicile in the holding that petitioner did not present ample
Municipality of General Macarthur, Eastern proof of a bona fide intention to establish
Samar, by 4 May 2006, a little over a year prior domicile at Ipil, Zamboanga Sibugay.
to the 14 May 2007 local elections, in which he
ran as a candidate for the Office of the Mayor
and in which he garnered the most number of
votes.
HELD:

NO. The COMELEC is incorrect. Jalosjos has


successfully proven by his acts of renouncing
Page 38 of 121
403 CASE DIGESTS | CONSTITUTIONAL LAW REVIEW | JUDGE ESTELA ALMA SINGCO | FIRST SEMESTER | 2020-2021

his Australian citizenship and by living in Ipil, lot in Ipil and a fish pond in San Isidro, Naga,
that he has changed his domicile to Zamboanga Sibugay. This, without a doubt is
Zamboanga Sibugay. sufficient to establish his intent to set his
domicile in Ipil, Zamboanga Sibugay.

53 Maquiling vs Comelec (April 16, 2013)


The LGC requires that a gubernatorial
candidate be a resident of the province for at
least one year before the elections. For the Respondent Arnado is a natural born Filipino
purposes of election laws, the requirement of citizen. However, as a consequence of his
residence is synonymous with domicile: i.e. he subsequent naturalization as a citizen of USA,
must have an intention to reside in a he lost his Filipino citizenship. Arnado applied
particulaar place, but must also have personal for repatriation under R.A. No. 9225 before the
presence coupled with conduct indicative of Consulate General of the Philippines in San
such intention. Francisco, USA and took the Oath of
Allegianceto the Republic of the Philippines on
The question of residence is a question of 10 July 2008. On the same day an order of
intention. To determine compliance with the approval of his citizenship retention and re-
residency/domicile requirement, jurisprudence acquisition was issued in his favour.
has laid down the following guideline:

(a) every person has a domicile or residence


somewhere In 2009, Arnado again took his Oath of
Allegiance to RP and executed an affidavit of
(b) where once established, that domicile renunciation of his foreign citizenship. On 30
remains until he acquires a new one; and November 2009, Arnado filed his certificate of
candidacy for Mayor of Kauswagan, Lanao Del
(c) a person can have but one domicile at a Norte. Respondent Linog Balua, another
time. mayoralty candidate, filed a petition to
disqualify Arnado and presented a record
indicating that Arnado has been using his US
The facts show that Jalosjos' domicile of origin
Passport in entering and departing the
was Quezon city. When he acquired Australian
Philippines.
citizenship, Australia became his domicile by
operation of law and by choice. On the other
hand, when he came to the Philippines in
November 2008 to live with his brother in
Zamboanga Sibugay, it is evident that Jalosjos COMELEC issued an order requiring the
did so with intent to change his domicile for respondent to personally file his answer. After
good. He left Australia, gave up his Australian Arnado failed to answer the petition, Balua
citizenship, and renounced his allegiance to moved to declare him in default. In 2010
that country and reacquired his old citizenship election, Arnado garnered the highest number
by taking an oath of allegiance to the of votes and was subsequently proclaimed as
Philippines. By his acts, Jalosjos forfeited his the winning candidate for Mayor. It was only
legal right to live in Australia, clearly proving after his proclamation that Arnado filed his
that he gave up his domicile there. And he has answer. COMELEC first division ruled for his
since lived nowhere else except in Ipil, disqualification. Petitioner Maquiling, another
Zamboanga Sibugay. candidate for mayor of Kausawagan, and who
garnered the second highest number of votes,
To hold that Jalosjos has not established a new intervened in the case and filed before the
domicile in Zamboanga Sibugay despite the COMELEC En Banc a motion for
loss of his domicile of origin (Quezon City) and reconsideration claiming that the cancellation
his domicile of choice and by operation of law of Arnado’s candidacy and the nullification of
(Australia) would violate the settled maxim his proclamation, him, as the legitimate
that a man must have a domicile or residence candidate who obtained the highest lawful
somewhere. votes should be proclaimed as the winner.

Neither can COMELEC conclude that Jalosjos


did not come to settle his domicile in Ipil since
he has merely been staying at his brother's COMELEC En Banc held that it shall continue
house. A candidate is not required to have a with the trial and hearing. However, it
house in order to establish his residence or reversed and set aside the ruling of first
domicile in that place. It is enough that he division and granted Arnado’s MR. Maquiling
should live there even if it be in a rented house filed the instant petition questioning the
or in the house of a friend or relative. To insist propriety of declaring Arnado qualified to run
that the candidate own the house where he for public office despite his continued use of a
lives would make property a qualification for US passport, and praying that he be
public office. What matters is that Jalosjos has proclaimed as the winner in the 2010
proved two things: actual physical presence in mayoralty race.
Ipil and an intention of making it his domicile.

As evidence, Jalosjos presented his next-door


neighbors who testified that he was physically Issue: Whether or not the use of a foreign
present in Ipil, he presented correspondence passport after renouncing foreign citizenship
with political leaders and local and national amount to undoing a renunciation earlier
party mates, furthermore, he is a registered made.
voter by final judgement of the RTC. The court
also noted that Jalosjos has since acquired a
Page 39 of 121
403 CASE DIGESTS | CONSTITUTIONAL LAW REVIEW | JUDGE ESTELA ALMA SINGCO | FIRST SEMESTER | 2020-2021

countries that he is an American citizen, with


all attendant rights and privileges granted by
Held: Yes. The Supreme Court ruled that the the United States of America. The renunciation
use of foreign passport after renouncing one’s of foreign citizenship is not a hollow oath that
foreign citizenship is a positive and voluntary can simply be professed at any time, only to
act of representation as to one’s nationality be violated the next day. It requires an
and citizenship; it does not divest Filipino absolute and perpetual renunciation of the
citizenship regained by repatriation but it foreign citizenship and a full divestment of all
recants the Oath of Renunciation required to civil and political rights granted by the foreign
qualify one to run for an elective position. country which granted the citizenship. While
Section 5(2) of The Citizenship Retention and the act of using a foreign passport is not one
Re-acquisition of the acts enumerated in Commonwealth Act
No. 63 constituting renunciation and loss of
Philippine citizenship, it is nevertheless an act
which repudiates the very oath of renunciation
required for a former Filipino citizen who is also
Act of 2003 provides:
a citizen of another country to be qualified to
run for a local elective position.
Those who retain or re-acquire Philippine
citizenship under this Act shall enjoy full civil
When Arnado used his US passport on 14 April
and political rights and be subject to all
2009, or just eleven days after he renounced
attendant liabilities and responsibilities under
his American citizenship, he recanted his Oath
existing laws of the Philippines and the
of Renunciation that he "absolutely and
following conditions:
perpetually renounce(s) all allegiance and
fidelity to the UNITED STATES OF AMERICA"
xxxx and that he "divest(s) himself of full
employment of all civil and political rights and
(2)Those seeking elective public in the privileges of the United States of America."
Philippines shall meet the qualification for
holding such public office as required by the We agree with the COMELEC En Banc that such
Constitution and existing laws and, at the time act of using a foreign passport does not divest
of the filing of the certificate of candidacy, Arnado of his Filipino citizenship, which he
make a personal and sworn renunciation of acquired by repatriation. However, by
any and all foreign before any public officer representing himself as an American citizen,
authorized to administer an oath. Arnado voluntarily and effectively reverted to
his earlier status as a dual citizen. Such
xxx reversion was not retroactive; it took place the
instant Arnado represented himself as an
American citizen by using his US passport. This
act of using a foreign passport after
Rommel Arnado took all the necessary steps to renouncing one’s foreign citizenship is fatal to
qualify to run for a public office. He took the Arnado’s bid for public office, as it effectively
Oath of Allegiance and renounced his foreign imposed on him a disqualification to run for an
citizenship. There is no question that after elective local position.
performing these twin requirements required
under Section 5(2) of R.A. No. 9225 or the The citizenship requirement for elective public
Citizenship Retention and Re-acquisition Act of office is a continuing one. It must be possessed
2003, he became eligible to run for public not just at the time of the renunciation of the
office. By renouncing his foreign citizenship, he foreign citizenship but continuously. Any act
was deemed to be solely a Filipino citizen, which violates the oath of renunciation opens
regardless of the effect of such renunciation the citizenship issue to attack.
under the laws of the foreign country.
Citizenship is not a matter of convenience. It
However, this legal presumption does not is a badge of identity that comes with
operate permanently and is open to attack attendant civil and political rights accorded by
when, after renouncing the foreign citizenship, the state to its citizens. It likewise demands
the citizen performs positive acts showing his the concomitant duty to maintain allegiance to
continued possession of a foreign citizenship. one’s flag and country. While those who
Arnado himself subjected the issue of his acquire dual citizenship by choice are afforded
citizenship to attack when, after renouncing the right of suffrage, those who seek election
his foreign citizenship, he continued to use his or appointment to public office are required to
US passport to travel in and out of the country renounce their foreign citizenship to be
before filing his certificate of candidacy on 30 deserving of the public trust. Holding public
November 2009. The pivotal question to office demands full and undivided allegiance to
determine is whether he was solely and the Republic and to no other.
exclusively a Filipino citizen at the time he filed
his certificate of candidacy, thereby rendering We therefore hold that Arnado, by using his US
him eligible to run for public office. passport after renouncing his American
citizenship, has recanted the same Oath of
Between 03 April 2009, the date he renounced Renunciation he took. Section 40(d) of the
his foreign citizenship, and 30 November Local Government Code applies to his
2009, the date he filed his COC, he used his situation. He is disqualified not only from
US passport four times, actions that run holding the public office but even from
counter to the affidavit of renunciation he had becoming a candidate in the May 2010
earlier executed. By using his foreign passport, elections.
Arnado positively and voluntarily represented
himself as an American, in effect declaring With Arnado being barred from even becoming
before immigration authorities of both a candidate, his certificate of candidacy is thus
Page 40 of 121
403 CASE DIGESTS | CONSTITUTIONAL LAW REVIEW | JUDGE ESTELA ALMA SINGCO | FIRST SEMESTER | 2020-2021

rendered void from the beginning. It could not No.


have produced any other legal effect except
that Arnado rendered it impossible to effect his The dust had long settled over the implications
disqualification prior to the elections because of a “green card” holder status on an elective
he filed his answer to the petition when the official's qualification for public office. We ruled
elections were conducted already and he was in Caasi v. Court of Appeals that a Filipino
already proclaimed the winner. citizen's acquisition of a permanent resident
status abroad constitutes an abandonment of
Arnado being a non-candidate, the votes cast his domicile and residence in the Philippines.
in his favor should not have been counted. This In short, the “green card” status in the USA is
leaves Maquiling as the qualified candidate a renunciation of one's status as a resident of
who obtained the highest number of votes. the Philippines. x x x x
Therefore, the rule on succession under the
Local Government Code will not apply. Domicile is classified into

(1) domicile of origin, which is acquired by


54 Ugdoracion, Jr. vs. COMELEC, 552 SCRA every person at birth;
231
(2) domicile of choice, which is acquired upon
abandonment ofthe domicile of origin; and
Jose Ugdoracion, Jr., (petitioner) and Ephraim
Tungol (respondent) were rival mayoralty (3) domicile by operation of law, which the law
candidates in the Municipality of Albuquerque, attributes to a person independently of his
Province of Bohol in the May 14, 2007 residence or intention. x x x x
elections. Both filed their respective
Certificates of Candidacy (COC).

The general rule is that the domicile of origin


is not easily lost; it is lost only when there is
On April 11, 2007, Tungol filed a Petition to an actual removal or change of domicile, a
Deny Due Course or Cancel the Certificate of bona fide intention of abandoning the former
Candidacy of Jose Ugdoracion, Jr., contending residence and establishing a new one, and acts
that Ugdoracion's declaration of eligibility for which correspond with such purpose. In the
Mayor constituted material misrepresentation instant case, however, Ugdoracion's
because Ugdoracion is actually a "green card" acquisition of a lawful permanent resident
holder or a permanent resident of the United status in the United States amounted to an
States of America (USA). Specifically, abandonment and renunciation of his status as
Ugdoracion stated in his COC that he had a resident of the Philippines; it constituted a
resided in Albuquerque, Bohol, Philippines for change from his domicile of origin, which was
forty-one years before May 14, 2007 and he is Albuquerque, Bohol, to a new domicile of
not a permanent resident or an immigrant to a choice, which is the USA.
foreign country.
The contention that Ugdoracion’s USA resident
status was acquired involuntarily, as it was
simply the result of his sister's beneficence,
Tungol’s contention: In our jurisdiction, does not persuade. Although immigration to
domicile is equivalent to residence, and he the USA through a petition filed by a family
retained his domicile of origin (Albuquerque, member (sponsor) is allowed by USA
Bohol) notwithstanding his ostensible immigration laws, the petitioned party is very
acquisition of permanent residency in the USA. much free to accept or reject the grant of
Also, his status as a “green card” holder was resident status. Permanent residency in the
not of his own making but a mere offshoot of USA is not conferred upon the unwilling; unlike
a petition filed by his sister.He admitted his citizenship, it is not bestowed by operation of
intermittent travels to the USA, but only to law. To reiterate, a person can have only one
visit his siblings, and short working stint residence or domicile at any given time. x x x
thereat to cover his subsistence for the x
duration of his stay.
We are not unmindful of the fact that
Ugdoracion appears to have won the election
as Mayor of Albuquerque, Bohol. Sadly,
On May 8, 2007, the COMELEC First Division winning the election does not substitute for the
promulgated ruled to cancel Ugdoracion's COC specific requirements of law on a person’s
and remove his name from the certified list of eligibility for public office which he lacked, and
candidates for the position of Mayor of does not cure his material misrepresentation
Albuquerque, Bohol. which is a valid ground for the cancellation of
his COC.

Any false representation of material fact stated


Issue: in the certificate of candidacy shall be a ground
for cancellation thereof. Section 74, in relation
Whether or not the COMELEC committed grave to Section 78 of the Code requires that the
abuse of discretion in cancelling Ugdoracion's facts stated in the certificate of candidacy must
COC for material misrepresentation. be true and any false representation therein of
a material fact shall be a ground for
cancellation thereof. The false representation
Ruling:
contemplated by Section 78 of the Code

Page 41 of 121
403 CASE DIGESTS | CONSTITUTIONAL LAW REVIEW | JUDGE ESTELA ALMA SINGCO | FIRST SEMESTER | 2020-2021

pertains to material fact, and is not simply an Philippine Army; Gen. Avelino I. Razon, Chief,
innocuous mistake. Philippine National Police (PNP); Gen. Edgardo
M. Doromal, Chief, Criminal Investigation and
Detention Group (CIDG); Sr. Supt. Leonardo
A. Espina, Chief, Police Anti-Crime and
The candidate’s misrepresentation in his/her Emergency Response; Gen. Joel Goltiao,
certificate of candidacy must not only refer to Regional Director, ARMM-PNP; and Gen.
a material fact (eligibility and qualifications for Ruben Rafael, Chief, Anti-Terror Task Force
elective office) but should evince a deliberate Comet [collectively referred to as petitioners].
intent to mislead, misinform or hide a fact
which would otherwise render a candidate On the same day the petition was filed, the CA
ineligible. A candidate’s disqualification to run immediately issued the Writ of Amparo, set
for public office does not necessarily constitute the case for hearing on January 7, 2008, and
material misrepresentation as a sole ground directed the petitioners to file their verified
for the cancellation of the certificate of return within seventy-two (72) hours from
candidacy. It must be made with an intention service of the writ.
to deceive the electorate as to one’ s
qualifications to run for public office. Winning
the elections does not substitute for the On March 7, 2008, the CA issued its decision
specific requirements of law on a person’s confirming that the disappearance of Tagitis
eligibility for public office which he lacked. was an “enforced disappearance” under the
Getting the plurality of votes needed does not United Nations (UN) Declaration on the
cure his/her material misrepresentation. Such Protection of All Persons from Enforced
misrepresentation may be used as a valid Disappearances.
ground for the cancellation of certificate of
candidacy. Issue: Whether or not the petition for writ of
amparo is proper in this case

Held: The Supreme Court ruled in the


affirmative.

INCORPORATION CLAUSE The framers of the Amparo Rule never


55 Razon vs Tagitis G.R. No. 182498 (Dec. 3, intended Section 5(c) to be complete in every
2009) detail in stating the threatened or actual
violation of a victim’s rights. As in any other
Razon vs. Tagitis initiatory pleading, the pleader must of course
state the ultimate facts constituting the cause
GR No. 182498 December 3, 2009 of action, omitting the evidentiary details.

To read the Rules of Court requirement on


pleadings while addressing the unique Amparo
Facts: Tagitis, a consultant for the World Bank situation, the test in reading the petition
and the Senior Honorary Counselor for the should be to determine whether it contains the
Islamic Development Bank (IDB) Scholarship details available to the petitioner under the
Programme, was last seen in Jolo, circumstances, while presenting a cause of
Sulu. Together with Arsimin Kunnong action showing a violation of the victim’s rights
(Kunnong), an IDB scholar, Tagitis arrived in to life, liberty and security through State or
Jolo by boat in the early morning of October private party action. The petition should
31, 2007 from a seminar in Zamboanga likewise be read in its totality, rather than in
City. They immediately checked-in at ASY terms of its isolated component parts, to
Pension House. Tagitis asked Kunnong to buy determine if the required elements – namely,
him a boat ticket for his return trip the of the disappearance, the State or private
following day to Zamboanga. When Kunnong action, and the actual or threatened violations
returned from this errand, Tagitis was no of the rights to life, liberty or security – are
longer around. The receptionist related that present.
Tagitis went out to buy food at around 12:30
in the afternoon and even left his room key In the present case, the petition amply recites
with the desk. Kunnong looked for Tagitis and in its paragraphs 4 to 11 the circumstances
even sent a text message to the latter’s under which Tagitis suddenly dropped out of
Manila-based secretary who did not know of sight after engaging in normal activities, and
Tagitis’ whereabouts and activities either; she thereafter was nowhere to be found despite
advised Kunnong to simply wait. efforts to locate him. The petition alleged,
too, under its paragraph 7, in relation to
On November 4, 2007, Kunnong and paragraphs 15 and 16, that according to
Muhammad Abdulnazeir N. Matli, a UP reliable information, police operatives were
professor of Muslim studies and Tagitis’ fellow the perpetrators of the abduction. It also
student counselor at the IDB, reported Tagitis’ clearly alleged how Tagitis’ rights to life, liberty
disappearance to the Jolo Police Station. On and security were violated when he was
November 7, 2007, Kunnong executed a sworn “forcibly taken and boarded on a motor vehicle
affidavit attesting to what he knew of the by a couple of burly men believed to be police
circumstances surrounding Tagitis’ intelligence operatives,” and then taken “into
disappearance. custody by the respondents’ police intelligence
operatives since October 30, 2007, specifically
More than a month later (on December 28, by the CIDG, PNP Zamboanga City, x x x held
2007), the respondent filed a Petition for the against his will in an earnest attempt of the
Writ of Amparo (petition) with the CA through police to involve and connect [him] with
her Attorney-in-Fact, Atty. Felipe P. different terrorist groups.”
Arcilla. The petition was directed against Lt.
Gen. Alexander Yano, Commanding General,
Page 42 of 121
403 CASE DIGESTS | CONSTITUTIONAL LAW REVIEW | JUDGE ESTELA ALMA SINGCO | FIRST SEMESTER | 2020-2021

These allegations, in our view, properly As the law now stands, extra-judicial killings
pleaded ultimate facts within the pleader’s and enforced disappearances in this
knowledge about Tagitis’ disappearance, the jurisdiction are not crimes penalized
participation by agents of the State in this separately from the component criminal acts
disappearance, the failure of the State to undertaken to carry out these killings and
release Tagitis or to provide sufficient enforced disappearances and are now
information about his whereabouts, as well as penalized under the Revised Penal Code and
the actual violation of his right to special laws.
liberty. Thus, the petition cannot be faulted
for any failure in its statement of a cause of Even without the benefit of directly applicable
action. substantive laws on extra-judicial killings and
enforced disappearances, however, the
If a defect can at all be attributed to the Supreme Court is not powerless to act under
petition, this defect is its lack of supporting its own constitutional mandate to
affidavit, as required by Section 5(c) of the promulgate “rules concerning the protection
Amparo Rule. Owing to the summary nature and enforcement of constitutional rights,
of the proceedings for the writ and to facilitate pleading, practice and procedure in all courts,”
the resolution of the petition, the Amparo Rule
incorporated the requirement for supporting Lest this Court intervention be misunderstood,
affidavits, with the annotation that these can we clarify once again that we do not rule on
be used as the affiant’s direct testimony. This any issue of criminal culpability for the
requirement, however, should not be read as extrajudicial killing or enforced
an absolute one that necessarily leads to the disappearance. This is an issue that requires
dismissal of the petition if not strictly criminal action before our criminal courts
followed. Where, as in this case, the petitioner based on our existing penal laws. Our
has substantially complied with the intervention is in determining whether an
requirement by submitting a verified petition enforced disappearance has taken place and
sufficiently detailing the facts relied upon, the who is responsible or accountable for this
strict need for the sworn statement that an disappearance, and to define and impose the
affidavit represents is essentially fulfilled. We appropriate remedies to address it. The
note that the failure to attach the required burden for the public authorities to discharge
affidavits was fully cured when the respondent in these situations, under the Rule on the Writ
and her witness (Mrs. Talbin) personally of Amparo, is twofold. The first is to ensure
testified in the CA hearings held on January 7 that all efforts
and 17 and February 18, 2008 to swear to and at disclosure andinvestigation are undertaken
flesh out the allegations of the petition. Thus, under pain of indirect contempt from this Court
even on this point, the petition cannot be when governmental efforts are less than what
faulted. the individual situations
require. The second is to address the
We reject the petitioners’ argument that the disappearance, so that the life of the victim is
respondent’s petition did not comply with the preserved and his or her liberty and security
Section 5(d) requirements of the Amparo Rule restored.
(that a prior investigation must have been
made), as the petition specifies in its Article 2 of the International Convention for
paragraph 11 that Kunnong and his the Protection of All Persons from Enforced
companions immediately reported Tagitis’ Disappearance defined enforced
disappearance to the police authorities in Jolo, disappearance as follows:
Sulu as soon as they were relatively certain
that he indeed had disappeared. The police, For the purposes of this
however, gave them the “ready answer” that Convention, “enforced
Tagitis could have been abducted by the Abu disappearance” is considered
Sayyaf group or other anti-government to be the arrest, detention,
groups. The respondent also alleged in abduction or any other form
paragraphs 17 and 18 of her petition that she of deprivation of liberty by
filed a “complaint” with the PNP Police Station agents of the State or by
in Cotobato and in Jolo, but she was told of “an persons or groups of persons
intriguing tale” by the police that her husband acting with the authorization,
was having “a good time with another support or acquiescence of
woman.” The disappearance was alleged to the State, followed by a
have been reported, too, to no less than the refusal to acknowledge the
Governor of the ARMM, followed by the deprivation of liberty or by
respondent’s personal inquiries that yielded concealment of the fate or
the factual bases for her petition. whereabouts of the
disappeared person, which
Based on these considerations, we rule that place such a person outside
the respondent’s petition for the Writ the protection of the law.
of Amparo is sufficient in form and substance
and that the Court of Appeals had every reason
to proceed with its consideration of the case. The Convention is the first universal human
rights instrument to assert that there is a right
The Amparo Rule expressly provides that the not to be subject to enforced
“writ shall cover extralegal killings and disappearance and that this right is non-
enforced disappearances or threats thereof.” derogable.
We note that although the writ specifically
covers “enforced disappearances,” this To date, the Philippines has neither signed nor
concept is neither defined nor penalized in this ratified the Convention, so that the country is
jurisdiction. not yet committed to enact any law penalizing
enforced disappearance as a crime. The
absence of a specific penal law, however, is not
Page 43 of 121
403 CASE DIGESTS | CONSTITUTIONAL LAW REVIEW | JUDGE ESTELA ALMA SINGCO | FIRST SEMESTER | 2020-2021

a stumbling block for action from this Court, as International Court to compel Guatemala (D)
heretofore mentioned; underlying every to recognize him as one of its national.
enforced disappearance is a violation of the Guatemala (D) challenged the validity of
constitutional rights to life, liberty and security Nottebohm’s (P) citizenship, the right of
that the Supreme Court is mandated by the Liechtenstein (P) to bring the action and
Constitution to protect through its rule-making alleged its belief that Nottebohm (P) remained
powers. a German national.

As a matter of human right and fundamental Issue: Must nationality be disregarded by


freedom and as a policy matter made in a UN other states where it is clear that it was a mere
Declaration, the ban on enforced device since the nationality conferred on a
disappearance cannot but have its effects on party is normally the concerns of that nation?
the country, given our own adherence
to “generally accepted principles of Ruling: NO. issues relating to citizenship are
international law as part of the law of the solely the concern of the granting nation. This
land.” is the general rule. But it does not mean that
other states will automatically accept the
We characterized “generally accepted conferring state’s designation unless it has
principles of international law” as norms of acted in conformity with the general aim of
general or customary international law that are forging a genuine bond between it and its
binding on all states. We held further: national aim. In this case, there was no
[G]enerally accepted principles of international relationship between Liechtenstein (P) and
law, by virtue of the incorporation clause of the Nottebohm (P). the change of nationality was
Constitution, form part of the laws of the land merely a subterfuge mandated by the war.
even if they do not derive from treaty Under this circumstance, Guatemala (D) was
obligations. The classical formulation in not forced to recognize it. Dismissed.
international law sees those customary rules
accepted as binding result from the A state putting forth a claim must establish a
combination [of] two elements: the locus standi for that purpose. Without
established, widespread, and interruption and continuously from the time of
consistent practice on the part of States; and the injury to the making of an award been a
apsychological element known as the opinion national of the state making the claim and
juris sive necessitates (opinion as to law or must not have been a national of the state
necessity). Implicit in the latter element is against whom the claim has been filed.
a belief that the practice in question is
rendered obligatory by the existence of a rule
of law requiring it. STATE IMMUNITY
57 Department of Agriculture v. NLRC, 227
While the Philippines is not yet formally bound SCRA 693, Nov. 11, 1993 [Vitug]
by the terms of the Convention on enforced
disappearance (or by the specific terms of the Facts: The Department of Agriculture
Rome Statute) and has not formally declared (petitioner) and Sultan Security Agency
enforced disappearance as a specific crime, entered into a contract for security services to
the above recital shows that enforced be provided by the latter to the said
disappearance as a State practice has been governmental entity. Pursuant to their
repudiated by the international community, so arrangements, guards were deployed by
that the ban on it is now a generally accepted Sultan Agency in the various premises of the
principle of international law, which we should petitioner.
consider a part of the law of the land, and
which we should act upon to the extent already Subsequently, several guards of the Sultan
allowed under our laws and the international Security Agency 􏰂led a complaint for
conventions that bind us. underpayment of wages against the petitioner
and Sultan Security Agency.

Executive Labor Arbiter rendered a decision


PRINCIPLE OF EFFECTIVE NATIONALITY finding herein petitioner jointly and severally
liable with sultan Security Agency for the
56 Nottebohm Case (Liechtenstein v. payment of the money claims.
Guatemala)
The Labor Arbiter issued a writ of execution,
Nottebohm case (Liechtenstein vs commanding the City Sheriff to enforce and
Guatemala) execute the judgment against the property of
the two respondents.
Facts: Nottebohn (P), a German by birth, lived
in Guatemala (D) for 34 years, retaining his A petition for injunction was filed by petitioner
German citizenship and family and business before the NLRC alleging that the attachment
ties with it. He however applied for or seizure of its property is invalid.
Liechtenstein (P) citizenship a month after the
outbreak of World War II. Nottebohm (P) had Issue: WON the petitioner, a government
no ties with Liechtenstein but intended to entity, waived its immunity from suit
remain in Guatemala. The naturalization
application was approved by Liechtenstein and Ruling:
impliedly waived its three-year. After this
approval, Nottebohm (P) travelled to The state may not be sued without its consent.
Liechtenstein and upon his return to The rule, in any case, is not really absolute for
Guatemala (D), he was refused entry because it does not say that the state may not be sued
he was deemed to be a German citizen. His under any circumstance. the doctrine only
Liechtenstein citizenship was not honored. conveys, "the state may not be sued without
Liechtenstein (P) thereby filed a suit before the its consent;" its clear import then is that the
Page 44 of 121
403 CASE DIGESTS | CONSTITUTIONAL LAW REVIEW | JUDGE ESTELA ALMA SINGCO | FIRST SEMESTER | 2020-2021

State may at times be sued. The States'


consent may be given either expressly or This motion was granted on the ground that
impliedly. Express consent may be made the Civil Aeronautics Administration not being
through a general law or a special law. a juridical person has no capacity to sue and
be sued and for that reason it cannot come
In this jurisdiction, the general law waiving the under the jurisdiction of the court.
immunity of the state from suit is found in Act
No. 3083, where the Philippine government Issue: WON the Civil Aeronautics
"consents and submits to be sued upon any Administration waived its immunity from suit
money claim involving liability arising from
contract, express or implied, which could serve Ruling: In the case of National Airports
as a basis of civil action between private Corporation v. Teodoro, the SC held that the
parties." Implied consent, on the other hand, Civil Aeronautics Administration may be sued
is conceded when the State itself commences and that the principle of state immunity from
litigation, thus opening itself to a counterclaim suit does not apply to it.
or when it enters into a contract. In this
situation, the government is deemed to have An obligation or liability of the state created by
descended to the level of the other contracting statute is enforceable against the officer or
party and to have divested itself of its agent charged with the duty to execute the
sovereign immunity. law. If there should be anything demandable
which had been paid or delivered to or
Not all contracts entered into by the collected by officers or agents of the state
government operate as a waiver of its non- without the authority of law, the action would
suability; distinction must still be made not be against the state but against the
between one which is executed in the exercise responsible officers or agents who received
of its sovereign functions and another which is what was not due the state or made the
done in its proprietary capacity. The restrictive unauthorized collection. Punishable acts or
application of State immunity is proper only omissions committed by officers or agents of
when the proceedings arise out of commercial the state are crimes and violations of law
transactions of the foreign sovereign, its perpetrated by such officers or agents and not
commercial activities or economic affairs. by the state. The same postulate may be
Stated differently, a State may be said to have applied to torts committed by officers or
descended to the level of an individual and can agents of the state. Nevertheless, if, where
thus be deemed to have tacitly given its and when the state or its government enters
consent to be sued only when it enters into into a contract, through its officers or agents,
business contracts. in furtherance of a legitimate aim and purpose
and pursuant to constitutional legislative
In the instant case, the petitioner has not authority, whereby mutual or reciprocal
pretended to have assumed a capacity apart benefits accrue and rights and obligations arise
from its being a governmental entity when it therefrom, and if the law granting the
entered into the questioned contract; nor that authority to enter into such contract does not
it could have, in fact, performed any act provide for or name the officer against whom
proprietary in character. action may be brought in the event of a breach
thereof, the state itself may be sued even
But, be that as it may, the claims of private without its consent, because by entering into a
respondents, i.e., for underpayment of wages, contract the sovereign state has descended to
holiday pay, overtime pay and similar other the level of the citizen and its consent to be
items, arising from the Contract for Security sued is implied from the very act of entering
Services, clearly constitute money claims. Act into such contract. If the dignity of the state,
No. 3083, aforecited, gives the consent of the the sacredness of the institution, the respect
State to be "sued upon any moneyed claim for the government are to be preserved and
involving liability arising from contract, the dragging of its name in a suit to be
express or implied, . . ." Pursuant, however, to prevented, the legislative department should
Commonwealth Act ("C.A.") No. 327, as name the officer or agent against whom the
amended by Presidential Decree ("P.D.") No. action may be brought in the event of breach
1445, the money claim should 􏰂first be of the contract entered into under its name
brought to the Commission on Audit. and authority. And the omission or failure of
58 Santos v. Santos, 92 Phil. 281 the legislative department to do so is no
obstacle or impediment for an individual or
Facts: Teodora Santos and her nieces citizen, who is aggrieved by the breach of the
complain that from 1945 to 1949 Leoncio contract, to bring an action against the state
Santos collected from the Army of the USA itself for the reasons already adverted to, to
rentals for the use and occupation of a parcel wit: the descent of the sovereign state to the
of land belonging to them and Leoncio Santos level of the individual or citizen with whom it
in common by inheritance from their ancestor. entered into a contract and its consent to be
They also complain that they made a demand sued implied from the act of entering into such
upon Leoncio Santos to have the lot contract.
partitioned among them but the latter refused
to do so, he having sold the lot to the 59 Kawananakoa v. Polybank, 205 U.S. 349
Administrator of the Civil Aeronautics
Administration who is now in possession Facts: Polyblank executed a mortgate to the
thereof and that the sale of the lot made by appellee, sister Albertina Kawananokoa.
Leoncio Santos to the Administrator of the Civil Polyblank pleaded that, after execution of the
Aeronautics Administration insofar as their mortgage, a part of the mortgaged land had
shares in the lot are concerned is null and void. been conveyed to them to one Damon, and by
Damon to the Territory of Hawaii, and was now
The Administrator of the Civil Aeronautics part of a public street.
Administration moved to dismiss the case
invoking the state’s immunity from suit.
Page 45 of 121
403 CASE DIGESTS | CONSTITUTIONAL LAW REVIEW | JUDGE ESTELA ALMA SINGCO | FIRST SEMESTER | 2020-2021

The bill originally made the territory of Hawaii 61 Republic v. Domingo, 657 SCRA 621
a party, but the territory demurred and the
plaintiffs dismissed their bill as to it before the Facts: Respondent filed 􏰂filed a Complaint for
above plea was argued. Speci􏰂c Performance against the DPWH.
Domingo averred that he entered into seven
Polyblank’s plea was overruled, and after contracts with the DPWH Region III for the
answer and hearing, the decree of foreclosure lease of his construction equipment to said
was made. The decree excepted from the sale government agency. Despite repeated
the land conveyed to the territory, and demands, Domingo asserted that the DPWH
directed a judgment for the sum remaining Region III failed to pay its obligations.
due in case the proceeds of the sale were
insufficient to pay the debt. The RTC rendered a decision in favor of
respondent. Thereafter, a writ of execution
Polyblank appealed from the decree and was issued against DPWH.
contended that the owners of the equity of
redemption (Kawanakoa and the territory of The Republic of the Philippines, represented by
Hawaii) in all parts of the mortgage must be the OSG filed a petition for annulment of
joined, and that no deficiency judgment should judgment of the RTC’s decision before the RTC
be entered until all the mortgage properties and averred that under the law, the statutory
have been sold. In aid of their contention they representatives of the government for
argue that the territory of Hawaii is liable to purposes of litigation are either the Solicitor
suit like a municipal corporation, irrespective General or the Legal Service Branch of the
of the permission given by its statues, which Executive Department concerned. Since no
does not extend to this case. summons was issued to either of said
representatives, the trial court never acquired
Issue: WON the territory of Hawaii is liable for jurisdiction over the Republic. However, the CA
suit. dismissed the petition.

Ruling: No. The territory could waive its The republic filed a petition before the SC
exemption and take no objection to the arguing that the service of summons upon the
proceedings. But, in the case at bar, the DPWH Region III alone was insufficient.
territory of Hawaii did object, and the question According to the Republic, the applicable rule
raised is whether the plaintiffs were bound to of procedure in this case is Section 13, Rule 14
yield. Some doubts have been expressed as to of the Rules of Court, which mandates that
the source of the immunity of a sovereign when the defendant is the Republic of the
power from suit without its own permission, Philippines, the service of summons may be
but the answer has been public property since effected on the O􏰂SG.
before the days of Hobbes. Leviathan. A
sovereign is exempt from suit not because of Issue: WON the trial court properly acquired
any formal conception or obsolete theory, but jurisdiction over the Republic of the Philippines
on the logical and practical ground that there
can be no legal right as against the authority Ruling: Under Rule 14, Section 13 of the 1997
that makes the law on which the right Rules of Procedure, where the defendant is the
depends. Republic of the Philippines, service of
summons must be made on the Solicitor
60 Providence Washington Insurance Co. v. General.
Republic of the Philippines, 29 SCRA 598
the Complaint for Specific Performance with
Facts: Petitioner filed appealed an order of the Damages 􏰂led by Domingo specifically named
lower court dismissing its suit for the non- as defendant the DPWH Region III. the DPWH
delivery of thirty cases of steel 􏰂les, which and its regional office are merely the agents of
cargo was insured by it against loss and the former (the Republic), which is the real
damage, naming as defendants the Republic of party in interest. Thus, as mandated by
the Philippines and the Bureau of Customs as Section 13, Rule 14 of the Rules of Court, the
the operator of the arrastre service. summons in this case should have been served
on the OSG.
Issue: WON the Bureau of Customs is immune
from suit 62 Sanders v. Veridiano G.R. No. L-46930
June 10, 1988
Ruling: In Mobil Philippines Exploration, Inc.
v. Customs Arrastre Service, the SC ruled that Facts: Petitioner Sanders was, at the time the
The Bureau of Customs, acting as part of the incident in question occurred, the special
machinery of the national government in the services director of the U.S. Naval Station.
operation of the arrastre service,. pursuant to Petitioner Moreau was the commanding
express legislative mandate and as a officer. Private respondents were American
necessary incident of its prime governmental Citizens and both employed as gameroom
function, is immune from suit, there being no attendants in the special services department
statute to the contrary. of the base.

The doctrine of non-suability thus holds The private respondents were advised that
undisputed sway. Its primacy appears to be their employment had been converted from
undeniable. For a suit of this character to permanent full-time to permanent part-time.
prosper, there must be a showing of consent Their reaction was to protest this conversion
either in express terms or by implication and to institute grievance proceedings
through the use of statutory language too plain conformably to the pertinent rules and
to be misinterpreted. regulations of the U.S. Department of Defense.
The result was a recommendation from the
hearing officer who conducted the proceedings
SUITS AGAINST GOVERNMENT AGENCIES
Page 46 of 121
403 CASE DIGESTS | CONSTITUTIONAL LAW REVIEW | JUDGE ESTELA ALMA SINGCO | FIRST SEMESTER | 2020-2021

for the reinstatement of the private gain but is an agency of the Republic
respondents to permanent full-time. performing governmental functions.

In a letter addressed to petitioner Moreau, Issue: WON the Bureau of Printing may be
Sanders disagreed with the hearing o􏰂fficer's sued
report and asked for the rejection of the
abovestated recommendation. Another Ruling: The Bureau of Printing is an o􏰂ffice of
different letter coming from Moreau was sent the Government. As such instrumentality of
to the Chief of Naval Personnel explaining the the Government, it operates under the direct
change of the private respondent's supervision of the Executive Secretary,
employment status and requesting O􏰂ffice of the President, and is charged with
concurrence therewith. the execution of all printing and binding,
including work incidental to those processes. It
The private respondent 􏰂led in the CFI a has no corporate existence, and its
complaint for damages against the herein appropriations are provided for in the General
petitioners claiming that letters contained Appropriations Act. Designed to meet the
libelous imputations that had exposed them to printing needs of the Government, it is
ridicule and caused them mental anguish. primarily a service bureau and is obviously,
not engaged in business or occupation for
Issue: Whether the petitioners were acting pecuniary profit.
officially or only in their private capacities
when they did the acts for which the private The additional work it executes for private
respondents have sued them for damages. parties is merely incidental to its function, and
although such work may be deemed
Ruling: Petitioners were performing official proprietary in character, there is no showing
duties. The mere allegation that a government that the employees performing said
functionary is being sued in his personal proprietary function are separate and distinct
capacity will not automatically remove him from those employed in its general
from the protection of the law of public governmental functions.
o􏰂fficers and, if appropriate, the doctrine of
state immunity. By the same token, the mere 64 Mobil Philippines Exploration, Inc. v.
invocation of o􏰂fficial character will not suffice Customs Arrastre Service, 18SCRA 1120
to insulate him from suability and liability for
an act imputed to him as a personal tort Facts: Four cases of rotary drill parts were
committed without or in excess of his shipped from abroad consigned to petitioner.
authority. These well-settled principles are The shipment arrived at the Port of Manila and
applicable not only to the o􏰂fficers of the local was discharged to the custody of the Customs
state but also where the person sued in its Arrastre Service, the unit of the Bureau of
courts pertains to the government of a foreign Customs then handling arrastre operations
state, as in the present case. therein. The Customs Arrastre Service later
delivered to the broker of the consignee three
The acts for which the petitioners are being cases only of the shipment.
called to account were performed by them in
the discharge of their official duties. Sanders, Petitioner filed a suit against the Customs
as director of the special services department Arrastre Service and the Bureau of Customs to
of NAVSTA, undoubtedly had supervision over recover the value of the undelivered case.
its personnel, including the private
respondents, and had a hand in their The defendants 􏰂led a motion to dismiss the
employment, work assignments, discipline, complaint on the ground that not being
dismissal and other related matters. It is not persons under the law, defendants cannot be
disputed that the letter he had written was in sued.They also contended that not all
fact a reply to a request from his superior, the government entities are immune from suit;
other petitioner, for more information that defendant Bureau of Customs as operator
regarding the case of the private respondents. of the arrastre service at the Port of Manila, is
As for Moreau, he was the immediate superior discharging proprietary functions and as such
of Sanders and directly answerable to Naval can be sued by private individuals.
Personnel matters involving the special
services department of the base. Issue: WON the Customs Arrastre Service and
Bureau of Customs are suable
63 Bureau of Printing v. Bureau of Printing
Employees Association, 1 SCRA 340 Ruling: NO. The fact that a non-corporate
government entity performs a function
Facts: Respondents filed a complaint against proprietary in nature does not necessarily
petitioners Bureau of Printing, Salvador and result in its being suable. If said non-
Ledesma. The complaint alleged that Salvador governmental function is undertaken as an
and Ledesma have been engaging in unfair incident to its governmental function, there is
labor practice by interfering with, or coercing no waiver thereby of the sovereign immunity
the employees of the Bureau of Printing, from suit extended to such government entity.
particularly the members of the complaining
association, in the exercise of their right to The Bureau of Customs, is part of the
self-organization and discriminating in regard Department of Finance, with no personality of
to hire and tenure of their employment in order its own apart from that of the national
to discourage them from pursuing their union government. Its primary function is
activities. governmental, that of assessing and collecting
lawful revenues from imported articles and all
Petitioners answered that the Bureau of other tariff and customs duties, fees, charges,
Printing has no juridical personality to sue and 􏰂fines and penalties.
be sued. That said Bureau of Printing is not an
industrial concern engaged for the purpose of
Page 47 of 121
403 CASE DIGESTS | CONSTITUTIONAL LAW REVIEW | JUDGE ESTELA ALMA SINGCO | FIRST SEMESTER | 2020-2021

Although said arrastre function may be


deemed proprietary, it is a necessary incident
of the primary and governmental function of
the Bureau of Customs, so that engaging in the 66 Air Transport Office v. Ramos, 664 SCRA
same does not necessarily render said Bureau 36
liable to suit. For otherwise, it could not
perform its governmental function without
necessarily exposing itself to suit. Sovereign Ramos discovered that a portion of his land
immunity, granted as to the end, should not was being used as part of the runway of Air
be denied as to the necessary means to that Transport Office. The parties agreed to sell the
end. affected portion. ATO failed to pay so Ramos
filed a collection case.
65 National Airports Corporation v. Teodoro,
91 Phil. 203 ATO moved to dismiss arguing that it cannot
be sued without the State’s consent; and the
sale was entered into in the performance of its
Corporation Law was made applicable to governmental functions.
National Airports Corporation. PAL paid landing
and parking fees to NAC for the use of one of Ruling:
its airports. The fees were rentals due to
Capitol Subdiv, the owner of the land used by
In Teodoro, the Court declared as proprietary
NAC as airports. NAC was abolished and
in nature the management and maintenance
replaced by Civil Aeronautics Administration.
of airport operations, among others. ATO as an
Capitol Subdiv was not paid so it sued PAL
agency of the Government was not performing
who, in turn, filed a third party complaint
a purely governmental or sovereign function,
against CAA. PAL argued that it paid NAC on
but was instead involved in the management
the belief that it was the lessee of the lands
and maintenance of the Loakan Airport, an
and that its successor, CAA, should be the one
activity that was not the exclusive prerogative
to pay Capitol Subdiv.
of the State in its sovereign capacity. Hence,
the ATO had no claim to the State’s immunity
Solgen moved to dismiss on the ground of non- from suit.
suability
Suits against State agencies with relation to
Ruling: matters in which they have assumed to act in
private or non-governmental capacity, and
CAA has the general power to execute various suits against certain corporations
contracts of any kind, to purchase property, created by the state for public purposes, but
and to grant concession rights, and to charge to engage in matters partaking more of the
landing fees, royalties on sales to aircraft of nature of ordinary business rather than
aviation gasoline, accessories and supplies, functions of a governmental or political
and rentals for the use of any property under character, are not regarded as suits against
its management. the state. The latter is true, although the state
The power to sue and be sued is implied from may own stock or property of such a
the power to transact private business. And if corporation for by engaging in business
it has the power to sue and be sued on its operations through a corporation, the state
behalf, the CAA with greater reason should divests itself so far of its sovereign character,
have the power to prosecute and defend suits and by implication consents to suits against
for and against the NAC, having acquired all the corporation
the properties, funds and choses in action and
assumed all the liabilities of the latter. 67 PNB v. CIR, 81 SCRA 314

Not all government entities, whether corporate A notice of garnishment was served on PNB.
or non-corporate, are immune to suits. What was sought to be garnished was the
Immunity from suits is determined by the money of the People's Homesite and Housing
character of the objects for which the entity Corporation deposited at petitioner's branch in
was organized. "Suits against state agencies Quezon City, to satisfy a decision of
with relation to matters in which they have respondent Court which had become final and
assumed to act in a private or non- executory.
governmental capacity, and various suits
against certain corporations created by the
PNB moved to quash on the ground that the
state for public purposes, but to engage in
funds subject of the garnishment "may be
matters partaking more of the nature of
public in character.”
ordinary business rather than functions of a
governmental or political character, are not
regarded as suits against the state. Is PNB’s allegation that the funds “may be
public in character” enough to prevent
The CAA comes under the category of a private garnishment?
entity. Although not a body corporate it was
created, like the NAC, not to maintain a Ruling:
necessary function of government, but to run
what is essentially a business, even if revenues
There was not even a categorical assertion to
be not its prime objective but rather the
that effect. It is only the possibility of its being
promotion of travel and the convenience of the
"public in character." The premise that the
travelling public. It is engaged in an enterprise
funds could be spoken of as public in character
which, far from being the exclusive prerogative
may be accepted in the sense that the People's
of state, may, more than the construction of
Homesite and Housing Corporation was a
public roads, be undertaken by private
government-owned entity but it does not
concerns.

Page 48 of 121
403 CASE DIGESTS | CONSTITUTIONAL LAW REVIEW | JUDGE ESTELA ALMA SINGCO | FIRST SEMESTER | 2020-2021

follow though that they were exempt from


garnishment. Ruling:

In Natl Shipyard vs CIR, a case with similar SSS' charter specifically provides that it can
facts, it was held that: “The allegation to the sue and be sued in Court. These words "sue
effect that the funds of the NASSCO are public and be sued" embrace all civil process incident
funds of the government, and that, as such, to a legal action. So that, even assuming that
the same may not be garnished, attached or the SSS, as it claims, enjoys immunity from
levied upon, is untenable for, as a government suit as an entity performing governmental
owned and controlled corporation. the functions, by virtue of the explicit provision of
NASSCO has a personality of its own, distinct the aforecited enabling law, the Government
and separate from that of the Government.” must be deemed to have waived immunity in
respect of the SSS, although it does not
It is well settled that when the government thereby concede its liability.
enters into commercial business, it abandons
its sovereign capacity and is to be treated like In so far as it is argued that to hold the SSS
any other corporation. By engaging in a liable for damages would be to deplete the
particular business thru the instrumentality of benefit funds available for its covered
a corporation, the government divests itself members, suffice it to say, that expenditures
pro hac vice of its sovereign character, so as of the System are not confined to the payment
to render the corporation subject to the rules of social security benefits.
of law governing private corporations.
In this case however, SSS should not be liable
for damages because it did not act maliciously
68 Rayo v.CFI Bulacan, 110 SCRA 460 in foreclosing the mortgage.

During a typhoon, NPC, through Chavez, The SSS was of the belief that it was acting in
simultaneously opened all three floodgates of the legitimate exercise of its right under the
the Angat Dam which resulted in the mortgage contract in the face of irregular
inundation of several towns, damage to payments made by private respondents, and
property, and deaths. placed reliance on the automatic acceleration
clause in the contract.
Complaints for damage were filed. NPC
answered that it was performing a purely SSS is only liable for nominal damages and
governmental function in the operation of atty’s fees as there was clear negligence on the
Angat Dam, hence, it can’t be sued without the part of SSS when they mistook the loan
express consent of the State. account of Socorro J. Cruz for that of private
respondent Socorro C. Cruz. Its attention was
Ruling: called to the error, but it adamantly refused to
acknowledge its mistake.
NPC can be sued because its charter says so.

It is not necessary to write an extended 70 Municipality of San Fernando, La Union v.


dissertation on whether or not the NPC Firme, 195 SCRA 692
performs a governmental function with respect
to the management and operation of the Angat A dump truck owned by the Municipality of San
Dam. Fernando got into a 3-way vehicular accident
with a truck and a jeepney resulting in
It is sufficient to say that the government has injuries/deaths of some passengers. A
organized a private corporation, put money in complaint for damages was filed against the
it and has allowed it to sue and be sued in any driver and owner of the jeepney. Jeepney then
court under its charter. As a government filed a third party complaint against the
owned and controlled corporation, it has a Municipality of SF and the driver of the dump
personality of its own, distinct and separate truck.
from that of the Government. Moreover, the
charter provision that the NPC can "sue and be Is the Municipality liable for the tort caused
sued in any court" is without qualification on by its employee?
the cause of action and accordingly it can
include a tort claim such as the one instituted Ruling:
by the petitioners.
Suability depends on the consent of the state
69 SSS v. Court Appeals, 120 SCRA 707 to be sued, liability on the applicable law and
the established facts. The circumstance that a
The Cruz spouses sued SSS for damages and state is suable does not necessarily mean that
attorney’s fees alleging, among other things, it is liable; on the other hand, it can never be
that SSS foreclosed the mortgage on their land held liable if it does not first consent to be
despite the fact that they had fully and sued. Liability is not conceded by the mere fact
religiously paid their monthly amortizations that the state has allowed itself to be sued.
and had not defaulted in any payment. When the state does waive its sovereign
immunity, it is only giving the plaintiff the
Issues: chance to prove, if it can, that the defendant
is liable.
1. Is SSS immune from suit? No. Its
Charter says it can be sued. Municipal corporations are suable because
2. May SSS be liable for torts? Yes, but their charters grant them the competence to
not in this case. No malice sue and be sued. Nevertheless, they are
generally not liable for torts committed by
Page 49 of 121
403 CASE DIGESTS | CONSTITUTIONAL LAW REVIEW | JUDGE ESTELA ALMA SINGCO | FIRST SEMESTER | 2020-2021

them in the discharge of governmental employment had been converted from


functions and can be held answerable only if it permanent full-time to permanent part-time.
can be shown that they were acting in a They protested this conversion and instituted
proprietary capacity. In permitting such grievance proceedings conformably to the
entities to be sued, the State merely gives the pertinent rules and regulations of the U.S. The
claimant the right to show that the defendant hearing officer recommended reinstatement of
was not acting in its governmental capacity their full-time status plus backwages but
when the injury was committed or that the petitioners, who were the director and
case comes under the exceptions recognized commanding officer of the Naval base,
by law. Failing this, the claimant cannot rejected the recommendation.
recover.
A letter purportedly corning from petitioner
The municipality cannot be held liable for Moreau as the commanding general was sent
the torts committed by its regular employee, to the Chief of Naval Personnel explaining the
who was then engaged in the discharge of change of the respondent's employment
governmental functions.Hence, the death status.
of the passenger imposed on the municipality
no duty to pay monetary compensation. Respondents claimed that the letters
Shauf v. CA contained libelous imputations and filed a
complaint for damages against Petitioners.
They made it clear the petitioners were being
Petitioner Shauf a Filipino by origin and sued in their private capacity. Nevertheless,
married to an American who is a member of petitioners moved to dismiss on the ground of
the USAF applied for the vacant position of state immunity.
Guidance Counselor at Clark Air Base, for
which she is eminently qualified. Ruling:

She was not hired despite her credentials. If the enforcement of the decision rendered
against the public officer will require an
Petitioner Shauf filed a complaint for damages affirmative act from the state, then it is a suit
against respondents Don Detwiler and against the sate.
Anthony Persi for the alleged discriminatory
acts in maliciously denying her application for Petitioners were performing their official
the position because of her nationality and sex. duties. As public officers, they cannot be sued
without the consent of the US who is ultimately
Respondents moved to dismiss on the ground liable.
that as officers of the United States Armed
Forces performing official functions in The acts for which the petitioners are being
accordance with the powers vested in them called to account were performed by them in
under the Philippine-American Military Bases the discharge of their official duties. Sanders,
Agreement, they are immune from suit. as director of the special services department,
had supervision over its personnel, including
Ruling: the respondents, and had a hand in their
employment, work assignments, discipline,
Shauf was refused appointment as Guidance dismissal and other related matters. It is not
Counselor by the respondents on account of disputed that the letter he had written was in
her sex, color and origin. fact a reply to a request from his superior, the
other petitioner, for more information
The doctrine of immunity from suit will not regarding the case of the respondents.
apply where the public official is being sued in
his private and personal capacity as an The letter dealt with the financial and
ordinary citizen. The cloak of protection budgetary problems of the department and
afforded the officers and agents of the contained recommendations for their solution,
government is removed the moment they are including the re-designation of the
sued in their individual capacity. This situation respondents There was nothing personal or
usually arises where the public official acts private about it. Given the official character of
without authority or in excess of the powers the above-described letters, the petitioners
vested in him. It is a well-settled principle of were, legally speaking, being sued as officers
law that a public official may be liable in his of the United States government. As they have
personal private capacity for whatever damage acted on behalf of that government, and within
he may have caused by his act done with the scope of their authority, it is that
malice and in bad faith, or beyond the scope of government, and not the petitioners
his authority or jurisdiction. personally, that is responsible for their acts.

The rationale for this ruling is that the doctrine 73 Wylie v. Rarang, 209 SCRA 357
of state immunity cannot be used as an
instrument for perpetrating an injustice. M. H. Wylie, in his capacity as assistant
administrative officer of the U.S. Naval Station
SUITS AGAINST PUBLIC OFFICERS supervised the publication of the POD was
published daily by the US Naval Base station.
71 Shauf v. Court of Appeals, 191 SCRA 713 One of the regular features of the POD was the
72 Sanders v. Veridiano G.R. No. L-46930 "action line inquiry."
June 10, 1988
One article mentions a certain "Auring", herein
respondent, as a disgrace to her division and
Respondents are American citizens employed to the Office of the Provost Marshal." The same
as gameroom attendants in the special article explicitly implies that Auring was
services department of the NAVSTA. Their
Page 50 of 121
403 CASE DIGESTS | CONSTITUTIONAL LAW REVIEW | JUDGE ESTELA ALMA SINGCO | FIRST SEMESTER | 2020-2021

consuming and appropriating for herself The doctrine of state immunity from suit
confiscated items like cigarettes and applies to complaints filed against public
foodstuffs. As a result of this article, Aurora officials for acts done in the performance of
was investigated by her supervisor. Before the their duties. The rule is that the suit must be
article came out, respondent had been the regarded as one against the state where
recipient of commendations by her superiors satisfaction of the judgment against the public
for honesty in the performance of her duties. official concerned will require the state itself to
perform a positive act, such as appropriation
MH Wylie wrote a letter of apology to Aurora of the amount necessary to pay the damages
for the ‘inadvertent’ publication. awarded to the plaintiff. The rule does not
apply where the public official is charged in his
Aurora filed a complaint for damages. official capacity for acts that are unlawful and
Petitioner moved to dismiss: 1) they acted in injurious to the rights of others. Public officials
the performance of their official functions as are not exempt, in their personal capacity,
officers of the US Navy and are immune from from liability arising from acts committed in
suit; 2) the US Naval Base is an bad faith.
instrumentality of the US govt and cannot be
sued without its consent We find, however, no evidence of such abuse
of authority on record. As earlier stated, Rizal
Park is beyond the commerce of man and,
Ruling:
thus, could not be the subject of a lease
contract. Admittedly, there was no written
Indeed, the imputation of theft in the article is contract. That private respondents were
a defamation against the character and allowed to occupy office and kiosk spaces in
reputation of Aurora. the park was only a matter of accommodation
by the previous administrator. This being so,
Petitioners are liable in their personal also admittedly, petitioner may validly
capacities. The libelous publication was ultra discontinue the accommodation extended to
vires; hence, beyond the scope of state private respondents, who may be ejected from
immunity. the park when necessary. Private respondents
cannot and does not claim a vested right to
The petitioners, however, were negligent continue to occupy Rizal Park.
because under their direction they issued the
publication without deleting the name 75 Chavez v. Sandiganbayan, 193 SCRA 282
"Auring." Such act or omission is ultra vires [1991]
and cannot be part of official duty. It was
a tortious act which ridiculed the private
respondent. As a result of the petitioners' act, FACTS:
Aurora according to the record, suffered
besmirched reputation, serious anxiety,
The petitioner challenges the resolutions dated
wounded feelings and social humiliation,
June 8, 1989 and November 2, 1989 of the
specially so, since the article was baseless and
Sandiganbayan issued in Civil Case No. 0033
false. The petitioners, alone, in their personal
which granted the motion of private
capacities are liable for the damages they
respondent Juan Ponce Enrile, one of the
caused her.
defendants in the civil case, to implead the
petitioner as additional party defendant in
74 Lansang v. Court of Appeals, G.R. 102667,
Enrile's counterclaim in the same civil case and
February 23, 2000
denied the petitioner's motion for
reconsideration.
FACTS: Private respondents were allegedly
awarded a “verbal contract of lease” in 1970
by the National Parks Development Committee On July 31, 1987, the Republic of the
(NPDC), a government initiated civic body Philippines, through the Presidential
engaged in the development of national parks, Commission on Good Government (PCGG)
including Rizal Park, which are outside the with the assistance of Solicitor General
commerce of man.With the change of Francisco Chavez filed with the respondent
government after the EDSA Revolution, the Sandiganbayan a complaint docketed as Civil
new Chairman of the NPDC, herein petitioner, Case No. 0033 against Eduardo Cojuangco, Jr.
sought to clean up Rizal Park. In a written and Juan Ponce Enrile, among others, for
notice petitioner terminated the so-called reconveyance, reversion and accounting,
verbal agreement with the General Assembly restitution and damages.
of the Blind, Inc. (GABI) and demanded that
the latter vacate the premises and the kiosks After the denial of his motion to dismiss,
it ran privately within the public park. respondent Enrile filed his answer with
compulsory counterclaim and cross-claim with
GABI filed an action for damages and damages.
injunction in the Regional Trial Court (RTC)
against petitioner, Villanueva, and “all persons The Republic filed its reply to the answer and
acting on their behalf”. The RTC later motion to dismiss the counterclaim. The
dismissed the case because it is actually one motion was opposed by respondent Enrile.
directed against the state which cannot be
sued without its consent. The Court of Appeals On January 30, 1989, respondent
reversed the decision of the RTC. Sandiganbayan issued a resolution, to wit:

ISSUE: Can there be a valid cause of action "The resolution of the Motion to Dismiss the
against the petitioner for his order to terminate Counterclaim against the Plaintiff government
the accommodation extended to GABI?

HELD: NO.
Page 51 of 121
403 CASE DIGESTS | CONSTITUTIONAL LAW REVIEW | JUDGE ESTELA ALMA SINGCO | FIRST SEMESTER | 2020-2021

is deferred until after trial, the grounds relied official duties where they have acted ultra
upon not appearing to be indubitable. vires or where there is a showing of bad faith.

On The matter of the additional parties Moreover, the petitioner's argument that the
(Solicitor General Chavez, Ex-PCGG Chairman immunity proviso under Section 4(a) of
Diaz, former Commissioners Doromal, Executive Order No. 1 also extends to him is
Rodrigo, Romero and Bautista), the propriety not well-taken. A mere invocation of the
of impleading them either under Sec. 14, Rule immunity clause does not ipso facto result in
6 or even under Sec. 12 as third-party the charges being automatically dropped.
defendant requires leave of Court to determine
the propriety thereof. No such leave has been Immunity from suit cannot institutionalize
sought. Consideration thereof cannot be irresponsibility and non-accountability nor
entertained at this time nor may therefore, the grant a privileged status not claimed by any
Motion to Dismiss the same be considered." other official of the Republic.
(Rollo, p. 329; Annex "H", Petition)
Where the petitioner exceeds his authority as
Respondent Enrile then requested leave from Solicitor General, acts in bad faith, or, as
the Sandiganbayan to implead the petitioner contended by the private respondent,
and the PCGG officials as party defendants for "maliciously conspir(es) with the PCGG
lodging this alleged "harassment suit" against commissioners in persecuting respondent
him. Enrile by filing against him an evidently
baseless suit in derogation of the latter's
The motion was granted in a resolution dated constitutional rights and liberties" (Rollo, p.
June 8, 1989, to wit: 417), there can be no question that a
complaint for damages may be filed against
"In respect to defendant Juan Ponce Enrile's him. High position in government does not
Manifestation and Motion dated February 23, confer a license to persecute or recklessly
1989, praying for leave to implead additional injure another. The actions governed by
parties to his counterclaim, the Court, finding Articles 19, 20, 21, and 32 of the Civil Code on
reason in the aforesaid Manifestation and Human Relations may be taken against public
Motion, grants leave to implead the officers or private citizens alike. The issue is
defendants named in the counterclaim and not the right of respondent Enrile to file an
admits defendant Juan Ponce Enrile's answer action for damages. He has the right. The
with counterclaim. issue is whether or not that action must be
filed as a compulsory counterclaim in the case
This is without prejudice to the defenses which filed against him.
said defendants may put forth individually or
in common, in their personal capacities or Under the circumstances of this case, we rule
otherwise." (Rollo, p. 27) that the charges pressed by respondent Enrile
for damages under Article 32 of the Civil Code
In a later resolution dated November 2, 1989, arising from the filing of an alleged harassment
respondent Sandiganbayan denied a motion to suit with malice and evident bad faith do not
reconsider the June 8, 1989 resolution. The constitute a compulsory counterclaim. To
dispositive portion of the resolution states: vindicate his rights, Senator Enrile has to file a
separate and distinct civil action for damages
against the Solicitor General.
"WHEREFORE, the Motions for Reconsideration
of the Solicitor General and former PCGG
officials Ramon Diaz, Quintin Doromal,
Orlando Romero, Ramon Rodrigo and Mary 76 Republic v. Sandoval, 220 SCRA 124,
Concepcion Bautista are denied, but, March 19, 1993, En Banc [Campos, Jr.]
considering these motions as in the nature of
motions to dismiss counterclaim/answers,
resolution of these motions is held in abeyance FACTS: Farmer-rallyists(KMP), led by its
pending trial on the merits." (Rollo, p. 31) national president, Jaime Tadeo, presented
their problems and demands, among which
Thereafter, all the PCGG officials filed their were: (a) giving lands for free to farmers; (b)
answer to the counterclaims invoking their zero retention of lands by landlords; and (c)
immunity from suits as provided in Section 4 stop amortizations of land payments; There
of Executive Order No. 1. Instead of filing an was a marchers-police confrontation which
answer, the petitioner comes to this Court resulted in the death of 12 rallyists and scores
assailing the resolutions as rendered with were wounded. As a result, then Pres. Aquino
grave abuse discretion amounting to lack of issued AO 11 creating the Citizens Mendiola
jurisdiction. Commission for the purpose of conducting an
investigation. The most significant
recommendation of the Commission was for
ISSUE: WON impleading the petitioner as
the heirs of the deceased and wounded victims
additional party defendant in the counterclaim
to be compensated by the government. Based
filed by respondent is tenable?
on such recommendation, the victims of
Mendiola massacre filed an action for damages
HELD: Presiding Justice Francis Garchitorena against the Republic and the military/police
correctly observed that there is no general officers involved in the incident.
immunity arising solely from occupying a
public office.
ISSUES: (1) Whether or not there is a valid
waiver of immunity(2) Whether or not the
The general rule is that public officials can be State is liable for damages
held personally accountable for acts claimed to
have been performed in connection with

Page 52 of 121
403 CASE DIGESTS | CONSTITUTIONAL LAW REVIEW | JUDGE ESTELA ALMA SINGCO | FIRST SEMESTER | 2020-2021

HELD: The Court held that there was no valid duties of his office if he is a special official. This
waiver of immunity as claimed by the concept does not apply to any executive agent
petitioners. The recommendation made by the who is an employee of the acting
Commission to indemnify the heirs of the administration and who on his own
deceased and the victims does not in any way responsibility performs the functions which are
mean that liability attaches to the State. AO 11 inherent in and naturally pertain to his office
merely states the purpose of the creation of and which are regulated by law and the
the Commission and, therefore, whatever is regulations. The driver of the ambulance of the
the finding of the Commission only serves as General Hospital was not a special agent; thus
the basis for a cause of action in the event any the Government is not liable
party decides to litigate the same. Thus, the
recommendation of the Commission does not
in any way bind the State. The State cannot be 78 Froilan v. Pan Oriental Shipping, G.R. No.
made liable because the military/police officers L‑6060, September 30, 1950
who allegedly were responsible for the death
and injuries suffered by the marchers acted
beyond the scope of their authority. It is a FACTS: For various reasons, among them the
settled rule that the State as a person can non-payment of installments the Shipping
commit no wrong. The military and police Commission cancelled its contract of sale with
officers who were responsible for the atrocities Fernando Froilan, took possession of the
can be held personally liable for damages as vessel FS-197 then chartered and delivered
they exceeded their authority, hence, the acts the vessel in question to defendant-appellant
cannot be considered official. Pan Oriental Shipping Company.

ISSUE: WON the lowe court erred in dismission


the counterclaim on the ground of alleged lack
IS THE RULE THAT THE STATE MAY NOT of jurisdiction over the intervenor Republic of
BE SUED AT ALL, ABSOLUTE? the Philippines

77 Merit v. Gov’t. of P.I., 34 Phil. 311


HELD: Yes. By filing its complaint in
intervention, the Government in effect waived
its right to non-suability.
FACTS: Merrit was riding a motorcycle along
Padre Faura Street when he was bumped by
the ambulance of the General Hospital. Merrit
79 Department of Agriculture v. NLRC, 227
sustained severe injuries rendering him unable
SCRA 693, Nov. 11, 1993 [Vitug]
to return to work. The legislature later enacted
Act 2457 authorizing Merritt to file a suit
against the Government in order to fix the
FACTS:
responsibility for the collision between his
motorcycle and the ambulance of the General
Hospital, and to determine the amount of the On April 1, 1989, the Department of
damages, if any, to which he is entitled. After Agriculture (DoA) office in Cagayan de Oro and
trial, the lower court held that the collision was Sultan Security Agency (SSA) entered into a
due to the negligence of the driver of the contract where the latter was to provide
ambulance. It then determined the amount of security services to the former. On September
damages and ordered the government to pay 13, 1990, several guards from SSA filed a
the same. complaint for underpayment of wages, non-
payment of 13th month pay, uniform
ISSUES: allowances, night shift differential pay, holiday
pay, as well as for damages, against DoA and
SSA. Both the DoA and SSA were subsequently
1. Did the Government, in enacting the Act
found guilty by the Executive Labor Arbiter,
2457, simply waive its immunity from suit or
which also held both of them liable for the
did it also concede its liability to the plaintiff?
payment of money claims amounting to
P266,483.91.
2. Is the Government liable for the negligent
act of the driver of the ambulance?
On July 18, 1991, the Labor Arbiter issued a
writ of execution. As a response, the DoA filed
HELD: a petition for injunction, prohibition, and
mandamus, with prayer for preliminary writ of
1. By consenting to be sued a state simply injunction, before the NLRC. The DoA's petition
waives its immunity from suit. It does not was dismissed.
thereby concede its liability to plaintiff, or
create any cause of action in his favor, or Following the dismissal of its petition before
extend its liability to any cause not previously the NLRC, DoA filed a petition before the SC
recognized. It merely gives a remedy to arguing that: (a) it was COA, not NLRC, that
enforce a preexisting liability and submits itself was supposed to have jurisdiction over money
to the jurisdiction of the court, subject to its claims against the Government pursuant to
right to interpose any lawful defense. Commonwealth Act No. 327 as amended by PD
No. 1445; and (b) that NLRC had disregarded
2. Under the Civil Code, the state is liable when the cardinal rule on the non-suability of the
it acts through a special agent, but not when State.
the damage should have been caused by the
official to whom properly it pertained to do the ISSUES:
act performed. A special agent is one who
receives a definite and fixed order or
commission, foreign to the exercise of the

Page 53 of 121
403 CASE DIGESTS | CONSTITUTIONAL LAW REVIEW | JUDGE ESTELA ALMA SINGCO | FIRST SEMESTER | 2020-2021

1. Whether or not it was COA that has


exclusive jurisdiction over money claims
80 EPG Construction Co. vs. Vigilar, 354
against the Government.
SCRA 566, March 16, 2001
2. Whether or not DoA, as an agency of the
State, is covered by the principle of the non-
FACTS:
suability of the State.

In 1983, the Ministry of Human Settlement


HELD:
(MHS), through the BLISS Development
Corporation, intiated a housing project on a
1. Yes, the Court ruled that money claims government property along the east bank of
against the Government should be filed before Manggahan Floodway in Pasig.
the Commission on Audit pursuant to CA Act
No. 327 as amended by PD No. 1445. In the
The MHS entered into a Memorandum of
instant case, underpayment of wages, holiday
Agreement (MOA) with Ministry of Public
pay, overtime pay, and other similar items
Works and Highways (MPWH) where the latter
arising from the Contract for Service clearly
undertook to develop the housing site and
constitute money claims. As such, the writ of
construct thereon 145 housing units.
execution issued by the Labor Arbiter and the
resolution issued by NLRC were reversed by
the Court in favor of DoA. By virtue of the MOA, MPWH forged individual
contracts with petitioners EPG, Ciper, Septa,
Phil. Plumbing, Home Construction, World
2. No, DoA cannot use the principle of non-
Builders, Glass World, Performance Builders,
suability of the State as an excuse not to be
and De Leon Araneta Construction for the
sued.
construction of the housing units. Under the
contracts, the scope of construction and
Section 3, Art. XVI of the 1987 Constitution funding covered only around "2/3 of each
states that "the State may not be sued without housing unit". Petitioners agreed to undertake
its consent." This principle reflects a and perform "additional constructions" for the
recognition of the sovereign character of the completion of the housing units despite the
State and an express affirmation of the fact that there was only a verbal promise, and
unwritten rule effectively insulating it from the not a written contract, by the MPWH
jurisdiction of the courts. As per Justice Undersecretary Aber Canlas that additional
Holmes, a sovereign State is exempt from funds will be available and forthcoming.
suits "not because of any formal conception or Unpaid balance for the additional constructions
obsolete theory, but on the logical and amounted to P5,918,315.63. Upon a demand
practical ground that there can be no legal letter from the petitioners, on November 14,
right as against the authority that makes the 1988, DPWH Asst. Secretary Madamba opined
law on which the right depends." that payment of petitioners' money claims
should be based on quantum meruit (what one
However, this privilege is not absolute; with its has earned) and should be forwarded to the
consent, the State can be sued. Commission on Audit (COA).

The Court clarifies that there are two kinds of In a Letter of the Undersecretary of Budget
consent: (1) express consent, which may be and Management dated December 20, 1994,
made either through a general law or a special the amount of P5,819,316.00 was then
law; and (2) implied consent, which is released for the payment of the petitioners'
conceded when the State either commences money claims under Advise of Allotment No.
litigation or enters into a contract. A4-1303-04-41-303. In an indorsement dated
December 27, 1995, the COA referred anew
But entering into a contract does not the money claims to the DPWH. In a letter
automatically mean the State can be sued. dated August 26, 1996, respondent Secretary
Once again, the Court clarifies that contracts Gregorio Vigilar denied the subject money
or agreements that constitute sovereign and claims.
governmental acts (jure imperii) cannot be the
subject of any lawsuit, while private, Petitioners filed before the RTC of QC, Branch
commercial, and proprietary acts (jure 226 a Petition for Mandamus to order the
gestionisis) can be made the subject of respondent to pay petitioners their money
litigation. claims plus damages and attorney's fees. In
addition the lower court denied the petition on
In the instant case, express consent was February 18, 1997.
provided for by Act No. 3083, which states that
"the Philippine government consents and ISSUE:
submits to be sued upon any money claims
involving liability arising from contract, Whether or not the implied, verbal contracts
express or implied, which could serve as a between the petitioners and then
basis of civil action between private parties." Undersecretary Canlas should be upheld?
And as cited earlier, money claims against the
Government should be filed before the
Whether or not the State is immune from suit?
Commission on Audit.

HOLDING:
At the same time, it is inarguable that DoA's
contract with SSA was clearly a proprietary
act, which essentially means that DoA cannot While the court agrees with the respondent
invoke the principle of the non-suability of the that the implied contracts are void, in view of
State. violation of applicable laws, auditing rules, and
lack of legal requirements, it still finds merit in

Page 54 of 121
403 CASE DIGESTS | CONSTITUTIONAL LAW REVIEW | JUDGE ESTELA ALMA SINGCO | FIRST SEMESTER | 2020-2021

the instant petition. The illegality of the that the National Government has not as yet
implied contracts proceeds from an express paid the value of the land which is being
declaration or prohibition by law, not from any utilized for public use."
intrinsic illegality. “Iin the interest of
substantial justice," petitioners-contractors' The lower court dismissed the complaint on
right to be compensated is upheld, applying January 30, 1969 stating that the case is
the principle of quantum meruit. Even the undoubtedly against the National Government
DPWH Asst. Sec. for Legal Affairs recommends and there is now showing that the Government
their compensation; even the DPWH Auditor has not consented to be sued in this case. The
did not object to the payment of the money petitioners appealed by certiorari to review the
claims decision and contended that they are entitled
for just compensation under the Art III, Sec. 1
The respondent may not conveniently hide (2) of the Constitution.
under the State's cloak of invincibility against
suit, considering that this principle yields to ISSUE:
certain settled exceptions. The State's
immunity cannot serve as an instrument Whether or not, the decision of the CFI of Cebu
perpetrating injustice. to dismiss the complaint by reason
Government immunity from suit correct?

81 Ministerio v. CFI, G.R. No. L-31635 August HELD:


31, 1971
NO. The doctrine of governmental immunity
from suit cannot be an instrument for
FACTS: perpetrating an injustice on a citizen. If there
were an observance of procedural regularity,
Petitioners as plaintiffs in a complaint filed with petitioners would not be in sad plaint they are
the Court of First Instance of Cebu, dated April now. It is unthinkable then that precisely there
13, 1966, sought the payment of just was a failure on what the law requires and the
compensation for a registered lot, containing petitioners has the right to demand from the
an area of 1045 square meters, alleging that Government what is due to them. The
in 1927 the National Government through its Supreme Court decided that the lower court’s
authorized representatives took physical and decision of dismissing the complaint is
material possession of it and used it for the reversed and the case remanded to the lower
widening of the Gorordo Avenue, a national court for proceedings in accordance with law.
road, Cebu City, without paying just
compensation and without any agreement,
either written or verbal. There was an
allegation of repeated demands for the
payment of its price or return of its possession, RESTRICTIVE DOCTRINE OF STATE
but defendants Public Highway Commissioner IMMUNITY FROM SUIT
and the Auditor General refused to restore its 82 Department of Agriculture v. NLRC, 227
possession. It was further alleged that on SCRA 693, Nov. 11, 1993 [Vitug]
August 25, 1965, the appraisal committee of
the City of Cebu approved Resolution No. 90,
appraising the reasonable and just price of Lot "The traditional rule of immunity exempts a
No. 647-B at P50.00 per square meter or a State from being sued in the courts of another
total price of P52,250.00. Thereafter, the State without its consent or waiver. This rule
complaint was amended on June 30, 1966 in is a necessary consequence of the principles of
the sense that the remedy prayed for was in independence and equality of States.
the alternative, either the restoration of However, the rules of International Law are
possession or the payment of the just not petrified; they are constantly developing
compensation. and evolving. And because the activities of
states havemultiplied, it has been necessary to
In the answer filed by defendants, now distinguish them - between sovereign and
respondents, through the then Solicitor governmental acts (jure imperii) and private,
General, now Associate Justice, Antonio P. commercial and proprietary acts (jure
Barredo, the principal defense relied upon was gestionis) The result is that State immunity
that the suit in reality was one against the now extends only to acts jure imperii. The
government and therefore should be restrictive application of State immunity is now
dismissed, no consent having been shown. the rule in the United States, the United
Then on July 11, 1969, the parties submitted Kingdom and other states in Western Europe.
a stipulation of facts to this effect: "That the
plaintiffs are the registered owners of Lot 647- The restrictive application of State immunity is
B of the Banilad estate described in the Survey proper only when the proceedings arise out of
plan RS-600 GLRO Record No. 5988 and more commercial transactions of the foreign
particularly described in Transfer Certificate of sovereign, its commercial activities or
Title No. RT-5963 containing an area of 1,045 economic affairs. Stated differently, a State
square meters; That the National Government may be said to have descended to the level of
in 1927 took possession of Lot 647-B Banilad an individual and can thus be deemed to have
estate, and used the same for the widening of tacitly given its consent to be sued only when
Gorordo Avenue; That the Appraisal it enters into business contracts. It does not
Committee of Cebu City approved Resolution apply where the contracts relates to the
No. 90, Series of 1965 fixing the price of Lot exercise of its sovereign functions. In this case
No. 647-B at P50.00 per square meter; That the projects are an integral part of the naval
Lot No. 647-B is still in the possession of the base which is devoted to the defense of both
National Government the same being utilized the United States and the Philippines,
as part of the Gorordo Avenue, Cebu City, and indisputably a function of the government of
Page 55 of 121
403 CASE DIGESTS | CONSTITUTIONAL LAW REVIEW | JUDGE ESTELA ALMA SINGCO | FIRST SEMESTER | 2020-2021

the highest order; they are not utilized for nor In this case, the projects are an integral part
dedicated to commercial or business of the naval base which is devoted to the
purposes." defense of the USA and Philippines which is,
indisputably, a function of the government. As
In the instant case, the Department of such, by virtue of state immunity, the courts
Agriculture has not pretended to have of the Philippines have no jurisdiction over the
assumed a capacity apart from its being a case for the US government has not given
governmental entity when it entered into the consent to the filing of this suit.
questioned contract; nor that it could have, in
fact, performed any act proprietary in 84 U.S. vs. Guinto, 182 SCRA 644
character.
FACTS:
But, be that as it may, the claims of private
The cases have been consolidated because
respondents, i.e., for underpayment of wages,
they all involve the doctrine of state immunity.
holiday pay, overtime pay and similar other
In GR No. 76607, private respondents
items, arising from the Contract for Security
regarding suing several officers of the US Air
Services, clearly constitute money claims. Act
Force in connection with the bidding for
No. 3083, aforecited, gives the consent of the
barbering services in Clark Air Base. In GR No.
State to be "sued upon any moneyed claim
80018, Luis Bautista was arrested following a
involving liability arising from contract,
buy-bust operation for a violation of the
express or implied, x x x" Pursuant, however,
Dangerous Drugs Act. Bautista then filed a
to Commonwealth Act ("C.A.") No. 327, as
complaint for damages claiming that because
amended by Presidential Decree ("P.D.") No.
of the acts of the respondents, he lost his job.
1445, the money claim should first be brought
In GR No. 79470, Fabian Genove filed a
to the Commission on Audit.
complaint for damages against petitioner for
his dismissal as cook in the US Air Force. In GR
No. 80258, complaint for damage was filed by
83 United States of America v. Ruiz (136 the respondents against petitioners for injuries
SCRA 487) allegedly sustained by plaintiffs. All cases
invoke the doctrine of state immunity as a
FACTS: ground to dismiss the same.
Sometime in May 1972, the United States
invited the submission of bids for certain naval
projects. Eligio de Guzman & Co. Inc.
responded to the invitation and submitted ISSUE:
bids. Subsequently, the company received two
telegrams requesting it to confirm its price. In Are the petitioners immune from suit?
June 1972, the company received a letter
which said that the company did not qualify to
receive an award for the projects. The
company then sued the United States of RULING:
America and individual petitioners demanding
It is clear that the petitioners in GR No. 80018
that the company perform the work on the
were acting in the exercise of their official
projects, or for the petitioners to pay damages
functions. They cannot be directly impleaded
and to issue a writ of preliminary injunction to
for the US government has not given its
restrain the petitioners from entering into
consent to be sued. In GR No. 79470,
contracts with third parties concerning the
petitioners are not immune because
project.
restaurants are commercial enterprises,
however, the claim of damages by Genove
ISSUE:
cannot be allowed on the strength of the
Does the court have jurisdiction over the case?
evidence presented. Barber shops are also
commercial enterprises operated by private
RULING:
persons, thus, petitioners in GR No. 76607
No, the court has no jurisdiction over the case.
cannot plead any immunity from the complaint
The rule of State immunity exempts a State
filed. In GR No. 80258, the respondent court
from being sued in the courts of another state
will have to receive the evidence of the alleged
without its consent or waiver. This is a
irregularity in the grant of the barbershop
necessary consequence of the principles of
concessions before it can be known in what
independence and equality of states. However,
capacity the petitioners were acting at the time
state immunity now extends only to
of the incident.
governmental acts of the state. It has been
necessary to distinguish them-between
sovereign and governmental acts (jure There is no question that the United States of
imperii) and private, commercial and America, like any other state, will be deemed
proprietary acts (jure gestionis). The result is to have impliedly waived its non-suability if it
that State immunity now extends only to acts has entered into a contract in its proprietary or
jure imperil. The restrictive application of private capacity. It is only when the contract
State immunity is proper only when the involves its sovereign or governmental
proceedings arise out of commercial capacity that no such waiver may be implied.
transactions of the foreign sovereign. Stated The restrictive application of State immunity is
differently, a State may be said to have proper only when the proceedings arise out of
descended to the level of an individual and can commercial transactions of the foreign
thus be deemed to have tacitly given its sovereign, its commercial activities or
consent to be sued only when it enters into economic affairs.
business contracts. It does not apply where
the contract relates to the exercise of its
sovereign functions.

Page 56 of 121
403 CASE DIGESTS | CONSTITUTIONAL LAW REVIEW | JUDGE ESTELA ALMA SINGCO | FIRST SEMESTER | 2020-2021

SCOPE OF PROHIBITION AGAINST SUING The petitioner cannot be held liable. It has
THE STATE already been remarked that municipal
85 Alvarez v. Commonwealth of the corporations are suable because their charters
Philippines, 65 Phil. 302, 313 (1938) grant them the competence to sue and be
FACTS: sued. Nevertheless, they are generally not
Plaintiff commenced a complaint of liable for torts committed by them in the
interpleader alleging that they are in discharge of governmental functions and can
possession and cultivated agricultural lands be held answerable only if it can be shown that
within San Pedro Tunasan, and they will pay they were acting in a proprietary capacity. In
rents as the court may determine to the permitting such entities to be sued, the State
following parties that would be adjudged as merely gives the claimant the right to show
owner of the subject property: to that the defendant was not acting in its
Commonwealth of the Philippines by Virtue of governmental capacity when the injury was
Escheat, Provincial of Laguna may have committed or that the case comes under the
interest of the Hacienda, to Municipality of San exceptions recognized by law. Failing this, the
pedro by virtue of escheat, to the Colegio de claimant cannot recover. The circumstance
san Jose also claims to be the owner of the that a state is suable does not necessarily
land and Carlos young also claims to be the mean that it is liable; on the other hand, it can
owner of the land. Plaintiffs conclude by asking never be held liable if it does not first consent
that the court order the defendants or to be sued. Liability is not conceded by the
interpleaders to litigate among themselves mere fact that the state has allowed itself to
over the ownership or dominion of the be sued. When the state does waive its
hacienda and thereafter determine by sovereign immunity, it is only giving the
judgment who is the rightful owner thereof plaintiff the chance to prove, if it can, that the
entitled to collect the rental from them defendant is liable.

Issue In the case at bar, the driver of the dump truck


May the Commonwealth of the Philippines be of the municipality insists that "he was on his
impleaded as defendant in a case of way to the Naguilian river to get a load of sand
interpleader? and gravel for the repair of San Fernando's
municipal streets." In the absence of any
Ruling: evidence to the contrary, the regularity of the
No, it cannot be impleaded as a defendant. performance of official duty is presumed
Speaking of the intervention of the pursuant to Section 3(m) of Rule 131 of the
Commonwealth of the Philippines, there is little Revised Rules of Court. Hence, We rule that
to be said. The question raised is already the driver of the dump truck was performing
settled in this jurisdiction. It is a fundamental duties or tasks pertaining to his office.
principle that the Government of the
Philippines, now the Commonwealth of the 87 UP v. Dizon, G.R. No. 171182, 679 SCRA
Philippines, as the supreme authority which 54, 23 August 2012, 1st Div. [Bersamin]
represents in this country the existing FACTS:
sovereignty, cannot be sued without its University of the Philippines (UP) entered into
consent. The prohibition holds true both in a General Construction Agreement with
case where it is joined as a defendant as well respondent Stern Builders Corporation (Stern
as in that where, as in the present, it is being Builders) for the construction and renovation
compelled to litigate against other persons of the buildings in the campus of the UP in Los
without its consent. There is no substantial Bas. UP was able to pay its first and second
difference between making it defend itself billing. However, the third billing worth
against it will in a case where it is a defendant P273,729.47 was not paid due to its
and compelling it, without its consent, to disallowance by the Commission on Audit
interplead in an action commenced by another (COA). Thus, Stern Builders sued the UP to
person. In one and the other case it is collect the unpaid balance. The RTC rendered
compelled, without its consent, to maintain a its decision ordering UP to pay Stern Builders.
suit or litigation, and this is what the legal Stern Builders filed in the RTC its motion for
principal prohibits. execution. Consequently, the sheriff served
notices of garnishment to the UPs depositary
86 Municipality of San Fernando, La Union v. banks and the RTC ordered the release of the
Firme, 195 SCRA 692 funds.
FACTS:
A passenger jeepney, a sand truck and a ISSUE
dump truck of the Municipality of San May the funds of UP be validly garnished?
Fernando, La Union collided. Due to the
impact, several passengers of the jeepney RULING:
including Laureano Baniña Sr. died. The heirs No, UP's funds, being government funds, are
of Baniña filed a complaint for damages not subject to garnishment. UP is a
against the owner and driver of the jeepney, government instrumentality, performing the
who, in turn, filed a Third Party Complaint States constitutional mandate of promoting
against the Municipality and its dump truck quality and accessible education. As a
driver, Alfredo Bislig. Municipality filed its government instrumentality, the UP
answer and raised the defense of non-suability administers special funds sourced from the
of the State. After trial, the court ruled in favor fees and income enumerated under Act No.
of the plaintiffs and ordered Municipality and 1870 and Section 1 of Executive Order No.
Bislig to pay jointly and severally the heirs of 714, and from the yearly appropriations, to
Baniña. achieve the purposes laid down by Section 2 of
Act 1870, as expanded in Republic Act No.
ISSUES: 9500.. All the funds going into the possession
Is the municipal corporation Liable? of the UP, including any interest accruing from
the deposit of such funds in any banking
RULING: institution, constitute a "special trust fund,"
Page 57 of 121
403 CASE DIGESTS | CONSTITUTIONAL LAW REVIEW | JUDGE ESTELA ALMA SINGCO | FIRST SEMESTER | 2020-2021

the disbursement of which should always be


aligned with the UPs mission and purpose, and
SUABILITY V. LIABILITY
should always be subject to auditing by the
COA. The funds of the UP are government 89 Republic v. NLRC G.R. No. 120385 October
funds that are public in character. They include 17, 1996
the income accruing from the use of real FACTS
property ceded to the UP that may be spent Some time in January, 1988, the sequestration
only for the attainment of its institutional order against PNEI was lifted to give way to
objectives. Hence, the funds subject of this the sale of its sale by the Asset Privatization
action could not be validly made the subject of Trust which, in the meanwhile, had taken over
writ of execution or garnishment. The adverse the management of the company. Yet, PNEI
judgment rendered against the UP in a suit to was still continuing to suffer deterioration of its
which it had impliedly consented was not financial condition. As a cost saving measure,
immediately enforceable by execution against the management committee recommended for
the UP, because suability of the State did not retrenchment .This resulted to various labor
necessarily mean its liability. complaints filed against PNEI. NLRC rendered
a decision holding PNEI and APT liable. In an
88 Meritt vs. Government, 31 SCRA 311, 318; alias writ of execution, it served a notice of
Republic vs. Villasor, 54 SCRA 84 garnishment on the Land bank of the
Philippines upon all credits, interest, bank
FACTS: deposits belonging to APT. The land bank
responded that since the funds of Apt, a
Merrit was riding a motorcycle along Padre government owned and controlled agency,
Faura Street when he was bumped by the were considered public in nature, they could
ambulance of the General Hospital. Merrit not be subject to garnishment.
sustained severe injuries rendering him unable
to return to work. The legislature later enacted ISSUE:
Act 2457 authorizing Merritt to file a suit Can the Apt be liable for the obligations of the
against the Government in order to fix the PNEI?
responsibility for the collision between his
motorcycle and the ambulance of the General RULING:
Hospital, and to determine the amount of the No, the APT cannot be liable for the obligations
damages, if any, to which he is entitled. After of PNEI. Under our jurisdiction, he State may
trial, the lower court held that the collision was not be sued without its consent is not really
due to the negligence of the driver of the absolute for it does not say that the state may
ambulance. It then determined the amount of not be sued under any circumstance. In this
damages and ordered the government to pay case, the law creating APT provides that it may
the same. “sue or be sued”.
Nonetheless, we have likewise since explained
that suability does not necessarily mean
liability on the part of the particular
ISSUES:
instrumentality or agency of the government;
Did the Government, in enacting the Act 2457, hence When the State gives its consent to be
simply waive its immunity from suit or did it sued, it does not thereby necessarily consent
also concede its liability to the plaintiff? to an unrestrained execution against it.
Tersely put, when the State waives its
immunity, all it does, in effect, is to give the
other party an opportunity to prove, if it can,
HELD: that the State has a liability. The universal rule
that where the State gives its consent to be
No, the government did not waive its liability sued by private parties either by general law
upon enacting 2457. As to the scope of or special law, it may limit claimant's action
legislative enactments permitting individuals only up to the completion of proceedings
to sue the state where the cause of action anterior to the stage of execution' and that the
arises out of either fort or contract, the rule is power of the Courts ends when the judgment
is rendered, since government funds and
“By consenting to be sued a state simply properties may not be seized under writs of
waives its immunity from suit. It does not execution or garnishment to satisfy such
thereby concede its liability to plaintiff, or judgments, is based on obvious considerations
create any cause of action in his favor, or of public policy. Disbursements of public funds
extend its liability to any cause not previously must be covered by the correspondent
recognized. It merely gives a remedy to appropriation as required by law. The functions
enforce a preexisting liability and submits itself and public services rendered by the State
to the jurisdiction of the court, subject to its cannot be allowed to be paralyzed or disrupted
right to interpose any lawful defense” by the diversion of public funds from their
legitimate and specific objects, as
appropriated by law.
In the case at bar, The plaintiff was authorized
90 Municipality of Hagonoy Bulacan v.
to bring this action against the Government "in
Dumdum G.R. No. 168289 March 22, 2010
order to fix the responsibility for the collision
Facts
between his motorcycle and the ambulance of
It was alleged that sometime in the middle of
the General Hospital and to determine the
the year 2000, respondent, doing business as
amount of the damages. The government did
KD Surplus was contacted by petitioner Ople.
not extends its liability under 2457. Hence, the
Respondent had entered into an agreement
government is not liable
with petitioner municipality through Ople for
the delivery of motor vehicles, which were
supposedly needed to carry out certain
developments for the delivery of the Motor
Page 58 of 121
403 CASE DIGESTS | CONSTITUTIONAL LAW REVIEW | JUDGE ESTELA ALMA SINGCO | FIRST SEMESTER | 2020-2021

Vehicles. Respondent agreed to deliver from May the funds of UP be validly garnished?
her principal place of business in Cebu City
twenty-one motor vehicles. However, despite RULING
having made several deliveries, Ople allegedly The funds of the UP are government funds that
did not heed respondent’s claim for payment. are public in character. They include the
The trial court issued the Writ of Preliminary income accruing from the use of real property
Attachment directing the sheriff "to attach the ceded to the UP that may be spent only for the
estate, real and personal properties" of attainment of its institutional objectives.
petitioners. Petitioners also filed a Motion to Hence, the funds subject of this action could
Dissolve and/or Discharge the Writ of not be validly made the subject of the RTC’s
Preliminary Attachment Already Issued, writ of execution or garnishment. The adverse
invoking immunity of the state from suit. judgment rendered against the UP in a suit to
which it had impliedly consented was not
ISSUE immediately enforceable by execution against
May the writ of preliminary Attachment be the UP, because suability of the State did not
discharged? necessarily mean its liability. A marked
distinction exists between suability of the
RULING State and its liability.
Yes, the writ of preliminary attachment may be
discharged. A distinction should first be made between
suability and liability. "Suability depends on
Be that as it may, a difference lies between the consent of the state to be sued, liability on
suability and liability. As held in City of the applicable law and the established facts.
Caloocan v. Allarde, where the suability of the The circumstance that a state is suable does
state is conceded and by which liability is not necessarily mean that it is liable; on the
ascertained judicially, the state is at liberty to other hand, it can never be held liable if it does
determine for itself whether to satisfy the not first consent to be sued. Liability is not
judgment or not. Execution may not issue conceded by the mere fact that the state has
upon such judgment, because statutes waiving allowed itself to be sued. When the state does
non-suability do not authorize the seizure of waive its sovereign immunity, it is only giving
property to satisfy judgments recovered from the plaintiff the chance to prove, if it can, that
the action. These statutes only convey an the defendant is liable.
implication that the legislature will recognize
such judgment as final and make provisions for
its full satisfaction. Thus, where consent to be
DIPLOMATIC IMMUNITY
sued is given by general or special law, the
implication thereof is limited only to the 92 Liang v. People, G.R. No. 125865, January
resultant verdict on the action before 28, 2000
execution of the judgment. The universal rule
that where the State gives its consent to be FACTS:
sued by private parties either by general or
special law, it may limit claimant’s action "only Petitioner Jeffrey Liang, an economist working
up to the completion of proceedings anterior to with the Asian Development Bank (ADB) , was
the stage of execution" and that the power of charged before the MeTC of Mandaluyong with
the Courts ends when the judgment is two counts of grave oral defamation for
rendered, since government funds and allegedly uttering defamatory words against a
properties may not be seized under writs of fellow ADB worker. Liang was arrested but
execution or garnishment to satisfy such later released. The next day, the judge
judgments, is based on obvious considerations received an "office of protocol" from the
of public policy. Disbursements of public funds Department of Foreign Affairs (DFA) stating
must be covered by the corresponding that Liang is covered by immunity from legal
appropriations as required by law. The process under Section 45 of the Agreement
functions and public services rendered by the between the ADB and the Philippine
State cannot be allowed to be paralyzed or Government. Based on the said protocol
disrupted by the diversion of public funds from communication, the judge, without notice to
their legitimate and specific objects. the prosecution, dismissed the two criminal
cases. The RTC set aside the MeTC ruling and
91 UP v. Dizon GR No. 171782, August 23, ordered the latter court to enforce the warrant
2012 of arrest it earlier issued. Liang appealed
FACTS: arguing that he is covered by immunity under
University of the Philippines (UP) entered into the Agreement.
a General Construction Agreement with
respondent Stern Builders Corporation (Stern ISSUE:
Builders) for the construction and renovation
of the buildings in the campus of the UP in Los Is Liang covered with immunity from legal
Bas. UP was able to pay its first and second process under Section 45 of the Agreement
billing. However, the third billing worth between the ADB and the Philippine
P273,729.47 was not paid due to its Government? - NO
disallowance by the Commission on Audit
(COA). Thus, Stern Builders sued the UP to RULING:
collect the unpaid balance. The RTC rendered
its decision ordering UP to pay Stern Builders. The petitioner’s case is not covered by the
Stern Builders filed in the RTC its motion for immunity. Courts cannot blindly adhere to the
execution. Consequently, the sheriff served communication from the DFA that the
notices of garnishment to the UPs depositary petitioner is covered by any immunity. It has
banks and the RTC ordered the release of the no binding effect in courts. The court needs to
funds. protect the right to due process not only of the
accused but also of the prosecution. Secondly,
ISSUE the immunity under Section 45 of the
Page 59 of 121
403 CASE DIGESTS | CONSTITUTIONAL LAW REVIEW | JUDGE ESTELA ALMA SINGCO | FIRST SEMESTER | 2020-2021

Agreement is not absolute, but subject to the act is private or acts jure gestionis (those that
exception that the acts must be done in are for profit), then immunity cannot be
“official capacity”. Hence, slandering a person invoked.
could not possibly be covered by the immunity
agreement because our laws do not allow the This Court has considered the following
commission of a crime, such as defamation, in transactions by a foreign state with private
the name of official duty. parties as acts jure imperii: (1) the lease by a
foreign government of apartment buildings for
Moreover, under the Vienna Convention on use of its military officers (Syquia v. Lopez, 84
Diplomatic Relations, a diplomatic agent, Phil. 312 [1949]; (2) the conduct of public
assuming petitioner is such, enjoys immunity bidding for the repair of a wharf at a United
from criminal jurisdiction of the receiving state States Naval Station (United States of America
except in the case of an action relating to any v. Ruiz, supra.); and (3) the change of
professional or commercial activity exercised employment status of base employees
by the diplomatic agent in the receiving state (Sanders v. Veridiano, 162 SCRA 88 [1988]).
outside his official functions. As already
mentioned above, the commission of a crime On the other hand, this Court has considered
is not part of official duty. the following transactions by a foreign state
93 Holy See v. Rosario, 238 SCRA 524 with private parties as acts jure gestionis: (1)
the hiring of a cook in the recreation center,
FACTS: consisting of three restaurants, a cafeteria, a
bakery, a store, and a coffee and pastry shop
Lot 5-A is registered under the name of the at the John Hay Air Station in Baguio City, to
petitioner The Holy See. This lot is contiguous cater to American servicemen and the general
to lots 5-B and 5-D registered in the name of public (United States of America v. Rodrigo,
Philippine Realty Corporation (PRC). These 182 SCRA 644 [1990]); and (2) the bidding for
three lots were sold through an agent Msgr. the operation of barber shops in Clark Air Base
Domingo Cirilos Jr. to Ramon Licup. Licup in Angeles City (United States of America v.
assigned his rights to private respondent Guinto, 182 SCRA 644 [1990]). The operation
Starbright Sales Ent. Inc. (SSEI). of the restaurants and other facilities open to
the general public is undoubtedly for profit as
Due to refusal of the squatters to vacate the a commercial and not a governmental activity.
lots, a dispute arose as to who of the parties By entering into the employment contract with
has the responsibility of eviction and clearing the cook in the discharge of its proprietary
the land. SSEI insists that petitioner should function, the United States government
clear the property of the squatters. Petitioner impliedly divested itself of its sovereign
refused and proposed that either SSEI immunity from suit.
undertake the eviction or that the earnest
money be returned. Msgr. Cirilos returned the Certainly, the mere entering into a contract by
P100,000.00 earnest money, and the property a foreign state with a private party cannot be
was sold to Tropicana Properties and the ultimate test. Such an act can only be the
Development Corporation (Tropicana). SSEI start of the inquiry. The logical question is
filed suit for annulment of sale, specific whether the foreign state is engaged in the
performance and damages against Msgr. activity in the regular course of business. If the
Cirilos, PRC, and Tropicana. The petitioner and foreign state is not engaged regularly in a
Msgr. Cirilos moved to dismiss for lack of business or trade, the particular act or
jurisdiction based on sovereign immunity from transaction must then be tested by its nature.
suit. It was denied on the ground that If the act is in pursuit of a sovereign activity,
petitioner ―shed off its sovereign immunity by or an incident thereof, then it is an act jure
entering into the business contract in question. imperii, especially when it is not undertaken
for gain or profit.
ISSUE:
In this case, the petitioner had denied that the
WON the Holy See can invoke its right to acquisition and subsequent disposal of the Lot
Sovereign Immunity to suit. - YES 5-A were made for profit. It claimed that it
acquired the property for its mission or the
RULING: Apostolic Nunciature in the Philippines. The lot,
allegedly, was acquired by donation from the
In the case at bar, lot 5-A was acquired as a Archdiocese of Manila for the purpose of
donation from the archdiocese of Manila for the building official residence of Papal Nuncio.
site of its mission or the Apostolic Nuniciature However, when the informal settlers refused to
in the Philippines. The subsequent disposal leave the property, the petitioner decided to
was made because the squatters living thereon dispose the property, not for commercial
made it impossible for petitioner to use it for purposes. The DFA intervened as they
the purpose of the donation. Petitioner did not established in a Memorandum and Certification
sell lot 5-A for profit or gain. the privilege of sovereign immunity of the
petitioner, stating that they are a duly
Generally, there are two accepted concepts of accredited diplomatic mission to the
sovereignty: a) classical or absolute theory, Philippines exempt from local jurisdiction and
wherein a sovereign cannot be made as has title to all rights, privileges and immunities
respondent to courts of another sovereign of a diplomatic mission or embassy in the
without its consent and; b) restrictive theory, country. When the plea of immunity has been
which puts conditions on when to recognize recognized by the executive department, such
immunity. shall be conclusive to courts

Under the restrictive theory, sovereign 94 China National Machinery Equipment


immunity is only recognized with regard to Corporation v. Sta. Maria, 665 SCRA 189
public acts or acts jure imperii (or those in
pursuant to governmental functions) . If the FACTS:
Page 60 of 121
403 CASE DIGESTS | CONSTITUTIONAL LAW REVIEW | JUDGE ESTELA ALMA SINGCO | FIRST SEMESTER | 2020-2021

government-owned corporation, it failed to


In September 2002, petitioner CNMEG entered adduce evidence that it has not consented to
into a memorandum of understanding (MOU) be sued under Chinese law. Thus, following
with North Luzon Railways Corp (Northrail) to this Court’s ruling in Deutsche Gesellschaft, in
conduct a feasibility study on the construction the absence of evidence to the contrary,
of a railway line from Manila to La Union CNMEG is to be presumed to be a government-
(Northrail Project). Subsequently, Export owned and -controlled corporation without an
Import Bank of China (EXIM Bank) and original charter. As a result, it has the capacity
Department of Finance entered into a MOU to sue and be sued under Section 36 of the
whereby China will extend a preferred buyer’s Corporation Code.
credit to the Philippines to finance the Northrail
Project. EXIM is to loan DOF $400 million
payable in 20 years with a 5-year grace period 95 JUSMAG v. NLRC, 239 SCRA 224
at the rate of 3% per annum. In December
2003, Northrail and CNMEG executed a FACTS:
contract for the construction of Phase I of the
Northrail Project. The contract price was Florento Sacramento (private respondent) was
pegged at $421,050,0003. The Philippine one of the 74 security assistance
Government then obtained a loan from EXIM supportpersonnel (SASP) working at the Joint
for $400,000 to finance the project. United States Military Assistance Group to
Respondents filed a complaint for annulment thePhilippines (JUSMAG- Phils.). He had been
of contract, alleging that the contract was void with JUSMAG from 1969- 1992. When
for being contrary to the Constitution, dismissed, he held the position of illustrator 2
Government Procurement Reform Act (RA and incumbent Pres. of JUSMAG Phils.-Filipino
9184), Government Auditing Code (PD 1445), Civilian Employees Association, a labor org.
and the Administrative Code. CNMEG filed a duly registered with DOLE. His services were
Motion to Dismiss dated 12 April 2006, arguing terminated allegedly due to the abolition of his
that the trial court did not have jurisdiction position. Sacramento filed a complaint with
over (a) its person, as it was an agent of the DOLE on the ground that he was illegally
Chinese government, making it immune from suspended and dismissed from service by
suit. JUSMAG. He asked for reinstatement. JUSMAG
filed a Motion to Dismiss invoking immunity
ISSUE: from suit. The Labor Arbiter in an Order,
dismissed the complaint for want of
WON CNMEG is entitled to immunity - NO jurisdiction. Sacramento appealed to the NLRC
which reversed the decision of the Labor
RULING: Arbiter and held that the petitioner had lost his
right not to be sued because (1) the principle
It was CNMEG that initiated the undertaking, of estoppel- JUSMAG failed to refute the
and not the Chinese government. The existence of employer- employee relationship
Feasibility Study was conducted not because of (2) Jusmag has waived its right to immunity
any diplomatic gratuity from or exercise of from suit when it hired the services to private
sovereign functions by the Chinese respondent. Hence, this petition.
government but was plainly a business
strategy employed by CNMEG with a view to ISSUE:
securing this commercial enterprise.
WON JUSMAG has immunity from suit. - YES
The use of the term “state corporation” to refer
to CNMEG was only descriptive of its nature as RULING:
a government-owned and/or -controlled
corporation, and its assignment as the Primary When JUSMAG took the services of the private
Contractor did not imply that it was acting on respondent, it was performing agovernmental
behalf of China in the performance of the function in behalf of the United States
latter’s sovereign functions. To imply pursuant to the Military Assistance Agreement
otherwise would result in an absurd situation, between the Phils. and the US. JUSMAG
in which all Chinese corporations owned by the consists of Air, Naval and Army groupand its
state would be automatically considered as primary task was to advise and assist the Phils.
performing governmental activities, even if on air force, army and navalmatters. A suit
they are clearly engaged in commercial or against JUSMAG is one against the United
proprietary pursuits. States government, and in the absence of any
waiver or consent of the latter to the suit, the
Even assuming arguendo that CNMEG complaint against JUSMAG cannot prosper.
performs governmental functions, such claim
does not automatically vest it with immunity. Immunity of State from suitis one of these
This view finds support in Malong v. Philippine universally recognized principles. In
National Railways, in which this Court held that international law, "immunity" is commonly
“immunity from suit is determined by the understood as an exemption of the state and
character of the objects for which the entity its organs from the judicial jurisdiction of
was organized.” another state. This is anchored on the principle
of the sovereign equality of states under which
In the case at bar, it is readily apparent that one state cannot assert jurisdiction over
CNMEG cannot claim immunity from suit, even another in violation of the maxim par in parem
if it contends that it performs governmental non habet imperium(an equal has no power
functions. Its designation as the Primary over an equal). As it stands now, the
Contractor does not automatically grant it application of the doctrine of immunity from
immunity, just as the term “implementing suit has been restricted to sovereign or
agency” has no precise definition for purposes governmental activities and does not extend to
of ascertaining whether GTZ was immune from commercial, private and proprietary acts.
suit. Although CNMEG claims to be a
Page 61 of 121
403 CASE DIGESTS | CONSTITUTIONAL LAW REVIEW | JUDGE ESTELA ALMA SINGCO | FIRST SEMESTER | 2020-2021

96 World Health Organization v. Aquino, 48 Arbitration Branch, Iloilo City. In these cases,
SCRA 242 the private respondents claim having been
wrongfully terminated from their employment
FACTS: by the petitioner.

Dr. Leonce Verstuyft was assigned by WHO to The petitioner, who claims to be an
its regional office in Manila as Acting Assistant international inter-government organization
Director of Health Services. His personal composed of various Southeast Asian
effects, contained in twelve (12) crates, were countries, filed a Motion to Dismiss, challenged
allowed free entry from duties and taxes. the jurisdiction of the public respondent in
Constabulary Offshore Action Center (COSAC) taking cognizance of the above cases.
suspected that the crates “contain large
quantities of highly dutiable goods” beyond the The private respondents, as well as
official needs of Verstuyft. Upon application of respondent labor arbiter, allege that the
the COSAC officers, Judge Aquino issued a petitioner is not immune from suit and
search warrant for the search and seizure of assuming that if, indeed, it is an international
the personal effects of Verstuyft. organization, it has, however, impliedly, if not
expressly, waived its immunity by belatedly
Secretary of Foreign Affairs Carlos P. Romulo raising the issue of jurisdiction.
advised Judge Aquino that Dr. Verstuyft is
entitled to immunity from search in respect for ISSUE:
his personal baggage as accorded to members
of diplomatic missions pursuant to the Host WON the petitioner is immune from suit. - YES
Agreement and requested that the search
warrant be suspended. The Solicitor General RULING:
accordingly joined Verstuyft for the quashal of
the search warrant but respondent judge It is beyond question that petitioner SEAFDEC
nevertheless summarily denied the quashal. is an international agency enjoying diplomatic
Verstuyft, thus, filed a petition for certiorari immunity. It has already been held in
and prohibition with the SC. WHO joined Southeast Asian Fisheries Development
Verstuyft in asserting diplomatic immunity. Center-Aquaculture Department vs. National
Labor Relations Commission (G.R. No. 86773,
ISSUE: 206 SCRA 283/1992). Petitioner Southeast
Asian Fisheries Development Center-
WON the personal effects of Verstuyft can be Aquaculture Department (SEAFDEC-AQD) is
exempted from search and seizure under the an international agency beyond the jurisdiction
diplomatic immunity. - YES of public respondent NLRC.

RULING: Being an intergovernmental organization,


SEAFDEC including its Departments (AQD),
The executive branch of the Phils has expressly enjoys functional independence and freedom
recognized that Verstuyft is entitled to from control of the state in whose territory its
diplomatic immunity, pursuant to the office is located. One of the basic immunities
provisions of the Host Agreement. The DFA of an international organization is immunity
formally advised respondent judge of the from local jurisdiction, i.e., that it is immune
Philippine Government's official position. The from the legal writs and processes issued by
Solicitor General, as principal law officer of the the tribunals of the country where it is found.
gorvernment, likewise expressly affirmed said The obvious reason for this is that the
petitioner's right to diplomatic immunity and subjection of such an organization to the
asked for the quashal of the search warrant. authority of the local courts would afford a
convenient medium thru which the host
It is a recognized principle of international law government may interfere in their operations
and under our system of separation of powers or even influence or control its policies and
that diplomatic immunity is essentially a decisions of the organization; besides, such
political question and courts should refuse to objection to local jurisdiction would impair the
look beyond a determination by the executive capacity of such body to discharge its
branch of the government, and where the plea responsibilities impartially on behalf of its
of diplomatic immunity is recognized and member-states.
affirmed by the executive branch of the
government as in the case at bar, it is then the
EXEMPTION FROM LEGAL REQUIREMENTS
duty of the courts to accept the claim of
immunity upon appropriate suggestion by the 98 Araneta v. Gatmaitan, 101 Phil. 323
principal law officer of the government, the
Solicitor General in this case, or other officer FACTS:
acting under his discretion. Courts may not so
exercise their jurisdiction by seizure and The President issued EO 22 - prohibiting the
detention of property, as to embarrass the use of trawls in San Miguel Bay, and the EO 66
executive arm of the government in and 80 as amendments to EO 22, as a
conducting foreign relations. response for the general clamor among the
majority of people living in the coastal towns
97 SEAFDEC v. NLRC, 241 SCRA 580 of San Miguel Bay that the said resources of
the area are in danger of major depletion
FACTS: because of the effects of trawl fishing.

Two labor cases were filed by the herein A group of Otter trawl operators took the
private respondents against the petitioner, matter to the court by filing a complaint for
Southeast Asian Fisheries Development Center injunction and/or declaratory relief with
(SEAFDEC), before the National Labor preliminary injunction with the Court of First
Relations Commission (NLRC), Regional Instance of Manila, docketed as Civil Case No.
Page 62 of 121
403 CASE DIGESTS | CONSTITUTIONAL LAW REVIEW | JUDGE ESTELA ALMA SINGCO | FIRST SEMESTER | 2020-2021

24867, praying that a writ of preliminary the government, is a corporation with a


injunction be issued to restrain the Secretary personality distinct from the Republic or any of
of Agriculture and Natural Resources and the its agencies or instrumentalities, and therefore
Director of Fisheries from enforcing said do not partake in the latter's exemption from
executive order; to declare the same null and the posting of appeal bonds.
void, and for such other relief as may be just
and equitable in the premises. ISSUE:

ISSUE: Whether Executive Orders Nos. 22, 66 WON BBC is exempt from posting an appeal
and 80 were valid, for the issuance thereof was bond - NO
not in the exercise of legislative powers unduly
delegated to the President. RULING:

RULING: As a general rule, the government and.all the


attached agencies with no legal personality
Yes. As already held by this Court, the true distinct from the former are exempt from
distinction between delegation of the power to posting appeal bonds, whereas government-
legislate and the conferring of authority or owned and controlled corporations (GOCCs)
discretion as to the execution of law consists are not similarly exempted. This distinction is
in that the former necessary involves a brought about by the very reason of the appeal
discretion as to what the law shall be, while in bond itself: to protect the presumptive
the latter the authority or discretion as to its judgment creditor against the insolvency of
execution has to be exercised under and in the presumptive judgment debtor. When the
pursuance of the law. The first cannot be done; State litigates, it is not required to put up an
to the latter no valid objection can be made. appeal bond because it is presumed to be
always solvent.
In the case of U. S. vs. Ang Tang Ho, 43 Phil.
1, We also held, the power to delegate - the This exemption, however, does not, as a
Legislature cannot delegate legislative power general rule, apply to GOCCs for the reason
to enact any law. If Act No. 2868 is a law unto that the latter has a personality distinct from
itself, and it does nothing more than to its shareholders. Thus, while a GOCCs majority
authorize the Governor-General to make rules stockholder, the State, will always be
and regulations to carry it into effect, then the presumed solvent, the presumption does not
Legislature created the law. There is no necessarily extend to the GOCC itself.
delegation of power and it is valid. On the However, when a' GOCC becomes a
other hand, if the act within itself does not "government machinery to carry out a
define a crime and is not complete, and some declared government policy," it becomes
legislative act remains to be done to make it a similarly situated as its majority stockholder as
law or a crime, the doing of which is vested in there is the assurance that the government
the Governor-General, the act is delegation of will necessarily fund its primary functions.
legislative power, is unconstitutional and void. Thus, a GOCC that is sued in relation to its
governmental functions may be, under
Congress provided under the Fisheries Act that appropriate circumstances, exempted from
a.) it is unlawful to take or catch fry or fish the payment of appeal fees.
eggs in the waters of the Philippines and b.) it
authorizes Sec. of Agriculture and Natural In the case at bar, BBC was organized as a
Resources to provide regulations/ restrictions private corporation, sequestered in the 1980's
as may be deemed necessary. The Act was and the ownership of which was subsequently
complete in itself and leaves it to the Sec. to transferred to the government in a
carry into effect its legislative intent. The compromise agreement. Further, it is stated in
President did nothing but show an anxious its Amended Articles of Incorporation that BBC
regard for the welfare of the inhabitants and has the following primary function:
dispose of issues of general concern which
were in consonance and strict conformity with To engage in commercial radio and television
law. broadcasting, and for this purpose, to
establish, operate and maintain such stations,
99 Banahaw Broadcasting Corporation v. both terrestrial and satellite or interplanetary,
Pacana, 649 SCRA 196 as may be necessary for broadcasting on a
network wide or international basis.
FACTS:
It is therefore crystal clear that BBC's function
On August 29, 1995, the DXWG personnel filed is purely commercial or proprietary and not
a complaint for illegal dismissal, ULP, and governmental. As such, BBC cannot be
reimbursement of unpaid CBA benefits against deemed entitled to an exemption from the
IBC and BBC. The Labor Arbiter rendered his posting of an appeal bond.
decision in favor of the employees and an
appeal was filed with the NLRC. The NLRC 100 Sarasola v. Trinidad, 40 Phil. 252
ordered BBC to post the required appeal bond
within 10 days from receipt of said Order, with FACTS:
a warning that noncompliance will cause the
dismissal of the appeal for non-perfection. Gregorio Sarasola filed a complaint for
Instead of complying with the Order to post injunction to restrain Wenceslao Trinidad, the
the required bond, BBC filed a Motion for Collector of Internal Revenue from the alleged
Reconsideration, alleging this time that since it illegal collection of taxes. CIR interposed a
is wholly owned by the Republic of the demurrer to the complaint stating that under
Philippines, it need not post an appeal bond. Section 1578 of the Administrative Code,
injunction is not available to restrain collection
The NLRC denied the motion. BBC appealed to of tax. CIR also alleged the complaint did not
the CA who ruled that BBC, though owned by entitle the plaintiff to the relief demanded as
Page 63 of 121
403 CASE DIGESTS | CONSTITUTIONAL LAW REVIEW | JUDGE ESTELA ALMA SINGCO | FIRST SEMESTER | 2020-2021

under Section 1579, it disallows interest on the public policy. The Philippine Government is not
internal revenue taxes in the sum alleged to a mere nominal party because it was
have been illegally collected. The judge of CFI exercising its sovereign functions or powers
Manila sustained the demurrer, basing his and was merely seeking to carry out a trust
decision from the case of Churchill v Rafferty. developed upon it when the Philippine Islands
Sarasola argued that the provisions under Sec was ceded to the United States. Finally, if said
1578-1579 are unconstitutional. loan was for ecclesiastical pious work, then
Spain would not exercise its civil capacities
ISSUE:
102 Republic v. Garcia, 679 SCRA 54
WON interest can be imposed against the The case cannot be found both in CD Asia
State in cases of recovery of taxes illegally and in the Syllabus
collected - YES
103 Arigo v. Swift, 735 SCRA 102 (2014)
RULING: Facts:

It is well settled both on principle and authority In 2013, the USS Guardian, a US ship, was on
that interest is not to be awarded against a its way to Indonesia when it ran aground the
sovereign government unless its consent has northwest side of South Shoal of the
been manifested by an Act of its Legislature or Tubbataha Reefs. Vice Admiral Scott Swift, US
by a lawful contract of its executive officers. If 7th Fleet Commander expressed regret for the
there be doubt upon the subject, that doubt incident in a press statement. Three months
must be resolved in favor of the State. The later, the US Navy-led salvage team had
State never pays interest unless she expressly finished removing the last piece of the
engages to do so. grounded ship from the coral reef. The
petitioners then filed this petition for the
Taxes only draw interest as do sums of money issuance of a Writ of Kalikasan against Swift
when expressly authorized. Interest cannot be and other officials, claiming that the grounding
recovered on an abatement unless the statute and salvaging operations caused and continue
provides for it. The only contrary dictum is that to cause environmental damage of such
where an illegal tax has been collected, the magnitude as to affect several provinces in the
citizen who has paid and is obliged to bring suit Visayas and Mindanao. They also seek a
against the collector is entitled to interest from directive from this Court for the institution of
the time of the illegal exaction. The distinction civil, administrative and criminal suits for acts
undoubtedly arises through the fiction that the committed in violation of environmental laws
suit is against the collector and not against the and regulations in connection with the
State, although the judgment is not to be paid grounding incident. Only the Philippine
by the collector but directly from the treasury. respondents filed a comment to the petition

The state is not amenable to judgments for Issue:


damages or costs without its consent. Our own
statute not only does not authorize interest but Whether or not the Supreme Court has
negatives the payment of interest. The law is jurisdiction over the US respondents who did
valid, or that the plaintiff has not proven such not submit any pleading or manifestation in
a case of irreparable injury as would warrant the case
the issuance of the extraordinary writ of
execution.
Ruling:
101 Government of the Philippine Islands v.
No. Under the Constitution, the State may not
Monte de Pieded,G.R. No. L-9959,
be sued without its consent. While the doctrine
December 13, 1916
appears to prohibit only suits against the state
without its consent, it is also applicable to
Facts:
complaints filed against officials of the state for
Spain paid $400,000 into the treasury of the
acts allegedly performed by them in the
Philippine Islands for the relief of those
discharge of their duties. The rule is that if the
damaged by an earthquake. Upon the petition
judgment against such officials will require the
of Monte de Piedad, an institution under the
state itself to perform an affirmative act to
control of the church, the Philippine
satisfy the same, such as the appropriation of
Government directed its treasurer to give
the amount needed to pay the damages
$80,000 of the relief fund in Four (4)4
awarded against them, the suit must be
installments. As a result, various petitions
regarded as against the state itself although it
were filed, including the heirs of those entitled
has not been formally impleaded. In such a
to the allotments. All prayed for the State to
situation, the state may move to dismiss the
bring suit against Monte de Piedad, and for it
complaint on the ground that it has been filed
to pay with interest. The Defendant appealed
without its consent. In this case, the US
since all its funds have been exhausted already
respondents were sued in their official capacity
on various jewelry loans.
as commanding officers of the US Navy who
had control and supervision over the USS
Issue:
Guardian and its crew. The alleged act or
Whether the government is the proper
authority to the cause of action
omission resulting in the unfortunate
Ruling: grounding of the USS Guardian on the TRNP
was committed while they were performing
official military duties. Considering that the
Yes. The Philippine government, as a trustee
satisfaction of a judgment against said officials
towards the funds could maintain the action
will require remedial actions and appropriation
since there has been no change of sovereignty.
of funds by the US government, the suit is
The state, as a sovereign, is the parens patriae
deemed to be one against the US itself. The
of the people. These principles are based upon
Page 64 of 121
403 CASE DIGESTS | CONSTITUTIONAL LAW REVIEW | JUDGE ESTELA ALMA SINGCO | FIRST SEMESTER | 2020-2021

principle of State immunity therefore bars the controlled by Filipinos. They invoke the
exercise of jurisdiction by this Court over the provisions of the Declaration of Principles and
persons of respondents Swift, Rice and State Policies under Article II of the 1987
Robling. Constitution But, as the Court explained in
Tañada v. Angara, the provisions of Article II
of the 1987 Constitution, the declarations of
principles and state policies, are not self-
executing. Legislative failure to pursue such
policies cannot give rise to a cause of action in
DECLARATION OF PRINCIPLES AND the courts. While Section 19, Article II of the
STATE POLICIES 1987 Constitution requires the development of
104 Espina v. Zamora, G.R. No. 143855, a self-reliant and independent national
September 21, 2010 economy effectively controlled by Filipino
Facts: entrepreneurs, it does not impose a policy of
Filipino monopoly of the economic
On March 7, 2000 President Joseph E. Estrada environment. The objective is simply to
signed into law Republic Act (R.A.) 8762, also prohibit foreign powers or interests from
known as the Retail Trade Liberalization Act of maneuvering our economic policies and ensure
2000. It expressly repealed R.A. 1180, which that Filipinos are given preference in all areas
absolutely prohibited foreign nationals from of development. the 1987 Constitution does
engaging in the retail trade business. R.A. not rule out the entry of foreign investments,
8762 also allows natural-born Filipino citizens, goods, and services. While it does not
who had lost their citizenship and now reside encourage their unlimited entry into the
in the Philippines, to engage in the retail trade country, it does not prohibit them either. In
business with the same rights as Filipino fact, it allows an exchange on the basis of
citizens. On October 11, 2000 petitioners equality and reciprocity, frowning only on
Magtanggol T. Gunigundo I, Michael T. foreign competition that is unfair. The key, as
Defensor, Gerardo S. Espina, Benjamin S. Lim, in all economies in the world, is to strike a
Orlando Fua, Jr., Prospero Amatong, Sergio balance between protecting local businesses
Apostol, Robert Ace S. Barbers, Enrique and allowing the entry of foreign investments
Garcia, Jr., Raul M. Gonzales, Jaime Jacob, and services
Apolinario Lozada, Jr., Leonardo Montemayor,
Ma. Elena Palma-Gil, Prospero Pichay, Juan Section 10, Article XII of the 1987 Constitution
Miguel Zubiri and Franklin Bautista, all gives Congress the discretion to reserve to
members of the House of Representatives, Filipinos certain areas of investments upon the
Filled the present petition, assailing the recommendation of the NEDA and when the
constitutionality of R.A. 8762 national interest requires. Thus, Congress can
determine what policy to pass and when to
pass it depending on the economic exigencies.
Issue: It can enact laws allowing the entry of
foreigners into certain industries not reserved
Whether or not petitioner lawmakers have the by the Constitution to Filipino citizens. In this
legal standing to challenge the case, Congress has decided to open certain
constitutionality of R.A. 8762 areas of the retail trade business to foreign
investments instead of reserving them
Whether or not R.A. 8762 is unconstitutional exclusively to Filipino citizens. The NEDA has
not opposed such policy. To the extent that
R.A. 8762, the Retail Trade Liberalization Act,
Ruling: lessens the restraint on the foreigners' right to
property or to engage in an ordinarily lawful
business, it cannot be said that the law
Legal standing or locus standi refers to the
amounts to a denial of the Filipinos' right to
right of a party to come to a court of justice
property and to due process of law. Filipinos
and make such a challenge. More particularly,
continue to have the right to engage in the
standing refers to his personal and substantial
kinds of retail business to which the law in
interest in that he has suffered or will suffer
question has permitted the entry of foreign
direct injury as a result of the passage of that
investors. Certainly, it is not within the
law. To put it another way, he must show that
province of the Court to inquire into the
he has been or is about to be denied some
wisdom of R.A. 8762 save when it blatantly
right or privilege to which he is lawfully entitled
violates the Constitution. But as the Court has
or that he is about to be subjected to some
said, there is no showing that the law has
burdens or penalties by reason of the law he
contravened any constitutional mandate. The
complains of. Here, there is no clear showing
Court is not convinced that the implementation
that the implementation of the Retail Trade
of R.A. 8762 would eventually lead to alien
Liberalization Act prejudices petitioners or
control of the retail trade business. Petitioners
inflicts damages on them, either as taxpayers
have not mustered any concrete and strong
or as legislators. Still the Court will resolve the
argument to support its thesis. The law itself
question they raise since the rule on standing
has provided strict safeguards on foreign
can be relaxed for non-traditional plaintiffs like
participation in that business
ordinary citizens, taxpayers, and legislators
when as in this case the public interest so
requires or the matter is of transcendental
105 Oposa v. Factoran, G.R. No. 101083, July
importance, of overarching significance to
30, 1993
society, or of paramount public interest.
Facts:

Petitioners mainly argue that R.A. 8762


The principal petitioners, all minors duly
violates the mandate of the 1987 Constitution
represented and joined by their respective
for the State to develop a self-reliant and
parents. Impleaded as an additional plaintiff is
independent national economy effectively
Page 65 of 121
403 CASE DIGESTS | CONSTITUTIONAL LAW REVIEW | JUDGE ESTELA ALMA SINGCO | FIRST SEMESTER | 2020-2021

the Philippine Ecological Network, Inc. (PENI),


a domestic, non-stock and non-profit The petitioners can file a class suit because
corporation organized for the purpose of, inter they represent their generation as well as
alia, engaging in concerted action geared for generations yet unborn. Their personality to
the protection of our environment and natural sue in behalf of the succeeding generations
resources. The petitioners alleged the can only be based on the concept of
respondent, Honorable Fulgencio S. Factoran, intergenerational responsibility insofar as the
Jr., then Secretary of the Department of right to a balanced and healthful ecology is
Environment and Natural Resources (DENR), concerned. Such a right, as hereinafter
continued approval of the Timber License expounded, considers the “rhythm and
Agreements (TLAs) to numerous commercial harmony of nature.” Nature means the created
logging companies to cut and deforest the world in its entirety. Such rhythm and
remaining forests of the country. Petitioners harmony indispensably include, inter alia, the
request the defendant, his agents, judicious disposition, utilization, management,
representatives and other persons acting in his renewal and conservation of the country’s
behalf to: forest, mineral, land, waters, fisheries,
wildlife, off-shore areas and other natural
Cancel all existing timber license agreements resources to the end that their exploration,
in the country; development and utilization be equitably
accessible to the present as well as future
Cease and desist from receiving, accepting, generations.
processing, renewing or approving new timber
license agreements. Every generation has a responsibility to the
next to preserve that rhythm and harmony for
Plaintiffs further assert that the adverse and the full enjoyment of a balanced and healthful
detrimental consequences of continued and ecology. Put a little differently, the minors’
deforestation are so capable of unquestionable assertion of their right to a sound environment
demonstration that the same may be constitutes, at the same time, the performance
submitted as a matter of judicial notice. This of their obligation to ensure the protection of
act of defendant constitutes a that right for the generations to come.
misappropriation and/or impairment of the
natural resource property he holds in trust for The Court does not agree with the trial court’s
the benefit of plaintiff minors and succeeding conclusions that the plaintiffs failed to allege
generations. Plaintiff have exhausted all with sufficient definiteness a specific legal right
administrative remedies with the defendant’s involved or a specific legal wrong committed,
office. On March 2, 1990, plaintiffs served and that the complaint is replete with vague
upon defendant a final demand to cancel all assumptions and conclusions based on
logging permits in the country. Defendant, unverified data.
however, fails and refuses to cancel the
existing TLA’s to the continuing serious The complaint focuses on one specific
damage and extreme prejudice of plaintiffs. fundamental legal right — the right to a
balanced and healthful ecology which, for the
Issue: first time in our nation’s constitutional history,
is solemnly incorporated in the fundamental
Whether or not the petitioners have the right law. Section 16, Article II of the 1987
to bring action to the judicial power of the Constitution explicitly provides:
Court.
Sec. 16. The State shall protect and advance
Whether or not the petitioners failed to allege the right of the people to a balanced and
in their complaint a specific legal right violated healthful ecology in accord with the rhythm
by the respondent Secretary for which any and harmony of nature.
relief is provided by law.
This right unites with the right to health which
Whether or not petitioners’ proposition to have is provided for in the preceding section of the
all the TLAs indiscriminately cancelled without same article:
the requisite hearing violates the requirements
of due process. Sec. 15. The State shall protect and promote
the right to health of the people and instill
Ruling: health consciousness among them.

In the resolution of the case, the Court held While the right to a balanced and healthful
that: ecology is to be found under the Declaration of
Principles and State Policies and not under the
The petitioners have the right to bring action Bill of Rights, it does not follow that it is less
to the judicial power of the Court.The case at important than any of the civil and political
bar is subject to judicial review by the Court. rights enumerated in the latter. Such a right
Justice Davide, Jr. precisely identified in his belongs to a different category of rights
opinion the requisites for a case to be altogether for it concerns nothing less than
subjected for the judicial review by the Court. self-preservation and self-perpetuation —
According to him, the subject matter of the aptly and fittingly stressed by the petitioners
complaint is of common interest, making this — the advancement of which may even be said
civil case a class suit and proving the existence to predate all governments and constitutions.
of an actual controversy. He strengthens this As a matter of fact, these basic rights need not
conclusion by citing in the decision Section 1, even be written in the Constitution for they are
Article 7 of the 1987 Constitution. assumed to exist from the inception of
humankind. The Court are not persuaded by
the trial court’s pronouncement. The

Page 66 of 121
403 CASE DIGESTS | CONSTITUTIONAL LAW REVIEW | JUDGE ESTELA ALMA SINGCO | FIRST SEMESTER | 2020-2021

respondent Secretary did not invoke in his section of the RH-IRR allows “contraceptives”
motion to dismiss the non-impairment clause. and recognizes as “abortifacient” only those
If he had done so, Justice Feliciano would have that primarily induce abortion or the
acted with utmost infidelity to the Government destruction of a fetus inside the mother's
by providing undue and unwarranted benefits womb or the prevention of the fertilized ovum
and advantages to the timber license holders to reach and be implanted in the mother's
because he would have forever bound the womb. This cannot be done. Evidently, with
Government to strictly respect the said the addition of the word “primarily,” in Section
licenses according to their terms and 3.0l(a) and G) of the RH-IRR is indeed ultra
conditions regardless of changes in policy and vires. It contravenes Section 4(a) of the RH
the demands of public interest and welfare. He Law and should, therefore, be declared
was aware that as correctly pointed out by the
petitioners, into every timber license must be invalid.
read Section 20 of the Forestry Reform Code
(P.D. No. 705) which provides that when the Section 15, Article II of the Constitution
national interest so requires, the President provides: The State shall protect and promote
may amend, modify, replace or rescind any the right to health of the people and instill
contract, concession, permit, licenses or any health consciousness among them. Contrary
other form of privilege granted herein. All to the OSG’s position, these provisions are
licenses may thus be revoked or rescinded by self-executing. At this point, the Court is of the
executive action. It is not a contract, property strong view that Congress cannot legislate that
or a property right protested by the due hormonal contraceptives and intra-uterine
process clause of the Constitution. devices are safe and non-abortifacient. The
provision in Section 9 covering the inclusion of
Hence, the instant Petition is hereby hormonal contraceptives, intra-uterine
GRANTED, and the challenged Order of devices, injectables, and other safe, legal,
respondent Judge of 18 July 1991 dismissing non-abortifacient and effective family planning
Civil Case No. 90-777 was set aside. The products and supplies by the National Drug
petitioners amend their complaint to implead Formulary in the EDL is not mandatory. There
as defendants the holders or grantees of the must first be a determination by the FDA that
questioned timber license agreements. they are in fact safe, legal, non-abortifacient
and effective family planning products and
supplies. There can be no predetermination by
106 Imbong v. Executive Secretary, G.R. No. Congress that the gamut of contraceptives are
204819, April 8, 2014 "safe, legal, non-abortifacient and effective"
without the proper scientific examination.
Facts:
Petitioners assailed the constitutionality of the 107 Legaspi v. CSC, G.R. No. L-72119, May 29,
Reproductive Health Law (RH Law), because, 1987
among others, it violates the right to life of the
unborn. Notwithstanding its declared policy Facts:
against abortion, the implementation of the RH The fundamental right of the people to
Law would authorize the purchase of hormonal information on matters of public concern is
contraceptives, intrauterine devices and invoked in this special civil action for
injectables which are abortives, in violation of Mandamus instituted by petitioner Valentin L.
Section 12, Article II of the Constitution which Legaspi against the Civil Service Commission.
guarantees protection of both the life of the The respondent had earlier denied Legaspi's
mother and the life of the unborn from request for information on the civil service
conception eligibilities of certain persons employed as
sanitarians in the Health Department of Cebu
Issue: City. These government employees, Julian
Sibonghanoy and Mariano Agas, had allegedly
Whether or RH Law violates the right to life of represented themselves as civil service
the unborn. (NO) eligibles who passed the civil service
examinations for sanitarians. Claiming that his
Ruling: right to be informed of the eligibilities of Julian
Sibonghanoy and Mariano Agas is guaranteed
The Framers of the Constitution did not intend by the Constitution, and that he has no other
to ban all contraceptives for being plain, speedy and adequate remedy to acquire
unconstitutional. Contraceptives that kill or the information, petitioner prays for the
destroy the fertilized ovum should be deemed issuance of the extraordinary writ of
an abortive and thus prohibited. Conversely, Mandamus to compel the respondent
contraceptives that actually prevent the union Commission to disclose said information.
of the male sperm and the female ovum, and
those that similarly take action prior to Issue:
fertilization should be deemed nonabortive,
and thus, constitutionally permissible. The Whether Writ of Mandamus will lie
clear and unequivocal intent of the Framers of
the 1987 Constitution in protecting the life of Ruling:
the unborn from conception was to prevent the
Legislature from enacting a measure legalizing To be given due course, a Petition for
abortion. A reading of the RH Law would show Mandamus must have been instituted by a
that it is in line with this intent and actually party aggrieved by the alleged inaction of any
proscribes abortion. While the Court has opted tribunal, corporation, board or person which
not to make any determination, at this stage, unlawfully excludes said party from the
when life begins, it finds that the RH Law itself enjoyment of a legal right. The petitioner in
clearly mandates that protection be afforded every case must therefore be an "aggrieved
from the moment of fertilization. However, the party" in the sense that he possesses a clear
Page 67 of 121
403 CASE DIGESTS | CONSTITUTIONAL LAW REVIEW | JUDGE ESTELA ALMA SINGCO | FIRST SEMESTER | 2020-2021

legal right to be enforced and a direct interest directly affect their lives, or simply because
in the duty or act to be performed. In the case such matters naturally arouse the interest of
before Us, the respondent takes issue on the an ordinary citizen. In the final analysis, it is
personality of the petitioner to bring this suit. for the courts to determine in a case by case
It is asserted that, the instant Petition is bereft basis whether the matter at issue is of interest
of any allegation of Legaspi's actual interest in or importance, as it relates to or affects the
the civil service eligibilities of Julian public. The information sought by the
Sibonghanoy and Mariano Agas. At most there petitioner in this case is the truth of the claim
is a vague reference to an unnamed client in of certain government employees that they are
whose behalf he had allegedly acted when he civil service eligibles for the positions to which
made inquiries on the subject but what is clear they were appointed concern of citizens to
upon the face of the Petition is that the ensure that government positions requiring
petitioner has firmly anchored his case upon civil service eligibility are occupied only by
the right of the people to information on persons who are eligibles. Public officers are at
matters of public concern, which, by its very all times accountable to the people even as to
nature, is a public right. It has been held thatil their eligibilities for their respective positions.
when the question is one of public right and But then, it is not enough that the information
the object of the mandamus is to procure the sought is of public interest. For Mandamus to
enforcement of a public duty, the people are lie in a given case, the information must not
regarded as the real party in interest and the be among the species exempted by law from
relator at whose instigation the proceedings the operation of the constitutional guarantee.
are instituted need not show that he has any
legal or special interest in the result, it being In the instant, case while refusing to confirm
sufficient to show that he is a citizen and as or deny the claims of eligibility, the respondent
such interested in the execution of the laws. has failed to cite any provision in the Civil
Service Law which would limit the petitioner's
From the foregoing, it becomes apparent that right to know who are, and who are not, civil
when a Mandamus proceeding involves the service eligibles. We take judicial notice of the
assertion of a public right, the requirement of fact that the names of those who pass the civil
personal interest is satisfied by the mere fact service examinations, as in bar examinations
that the petitioner is a citizen, and therefore, and licensure examinations for various
part of the general "public" which possesses professions, are released to the public. Hence,
the right. "Public" is a comprehensive, all- there is nothing secret about one's civil service
inclusive term. Properly construed, it eligibility, if actually possessed. Petitioner's
embraces every person. To say that only those request is, therefore, neither unusual nor
who have a present and existing interest of a unreasonable. And when, as in this case, the
pecuniary character in the particular government employees concerned claim to be
information sought are given the right of civil service eligibles, the public, through any
inspection is to make an unwarranted citizen, has a right to verify their professed
distinction The petitioner, being a citizen who, eligibilities from the Civil Service Commission.
as such is clothed with personality to seek The civil service eligibility of a sanitarian being
redress for the alleged obstruction of the of public concern, and in the absence of
exercise of the public right. express limitations under the law upon access
to the register of civil service eligibles for said
We find no cogent reason to deny his standing position, the duty of the respondent
to bring the present suit. The incorporation in Commission to confirm or deny the civil service
the Constitution of a guarantee of access to eligibility of any person occupying the position
information of public concern is a recognition becomes imperative. Mandamus, therefore
of the essentiality of the free flow of ideas and lies.
information in a democracy But the
constitutional guarantee to information on
matters of public concern is not absolute. It 108 Manila Prince Hotel v. GSIS, G.R. No.
does not open every door to any and all 122156, February 3, 1997.
information. Under the Constitution, access to
official records, papers, etc., are "subject to Facts:
limitations as may be provided by law" (Art. GSIS, pursuant to the privatization program of
III, Sec. 7, second sentence). The law may the Philippine Government decided to sell
therefore exempt certain types of information through public bidding issued and outstanding
from public scrutiny, such as those affecting shares of respondent Manila Hotel Corporation
national security of the desired information. (MHC). Two bidders participated: Manila Prince
However, as already discussed, this does not Hotel Corporation, a Filipino corporation, which
give the agency concerned any discretion to offered to buy the shares at P41.58 per share,
grant or deny access. In case of denial of and Renong Berhad, a Malaysian firm, which
access, the government agency has the bid for the same number of shares at P44.00
burden of showing that the information per share. Pending the declaration of Renong
requested is not of public concern, or, if it is of Berhard as the winning bidder/strategic
public concern, that the same has been partner and the execution of the necessary
exempted by law from the operation of the contracts, Manila Prince matched the bid price
guarantee. To hold otherwise will serve to of P44.00 per share. Perhaps apprehensive
dilute the constitutional right. that GSIS has disregarded the tender of the
matching bid, Manila Prince came to the
In determining whether or not a particular Supreme Court on prohibition and mandamus.
information is of public concern there is no
rigid test which can be applied. "Public
concern" like "public interest" is a term that Issue:
eludes exact definition. Both terms embrace a
broad spectrum of subjects which the public
may want to know, either because these
Page 68 of 121
403 CASE DIGESTS | CONSTITUTIONAL LAW REVIEW | JUDGE ESTELA ALMA SINGCO | FIRST SEMESTER | 2020-2021

member-countries on the same footing as


Whether GSIS is mandated to abide the Filipinos and local products” and (2) that the
dictates of the Constitution on National WTO “intrudes, limits and/or impairs” the
Economy and Patrimony. constitutional powers of both Congress and the
Supreme Court.

Ruling: Issue:
Whether provisions of the Agreement
YES. It should be stressed that while the Establishing the World Trade Organization
Malaysian firm offered the higher bid it is not unduly limit, restrict and impair Philippine
yet the winning bidder. The bidding rules sovereignty specifically the legislative power
expressly provide that the highest bidder shall which, under Sec. 2, Article VI, 1987 Philippine
only be declared the winning bidder after it has Constitution is ‘vested in the Congress of the
negotiated and executed the necessary Philippines
contracts, and secured the requisite approvals.
Since the Filipino First Policy provision of the
Constitution bestows preference on qualified Ruling:
Filipinos the mere tending of the highest bid is
not an assurance that the highest bidder will No, the WTO agreement does not unduly limit,
be declared the winning bidder. Resultantly, restrict, and impair the Philippine sovereignty,
respondents are not bound to make the award particularly the legislative power granted by
yet, nor are they under obligation to enter into the Philippine Constitution. The Senate was
one with the highest bidder. For in choosing acting in the proper manner when it concurred
the awardee, respondents are mandated to with the President’s ratification of the
abide by the dictates of the 1987 Constitution agreement.
the provisions of which are presumed to be
known to all the bidders and other interested While sovereignty has traditionally been
parties. deemed absolute and all-encompassing on the
domestic level, it is however subject to
Adhering to the doctrine of constitutional restrictions and limitations voluntarily agreed
supremacy, the subject constitutional to by the Philippines, expressly or impliedly, as
provision is, as it should be, impliedly written a member of the family of nations.
in the bidding rules issued by respondent Unquestionably, the Constitution did not
GSIS, lest the bidding rules be nullified for envision a hermit-type isolation of the country
being violative of the Constitution. It is a basic from the rest of the world. In its Declaration of
principle in constitutional law that all laws and Principles and State Policies, the Constitution
contracts must conform with the fundamental “adopts the generally accepted principles of
law of the land. Those which violate the international law as part of the law of the land,
Constitution lose their reason for being. and adheres to the policy of peace, equality,
justice, freedom, cooperation and amity, with
Certainly, the constitutional mandate itself is all nations.”
reason enough not to award the block of
shares immediately to the foreign bidder By the doctrine of incorporation, the country is
notwithstanding its submission of a higher, or bound by generally accepted principles of
even the highest, bid. In fact, we cannot international law, which are considered to be
conceive of a stronger reason than the automatically part of our own laws. One of the
constitutional injunction itself. oldest and most fundamental rules in
international law is pacta sunt servanda —
international agreements must be performed
in good faith. “A treaty engagement is not a
INCORPORATION CLAUSE mere moral obligation but creates a legally
109 Tanada vs. Angara, May 2, 1997 binding obligation on the parties x x x. A state
which has contracted valid international
Facts: obligations is bound to make in its legislations
such modifications as may be necessary to
Petitioners prayed for the nullification, on ensure the fulfillment of the obligations
constitutional grounds, of the concurrence of undertaken.”
the Philippine Senate in the ratification by the
President of the Philippines of the Agreement By their inherent nature, treaties really limit or
Establishing the World Trade Organization restrict the absoluteness of sovereignty. By
(WTO Agreement, for brevity) and for the their voluntary act, nations may surrender
prohibition of its implementation and some aspects of their state power in exchange
enforcement through the release and for greater benefits granted by or derived from
utilization of public funds, the assignment of a convention or pact. After all, states, like
public officials and employees, as well as the individuals, live with coequals, and in pursuit
use of government properties and resources of mutually covenanted objectives and
by respondent-heads of various executive benefits, they also commonly agree to limit the
offices concerned therewith. exercise of their otherwise absolute rights.
Thus, treaties have been used to record
They contended that WTO agreement violates agreements between States concerning such
the mandate of the 1987 Constitution to widely diverse matters as, for example, the
“develop a self-reliant and independent lease of naval bases, the sale or cession of
national economy effectively controlled by territory, the termination of war, the
Filipinos x x x (to) give preference to qualified regulation of conduct of hostilities, the
Filipinos (and to) promote the preferential use formation of alliances, the regulation of
of Filipino labor, domestic materials and locally commercial relations, the settling of claims,
produced goods” as (1) the WTO requires the the laying down of rules governing conduct in
Philippines “to place nationals and products of peace and the establishment of international
Page 69 of 121
403 CASE DIGESTS | CONSTITUTIONAL LAW REVIEW | JUDGE ESTELA ALMA SINGCO | FIRST SEMESTER | 2020-2021

organizations. The sovereignty of a state It should be emphasized that under our


therefore cannot in fact and in reality be Constitution, the power to ratify is vested in
considered absolute. Certain restrictions enter the President, subject to the concurrence of
into the picture: (1) limitations imposed by the the Senate. The role of the Senate, however,
very nature of membership in the family of is limited only to giving or withholding its
nations and (2) limitations imposed by treaty consent, or concurrence, to the ratification.
stipulations. As aptly put by John F. Kennedy, Hence, it is within the authority of the
“Today, no nation can build its destiny alone. President to refuse to submit a treaty to the
The age of self-sufficient nationalism is over. Senate or, having secured its consent for its
The age of interdependence is here.” ratification, refuse to ratify it.|||
111 Mejoff vs. Director of Prisons G.R. No. L-
The WTO reliance on “most favored nation,”
4254 September 26, 1951
“national treatment,” and “trade without
discrimination” cannot be struck down as
Facts.
unconstitutional as in fact they are rules of
The petitioner Boris Mejoff is an alien of
equality and reciprocity that apply to all WTO
Russian descent who was brought to the
members. Aside from envisioning a trade
Philippines as a secret operative by the
policy based on “equality and reciprocity,” the
Japanese forces during the latter's regime in
fundamental law encourages industries that
these Islands. Upon liberation he was arrested
are “competitive in both domestic and foreign
as a Japanese spy. Later he was handed to the
markets,” thereby demonstrating a clear policy
Commonwealth Government for disposition.
against a sheltered domestic trade
Thereafter the People's Court ordered his
environment, but one in favor of the gradual
release. But the Deportation Board taking his
development of robust industries that can
case up, found that having no travel
compete with the best in the foreign markets.
documents Mejoff was illegally in this country,
Indeed, Filipino managers and Filipino
and consequently referred the matter to the
enterprises have shown capability and tenacity
immigration authorities. Years later however
to compete internationally. And given a free
he was not yet deported because no ship or
trade environment, Filipino entrepreneurs and
country would take the petitioner.|
managers in Hongkong have demonstrated the
Filipino capacity to grow and to prosper against
Issue.
the best offered under a policy of laissez faire.
Whether or not Mejoff, a foreign national who
WHEREFORE, the petition is DISMISSED for
is not an enemy, may be indefinitely kept in
lack of merit.
detention.

Ruling.
110 Pimentel vs. Ermita, 462 SCRA 622, July
No.
6, 2005
By its Constitution (Art. II, Sec. 3) the
Facts.
Philippines "adopts the generally accepted
The petitioners filed the case to compel the
principles of international law as part of the law
respondents to transmit the signed text of the
of Nation." And in a resolution entitled
Rome Statute to the Senate for ratification.
"Universal Declaration Of Human Rights" and
The Rome Statute established the
approved by the General Assembly of the
International Criminal Court which "shall have
United Nations of which the Philippines is a
the power to exercise its jurisdiction over
member, at its plenary meeting on December
persons for the most serious crimes of
10, 1948, the right to life and liberty and all
international concern . . . and shall be
other fundamental rights as applied to all
complementary to the national criminal
human beings were proclaimed. These rights
jurisdictions." Its jurisdiction covers the crime
are available to him.
of genocide, crimes against humanity, war
crimes and the crime of aggression as defined
112 Kuroda vs. Jalandoni G.R. No. L-2662
in the Statute.
March 26, 1949
Issue.
Facts.
Whether or not the respondents may be Petitioner Shigenori Kuroda was charged
compelled to transmit the case to the Senate before the Philippine Military Commission of
for ratification war crimes. He questioned the
Ruling. constitutionality of E.O. No. 68 that created
the National War Crimes Office and prescribed
No. There is no legal obligation to ratify a rules on the trial of accused war criminals. He
treaty. contended the Philippines is not a signatory to
the Hague Convention on Rules and
The signing of the treaty and the ratification
Regulations covering Land Warfare and
are two separate and distinct steps in the
therefore he is charged of crimes not based on
treaty-making process. As earlier discussed,
law, national and international.
the signature is primarily intended as a means
of authenticating the instrument and as a
Issue.
symbol of the good faith of the parties. It is
Whether or not EO 68 is valid and
usually performed by the state's authorized
constitutional.
representative in the diplomatic mission.
Ratification, on the other hand, is the formal
Ruling.
act by which a state confirms and accepts the
YES, E.O. No. 68 valid and constitutional.
provisions of a treaty concluded by its
representative. It is generally held to be an
Article 2 of our Constitution provides in its
executive act, undertaken by the head of the
section 3, that –
state or of the government.
The Philippines renounces war as an
instrument of national policy and adopts the
Page 70 of 121
403 CASE DIGESTS | CONSTITUTIONAL LAW REVIEW | JUDGE ESTELA ALMA SINGCO | FIRST SEMESTER | 2020-2021

generally accepted principles of international necessity). Implicit in the latter element is a


law as part of the law of the nation. belief that the practice in question is rendered
obligatory by the existence of a rule of law
In accordance with the generally accepted requiring it.
principle of international law -- Hague
Convention, the Geneva Convention and There is a widespread practice among states
significant precedents of international accepting in principle the need for such
jurisprudence- all those person military or recognition and enforcement, albeit subject to
civilian who have been guilty of planning limitations of varying degrees. The fact that
preparing or waging a war of aggression and there is no binding universal treaty governing
of the commission of crimes and offenses the practice is not indicative of a widespread
consequential and incidental thereto in rejection of the principle, but only a
violation of the laws and customs of war, of disagreement as to the imposable specific
humanity and civilization are held accountable rules governing the procedure for recognition
therefor. Consequently in the promulgation and enforcement.||| Aside from the
and enforcement of EO No. 68 the President of widespread practice, it is indubitable that the
the Philippines has acted in conformity with the procedure for recognition and enforcement is
generally accepted and policies of international embodied in the rules of law, whether
law which are part of our Constitution. statutory or jurisprudential, adopted in various
foreign jurisdictions. In the Philippines, this is
Petitioner argues that respondent Military evidenced primarily by Section 48, Rule 39 of
Commission has no jurisdiction to try the Rules of Court which has existed in its
petitioner for acts committed in violation of the current form since the early 1900s.
Hague Convention and the Geneva Convention
because the Philippines is not a signatory to There may be distinctions as to the rules
the first and signed the second only in 1947. adopted by each particular state, but they all
Yet, these rules and principles were accepted prescind from the premise that there is a rule
by the two belligerent nations the United State of law obliging states to allow for, however
and Japan who were signatories to the two generally, the recognition and enforcement of
Convention. Such rule and principles therefore a foreign judgment. The bare principle, to our
form part of the law of our nation even if the mind, has attained the status of opinio juris in
Philippines was not a signatory to the international practice.
conventions embodying them since the
Constitution has been deliberately general and Rules of procedure are promulgated by the
extensive in its scope and is not confined to Supreme Court, and could very well be
the recognition of rule and principle of abrogated or revised by the high court itself.
international law as contained in treaties to Yet the Supreme Court is obliged, as are all
which our government may have been or shall State components, to obey the laws of the
be a signatory. land, including generally accepted principles of
international law which form part thereof, such
113 Mijares, et al. vs. Javier, et al., April 12, as those ensuring the qualified recognition and
2005 enforcement of foreign judgments.

Facts. 114 Llamanzares vs. COMELEC, G.R. No.


Invoking the Alien Tort Act, petitioners 221697, March 8, 2016
Mijares, et al.*, all of whom suffered human
rights violations during the Marcos era, Facts.
obtained a Final Judgment in their favor Petitioner Mary Grace Natividad S. Poe-
against the Estate of the late Ferdinand Marcos Llamanzares was found abandoned as a
amounting to roughly 1.9 Billion U.S. Dollars newborn infant in the Parish Church of Jaro,
in compensatory and exemplary damages for Iloilo on Sept. 3, 1968. After passing the
tortuous violations of international law in the parental care and custody over petitioner by
US District Court of Hawaii. This Final Edgardo Militar to Emiliano Militar and his wife,
Judgment was affirmed by the US Court of she has been reported and registered as a
Appeals. foundling and issued a Foundling Certificate
and Certificate of Live Birth, thus was given the
Issue. name, Mary Grace Natividad Contreras Militar.
Whether or not such foreign judgment may be She was then adopted by Fernando Poe Jr. and
enforced under the law Susan Roces.

Ruling. Issue.
Yes. Whether or not she is considered as a natural
born-citizen of the Philippines, pursuant to
There is no obligatory rule derived from international law.
treaties or conventions that requires the
Philippines to recognize foreign judgments, or Ruling.
allow a procedure for the enforcement thereof. Yes.
However, generally accepted principles of
international law, by virtue of the Foundlings are citizens under international
incorporation clause of the Constitution, form law. Under the 1987 Constitution, an
part of the laws of the land even if they do not international law can become part of the
derive from treaty obligations. The classical sphere of domestic law either by
formulation in international law sees those transformation or incorporation. The
customary rules accepted as binding result transformation method requires that an
from the combination two elements: the international law be transformed into a
established, widespread, and consistent domestic law through a constitutional
practice on the part of States; and a mechanism such as local legislation. On the
psychological element known as the opinion other hand, generally accepted principles of
juris sive necessitates (opinion as to law or international law, by virtue of the
Page 71 of 121
403 CASE DIGESTS | CONSTITUTIONAL LAW REVIEW | JUDGE ESTELA ALMA SINGCO | FIRST SEMESTER | 2020-2021

incorporation clause of the Constitution, form aerial domain and the atmospheric space
part of the laws of the land even if they do not above it, embracing the Mindanao-Sulu-
derive from treaty obligations. Generally Palawan geographic region. With regard to
accepted principles of international law include governance, on the other hand, a shared
international custom as evidence of a general responsibility and authority between the
practice accepted as law, and general Central Government and BJE was provided.
principles of law recognized by civilized The relationship was described as
nations. International customary rules are ―associative. With the formulation of the
accepted as binding as a result from the MOA-AD, petitioners aver that the negotiation
combination of two elements: the established, and finalization of the MOA-AD violates
widespread, and consistent practice on the constitutional and statutory provisions on
part of States; and a psychological element public consultation, as mandated by Executive
known as the opinion juris sive necessitates Order No. 3, and right to information. They
(opinion as to law or necessity). Implicit in the further contend that it violates the Constitution
latter element is a belief that the practice in and laws. Hence, the filing of the petition.
question is rendered obligatory by the
existence of a rule of law requiring it. Issue.
Whether or not the right to self-determination
The common thread of the UDHR, UNCRC and is a generally accepted principle of
ICCPR is to obligate the Philippines to grant international law.
nationality from birth and ensure that no child
is stateless. Ruling.
Yes.
The principles found in two conventions, while International law has long recognized the right
yet unratified by the Philippines, are generally to self-determination of "peoples", understood
accepted principles of international law. not merely as the entire population of a State
but also a portion thereof. "the right of a
That the Philippines is not a party to the 1930 people to self-determination is now so widely
Hague Convention nor to the 1961 Convention recognized in international conventions that
on the Reduction of Statelessness does not the principle has acquired a status beyond
mean that their principles are not binding. 'convention' and is considered a general
While the Philippines is not a party to the 1930 principle of international law".
Hague Convention, it is a signatory to the
Universal Declaration on Human Rights, Article Among the conventions referred to are the
15 (1) of which effectively affirms Article 14 of International Covenant on Civil and Political
the 1930 Hague Convention. Article 2 of the Rights 161 and the International Covenant on
1961 "United Nations Convention on the Economic, Social and Cultural Rights 162
Reduction of Statelessness" merely "gives which state, in Article 1 of both covenants, that
effect" to Article 15 (1) of the UDHR.| all peoples, by virtue of the right of self-
|| determination, "freely determine their political
115 The Province of North Cotabato v. The status and freely pursue their economic,
Government of the Republic of the social, and cultural development".
Philippines Peace Panel, GR No. 183591,
The people's right to self-determination should
October 14, 2008
not, however, be understood as extending to a
unilateral right of secession. The UN General
Facts.
Assembly adopted the United Nations
President Gloria Macapagal-Arroyo, in line with
Declaration on the Rights of Indigenous
the government‘s policy of pursuing peace
Peoples (UN DRIP) through General Assembly
negotiations with the Moro Islamic Liberation
Resolution 61/295. The vote was 143 to 4, the
Front (MILF), asked Prime Minister Mahathir
Philippines being included among those in
Mohammad to convince the MILF to continue
favor. The Declaration clearly recognized the
negotiating with the government. MILF,
right of indigenous peoples to self-
thereafter, convened its Central Committee
determination, encompassing the right to
and decided to meet with the Government of
autonomy or self-government.
the Republic of the Philippines (GRP). Formal
peace talks were held in Libya which resulted Assuming that the UN DRIP, like the Universal
to the crafting of the GRP-MILF Tripoli Declaration on Human Rights, must now be
Agreement on Peace (Tripoli Agreement 2001) regarded as embodying customary
which consists of three (3) aspects: a.) international law, the obligations enumerated
security aspect; b.) rehabilitation aspect; and therein do not strictly require the Republic to
c.) ancestral domain aspect. Various grant the Bangsamoro people, through the
negotiations were held which led to the instrumentality of the BJE, the particular rights
finalization of the Memorandum of Agreement and powers provided for in the MOA-AD. Even
on the Ancestral Domain (MOA-AD). The said the more specific provisions of the UN DRIP are
memorandum was set to be signed last August general in scope, allowing for flexibility in its
5, 2008. In its body, it grants ―the authority application by the different States.
and jurisdiction over the Ancestral Domain and
Ancestral Lands of the Bangsamoro to the Moreover, the UN DRIP, while upholding the
Bangsamoro Juridical Entity (BJE). The latter, right of indigenous peoples to autonomy, does
in addition, has the freedom to enter into any not obligate States to grant indigenous
economic cooperation and trade relation with peoples the near-independent status of an
foreign countries. ―The sharing between the associated state.
Central Government and the BJE of total Even if the UN DRIP were considered as part
production pertaining to natural resources is to of the law of the land pursuant to Article II,
be 75:25 in favor of the BJE. The MOA-AD Section 2 of the Constitution, it would not
further provides for the extent of the territory suffice to uphold the validity of the MOA-AD so
of the Bangsamoro. It describes it as ―the as to render its compliance with other laws
land mass as well as the maritime, terrestrial, unnecessary.
fluvial and alluvial domains, including the
Page 72 of 121
403 CASE DIGESTS | CONSTITUTIONAL LAW REVIEW | JUDGE ESTELA ALMA SINGCO | FIRST SEMESTER | 2020-2021

It is, therefore, clear that the MOA-AD on September 11, 2001. Arthur D. Lim and
contains numerous provisions that cannot be Paulino P. Ersando as citizens, lawyers and
reconciled with the Constitution and the laws taxpayers filed a petition for certiorari and
as presently worded. prohibition attacking the constitutionality of
the joint exercise. Partylists Sanlakas and
Partido Ng Manggagawa as residents of
116 Ang Ladlad LGBT Party v. COMELEC, GR Zamboanga and Sulu directly affected by the
No.190582, April 8, 2010 operations filed a petition-in-intervention.
Facts.
Petitioner is a national organization which Issue.
represents the lesbians, gays, bisexuals, and Whether or not the State may take into
trans-genders. It filed a petition for cognizance the validity of the VFA
accreditation as a party-list organization to
public respondent. However, due to moral Ruling.
grounds, the latter denied the said petition. To Yes.
buttress their denial, COMELEC cited certain From the perspective of public international
biblical and quranic passages in their decision. law, a treaty is favored over municipal law
It also stated that since their ways are immoral pursuant to the principle of pacta sunt
and contrary to public policy, they are servanda. Hence, "[e]very treaty in force is
considered nuisance. In fact, their acts are binding upon the parties to it and must be
even punishable under the Revised Penal Code performed by them in good faith." Further, a
in its Article 201. party to a treaty is not allowed to "invoke the
provisions of its internal law as justification for
Issue. its failure to perform a treaty." Our
Whether or not the Yogyakarta Principles (the Constitution espouses the opposing view.
Application of International Human Rights Law Witness our jurisdiction as stated in Section 5
In Relation to Sexual Orientation and Gender of Article VIII, In Ichong v. Hernandez, we
Identity) are considered as generally accepted ruled that the provisions of a treaty are always
principles of international law. subject to qualification or amendment by a
subsequent law, or that it is subject to the
Ruling. police power of the State. The foregoing
No. premises leave us no doubt that US forces are
The court is not prepared to declare that these prohibited from engaging in an offensive war
Yogyakarta Principles contain norms that are on Philippine territory.
obligatory on the Philippines. There are 118 Ichong vs. Hernandez, 101 Phil. 1155 [1957]
declarations and obligations outlined in said
Principles which are not reflective of the Facts.
current state of international law, and do not Petitioner Inchong, for and in his own behalf
find basis in any of the sources of international and on behalf of other alien resident
law enumerated under Article 38 (1) of the corporations and partnerships adversely
Statute of the International Court of Justice. affected by the provisions of Republic Act. No.
1180, brought this action to obtain a judicial
These Yogyakarta Principles, consisting of a declaration that said Act is unconstitutional,
declaration formulated by various international and to enjoin the Secretary of Finance and all
law professors, are — at best — de lege other persons acting under him, particularly
ferenda — and do not constitute binding city and municipal treasurers, from enforcing
obligations on the Philippines. Indeed, so much its provisions. Inchong attacks the
of contemporary international law is constitutionality of the Act, contending that:
characterized by the "soft law" nomenclature, (1) it denies to alien residents the equal
i.e., international law is full of principles that protection of the laws and deprives of their
promote international cooperation, harmony, liberty and property without due process of law
and respect for human rights, most of which ; (2) the subject of the Act is not expressed or
amount to no more than well-meaning desires, comprehended in the title thereof; (3) the Act
without the support of either State practice or violates international and treaty obligations of
opinio juris. the Republic of the Philippines; (4) the
provisions of the Act against the transmission
by aliens of their retail business thru
hereditary succession, and those requiring
117 Lim vs. Exec. Sec., April 11, 2002 100% Filipino capitalization for a corporation
or entity to entitle it to engage in the retail
Facts. business, violate the spirit of Sections 1 and 5,
Pursuant to the Visiting Forces Agreement Article XIII and Section 8 of Article XIV of the
(VFA) signed in 1999, personnel from the Constitution.
armed forces of the United States of America
started arriving in Mindanao to take partin Issue.
"Balikatan 02-1” on January 2002. The Whether the RA No. 1180 violates international
Balikatan 02-1 exercises involves the law
simulation of joint military maneuvers
pursuant to the Mutual Defense Treaty, a Ruling.
bilateral defense agreement entered into by No.
the Philippines and the United States in 1951.
The exercise is rooted from the international The law does not violate international treaties
anti-terrorism campaign declared by President and obligations. The United Nations Charter
George W. Bush in reaction to the 3 imposes no strict or legal obligations regarding
commercial aircrafts hijacking that smashed the rights and freedom of their subjects (Jans
into twin towers of the World Trade Center in Kelsen, The Law of the United Nations, 1951
New York City and the Pentagon building in ed., pp. 29-32), and the Declaration of Human
Washington, D.C. allegedly by the al-Qaeda Rights contains nothing more than a mere
headed by the Osama bin Laden that occurred recommendation, or a common standard of
Page 73 of 121
403 CASE DIGESTS | CONSTITUTIONAL LAW REVIEW | JUDGE ESTELA ALMA SINGCO | FIRST SEMESTER | 2020-2021

achievement for all peoples and all nations. private respondents due process rights,
The Treaty of Amity between the Republic of although not guaranteed by statute or by
the Philippines and the Republic of China of treaty, are protected by constitutional
April 18, 1947 guarantees equality of guarantees. We would not be true to the
treatment to the Chinese nationals "upon the organic law of the land if we choose strict
same terms as the nationals of any other construction over guarantees against the
country". But the nationals of China are not deprivation of liberty. That would not be in
discriminated against because nationals of all keeping with the principles of democracy on
other countries, except those of the United which our Constitution is premised.
States, who are granted special rights by the
Constitution, are all Prohibited from engaging
in the retail trade. But even supposing that the
CIVILIAN SUPREMACY OVER THE
law infringes upon the said treaty, the treaty
MILITARY
is always subject to qualification or
120 People v. Lagman (G.R. No. L-45893, July
amendment by a subsequent law (U.S. vs.
13, 1938)
Thompson, 258, Fed. 257, 260), and the same
FACTS:
may never curtail or restrict the scope of the
Appellants Tranquilino Lagman and Primitivo
police power of the State (Palston vs.
de Sosa are charged with a violation of section
Pennsylvania 58 L. ed., 539).
60 of Commonwealth Act No. 1, known as the
National Defense Law. It is alleged that these
119 Sec. of Justice vs. Lantion G.R. No.
two appellants, being Filipinos and having
139465 January 18, 2000
reached the age of twenty years in 1936,
willfully and unlawfully refused to register in
FACTS:
the military service between the 1st and 7th of
The Department of Justice received from the
April of said year, even though they had been
Department of Foreign Affairs a request from
required to do so. The two appellants were
the United States for the extradition of Mark
duly notified to appear before the Acceptance
Jimenez to the United States pursuant to PD
Board in order to register for military service
No. 1609 prescribing the procedure for
but still did not register up to the date of the
extradition of persons who have committed a
filing of the information. Appellants argue that
crime in a foreign country. Jimenez requested
they did not register because de Sosa is
for copies of the request and that he be given
fatherless and has a mother and a brother
ample time to comment on the said request.
eight years old to support, and Lagman also
The petitioners denied the request pursuant to
has a father to support, has no military
the RP-US Extradition Treaty.
learnings, and does not wish to kill or be killed.
The Court of First Instance sentenced them
ISSUE:
both to one month and one day of
Whether or not treaty stipulations must take
imprisonment, with the costs.
precedence over an individual’s due process
rights
ISSUE:
WON the National Defense Law was
HELD:
constitutional by virtue of Section 2, Article II
No. The human rights of person and the rights
of the Constitution
of the accused guaranteed in the Constitution
should take precedence over treaty rights
claimed by a contracting party, the doctrine of
incorporation is applied whenever municipal
HELD:
tribunals are confronted with a situation where
Yes. Decision of CFI affirmed. The National
there is a conflict between a rule of the
Defense Law, in so far as it establishes
international law and the constitution. Efforts
compulsory military service, does not go
must first be made in order to harmonize the
against this constitutional provision but is, on
provisions so as to give effect to both but, if
the contrary, in faithful compliance therewith.
the conflict is irreconcilable, the municipal law
The duty of the Government to defend the
must be upheld. The fact that international law
State cannot be performed except through an
has been made part of the law of the land does
army. To leave the organization of an army to
not pertain to or imply the primacy of
the will of the citizens would be to make this
international law over municipal law in the
duty of the Government excusable should
municipal sphere. In states where the
there be no sufficient men who volunteer to
constitution is the highest law of the land, both
enlist therein. The right of the government to
statutes and treaties may be invalidated if they
require compulsory military services is a
are in conflict with the constitution.
consequence of its duty to defend the state
and is reciprocal with its duty to defend the
In the case at bar, private respondent does not
life, liberty, and property of its citizens.
only face a clear and present danger of loss of
Republic Act No. 9163 establishing a National
property or employment but of liberty itself,
Service Training Program amends RA 7077
which may eventually lead to his forcible
and institutionalizes citizen training to
banishment to a foreign land. The convergence
encompass not just military preparedness but
of petitioners favorable action on the
also literacy and civic welfare.
extradition request and the deprivation of
private respondents liberty is easily
comprehensible. In US cases, it was stated that the right of the
Government to require compulsory military
We have ruled time and again that this Courts service is a consequence of its duty to defend
equity jurisdiction, which is aptly described as the State; and, that a person may be
"justice outside legality," may be availed of compelled by force to take his place in the
only in the absence of, and never against, ranks of the army of his country, and risk the
statutory law or judicial pronouncements. The chance of being shot down in its defense. What
constitutional issue in the case at bar does not justifies compulsory military service is the
even call for "justice outside legality," since defense of the State, whether actual or
Page 74 of 121
403 CASE DIGESTS | CONSTITUTIONAL LAW REVIEW | JUDGE ESTELA ALMA SINGCO | FIRST SEMESTER | 2020-2021

whether in preparation to make it more Whether or not habeas corpus is not the proper
effective, in case of need. The circumstances mode to question conditions of confinement
of the appellants do not excuse them from
their duty to present themselves before the
Acceptance Board because they can obtain the HELD:
proper pecuniary allowance to attend to these Article 70 of the Articles of War grants
family responsibilities. discretion to military authorities over the
imposition of arrest or confinement of persons
subject to military law charged with crime or
121 Aquino v. Esperon (G.R. No. 174994, with serious offense.
August 31, 2007)
While it is true that the extraordinary writ of
FACTS: habeas corpus is the appropriate remedy to
Major Aquino, along with several military men, inquire into questions of violations of
allegedly met at the rest house of Captain constitutional right, this Court, however, does
Aldomovar near Camp Tecson, San Miguel, not find the conditions of Major Aquino’s
Bulacan to plot a breach of the Camp Defense confinement to be a proper subject of inquiry
Plan of Camp General Emilio Aguinaldo and to in the instant Petition.
take over Camp Aquinaldo, as well as the
Headquarters of the Philippine Army. In the As a rule, therefore, the writ of habeas corpus
wake of the group’s alleged withdrawal of does not extend into questions of conditions of
support from the AFP chain of command and confinement; but only to the fact and duration
the administration of President Gloria of confinement. The high prerogative writ of
Macapagal-Arroyo, Major Aquino was ordered habeas corpus was devised and exists as a
arrested and confined at the Intelligence speedy and effectual remedy to relieve
Service Group of the Philippine Army in Fort persons from unlawful restraint. Its object is to
Bonifacio, Taguig, upon the order of Lt. Gen. inquire into the legality of one’s detention, and
Hermogenes Esperon, who was then the if found illegal, to order the release of the
Commanding General of the Philippine Army. detainee. It is not a means for the redress of
On the same day, Lt. Gen. Esperon ordered the grievances or to seek injunctive relief or
Army Inspector General to conduct an damages. We reiterate the pronouncement of
investigation on the matter. For this purpose, this Court in Alejano v. Cabuay
a panel of investigators was formed. During
the investigation, Major Aquino denied the The ruling in this case, however, does not
accusations hurled against him. The panel of foreclose the right of detainees and convicted
investigators, found that the troop movement prisoners from petitioning the courts for the
by some military personnel from their redress of grievances. Regulations and
respective stations to Manila was illegal, conditions in detention and prison facilities
implicating Major Aquino. that violate the Constitutional rights of the
detainees and prisoners will be reviewed by
The panel recommended that all implicated the courts on a case-by-case basis. The courts
officers therein mentioned be immediately could afford injunctive relief or damages to the
relieved from their respective posts; and detainees and prisoners subjected to arbitrary
appropriate charges be filed before the and inhumane conditions. However, habeas
General Court Martial against Major Aquino, corpus is not the proper mode to question
among other military officers/personnel, for conditions of confinement. The writ of habeas
violations of Article 67 (Attempting to Begin or corpus will only lie if what is challenged is the
Create Mutiny); and Article 97 (Disorders and fact or duration of confinement.
Neglects Prejudicial to Good Order and Military
Discipline) of the Articles of War. 122 Kapunan v. Quisumbing (G.R. Nos.
148213–17A, March 13, 2009)
The Judge Advocate General’s Office (JAGO)
found the existence of probable cause against FACTS:
Major Aquino, among other military officers. The petitioners, Edgardo Kapunan and Oscar
Lt. Gen. Esperon issued an Order to the Legaspi charges with the killing of KMU
Commanding Officer to exercise custodial Chairman Rolando Olalia and his driver Leonor
responsibility of Major Aquino, together with Alay-ay. On June 1986 Olalia and Alay-ay dead
the other implicated military personnel, and to body was found. The murder case of Olalia is
place them in confinement at the Philippine a controversial case during that time, Oliala is
Army Detention Center, Camp Capinpin, a profile individual being the Chairman of the
Tanay, Rizal. The same Order also designated KMU at the time of his death.
the aforementioned Commanding Officer to
exercise direct supervision and control over On Nov. 1998, private respondents Feliciano
the concerned detainees. Olalia and Perlina Alay-ay, filed a complaint
letter to Department of Justice for the alleged
Petitioner filed a Petition for Habeas Corpus complex kidnapping and killing of Olalia and
with the CA, praying that the AFP Chief of Staff Alay-ay against Edgardo Kapunan and Oscar
and the Commanding General of the Philippine Legaspi and other men and officers of Phil.
Army, or whoever are acting in their place and National Police and the AFP.
stead, be directed to immediately produce the
body of Major Aquino and explain forthwith Sec. Serafin Cuevas, the Secretary of
why he should not be set at liberty without Department of Justice, created a Panel that
delay. The CA denied the petition, ruling that were tasked to conduct a preliminary
the remedy of the writ of habeas corpus is investigation of Olalia case. The petitioner filed
futile because charges had already been a motion to dismiss in Department of Justice
preferred against Major Aquino. Petitioner’s on the ground that the Amnesty granted to
MR was denied as well. them by the National Amnesty Commission
extinguishes their criminal liability under
ISSUE:
Page 75 of 121
403 CASE DIGESTS | CONSTITUTIONAL LAW REVIEW | JUDGE ESTELA ALMA SINGCO | FIRST SEMESTER | 2020-2021

Proclamation 347 issued by Pres. Fidel V. that the evidence of guilt is strong, whether
Ramos entitled, “Granting of Amnesty to the ascertained in a hearing of an application for
rebels, insurgents and all other persons, who bail or imported from a trial court’s judgment
may or may be committed crimes against of conviction, justifies the detention of an
public order and crimes committed in accused as a valid curtailment of his right to
furtherance of political ends. provisional liberty. This accentuates the
The petitioners filed a motion for certiorari. proviso that the denial of the right to bail in
such cases is “regardless of the stage of the
ISSUES: criminal action.” Such justification for
Whether or not the amnesty granted to confinement with its underlying rationale of
Kapunan and Legaspi, extinguishes their public self-defense applies equally to detention
criminal liability in Olalia case? prisoners like Trillanes. The Court in People v.
Hon. Maceda said that all prisoners whether
HELD: under preventive detention or serving final
The Panel created by the Department of sentence can not practice their profession nor
Justice refused to consider the defense of engage in any business or occupation, or hold
Amnesty of the petitioners on the ground that office, elective or appointive, while in
the document presented pertaining to the detention. This is a necessary consequence of
Amnesty failed to show that the Olalia murder arrest and detention.
case was one of the crimes for which the
amnesty was applied for. The Court of Appeals The case against Trillanes is not administrative
also dismissed the petition, finding no grave of in nature. And there is no “prior term” to speak
abuse of discretion on the Panel created by the of.
DOJ, The Appellate Court refused to rule on the
applicability of Amnesty issued to Kapunan In a plethora of cases, the Court categorically
and Legapi. held that the doctrine of condonation does not
Finally the Supreme court dismissed the apply to criminal cases. Election, or more
petition for certiorari on the ground that the precisely, re-election to office, does not
Amnesty granted to Kapunan and Legaspi obliterate a criminal charge. The performance
pertains only to the crimes against rebellion of legitimate and even essential duties by
and not covered crime of murder Olalia and public officers has never been an excuse to
Alay-ay case. free a person validly in prison.
General grant of amnesty exonerates the
accused (former military personnel) for their
participation in the 1987 and 1989 coup
124 Gudani and Balutan v. Senga (G.R. No.
attempts, but not for murder (in this case, the
170165, August 15, 2006)
murder of Kilusang Mayo Uno leader Rolando
Olalia and his driver Leonor Alay-ay).
FACTS:
Senator Rodolfo Biazon invited several senior
officers of the AFP, including Gen. Gudani and
123 Trillanes v. Pimentel (G.R. No. 179817, Col. Balutan, to appear at a public hearing
June 27, 2008) before the Senate Committee on National
Defense and Security to shed light on the
FACTS: “Hello Garci” controversy. Gudani and Balutan
On July 27, 2003, more than 300 heavily were directed by AFP Chief of Staff Gen.
armed soldiers led by junior officers of the Senga, per instruction of Pres. Arroyo, not
Armed Forces of the Philippines (AFP) stormed testify before the Committee. On the very day
into the Oakwood Premier Apartments in of the hearing, President Gloria-Macapagal-
Makati City and publicly demanded the Arroyo issued Executive Order No. 464
resignation of the President and key national enjoining officials of the executive department
officials. After a series of negotiations, military including the military establishment from
soldiers surrendered that evening. In the appearing in any legislative inquiry without her
aftermath of such event dubbed as the approval. However, the two testified before
Oakwood Incident, petitioner Antonio F. the Senate, prompting Gen. Senga to issue an
Trillanes IV was charged with coup d’état order directing Gudani and Balutan to appear
before the Regional Trial Court of Makati. Four before the Office of the Provost Marshal
years later, Trillanes remained in detention General for investigation. The following day,
and won a seat in the Senate. Before starting Gen. Gudani was compulsorily retired from
his term, Trillanes filed with RTC an Omnibus military service. After investigation, the OPMG
Motion for Leave of Court to be Allowed to recommended that the two be charged with
Attend Senate Sessions and Related Requests. violation of Article of War 65, on willfully
Trillanes requested to be allowed to attend disobeying a superior officer. Thus, Gudani and
senate sessions and fulfill his functions as Balutan filed a petition for certiorari and
senator. The RTC however denied his motion. prohibition seeking that the order of President
Thus, he filed Petition for Certiorari with the Arroyo be declared unconstitutional; and the
Supreme Court to set aside orders of the RTC. charges against them be quashed; and Gen.
Senga and their successors-in-interest or
ISSUE: persons acting for and on their behalf or
orders, be permanently enjoined from
Whether or not Trillanes‘ election as senator proceeding against them, as a consequence of
provides legal justification to allow him to work their having testified before the Senate.
and serve his mandate as senator.

HELD:

It is uncontroverted that petitioner’s ISSUE:


application for bail and for release on
recognizance was denied. The determination
Page 76 of 121
403 CASE DIGESTS | CONSTITUTIONAL LAW REVIEW | JUDGE ESTELA ALMA SINGCO | FIRST SEMESTER | 2020-2021

May the President prevent a member of the What is in here is mutual support and
armed forces from testifying before a cooperation between the military and the
legislative inquiry? civilian authorities, not a derogation of civilian
supremacy. Wherefore, the petition is hereby
HELD: dismissed.
Yes. The President has constitutional authority
to do so, by virtue of her power as
commander-in-chief, and that as a
consequence a military officer who defies such
SEPARATION OF THE CHURCH AND STATE
an injunction is liable under military justice.
Our ruling that the President could, as a 126 Estrada vs. Escritor, June 22, 2006
general rule, require military officers to seek
presidential approval before appearing before
Congress is based foremost on the notion that FACTS: Escritor is a court interpreter since
a contrary rule unduly diminishes the 1999 in the RTC of Las Pinas City. She has
prerogatives of the President as commander- been living with Quilapio, a man who is not her
in-chief. Congress holds significant control husband, for more than twenty-five years and
over the armed forces in matters such as had a son with him as well. Respondent’s
budget appropriations and the approval of husband died a year before she entered into
higher-rank promotions, yet it is on the the judiciary while Quilapio is still legally
President that the Constitution vests the title married to another woman.
as commander-in-chief and all the
prerogatives and functions pertaining to the
Complainant Estrada requested the Judge of
position. Again, the exigencies of military
said RTC to investigate respondent. According
discipline and the chain of command mandate
to complainant, respondent should not be
that the President's ability to control the
allowed to remain employed therein for it will
individual members of the armed forces be
appear as if the court allows such act.
accorded the utmost respect. Where a military
officer is torn between obeying the President
and obeying the Senate, the Court will without Respondent claims that their conjugal
hesitation affirm that the officer has to choose arrangement is permitted by her religion—the
the President. After all, the Constitution Jehovah’s Witnesses and the Watch Tower and
prescribes that it is the President, and not the the Bible Trace Society. They allegedly have a
Senate, who is the commander-in-chief of the ‘Declaration of Pledging Faithfulness’ under the
armed forces. approval of their congregation. Such a
declaration is effective when legal
125 IBP v. Zamora (G.R. No. 141284, August impediments render it impossible for a couple
15, 2000) to legalize their union.

FACTS: ISSUE:

The President ordered the PNP and the Marines Whether or not the State could penalize
to conduct joint visibility patrols for the respondent for such conjugal arrangement.
purpose of crime prevention and suppression.
On January 17, 2000, the IBP filed the instant HELD:
petition to declare the deployment of
Philippines Marines unconstitutional, thus null No. The State could not penalize respondent
and void alleging that no emergency situation for she is exercising her right to freedom of
would justify the employment of soldiers for religion. The free exercise of religion is
law enforcement work and that the same is in specifically articulated as one of the
derogation of Article II Section 3 of the fundamental rights in our Constitution. The
Constitution. State’s interest in enforcing its prohibition
cannot be merely abstract or symbolic in order
ISSUE: to be sufficiently compelling to outweigh a free
exercise claim. In the case at bar, the State
Whether or not the joint visibility patrols has not evinced any concrete interest in
violate the Constitutional provisions on civilian enforcing the concubinage or bigamy charges
supremacy over the military against respondent or her partner. Thus the
State’s interest only amounts to the symbolic
HELD: preservation of an unenforced prohibition.
Furthermore, a distinction between public and
Civilian authority (Section 3, Article II) is not secular morality and religious morality should
defeated in a joint task force between the PNP be kept in mind. The jurisdiction of the Court
and the Philippine Marines for the enforcement extends only to public and secular morality.
of law and order in Metro Manila as long as
control is left to the PNP. The calling of the The Court further states that our Constitution
Marines, in this case, constitutes permissible adheres the benevolent neutrality approach
use of military assets for civilian enforcement that gives room for accommodation of
and that it does not contravene Section 3, religious exercises as required by the Free
Article II of the Constitution. The limited Exercise Clause. This benevolent neutrality
participation of the Marines is evident in the could allow for accommodation of morality
Letter of Instruction which sufficiently provides based on religion, provided it does not offend
the metes and bounds of the Marines’ compelling state interests. Assuming arguendo
authority. The deployment of the Marines to that the OSG has proved a compelling state
assist the PNP does not unmake the civilian interest, it has to further demonstrate that the
character of the police force. The real authority state has used the least intrusive means
in these operations is lodged with the head of possible so that the free exercise is not
a civilian institution and not with the military. infringed any more than necessary to achieve
Page 77 of 121
403 CASE DIGESTS | CONSTITUTIONAL LAW REVIEW | JUDGE ESTELA ALMA SINGCO | FIRST SEMESTER | 2020-2021

the legitimate goal of the state. Thus the health of Muslim Filipinos can be effectively
conjugal arrangement cannot be penalized for protected by assigning to OMA the exclusive
it constitutes an exemption to the law based power to issue halal certifications. The
on her right to freedom of religion. protection and promotion of the Muslim
Filipinos right to health are already provided
for in existing laws and ministered to by
government agencies charged with ensuring
127 Islamic Da’Wah Council of the Philippines that food products released in the market are
vs. Office of the Executive Secretary, July fit for human consumption, properly labeled
9, 2003 and safe. Unlike EO 46, these laws do not
encroach on the religious freedom of Muslims.
FACTS:
Petitioner is a non-governmental organization
internationally accredited to issue halal
certifications in the Philippines. To carry out its
functions, it formulated internal rules and
procedures based on the Qur’an and Sunnah
for food analysis and inspection, and began to
issue certifications to qualified products and
THE FILIPINO FIRST POLICY
food manufacturers for a fee.
Later, respondent Office issued Executive 128 Manila Prince Hotel v. GSIS, 267 SCRA
Order 46 which created the Philippine Halal 408 (1997) (Bellosillo)
Certification Scheme. Such order vested
exclusive authority on the Office on Muslim Facts:
Affairs (OMA) to issue halal certificates and Pursuant to the privatization program of the
perform other related regulatory activities. Philippine Government, the GSIS sold in public
OMA then warned Muslim consumers to buy auction its stake in Manila Hotel Corporation
only products with its official halal certification (MHC). Only 2 bidders participated: petitioner
since those without said certification had not Manila Prince Hotel Corporation, a Filipino
been subjected to careful analysis and corporation, which offered to buy 51% of the
therefore could contain pork. It also began to MHC or 15,300,000 shares at P41.58 per
send letters to food manufacturers asking share, and Renong Berhad, a Malaysian firm,
them to secure the halal certification only from with ITT-Sheraton as its hotel operator, which
OMA lest they violate the order. As a result, bid for the same number of shares at P44.00
the IDCP lost revenues after food per share, or P2.42 more than the bid of
manufacturers stopped securing certifications petitioner.
from it. Hence, petitioner filed a petition to
nullify EO 46, contending that it is Petitioner filed a petition before the Supreme
unconstitutional for the government to Court to compel the GSIS to allow it to match
formulate policies and guidelines on the halal the bid of Renong Berhad. It invoked the
certification scheme because it is a function Filipino First Policy enshrined in §10,
that only religious organizations can lawfully & paragraph 2, Article XII of the 1987
validly perform for the Muslims. Constitution, which provides that “in the grant
of rights, privileges, and concessions covering
ISSUE: the national economy and patrimony, the
Whether or not EO 46 is unconstitutional. State shall give preference to qualified
Filipinos.”
RULING:
Yes. Classifying a food product as halal is a Issues:
religious function because the standards used 1. Whether §10, paragraph 2, Article XII of
are drawn from the Qur’an & Islamic beliefs. the 1987 Constitution is a self-executing
By giving OMA the exclusive power to classify provision and does not need implementing
food products as halal, EO 46 encroached on legislation to carry it into effect;
the religious freedom of Muslim organizations 2. Assuming §10, paragraph 2, Article XII is
to interpret for Filipino Muslims what food self-executing, whether the controlling shares
products are fit for Muslim consumption; by of the Manila Hotel Corporation form part of
arrogating to itself the task of issuing halal our patrimony as a nation;
certifications, the State has in effect forced 3. Whether GSIS is included in the term
Muslims to accept its own interpretation of the “State,” hence, mandated to implement §10,
Qur’an & Sunnah on halal food. paragraph 2, Article XII of the Constitution;
and
Only the prevention of an immediate & grave 4. Assuming GSIS is part of the State,
danger to the security and welfare of the whether it should give preference to the
community can justify the infringement of petitioner, a Filipino corporation, over Renong
religious freedom. If the government fails to Berhad, a foreign corporation, in the sale of
show the seriousness & immediacy of the the controlling shares of the Manila Hotel
threat, State intrusion is constitutionally Corporation.
unacceptable. In a society with a democratic
framework like ours, the State must minimize Ruling:
its interference with the affairs of its citizens [The Court, voting 11-4, DISMISSED the
and instead allow them to exercise reasonable petition.]
freedom of personal and religious activity.
1. YES, §10, paragraph 2, Article XII of
In the case at bar, the Court finds no the 1987 Constitution is a self-executing
compelling justification for the government to provision and does not need
deprive Muslim organizations, like herein implementing legislation to carry it into
petitioner, of their religious right to classify a effect.
product as halal, even on the premise that the

Page 78 of 121
403 CASE DIGESTS | CONSTITUTIONAL LAW REVIEW | JUDGE ESTELA ALMA SINGCO | FIRST SEMESTER | 2020-2021

Sec. 10, second par., of Art XII is couched in For more than eight (8) decades Manila Hotel
such a way as not to make it appear that it is has bore mute witness to the triumphs and
non-self-executing but simply for purposes of failures, loves and frustrations of the Filipinos;
style. But, certainly, the legislature is not its existence is impressed with public interest;
precluded from enacting further laws to its own historicity associated with our struggle
enforce the constitutional provision so long as for sovereignty, independence and
the contemplated statute squares with the nationhood. Verily, Manila Hotel has become
Constitution. Minor details may be left to the part of our national economy and patrimony.
legislature without impairing the self- For sure, 51% of the equity of the MHC comes
executing nature of constitutional provisions. within the purview of the constitutional shelter
for it comprises the majority and controlling
xxx xxx stock, so that anyone who acquires or owns
xxx the 51% will have actual control and
management of the hotel. In this instance,
Respondents . . . argue that the non-self- 51% of the MHC cannot be disassociated from
executing nature of Sec. 10, second par., of the hotel and the land on which the hotel
Art. XII is implied from the tenor of the first edifice stands. Consequently, we cannot
and third paragraphs of the same section sustain respondents’ claim that the Filipino
which undoubtedly are not self-executing. The First Policy provision is not applicable since
argument is flawed. If the first and third what is being sold is only 51% of the
paragraphs are not self-executing because outstanding shares of the corporation, not the
Congress is still to enact measures to Hotel building nor the land upon which the
encourage the formation and operation of building stands.
enterprises fully owned by Filipinos, as in the
first paragraph, and the State still needs 3. YES, GSIS is included in the term
legislation to regulate and exercise authority “State,” hence, it is mandated to
over foreign investments within its national implement §10, paragraph 2, Article XII
jurisdiction, as in the third paragraph, then a of the Constitution.
fortiori, by the same logic, the second
paragraph can only be self-executing as it does It is undisputed that the sale of 51% of the
not by its language require any legislation in MHC could only be carried out with the prior
order to give preference to qualified Filipinos approval of the State acting through
in the grant of rights, privileges and respondent Committee on Privatization. [T]his
concessions covering the national economy fact alone makes the sale of the assets of
and patrimony. A constitutional provision may respondents GSIS and MHC a “state action.”
be self-executing in one part and non-self- In constitutional jurisprudence, the acts of
executing in another. persons distinct from the government are
considered “state action” covered by the
xxx. Sec. 10, second par., Art. XII of the 1987 Constitution (1) when the activity it engages in
Constitution is a mandatory, positive is a “public function;” (2) when the
command which is complete in itself and which government is so significantly involved with
needs no further guidelines or implementing the private actor as to make the government
laws or rules for its enforcement. From its very responsible for his action; and, (3) when the
words the provision does not require any government has approved or authorized the
legislation to put it in operation. It is per se action. It is evident that the act of respondent
judicially enforceable. When our Constitution GSIS in selling 51% of its share in respondent
mandates that [i]n the grant of rights, MHC comes under the second and third
privileges, and concessions covering national categories of “state action.” Without doubt
economy and patrimony, the State shall give therefore the transaction, although entered
preference to qualified Filipinos, it means just into by respondent GSIS, is in fact a
that - qualified Filipinos shall be preferred. transaction of the State and therefore subject
And when our Constitution declares that a to the constitutional command.
right exists in certain specified circumstances
an action may be maintained to enforce such When the Constitution addresses the State it
right notwithstanding the absence of any refers not only to the people but also to the
legislation on the subject; consequently, if government as elements of the State. After
there is no statute especially enacted to all, government is composed of three (3)
enforce such constitutional right, such right divisions of power - legislative, executive and
enforces itself by its own inherent potency and judicial. Accordingly, a constitutional mandate
puissance, and from which all legislations must directed to the State is correspondingly
take their bearings. Where there is a right directed to the three (3) branches of
there is a remedy. Ubi jus ibi remedium. government. It is undeniable that in this case
the subject constitutional injunction is
addressed among others to the Executive
2. YES, the controlling shares of the Department and respondent GSIS, a
Manila Hotel Corporation form part of our government instrumentality deriving its
patrimony as a nation. authority from the State.

In its plain and ordinary meaning, the term


patrimony pertains to heritage. When the 4. YES, GSIS should give preference to
Constitution speaks of national patrimony, it the petitioner in the sale of the controlling
refers not only to the natural resources of the shares of the Manila Hotel Corporation.
Philippines, as the Constitution could have
very well used the term natural resources, but It should be stressed that while the Malaysian
also to the cultural heritage of the Filipinos. firm offered the higher bid it is not yet the
winning bidder. The bidding rules expressly
xxx xxx provide that the highest bidder shall only be
xxx declared the winning bidder after it has
negotiated and executed the necessary
Page 79 of 121
403 CASE DIGESTS | CONSTITUTIONAL LAW REVIEW | JUDGE ESTELA ALMA SINGCO | FIRST SEMESTER | 2020-2021

contracts, and secured the requisite approvals. same shares to Parallax Venture who won with
Since the Filipino First Policy provision of the a bid of P25.6 billion or US$510 million.
Constitution bestows preference on qualified
Filipinos the mere tending of the highest bid is Thereafter, First Pacific announced that it
not an assurance that the highest bidder will would exercise its right of first refusal as a
be declared the winning bidder. Resultantly, PTIC stockholder and buy the 111,415 PTIC
respondents are not bound to make the award shares by matching the bid price of Parallax.
yet, nor are they under obligation to enter into On 14 February 2007, First Pacific, through its
one with the highest bidder. For in choosing subsidiary, MPAH, entered into a Conditional
the awardee respondents are mandated to Sale and Purchase Agreement of the 111,415
abide by the dictates of the 1987 Constitution PTIC shares, or 46.125 percent of the
the provisions of which are presumed to be outstanding capital stock of PTIC, with the
known to all the bidders and other interested Philippine Government for the price of
parties. P25,217,556,000 or US$510,580,189. The
sale was completed on 28 February 2007.
xxx xxx
xxx Since PTIC is a stockholder of PLDT, the sale
by the Philippine Government of 46.125
Paragraph V. J. 1 of the bidding rules provides percent of PTIC shares is actually an indirect
that [i]f for any reason the Highest Bidder sale of 12 million shares or about 6.3 percent
cannot be awarded the Block of Shares, GSIS of the outstanding common shares of PLDT.
may offer this to other Qualified Bidders that With the sale, First Pacific common
have validly submitted bids provided that shareholdings in PLDT increased from 30.7
these Qualified Bidders are willing to match the percent to 37 percent, thereby increasing the
highest bid in terms of price per share. common shareholdings of foreigners in PLDT
Certainly, the constitutional mandate itself is to about 81.47 percent. This, according to
reason enough not to award the block of petitioner, violates Section 11, Article XII of
shares immediately to the foreign bidder the 1987 Philippine Constitution which limits
notwithstanding its submission of a higher, or foreign ownership of the capital of a public
even the highest, bid. In fact, we cannot utility to not more than 40 percent.
conceive of a stronger reason than the
constitutional injunction itself. On 28 February 2007, petitioner filed the
instant petition for prohibition, injunction,
In the instant case, where a foreign firm declaratory relief, and declaration of nullity of
submits the highest bid in a public bidding sale of the 111,415 PTIC shares.
concerning the grant of rights, privileges and
concessions covering the national economy ISSUE: Does the term "capital" in Section 11,
and patrimony, thereby exceeding the bid of a Article XII of the Constitution refer to the total
Filipino, there is no question that the Filipino common shares only or to the total
will have to be allowed to match the bid of the outstanding capital stock of PLDT, a public
foreign entity. And if the Filipino matches the utility?
bid of a foreign firm the award should go to the
Filipino. It must be so if we are to give life and HELD: Section 11, Article XII (National
meaning to the Filipino First Policy provision of Economy and Patrimony) of the 1987
the 1987 Constitution. For, while this may Constitution mandates the Filipinization
neither be expressly stated nor contemplated of public utilities, to wit:
in the bidding rules, the constitutional fiat is
omnipresent to be simply disregarded. To Section 11. No franchise, certificate, or any
ignore it would be to sanction a perilous other form of authorization for the operation of
skirting of the basic law. a public utility shall be granted except to
citizens of the Philippines or to corporations or
129 Wilson P. Gamboa v. Finance Secretary associations organized under the laws of the
Margarito B. Teves, et al., G.R. No. Philippines, at least sixty per centum of whose
176579, June 28, 2011, En Banc (Carpio) capital is owned by such citizens; nor shall
such franchise, certificate, or authorization be
Facts: exclusive in character or for a longer period
than fifty years. Neither shall any such
In 1969, General Telephone and Electronics franchise or right be granted except under the
Corporation (GTE), sold 26 percent of the condition that it shall be subject to
outstanding common shares of PLDT to amendment, alteration, or repeal by the
Philippine Telecommunications Investment Congress when the common good so requires.
Corporation (PTIC). In 1977, Prime Holdings, The State shall encourage equity participation
Inc. (PHI) became the owner of 111,415 in public utilities by the general public. The
shares of stock of PTIC. In 1986, the 111,415 participation of foreign investors in the
shares of stock of PTIC held by PHI were governing body of any public utility enterprise
sequestered by the Presidential Commission shall be limited to their proportionate share in
on Good Government (PCGG). The 111,415 its capital, and all the executive and managing
PTIC shares, which represent about 46.125 officers of such corporation or association
percent of the outstanding capital stock of must be citizens of the Philippines.
PTIC, were later declared by this Court to be
owned by the Republic of the Philippines.
The intent of the framers of the Constitution in
In 1999, First Pacific, a Bermuda-registered imposing limitations and restrictions on fully
acquired the remaining 54 percent of the nationalized and partially nationalized
outstanding capital stock of PTIC. On 20 activities is for Filipino nationals to be always
November 2006, the Inter-Agency in control of the corporation undertaking said
Privatization Council (IPC) of the Philippine activities. Otherwise, if the Trial Court ruling
Government through a public bidding sold the upholding respondent's arguments were to be
given credence, it would be possible for the
Page 80 of 121
403 CASE DIGESTS | CONSTITUTIONAL LAW REVIEW | JUDGE ESTELA ALMA SINGCO | FIRST SEMESTER | 2020-2021

ownership structure of a public utility medicine in the Philippines on the ground that
corporation to be divided into one percent the Board "believes that no genuine reciprocity
(1%) common stocks and ninety-nine percent can be found in the law of Japan... as there is
(99%) preferred stocks. Following the Trial no Filipino or foreigner who can possibly
Court ruling adopting respondent's arguments, practice there." [9]
the common shares can be owned entirely by
foreigners thus creating an absurd situation Respondent then filed a Petition for Certiorari
wherein foreigners, who are supposed to be and Mandamus against the Board before the
minority shareholders, control the public utility RTC of Manila on June 24, 1993, which petition
corporation. was amended on February 14, 1994 to implead
the PRC through its Chairman.
The term "capital" in Section 11, Article XII of
On October 19, 2003, the RTC rendered its
the Constitution refers only to shares of stock
Decision finding that respondent had
entitled to vote in the election of directors, and
adequately proved that the medical laws of
thus in the present case only to common
Japan allow foreigners like Filipinos to be
shares, and not to the total outstanding capital
granted license and be admitted into the
stock comprising both common and non-voting
practice of medicine under the principle of
preferred shares.
reciprocity; and that the
Indisputably, one of the rights of a stockholder Board had a ministerial duty of issuing the
is the right to participate in the control or Certificate of Registration and license to
management of the corporation. This is respondent, as it was shown that he had
exercised through his vote in the election of substantially complied with the requirements
directors because it is the board of directors under the law.[12] The RTC then ordered the
that controls or manages the corporation. In Board to issue in favor of... respondent the
the absence of provisions in the articles of corresponding Certificate of Registration
incorporation denying voting rights to and/or license to practice medicine in the
preferred shares, preferred shares have the Philippines.[13]
same voting rights as common shares.
However, preferred shareholders are often The Board and the PRC (petitioners) appealed
excluded from any control, that is, deprived of the case to the CA, stating that while
the right to vote in the election of directors and respondent submitted documents showing
on other matters, on the theory that the that foreigners are allowed to practice
preferred shareholders are merely investors in medicine in Japan, it was not shown that the
the corporation for income in the same manner conditions for the practice of medicine there
as bondholders. In fact, under the Corporation are practical and... attainable by a foreign
Code only preferred or redeemable shares can applicant, hence, reciprocity was not
be deprived of the right to vote. Common established; also, the power of the PRC and
shares cannot be deprived of the right to vote the Board to regulate and control the practice
in any corporate meeting, and any provision in of medicine is discretionary and not
the articles of incorporation restricting the ministerial, hence, not compellable by a writ of
right of common shareholders to vote is mandamus.[14]
invalid.
The CA denied the appeal and affirmed the
Considering that common shares have voting ruling of the RTC.[15]
rights which translate to control, as opposed to
preferred shares which usually have no voting Hence, herein petition
rights, the term "capital" in Section 11, Article Issues:
XII of the Constitution refers only to common
shares. However, if the preferred shares also WHETHER THE COURT OF APPEALS
have the right to vote in the election of COMMITTED A REVERSIBLE ERROR IN
directors, then the term "capital" shall include FINDING THAT RESPONDENT HAD
such preferred shares because the right to ESTABLISHED THE EXISTENCE OF
participate in the control or management of RECIPROCITY IN THE PRACTICE OF MEDICINE
the corporation is exercised through the right BETWEEN THE PHILIPPINES AND JAPAN.
to vote in the election of directors. In short,
the term "capital" in Section 11, Article XII of Ruling:
the Constitution refers only to shares of stock
that can vote in the election of directors. The Court denies the petition for lack of merit.

R.A. No. 2382, which provides who may be


This interpretation is consistent with the intent
candidates for the medical board
of the framers of the Constitution to place in
examinations, merely requires a foreign citizen
the hands of Filipino citizens the control and
to submit competent and conclusive
management of public utilities. Thus, 60
documentary evidence, confirmed by the
percent of the "capital" assumes, or should
Department of Foreign Affairs (DFA), showing
result in, "controlling interest" in the
that his country's existing laws... permit
corporation and thus in the present case, only
citizens of the Philippines to practice medicine
to common shares, and not to the total
under the same rules and regulations
outstanding capital stock (common and non-
governing citizens thereof.
voting preferred shares).
Section (j) of P.D. No. 223 also defines the
130 Board of Medicine v. Ota, G.R. No. extent of PRC's power to grant licenses, i.e., it
166097, July 14 may, upon recommendation of the board,
approve the registration and authorize the
Facts: issuance of a certificate of registration with or
without examination to a foreigner who is...
Board of Medicine (Board) of the PRC, in a
registered under the laws of his country,
letter dated March 8, 1993, denied
provided the following conditions are met: (1)
respondent's request for a license to practice
Page 81 of 121
403 CASE DIGESTS | CONSTITUTIONAL LAW REVIEW | JUDGE ESTELA ALMA SINGCO | FIRST SEMESTER | 2020-2021

that the requirement for the registration or


licensing in said foreign state or country are 2. SUBSTANTIVE ISSUES:
substantially the same as those required and Whether the RH law is unconstitutional:
contemplated by the laws of the Philippines;
(2) that... the laws of such foreign state or Right to Life
country allow the citizens of the Philippines to Right to Health
practice the profession on the same basis and Freedom of Religion and the Right to Free
grant the same privileges as the subject or Speech
citizens of such foreign state or country; and The Family
(3) that the applicant shall submit competent Freedom of Expression and Academic Freedom
and... conclusive documentary evidence, Due Process
confirmed by the DFA, showing that his Equal Protection
country's existing laws permit citizens of the Involuntary Servitude
Philippines to practice the profession under the Delegation of Authority to the FDA
rules and regulations governing citizens Autonomy of Local Governments / ARMM
thereof.
RULING:
The said provision further states that the PRC
is authorized to prescribe additional Before delving into the constitutionality of the
requirements or grant certain privileges to RH Law and its implementing rules, it
foreigners seeking registration in the behooves the Court to resolve some
Philippines if the same privileges are granted procedural impediments.
to or some additional requirements are
required of... citizens of the Philippines in The petition no doubt raises a justiciable
acquiring the same certificates in his country. controversy. Where an action of the legislative
branch is seriously alleged to have infringed
Nowhere in said statutes is it stated that the
the Constitution, it becomes not only the right
foreign applicant must show that the
but in fact the duty of the judiciary to settle
conditions for the practice of medicine in said
the dispute. “The question thus posed is
country are practical and attainable by
judicial rather than political. The duty (to
Filipinos. Neither is it stated that it must first
adjudicate) remains to assure that the
be proven that a Filipino has been... granted
supremacy of the Constitution is upheld. Once
license and allowed to practice his profession
a controversy as to the application or
in said country before a foreign applicant may
interpretation of constitutional provision is
be given license to practice in the Philippines.
raised before this Court (as in the instant
case), it becomes a legal issue which the Court
is bound by constitutional mandate to decide.
THE RIGHT TO LIFE OF THE UNBORN In the scholarly estimation of former Supreme
FROM CONCEPTION Court Justice Florentino Feliciano, “judicial
review is essential for the maintenance and
131 James M. Imbong, et al. v. Hon. Paquito enforcement of the separation of powers and
N. Ochoa, Jr., et al., GR No. 204819, April the balancing of powers among the three great
8, 2014, En Banc [Mendoza] departments of government through the
definition and maintenance of the boundaries
FACTS: of authority and control between them.” To
him, judicial review is the chief, indeed the
Shortly after the President placed his only, medium of participation – or instrument
imprimatur on Republic Act (R.A.) No. 10354, of intervention – of the judiciary in that
otherwise known as the Responsible balancing operation. Lest it be misunderstood,
Parenthood and Reproductive Health Act of it bears emphasizing that the Court does not
2012 (RH Law), challengers from various have the unbridled authority to rule on just any
sectors of society came knocking on the doors and every claim of constitutional violation.
of the Court, beckoning it to wield the sword Jurisprudence is replete with the rule that the
that strikes down constitutional disobedience. power of judicial review is limited by four
Aware of the profound and lasting impact that exacting requisites, viz : (a) there must be an
its decision may produce, the Court now faces actual case or controversy; (b) the petitioners
the controversy, as presented in fourteen (14) must possess locus standi; (c) the question of
petitions and two (2) petitions-in-intervention. constitutionality must be raised at the earliest
opportunity; and (d) the issue of
The petitioners are one in praying that constitutionality must be the lis mota of the
the entire RH Law be declared case.
unconstitutional. Even a singular violation of the Constitution
and/or the law is enough to awaken judicial
ISSUES: duty. In this case, the Court is of the view that
an actual case or controversy exists and that
After a scrutiny of the various arguments and the same is ripe for judicial determination.
contentions of the parties, the Court has Considering that the RH Law and its
synthesized and refined them to the following implementing rules have already taken effect
principal issues: and that budgetary measures to carry out the
law have already been passed, it is evident
PROCEDURAL ISSUES: that the subject petitions present a justiciable
1. Whether the Court may exercise its power controversy. When an action of the legislative
of judicial review over the controversy. branch is seriously alleged to have infringed
Power of Judicial Review the Constitution, it not only becomes a right,
Actual Case or Controversy but also a duty of the Judiciary to settle the
Facial Challenge dispute.
Locus Standi
Declaratory Relief The Court is not persuaded. In United States
One Subject/One Title Rule (US) constitutional law, a facial challenge, also
Page 82 of 121
403 CASE DIGESTS | CONSTITUTIONAL LAW REVIEW | JUDGE ESTELA ALMA SINGCO | FIRST SEMESTER | 2020-2021

known as a First Amendment Challenge, is one and conception are two distinct and successive
that is launched to assail the validity of stages in the reproductive process. They are
statutes concerning not only protected speech, not identical and synonymous.” Citing a letter
but also all other rights in the First of the WHO, he wrote that medical authorities
Amendment. These include religious freedom, confirm that the implantation of the fertilized
freedom of the press, and the right of the ovum is the commencement of conception and
people to peaceably assemble, and to petition it is only after implantation that pregnancy can
the Government for a redress of grievances. be medically detected. This theory of
After all, the fundamental right to religious implantation as the beginning of life is devoid
freedom, freedom of the press and peaceful of any legal or scientific mooring. It does not
assembly are but component rights of the right pertain to the beginning of life but to the
to one’s freedom of expression, as they are viability of the fetus. The fertilized
modes which one’s thoughts are externalized. ovum/zygote is not an inanimate object – it is
In this jurisdiction, the application of doctrines a living human being complete with DNA and
originating from the U.S. has been generally 46 chromosomes. Implantation has been
maintained, albeit with some modifications. conceptualized only for convenience by those
While this Court has withheld the application who had population control in mind. To adopt
of facial challenges to strictly penal statues, it it would constitute textual infidelity not only to
has expanded its scope to cover statutes not the RH Law but also to the Constitution. It is
only regulating free speech, but also those the Court’s position that life begins at
involving religious freedom, and other fertilization, not at implantation. When a
fundamental rights. Verily, the framers of Our fertilized ovum is implanted in the uterine wall,
Constitution envisioned a proactive Judiciary, its viability is sustained but that instance of
ever vigilant with its duty to maintain the implantation is not the point of beginning of
supremacy of the Constitution. life.
A component to the right to life is the
The transcendental importance of the issues constitutional right to health. In this regard,
involved in this case warrants that we set aside the Constitution is replete with provisions
the technical defects and take primary protecting and promoting the right to health.
jurisdiction over the petition at bar. One These provisions are self-executing. Unless the
cannot deny that the issues raised herein have provisions clearly express the contrary, the
potentially pervasive influence on the social provisions of the Constitution should be
and moral well being of this nation, specially considered self-executory. There is no need for
the youth; hence, their proper and just legislation to implement these self-executing
determination is an imperative need. This is in provisions.
accordance with the well-entrenched principle
that rules of procedure are not inflexible tools In Manila Prince Hotel v. GSIS, it was stated:
designed to hinder or delay, but to facilitate x x x Hence, unless it is expressly provided
and promote the administration of justice. that a legislative act is necessary to enforce a
Their strict and rigid application, which would constitutional mandate, the presumption now
result in technicalities that tend to frustrate, is that all provisions of the constitution are
rather than promote substantial justice, must self-executing. If the constitutional provisions
always be eschewed. Considering that it is the are treated as requiring legislation instead of
right to life of the mother and the unborn self-executing, the legislature would have the
which is primarily at issue, the Court need not power to ignore and practically nullify the
wait for a life to be taken away before taking mandate of the fundamental law. This can be
action. cataclysmic. That is why the prevailing view is,
Where the case has far-reaching implications as it has always been, that –… in case of doubt,
and prays for injunctive reliefs, the Court may the Constitution should be considered self-
consider them as petitions for prohibition executing rather than non-self-executing. . . .
under Rule 65.
Unless the contrary is clearly intended, the
The RH Law does not violate the one provisions of the Constitution should be
subject/one bill rule. It is well-settled that the considered self-executing, as a contrary rule
“one title-one subject” rule does not require would give the legislature discretion to
the Congress to employ in the title of the determine when, or whether, they shall be
enactment language of such precision as to effective. These provisions would be
mirror, fully index or catalogue all the contents subordinated to the will of the lawmaking
and the minute details therein. The rule is body, which could make them entirely
sufficiently complied with if the title is meaningless by simply refusing to pass the
comprehensive enough as to include the needed implementing statute.
general object which the statute seeks to
effect, and where, as here, the persons It is not within the province of the Court to
interested are informed of the nature, scope determine whether the use of contraceptives
and consequences of the proposed law and its or one’s participation in the support of modern
operation. Moreover, this Court has invariably reproductive health measures is moral from a
adopted a liberal rather than technical religious standpoint or whether the same is
construction of the rule “so as not to cripple or right or wrong according to one’s dogma or
impede legislation.” In this case, a textual belief. For the Court has declared that matters
analysis of the various provisions of the law dealing with “faith, practice, doctrine, form of
shows that both “reproductive health” and worship, ecclesiastical law, custom and rule of
“responsible parenthood” are interrelated and a church … are unquestionably ecclesiastical
germane to the overriding objective to control matters which are outside the province of the
the population growth. civil courts.” The jurisdiction of the Court
extends only to public and secular morality.
AS TO SUBSTANTIVE ISSUES: Whatever pronouncement the Court makes in
The Court cannot subscribe to the theory the case at bench should be understood only
advocated by Hon. Lagman that life begins at in this realm where it has authority. Stated
implantation. According to him, “fertilization otherwise, while the Court stands without
Page 83 of 121
403 CASE DIGESTS | CONSTITUTIONAL LAW REVIEW | JUDGE ESTELA ALMA SINGCO | FIRST SEMESTER | 2020-2021

authority to rule on ecclesiastical matters, as reproductive health education. One can only
vanguard of the Constitution, it does have speculate on the content, manner and medium
authority to determine whether the RH Law of instruction that will be used to educate the
contravenes the guarantee of religious adolescents and whether they will contradict
freedom. Consequently, the petitioners are the religious beliefs of the petitioners and
misguided in their supposition that the State validate their apprehensions. Thus,
cannot enhance its population control program considering the premature nature of this
through the RH Law simply because the particular issue, the Court declines to rule on
promotion of contraceptive use is contrary to its constitutionality or validity.
their religious beliefs. Indeed, the State is not A statute or act suffers from the defect of
precluded to pursue its legitimate secular vagueness when it lacks comprehensible
objectives without being dictated upon by the standards that men of common intelligence
policies of any one religion. One cannot refuse must necessarily guess its meaning and differ
to pay his taxes simply because it will cloud his as to its application. It is repugnant to the
conscience. The demarcation line between Constitution in two respects: (1) it violates due
Church and State demands that one render process for failure to accord persons,
unto Caesar the things that are Caesar’s and especially the parties targeted by it, fair notice
unto God the things that are God’s. The Court of the conduct to avoid; and (2) it leaves law
is of the view that the obligation to refer enforcers unbridled discretion in carrying out
imposed by the RH Law violates the religious its provisions and becomes an arbitrary flexing
belief and conviction of a conscientious of the Government muscle. Moreover, in
objector. Once the medical practitioner, determining whether the words used in a
against his will, refers a patient seeking statute are vague, words must not only be
information on modern reproductive health taken in accordance with their plain meaning
products, services, procedures and methods, alone, but also in relation to other parts of the
his conscience is immediately burdened as he statute. It is a rule that every part of the
has been compelled to perform an act against statute must be interpreted with reference to
his beliefs. As Commissioner Joaquin A. Bernas the context, that is, every part of it must be
(Commissioner Bernas) has written, “at the construed together with the other parts and
basis of the free exercise clause is the respect kept subservient to the general intent of the
for the inviolability of the human conscience. whole enactment.

The Court is of the strong view that the To provide that the poor are to be given
religious freedom of health providers, whether priority in the government’s reproductive
public or private, should be accorded primacy. health care program is not a violation of the
Accordingly, a conscientious objector should equal protection clause. In fact, it is pursuant
be exempt from compliance with the mandates to Section 11, Article XIII of the Constitution
of the RH Law. If he would be compelled to act which recognizes the distinct necessity to
contrary to his religious belief and conviction, address the needs of the underprivileged by
it would be violative of “the principle of non- providing that they be given priority in
coercion” enshrined in the constitutional right addressing the health development of the
to free exercise of religion. people. Thus: Section 11. The State shall
The same holds true with respect to non- adopt an integrated and comprehensive
maternity specialty hospitals and hospitals approach to health development which shall
owned and operated by a religious group and endeavor to make essential goods, health and
health care service providers. Considering that other social services available to all the people
Section 24 of the RH Law penalizes such at affordable cost. There shall be priority for
institutions should they fail or refuse to comply the needs of the underprivileged, sick, elderly,
with their duty to refer under Section 7 and disabled, women, and children. The State shall
Section 23(a)(3), the Court deems that it must endeavor to provide free medical care to
be struck down for being violative of the paupers. It should be noted that Section 7 of
freedom of religion. the RH Law prioritizes poor and marginalized
couples who are suffering from fertility issues
The same applies to Section 23(a)(l) and and desire to have children. There is,
(a)(2) in relation to Section 24, considering therefore, no merit to the contention that the
that in the dissemination of information RH Law only seeks to target the poor to reduce
regarding programs and services and in the their number. While the RH Law admits the use
performance of reproductive health of contraceptives, it does not, as elucidated
procedures, the religious freedom of health above, sanction abortion. As Section 3(1)
care service providers should be respected. explains, the “promotion and/or stabilization
The punishment of a healthcare service of the population growth rate is incidental to
provider, who fails and/or refuses to refer a the advancement of reproductive health.”
patient to another, or who declines to perform
reproductive health procedure on a patient The notion of involuntary servitude connotes
because incompatible religious beliefs, is a the presence of force, threats, intimidation or
clear inhibition of a constitutional guarantee other similar means of coercion and
which the Court cannot allow. compulsion. A reading of the assailed
provision, however, reveals that it only
The State cannot, without a compelling state encourages private and non- government
interest, take over the role of parents in the reproductive healthcare service providers to
care and custody of a minor child, whether or render pro bono service. Other than non-
not the latter is already a parent or has had a accreditation with PhilHealth, no penalty is
miscarriage. Only a compelling state interest imposed should they choose to do otherwise.
can justify a state substitution of their parental Private and non-government reproductive
authority. healthcare service providers also enjoy the
Any attack on the validity of Section 14 of the liberty to choose which kind of health service
RH Law is premature because the Department they wish to provide, when, where and how to
of Education, Culture and Sports has yet to provide it or whether to provide it all. Clearly,
formulate a curriculum on age-appropriate therefore, no compulsion, force or threat is
Page 84 of 121
403 CASE DIGESTS | CONSTITUTIONAL LAW REVIEW | JUDGE ESTELA ALMA SINGCO | FIRST SEMESTER | 2020-2021

made upon them to render pro bono service


against their will. While the rendering of such FACTS:
service was made a prerequisite to Following the campaign of President Rodrigo
accreditation with PhilHealth, the Court does Roa Duterte to implement a nationwide curfew
not consider the same to be an unreasonable for minors, several local governments in Metro
burden, but rather, a necessary incentive Manila started to strictly implement their
imposed by Congress in the furtherance of a curfew ordinances on minors through police
perceived legitimate state interest. Consistent operations which were publicly known as part
with what the Court had earlier discussed, of "Oplan Rody."
however, it should be emphasized that
conscientious objectors are exempt from this Petitioners,spearheaded by the Samahan ng
provision as long as their religious beliefs and mga Progresibong Kabataan (SPARK)- an
convictions do not allow them to render association of young adults and minors that
reproductive health service, pro bona or aims to forward a free and just society, in
otherwise. particular the protection of the rights and
welfare of the youth and minors[10] - filed this
The Court finds nothing wrong with the present petition, arguing that the Curfew
delegation. The FDA does not only have the Ordinances are unconstitutional because they:
power but also the competency to evaluate, (a) result in arbitrary and discriminatory
register and cover health services and enforcement, and thus, fall under the void for
methods. It is the only government entity vagueness doctrine; (b) suffer from
empowered to render such services and highly overbreadth by proscribing or impairing
proficient to do so. It should be understood legitimate activities of minors during curfew
that health services and methods fall under the hours; (c) deprive minors of the right to liberty
gamut of terms that are associated with what and the right to travel without substantive due
is ordinarily understood as “health products.” process; and (d) deprive parents of their
Being the country’s premiere and sole agency natural and primary right in rearing the youth
that ensures the safety of food and medicines without substantive due process. In addition,
available to the public, the FDA was equipped petitioners assert that the Manila Ordinance
with the necessary powers and functions to contravenes RA 9344, as amended by RA
make it effective. Pursuant to the principle of 10630.
necessary implication, the mandate by
Congress to the FDA to ensure public health More specifically, petitioners posit that the
and safety by permitting only food and Curfew Ordinances encourage arbitrary and
medicines that are safe includes “service” and discriminatory enforcement as there are no
“methods.” From the declared policy of the RH clear provisions or detailed standards on how
Law, it is clear that Congress intended that the law enforcers should apprehend and properly
public be given only those medicines that are determine the age of the alleged curfew
proven medically safe, legal, non- violators.[13] They further argue that the law
abortifacient, and effective in accordance with enforcer's apprehension depends only on his
scientific and evidence-based medical research physical assessment, and, thus, subjective and
standards. The philosophy behind the based only on the law enforcer's visual
permitted delegation was explained in assessment of the alleged curfew violator.
Echagaray v. Secretary of Justice, as follows:
While petitioners recognize that the Curfew
The reason is the increasing complexity of the Ordinances contain provisions indicating the
task of the government and the growing activities exempted from the operation of the
inability of the legislature to cope directly with imposed curfews, i.e., exemption of working
the many problems demanding its attention. students or students with evening class, they
The growth of society has ramified its activities contend that the lists of exemptions do not
and created peculiar and sophisticated cover the range and breadth of legitimate
problems that the legislature cannot be activities or reasons as to why minors would
expected reasonably to comprehend. be out at night, and, hence, proscribe or impair
Specialization even in legislation has become the legitimate activities of minors during
necessary. To many of the problems attendant curfew hours.
upon present day undertakings, the legislature
may not have the competence, let alone the Petitioners likewise proffer that the Curfew
interest and the time, to provide the required Ordinances: (a) are unconstitutional as they
direct and efficacious, not to say specific deprive minors of the right to liberty and the
solutions. right to travel without substantive due
process; and (b) fail to pass the strict scrutiny
A reading of the RH Law clearly shows that test, for not being narrowly tailored and for
whether it pertains to the establishment of employing means that bear no reasonable
health care facilities, the hiring of skilled health relation to their purpose. They argue that the
professionals, or the training of barangay prohibition of minors on streets during curfew
health workers, it will be the national hours will not per se protect and promote the
government that will provide for the funding of social and moral welfare of children of the
its implementation. Local autonomy is not community.
absolute. The national government still has the
say when it comes to national priority Furthermore, petitioners claim that the Manila
programs which the local government is called Ordinance, particularly Section 4 thereof,
upon to implement like the RH Law. contravenes Section 57-A of RA 9344, as
amended, given that the cited curfew provision
imposes on minors the penalties of
imprisonment, reprimand, and admonition.
RIGHTS OF PARENTS IN THE REARING OF
They contend that the imposition of penalties
THEIR CHILDREN
contravenes RA 9344's express command that
no penalty shall be imposed on minors for
132 SPARK vs. Quezon City, et al
curfew violations.
Page 85 of 121
403 CASE DIGESTS | CONSTITUTIONAL LAW REVIEW | JUDGE ESTELA ALMA SINGCO | FIRST SEMESTER | 2020-2021

"This affirmative process of teaching, guiding,


Lastly, petitioners submit that there is no and inspiring by precept and example is
compelling State interest to impose curfews essential to the growth of young people into
contrary to the parents' prerogative to impose mature, socially responsible citizens."
them in the exercise of their natural and
primary right in the rearing of the youth, and While parents have the primary role in child-
that even if a compelling interest exists, less rearing, it should be stressed that "when
restrictive means are available to achieve the actions concerning the child have a relation to
same. In this regard, they suggest massive the public welfare or the well-being of the
street lighting programs, installation of CCTVs child, the [S]tate may act to promote these
(closed-circuit televisions) in public streets, legitimate interests." Thus, "[i]n cases in
and regular visible patrols by law enforcers as which harm to the physical or mental health of
other viable means of protecting children and the child or to public safety, peace, order, or
preventing crimes at night. They further opine welfare is demonstrated, these legitimate
that the government can impose more state interests may override the parents'
reasonable sanctions, i.e., mandatory parental qualified right to control the upbringing of their
counseling and education seminars informing children."
the parents of the reasons behind the curfew,
and that imprisonment is too harsh a penalty [W]here minors are involved, the State acts as
for parents who allowed their children to be out parens patriae. To it is cast the duty of
during curfew hours. protecting the rights of persons or individual
who because of age or incapacity are in an
Issue: unfavorable position, vis-a- vis other parties.
Whether it violates the Right of Parents to Rear Unable as they are to take due care of what
their Children. (Answering this issue only as it concerns them, they have the political
falls in this particular topic) community to look after their welfare. This
obligation the state must live up to. It cannot
Ruling: be recreant to such a trust. As was set forth in
an opinion of the United States Supreme
Petitioners are NOT CORRECT that the Curfew Court: "This prerogative of parens patriae is
Ordinances are unconstitutional because they inherent in the supreme power of every State,
deprive parents of their natural and primary x x x."(Emphases and underscoring supplied)
right in the rearing of the youth without
substantive due process. In this regard, they The Curfew Ordinances are but examples of
assert that this right includes the right to legal restrictions designed to aid parents in
determine whether minors will be required to their role of promoting their children's well-
go home at a certain time or will be allowed to being.
stay late outdoors. Given that the right to
impose curfews is primarily with parents and At this juncture, it should be emphasized that
not with the State, the latter's interest in the Curfew Ordinances apply only when the
imposing curfews cannot logically be minors are not - whether actually or
compelling. constructively (as will be later discussed) -
accompanied by their parents. This serves as
Section 12, Article II of the 1987 Constitution an explicit recognition of the State's deference
articulates the State's policy relative to the to the primary nature of parental authority and
rights of parents in the rearing of their the importance of parents' role in child-
children: rearing. Parents are effectively given
unfettered authority over their children's
Section 12. The State recognizes the sanctity conduct during curfew hours when they are
of family life and shall protect and strengthen able to supervise them. Thus, in all actuality,
the family as a basic autonomous social the only aspect of parenting that the Curfew
institution. It shall equally protect the life of Ordinances affects is the parents' prerogative
the mother and the life of the unborn from to allow minors to remain in public places
conception. The natural and primary right and without parental accompaniment during the
duty of parents in the rearing of the youth for curfew hours.
civic efficiency and the development of moral
character shall receive the support of the Finally, it may be well to point out that the
Government. (Emphasis and underscoring Curfew Ordinances positively influence
supplied.) children to spend more time at home.
As may be gleaned from this provision, the Consequently, this situation provides parents
rearing of children (i.e., referred to as the with better opportunities to take a more active
"youth") for civic efficiency and the role in their children's upbringing.
development of their moral character are
characterized not only as parental rights, but
BALANCED & HEALTHFUL ECOLOGY
also as parental duties. This means that
parents are not only given the privilege of 133 Prov. of Rizal vs. Exec. Sec., December 13,
exercising their authority over their children; 2005
they are equally obliged to exercise this
authority conscientiously. The duty aspect of Facts:
this provision is a reflection of the State's This is a petition filed by the Province of
independent interest to ensure that the youth Rizal, the municipality of San Mateo, and
would eventually grow into free, independent, various concerned citizens for review on
and well-developed citizens of this nation. For certiorari of the Decision of the Court of
indeed, it is during childhood that minors are Appeals, denying, for lack of cause of action,
prepared for additional obligations to society. the petition for certiorari, prohibition and
"[T]he duty to prepare the child for these mandamus with application for a temporary
[obligations] must be read to include the restraining order/writ of preliminary injunction
inculcation of moral standards, religious assailing the legality and constitutionality of
beliefs, and elements of good citizenship." Proclamation No. 635.
Page 86 of 121
403 CASE DIGESTS | CONSTITUTIONAL LAW REVIEW | JUDGE ESTELA ALMA SINGCO | FIRST SEMESTER | 2020-2021

notified the MMDA that they would oppose any


At the height of the garbage crisis plaguing further attempt to dump garbage in their
Metro Manila and its environs, parts of the province.
Marikina Watershed Reservation were set
aside by the Office of the President [President Moreover, Section 447, which enumerates
Ramos], through Proclamation No. 635, for the powers, duties and functions of the
use as a sanitary landfill and similar waste municipality, grants the sangguniang bayan
disposal applications. the power to, among other things, “enact
ordinances, approve resolutions and
The petitioners opposed the implementation appropriate funds for the general welfare of
of said order since the creation of dump site the municipality and its inhabitants pursuant
under the territorial jurisdiction would to Section 16 of th(e) Code.” These include:
compromise the health of their constituents.
Moreso, the the dump site is to be constructed (1) Approving ordinances and passing
in Watershed reservation. resolutions to protect the environment and
Through their concerted efforts of the impose appropriate penalties for acts which
officials and residents of Province of Rizal and endanger the environment, such as dynamite
Municipality of San Mateo, the dump site was fishing and other forms of destructive fishing,
closed. However, during the term of President illegal logging and smuggling of logs,
Estrada in 2003, the dumpsite was reopened. smuggling of natural resources products and
of endangered species of flora and fauna, slash
A temporary restraining order was then and burn farming, and such other activities
filed. Although petitioners did not raised the which result in pollution, acceleration of
question that the project was not consulted eutrophication of rivers and lakes, or of
and approved by their appropriate ecological imbalance; [Section 447 (1)(vi)]
Sanggunian, the court take it into
consideration since a mere MOA does not (2) Prescribing reasonable limits and restraints
guarantee the dump site’s permanent closure. on the use of property within the jurisdiction
of the municipality, adopting a comprehensive
Issue: land use plan for the municipality, reclassifying
Whether or not the consultation and land within the jurisdiction of the city, subject
approval of the Province of Rizal and to the pertinent provisions of this Code,
municipality of San Mateo is needed before the enacting integrated zoning ordinances in
implementation of the project.. consonance with the approved comprehensive
land use plan, subject to existing laws, rules
Ruling: and regulations; establishing fire limits or
The court reiterated again that "the earth zones, particularly in populous centers; and
belongs in usufruct to the living." regulating the construction, repair or
modification of buildings within said fire limits
Yes, as lucidly explained by the court: or zones in accordance with the provisions of
contrary to the averment of the respondents, this Code;[Section 447 (2)(vi-ix)]
Proclamation No. 635, which was passed on 28
August 1995, is subject to the provisions of the (3) Approving ordinances which shall ensure
Local Government Code, which was approved the efficient and effective delivery of the basic
four years earlier, on 10 October 1991. services and facilities as provided for under
Section 17 of this Code, and in addition to said
Section 2(c) of the said law declares that it services and facilities, …providing for the
is the policy of the state- "to require all establishment, maintenance, protection, and
national agencies and offices to conduct conservation of communal forests and
periodic consultation with appropriate local watersheds, tree parks, greenbelts,
government units, non-governmental and mangroves, and other similar forest
people's organization, and other concerned development projects ….and, subject to
sectors of the community before any project or existing laws, establishing and providing for
program is implemented in their respective the maintenance, repair and operation of an
jurisdiction." Likewise Section 27 requires efficient waterworks system to supply water
prior consultations before a program shall be for the inhabitants and purifying the source of
implemented by government authorities and the water supply; regulating the construction,
the prior approval of the Sanggunian is maintenance, repair and use of hydrants,
obtained." Corollarily as held in Lina , Jr. v. pumps, cisterns and reservoirs; protecting the
Paño, Section 2 (c), requiring consultations purity and quantity of the water supply of the
with the appropriate local government units, municipality and, for this purpose, extending
should apply to national government projects the coverage of appropriate ordinances over
affecting the environmental or ecological all territory within the drainage area of said
balance of the particular community water supply and within one hundred (100)
implementing the project. meters of the reservoir, conduit, canal,
aqueduct, pumping station, or watershed used
Relative to the case, during the oral in connection with the water service; and
arguments at the hearing for the temporary regulating the consumption, use or wastage of
restraining order, Director Uranza of the MMDA water.”[Section 447 (5)(i) & (vii)]
Solid Waste Management Task Force declared
before the Court of Appeals that they had Briefly stated, under the Local Government
conducted the required consultations. Code, two requisites must be met before a
However, the ambivalence of his reply was national project that affects the environmental
brought to the fore when at the height of the and ecological balance of local communities
protest rally and barricade made by the can be implemented:
residents of petitioners to stop dump trucks (1) prior consultation with the affected
from reaching the site, all the municipal local communities, and
mayors of the province of Rizal openly (2)prior approval of the project by the
declared their full support for the rally and appropriate sanggunian.
Page 87 of 121
403 CASE DIGESTS | CONSTITUTIONAL LAW REVIEW | JUDGE ESTELA ALMA SINGCO | FIRST SEMESTER | 2020-2021

Further, they contended that since the


Absent either of these mandatory scientific evidence as to the safety of Bt talong
requirements, the project’s implementation is remained insufficient or uncertain, and that
illegal. preliminary scientific evaluation shows
reasonable grounds for concern, the
134 International Service for the Acquisition precautionary principle should be applied and,
of Agri-Biotech Applications. Inc. v. thereby, the field trials be enjoined.
Greenpeace Southeast Asia (Philippines),
et al., December 8, 2015 On May 2, 2012, the Court issued... a Writ of
Kalikasan against petitioners (except UPLB
Facts:
May 17, 2013, the CA ruled in favor of
From 2007 to 2009, petitioner University of the respondents and directed petitioners to
Philippines Los Baiios (UPLB), the permanently cease and desist from conducting
implementing institution of the field trials, the Bt talong field trials.
conducted a contained experiment on Bt
talong under the supervision of the National Issues:
Committee on Biosafety of the Philippines
(NCBP) the case should have been dismissed for
mootness in view of the completion and
The NCBP, created under Executive Order No. termination of the Bt talong field trials and the
(EO) 430,... is the regulatory body tasked to: expiration of the Biosafety Permits;... b) the
(a) "identify and evaluate potential hazards Court should not have ruled on the validity of
involved in initiating genetic engineering DAO 08-2002 as it was not raised as an issue
experiments or the introduction of new species
and genetically engineered organisms and Ruling:
recommend measures to minimize risks"; and
(b) "formulate and review national policies and The Proceedings Before the Court
guidelines on biosafety, such as the safe
conduct of work on genetic engineering, pests Agreeing with the CA, the Court held that the
and their genetic materials for the protection precautionary principle applies in this case
of public health, environment[,] and since the risk of harm from the field trials of Bt
personnel^] and supervise the talong remains uncertain and there exists a
implementation thereof." possibility of serious and irreversible harm.
The Court observed that eggplants are a staple
Upon the completion of the contained vegetable in the country that is mostly grown
experiment, the NCBP issued a Certificate... by small-scale farmers who are poor and
therefor stating that all biosafety measures marginalized; thus, given the country's rich
were complied with, and no untoward incident biodiversity, the consequences of
had occurred contamination and genetic pollution would be
disastrous and irreversible.
On March 16, 2010 and June 28, 2010, the
Bureau of Plant Industries (BPI) issued two The Court likewise agreed with the CA in not
(2)-year Biosafety Permits... for field testing of dismissing the case for being moot and
Bt talong academic despite the completion and
tennination of the Bt talong field trials, on
Consequently, field testing proceeded in account of the following exceptions to the
approved trial sites in North Cotabato, mootness principle: (a) the exceptional
Pangasinan, Camarines Sur, Davao City, and character of the situation and the paramount
Laguna. public interest is involved; and (b) the case is
capable of repetition yet evading review.
On April 26, 2012, respondents Greenpeace
Southeast Asia (Philippines) (Greenpeace), Thus, the Court permanently enjoined the field
Magsasaka at Siyentipiko sa Pagpapaunlad ng testing of Bt talong. In addition, it declared
Agrikultura (MASIPAG), and others DAO 08-2002 null and void for failure to
(respondents) filed before the Court a Petition consider the provisions of the NBF. The Court
for Writ of Continuing Mandamus and Writ of also temporarily enjoined any application for
Kalikasan with Prayer for the Issuance of a contained use, field testing, propagation,
Temporary Environmental Protection Order commercialization, and importation of
(TEPO) genetically modified organisms until a new
administrative order is promulgated in
(petition for Writ of Kalikasan) against herein accordance with law.
petitioners... alleging that the Bt talong field
trials violated their constitutional right to The Court's Ruling
health and a balanced ecology considering,
among others, that: (a) the Environmental The Court grants the motions for
Compliance Certificate (ECC), as required by reconsideration on the ground of mootness.
Presidential Decree No. (PD) 1151,... was not
secured prior to the field trials;... b) the As a rule, the Court may only adjudicate
required public consultations under the Local actual, ongoing controversies.
Government Code (LGC) were not complied
with;... and (c) as a regulated article under Accordingly, the Court is not empowered to
DAO 08-2002, Bt talong is presumed harmful decide moot questions or abstract
to human health and the environment, and propositions, or to declare principles or rules
that there is no independent, peer-reviewed of law which cannot affect the result as to the
study showing its safety for human thing in issue in the case before it. In other
consumption and the environment words, when a case is moot, it becomes non-
justiciable

Page 88 of 121
403 CASE DIGESTS | CONSTITUTIONAL LAW REVIEW | JUDGE ESTELA ALMA SINGCO | FIRST SEMESTER | 2020-2021

An action is considered "moot" when it no may perceivably benefit. Any future threat to
longer presents a justiciable controversy the right of herein respondents or the public in
because the issues involved have become general to a healthful and balanced ecology is
academic or dead or when the matter in therefore more imagined than real
dispute has already been resolved and hence,
one is not entitled to judicial intervention In fact, it would appear to be more beneficial
unless the issue is likely to be raised again to the public to stay a verdict on the safeness
between the parties. There is nothing for the of Bt talong - or GMOs, for that matter - until
court to resolve as the determination thereof an actual and justiciable case properly
has been overtaken by subsequent events presents itself before the Court

Nevertheless, case law states that the Court II. The case is not one capable of repetition vet
will decide cases, otherwise moot, if: first, evading review.Likewise, contrary to the
there is a grave violation of the Constitution; Court's earlier ruling... these cases do not fall
second, the exceptional character of the under the "capable of repetition yet evading
situation and the paramount public interest are review" exception.
involved; third, when the constitutional issue
raised requires formulation of controlling Under DAO 08-2002, no specific guidelines
principles to guide the bench, the bar, and the were used in the conduct of risk assessment,
public; and fourth, the case is capable of and the DA was allowed to consider the expert
repetition yet evading review advice of, and guidelines developed by,
relevant international organizations and
Thus, jurisprudence recognizes these four regulatory authorities of countries with
instances as exceptions to the mootness significant experience in the regulatory
principle. supervision of the regulated article.

In the December 8, 2015 Decision of the However, under JDC 01-2016, the CODEX
Court, it was held that (a) the present case is Alimentarius Guidelines was adopted to govern
of exceptional character and paramount public the risk assessment of activities involving the
interest is involved, and (b) it is likewise research, development, handling and use,
capable of repetition yet evading review. transboundary movement, release into the
Hence, it was excepted from the mootness environment, and management of genetically
principle modified plant and plant products derived from
the use of modern biotechnology.
However, upon a closer scrutiny of the parties'
arguments, the Court reconsiders its ruling Based on the foregoing, it is apparent that the
and now finds merit in petitioners' assertion regulatory framework now applicable in
that the case should have been dismissed for conducting risk assessment in matters
being moot and academic, and that the involving the research, development,
aforesaid exceptions to the said rule should not handling, movement, and release into the
have been applied. environment of genetically modified plant and
plant products derived from the use of modern
I. On the paramount public interest exception. biotechnology is substantially different from
that which was applied to the subject field
However, a survey of cases would show that, trials. In this regard, it cannot be said that the
as a common guidepost for application, there present case is one capable of repetition yet
should be some perceivable benefit to the evading review.
public which demands the Court to proceed
with the resolution of otherwise moot the Court discerns that there are two (2)
questions. factors to be considered before a case is
deemed one capable of repetition yet evading
no perceivable benefit to the public - whether review: (1) the challenged action was in its
rational or practical - may be gained by duration too short to be fully litigated prior to
resolving respondents' petition for Writ of its cessation or expiration; and (2) there was
Kalikasan on the merits. a reasonable expectation that the same
complaining party would be subjected to the
To recount, these cases, which stemmed from same action.
herein respondents petition for Writ of
Kalikasan, were mooted by the undisputed Here, respondents cannot claim that the
expiration of the Biosafety Permits issued by duration of the subject field tests was too short
the BPI and the completion and tennination of to be fully litigated. It must be emphasized
the Bt talong field trials subject of the same that the Biosafety Permits for the subject field
tests were issued on March 16, 2010 and June
These incidents effectively negated the 28, 2010, and were valid for two (2) years.
necessity for the reliefs sought by respondents However, as aptly pointed out by Justice
in their petition for Writ of Kalikasan as there Leonen, respondents filed their petition for
was no longer any field test to enjoin. Hence, Writ of Kalikasan only on April 26, 2012 -just
at the time the CA rendered its Decision dated a few months before the Biosafety Permits
May 17, 2013, the reliefs petitioner sought and expired and when the field testing activities
granted by the CA were no longer capable of were already over.[108] Obviously, therefore,
execution. the cessation of the subject field tests before
the case could be resolved was due to
As the matter never went beyond the field respondents' own inaction.
testing phase, none of the foregoing tasks
related to propagation were pursued or the Moreover, the situation respondents complain
requirements therefor complied with. Thus, of is not susceptible to repetition. As discussed
there are no guaranteed after-effects to the above, DAO 08-2002 has already been
already concluded Bt talong field trials that superseded by JDC 01-2016. Hence, future
demand an adjudication from which the public applications for field testing will be governed
Page 89 of 121
403 CASE DIGESTS | CONSTITUTIONAL LAW REVIEW | JUDGE ESTELA ALMA SINGCO | FIRST SEMESTER | 2020-2021

by JDC 01-2016 which, as illustrated, adopts a methods of pesticides application because it


regulatory framework that is substantially exposed the residents to a higher degree of
different from that of DAO 08-2002. health risk caused by aerial drift;[15] and that
the ordinance enjoyed the presumption of
In fact, in relation to the latter, it is observed constitutionality, and could be invalidated only
that the Court should not have even delved upon a clear showing that it had violated the
into the constitutionality of DAO 08-2002 as it Constitution.
was merely collaterally challenged by
respondents, based on the constitutional On January 9, 2009, the CA promulgated its
precepts of the people's rights to information assailed decision reversing the judgment of
on matters of public concern, to public the RTC.[22] It declared Section 5 of
participation, to a balanced and healthful Ordinance No. 0309-07 as void and
ecology, and to health. unconstitutional for being unreasonable and
oppressive;
All told, with respondents' petition for Writ of
Kalikasan already mooted by the expiration of The CA did not see any established relation
the Biosafety Permits and the completion of between the purpose of protecting the public
the field trials subject of these cases, and with and the environment against the harmful
none of the exceptions to the mootness effects of aerial spraying, on one hand, and the
principle properly attending, the Court grants imposition of the ban against aerial spraying of
the instant motions for reconsideration and all forms of substances, on the other.
hereby dismisses the aforesaid petition. With
this pronouncement, no discussion on the Issues:
substantive merits of the same should be
made. whether or not Ordinance No. 0309-07 is
unconstitutional on due process and equal
Principles: protection grounds for being unreasonable and
oppressive, and an invalid exercise of police
At this point, the Court discerns that there are power: (a) in imposing a ban on aerial
two (2) factors to be considered before a case spraying as an agricultural practice in Davao
is deemed one capable of repetition yet City under Section 5; (b) in decreeing a 3-
evading review: (1) the challenged action was month transition-period to shift to other
in its duration too short to be fully litigated modes of pesticide application under Section
prior to its cessation or expiration; and (2) 5; and (c) in requiring the maintenance of the
there was a reasonable expectation that the 30-meter buffer zone under Section 6 thereof
same complaining party would be subjected to in all agricultural lands in Davao City.
the same action.
Ruling:
135 Mosqueda, et al. v. Pilipino Banana
Growers & Exporters Association, Inc., et The Sangguniang Bayan of Davao City enacted
al., GR No. 189185, August 16, 2016 Ordinance No. 0309-07under its corporate
powers... the right to a balanced and healthful
Facts: ecology under Section 16 is an issue of
transcendental importance with
After several committee hearings and intergenerational implications. It is under this
consultations with various stakeholders, the milieu that the questioned ordinance should be
Sangguniang Panlungsod of Davao City appreciated.
enacted Ordinance No. 0309, Series of 2007,
to impose a ban against aerial spraying as an Advancing the interests of the residents who
agricultural practice by all agricultural entities are vulnerable to the alleged health risks due
within Davao City to their exposure to pesticide drift justifies the
motivation behind the enactment of the
The Pilipino Banana Growers and Exporters ordinance. The City of Davao has the authority
Association, Inc. (PBGEA) and two of its to enact pieces of legislation that will promote
members, namely: Davao Fruits Corporation the general welfare, specifically the health of
and Lapanday Agricultural and Development its constituents. Such authority should not be
Corporation (PBGEA, et al.), filed their petition construed, however, as a valid license for the
in the RTC to challenge the constitutionality of City of Davao to enact any ordinance it deems
the ordinance fit to discharge its mandate. A thin but well-
defined line separates authority to enact
They alleged that the ordinance exemplified legislations from the method of accomplishing
the unreasonable exercise of police power; the same.
violated the equal protection clause;
amounted to the confiscation of property Ordinance No. 0309-07 violates the Due
without due process of law; and lacked Process Clause
publication pursuant] to Section 511[6] of
Republic Act No. 7160 A valid ordinance must not only be enacted
within the corporate powers of the local
On September 22, 2007, after trial, the RTC government and passed according to the
rendered judgment declaring Ordinance No. procedure prescribed by law.[108] In order to
0309-07 valid and constitutional declare it as a valid piece of local legislation, it
must also comply with the following
The RTC opined that the City of Davao had substantive requirements, namely: (1) it must
validly exercised police power[13] under the not contravene the Constitution or any
General Welfare Clause of the Local statute; (2) it must be fair, not oppressive; (3)
Government Code;[14] that the ordinance, it must not be partial or discriminatory; (4) it
being based on a valid classification, was must not prohibit but may regulate trade; (5)
consistent with the Equal Protection Clause; it must be general and consistent with public
that aerial spraying was distinct from other policy; and (6) it must not be
Page 90 of 121
403 CASE DIGESTS | CONSTITUTIONAL LAW REVIEW | JUDGE ESTELA ALMA SINGCO | FIRST SEMESTER | 2020-2021

unreasonable.[109]In the State's exercise of the law demands that the State governs
police power, the property rights of individuals impartially, and not to draw distinctions
may be subjected to restraints and burdens in between individuals solely on differences that
order to fulfill the objectives of the are irrelevant to the legitimate governmental
Government.[110] A local government unit is objective.
considered to have properly exercised its
police powers only if it satisfies the following Equal treatment neither requires universal
requisites, to wit: (1) the interests of the application of laws to all persons or things
public generally, as distinguished from those without distinction,[120] nor intends to
of a particular class, require the interference of prohibit legislation by limiting the object to
the State; and (2) the means employed are which it is directed or by the territory in which
reasonably necessary for the attainment of the it is to operate.[121] The guaranty of equal
object sought to be accomplished and not protection envisions equality among equals
unduly oppressive.[111] The first requirement determined according to a valid
refers to the Equal Protection Clause of the classification.[122] If the groupings are
Constitution; the second, to the Due Process characterized by substantial distinctions that
Clause of the Constitution.[112]Substantive make real differences, one class may be
due process requires that a valid ordinance treated and regulated differently from
must have a sufficient justification for the another.[123] In other word, a valid
Government's action.[113] This means that in classification must be: (1) based on
exercising police power the local government substantial distinctions; (2) germane to the
unit must not arbitrarily, whimsically or purposes of the law; (3) not limited to existing
despotically enact the ordinance regardless of conditions only; and (4) equally applicable to
its salutary purpose. So long as the ordinance all members of the class.
realistically serves a legitimate public purpose,
and it employs means that are reasonably In our view, the petitioners correctly argue
necessary to achieve that purpose without that the rational basis approach appropriately
unduly oppressing the individuals regulated, applies herein. Under the rational basis test,
the ordinance must survive a due process we shall: (1) discern the reasonable
challenge. relationship between the means and the
purpose of the ordinance; and (2) examine
The required civil works for the conversion to whether the means or the prohibition against
truck-mounted boom spraying alone will aerial spraying is based on a substantial or
consume considerable time and financial reasonable distinction. A reasonable
resources given the topography and classification includes all persons or things
geographical features of the plantations.[117] similarly situated with respect to the purpose
As such, the conversion could not be of the law.
completed within the short timeframe of three
months. Requiring the respondents and other Davao City justifies the prohibition against
affected individuals to comply with the aerial spraying by insisting that the occurrence
consequences of the ban within the three- of drift causes inconvenience and harm to the
month period under pain of penalty like fine, residents and degrades the environment.
imprisonment and even cancellation of Given this justification, does the ordinance
business permits would definitely be satisfy the requirement that the classification
oppressive as to constitute abuse of police must rest on substantial distinction?We
power. answer in the negative.

The respondents posit that the requirement of The occurrence of pesticide drift is not limited
maintaining a buffer zone under Section 6 of to aerial spraying but results from the conduct
the ordinance violates due process for being of any mode of pesticide application. Even
confiscatory; and that the imposition unduly manual spraying or truck-mounted boom
deprives all agricultural landowners within spraying produces drift that may bring about
Davao City of the beneficial use of their the same inconvenience, discomfort and
property that amounts to taking without just alleged health risks to the community and to
compensation. the environment.[141] A ban against aerial
spraying does not weed out the harm that the
The position of the respondents is untenable. ordinance seeks to achieve.[142] In the
process, the ordinance suffers from being
In City of Manila v. Laguio, Jr.,[118] we have "underinclusive" because the classification
thoroughly explained that taking only becomes does not include all individuals tainted with the
confiscatory if it substantially divests the same mischief that the law seeks to
owner of the beneficial use of its property eliminate.[143] A classification that is
drastically underinclusive with respect to the
Ordinance No. 0309-07 violates the Equal purpose or end appears as an irrational means
Protection Clause to the legislative end because it poorly serves
the intended purpose of the law.
The constitutional right to equal protection
requires that all persons or things similarly WHEREFORE, the Court DENIES the
situated should be treated alike, both as to consolidated petitions for review on certiorari
rights conferred and responsibilities imposed. for their lack of merit; AFFIRMS the decision
It requires public bodies and institutions to promulgated on January 9, 2009 in C.A.-G.R.
treat similarly situated individuals in a similar CV No. 01389-MIN. declaring Ordinance No.
manner. The guaranty equal protection 0309-07 UNCONSTITUTIONAL;
secures every person within the State's
jurisdiction against intentional and arbitrary Principles:
discrimination, whether occasioned by the
express terms of a statue or by its improper Constitutional Law... the right to a balanced
execution through the State's duly constituted and healthful ecology under Section 16 is an
authorities. The concept of equal justice under
Page 91 of 121
403 CASE DIGESTS | CONSTITUTIONAL LAW REVIEW | JUDGE ESTELA ALMA SINGCO | FIRST SEMESTER | 2020-2021

issue of transcendental importance with The petitioners in G.R. No. 180771 are the
intergenerational implications. “Resident Marine Mammals” which inhibit the
waters in and around the Tañon Strait, joined
Political Law... taking only becomes by “Stewards” Gloria Estenzo Ramos and
confiscatory if it substantially divests the Rose-Liza Eisma-Osorio as their legal
owner of the beneficial use of its property. guardians and friends seeking their protection.
Also impleaded as unwilling co-petitioner is
136 Resident Marine Mammals of the former President Gloria Macapagal-Arroyo.
Protected Seascape Tanon straits, et al. v.
Secretary Angelo Reyes, et al., GR In G.R. No. 181527, the petitioners are
No.180771, April 21, 2016 the Central Visayas Fisherfolk
Development Center (FIDEC), a non-stock,
FACTS: non-profit, non-governmental organization
established for the welfare of the marginal
On 13 June 2002, the Government of the fisherfolk in Region VII and
Philippines, acting through the Department of representatives of the subsistence fisherfolk
Energy (DOE) entered into a Geophysical of the municipalities of Aloguinsan and
Survey and Exploration Contract-102 (GSEC- Pinamungajan, Cebu.
102) with Japan Petroleum Exploration Co.,
Ltd. (JAPEX). Their contentions are:
A study made after the seismic survey showed
The studies included surface geology, sample that there is a drastic reduce in fish catch by
analysis, and reprocessing of seismic and 50-70% attributable to the destruction of the
magnetic data. Geophysical and satellite “payao” or the artificial reef.
surveys as well as oil and gas sampling in
Tañon Strait was conducted. The ECC obtained by the respondents
is invalid because there is no public
On 12 December 2004, DOE and JAPEX consultations and discussions prior to its
converted GSEC-102 to Service Contract No. issuance.
46 (SC-46) for the exploration, development,
and production of petroleum resources in a SC-46 is null and void for having violated
block covering approximately 2,850 sqm. Section 2, Article XII of the 1987 Constitution,
offshore the Tañon Strait. considering that there is no general law
prescribing the standard or uniform terms,
From 9-18 May 2005, JAPEX conducted conditions, and requirements for service
seismic surveys in and around Tañon Strait, contracts involving oil exploration and
including a multi-channel sub-bottom extraction
profiling covering approximately 751 kms.
to determine the area’s underwater FIDEC alleges that it was barred from entering
composition. and fishing within a 7-kilometer radius from
the point where the oilrig was located, an area
During the 2nd sub-phase of the project, JAPEX grated than the 1.5-kilometer radius exclusion
committed to drill one exploration well. Since zone stated in the Initial Environmental
the same was to be drilled in the marine Examination
waters of Aloguisan and Pinamungajan where
the Tañon Strait was declared a protected The respondents in both petitions are: the late
seascape in 1988, JAPEX agreed to comply Angelo T. Reyes, DOE Secretary; Jose L.
with the Environmental Impact Atienza, DENR Secretary; Leonardo
Assessment requirements under Sibbaluca, DENR-Region VII Director and
Presidential Decree No. 1586 (PD 1586), Chairman of Tañon Strait PAMB; JAPEX, a
entitled “Establishing an Environmental Japanese company; and Supply Oilfield
Impact Statement System, Including Other Services, Inc. (SOS) as the alleged Philippine
Environmental Management Related Measures agent of JAPEX. Their counter-allegations
and For Other Purposes.” are:The “Resident Marine Mammals” and
“Stewards” have no legal standing to file the
On 31 January 2007, the Protected Area petition. SC-46 is constitutional. The ECC was
Management Board (PAMB) of the Tañon Strait legally issued. The case is moot and academic
issued Resolution No. 2007-01 where it since SC-46 is mutually terminated on 21 June
adopted the Initial Environmental Examination 2008.
commissioned by JAPEX, and favourably
recommended the approval of the latter’s ISSUES
application for an Environmental Compliance 1. WON the case is moot and academic
Certificate (ECC). 2. WON Petitioners have a legal standing
3. WON SC-46 is unconstitutional
On 6 March 2007, DENR-EMB Region VII
granted an ECC to DOE and JAPEX for the RULING
offshore oil and gas exploration project in 1. No. The Court makes clear that the “moot
Tañon Strait. and academic” principle is not a magic formula
that can automatically dissuade the courts in
From 16 November 2007 to 8 February 2008, resolving a case. Despite the termination of
JAPEX drilled an exploratory well with a depth SC-46, the Court deems it necessary to resolve
of 3,150 meters near Pinamungajan town. the consolidated petitions as it falls within the
exceptions. Both petitioners allege that SC-46
On 17 December 2007, two separate original is violative of the Constitution, the
petitions were filed commonly seeking that the environmental and livelihood issues raised
implementation of SC-46 be enjoined for undoubtedly affect the public’s interest, and
violation of the 1987 Constitution. the respondents’ contested actions are capable
of repetition.

Page 92 of 121
403 CASE DIGESTS | CONSTITUTIONAL LAW REVIEW | JUDGE ESTELA ALMA SINGCO | FIRST SEMESTER | 2020-2021

2. Yes. In our jurisdiction, locus standi in this provision, within thirty days from its
environmental cases has been given a execution.” (Emphases supplied)
more liberalized approach. The Rules of
Procedure for Environmental Cases allow The disposition, exploration, development,
for a “citizen suit,” and permit any Filipino exploitation, and utilization of indigenous
citizen to file an action before our courts petroleum in the Philippines are governed by
for violation of our environmental laws on the Presidential Decree No. 87 (PD 87) or the Oil
principle that humans are stewards of nature: Exploration and Development Act of 1972.
Although the Court finds that PD 87 is
“Section 5. Citizen suit. – sufficient to satisfy the requirement of a
Any Filipino citizen in representation of general law, the absence of the two other
others, including minors or generations yet conditions, that the President be a signatory to
unborn, may file an action SC-46, and that the Congress be notified of
to enforce rights or obligations under such contract, renders it null and void. SC-46
environmental laws. appears to have been entered into and signed
by the DOE through its then Secretary Vicente
Upon the filing of a citizen suit, the court shall S. Perez, Jr. Moreover, public respondents
issue an order which shall contain a brief have neither shown nor alleged that Congress
description of the cause of action and the was subsequently notified of the execution of
reliefs prayed for, requiring all interested such contract.
parties to manifest their interest to
intervene in the case Service contracts involving the exploitation,
within fifteen (15) days from notice thereof. development, and utilization of our natural
The plaintiff may publish the order once in resources are of paramount interest to the
a newspaper of general circulation in present and future generations. Hence,
the Philippines or furnish all affected safeguards were out in place to insure that the
barangays copies of said order. guidelines set by law are meticulously
observed and likewise eradicate the corruption
Citizen suits filed under R.A. No. 8749 that may easily penetrate departments and
and R.A. No. 9003 shall be governed by agencies by ensuring that the President
their respective provisions. (Emphasis has authorized or approved of the
supplied)” service
contracts herself.
Although the petition was filed in 2007, years
before the effectivity of the Rules of Even under the provisions of PD 87, it is
Procedure for Environmental Cases, it required that the Petroleum Board, now the
has been consistently held that rules of DOE, obtain the President’s approval for the
procedure may be retroactively applied to execution of any contract under said statute.
actions pending and undetermined at the time
of their passage and will not violate The SC likewise ruled on the legality of SC-46
any right of a person who may feel that vis-à-vis other pertinent laws to serve as a
he is adversely affected, inasmuch as there guide for the Government when executing
is no vested rights in rules of procedure. service contracts. Under Proclamation No.
Moreover, even before the Rules of Procedure 2146, the Tañon Strait is an
for Environmental Cases became effective, the environmentally critical
SC had already taken a permissive position on area, having been declared as a protected area
the issue of in 1998; therefore, any activity outside the
locus standi in environmental cases. scope of its management plan may only be
implemented pursuant to an ECC secured after
In Oposa, the SC allowed the suit to be undergoing an Environment Impact
brought in the name of generations yet unborn Assessment (EIA) to determine the effects of
“based on the concept of intergenerational such activity on its ecological system.
responsibility insofar as the right to a balanced
and healthful ecology is concerned.” It is also Public respondents admitted that JAPEX only
worth noting that the Stewards in the present started to secure an ECC prior to the 2nd sub-
case are joined as real parties in the Petition phase of SC-46, which required the drilling of
and not just in representation of the named the exploration well. This means that no
cetacean species. environmental impact evaluation was done
when the seismic surveys were conducted.
3. Yes. Section 2, Article XII of the 1987 Unless the seismic surveys are part of the
Constitution provides in part: management plan of the Tañon
“ The President may enter into Strait, such surveys were done in violation of
agreement with foreign-owned corporations Section 12 of NIPAS Act and Section 4 of
involving either technical or financial Presidential Decree No. 1586.
assistance for large-scale exploration,
development, and utilization of minerals, While PD 87 may serve as the general law
petroleum, and other mineral oils according to upon which a service contract for petroleum
the general terms and conditions provided exploration and extraction may be authorized,
by law, based on real contributions to the exploitation and utilization of this energy
the resource in the present case may be allowed
economic growth and general welfare of only through a law passed by Congress, since
the country. the Tañon Strait is a NIPAS area. Since there
is no such law specifically allowing oil
In such agreements, the State shall exploration and/or extraction in the
promote the development and use of Tañon Strait, no energy resource
local scientific and technical resources. The exploitation and utilization may be done in said
President shall notify the Congress of every protected seascape.
contract entered into in accordance with

Page 93 of 121
403 CASE DIGESTS | CONSTITUTIONAL LAW REVIEW | JUDGE ESTELA ALMA SINGCO | FIRST SEMESTER | 2020-2021

situations which are different in fact or opinion


to be treated in law as though they were the
LOCAL AUTONOMY
same.
137 Basco vs. Pagcor G.R. No. 91649 May 14,
1991 (4) No. The judiciary does not settle policy
issues. The Court can only declare what the
FACTS: The PH Amusement and Gaming Corp. law is and not what the law should be. Under
was created by PD 1067-A and granted a our system of government, policy issues are
franchise under PD 1067-B. Subsequently, within the domain of the political branches of
under PD 1869, the Government enabled it to government and of the people themselves as
regulate and centralize all games of chance the repository of all state power. On the issue
authorized by existing franchise or permitted of monopoly, the same is not necessarily
by law, under declared policy. But the prohibited by the Constitution. The state must
petitioners think otherwise, that is why, they still decide whether public interest demands
filed the instant petition seeking to annul the that monopolies be "regulated" or prohibited.
PAGCOR Charter — PD 1869, because it is Again, this is a matter of policy for the
allegedly contrary to morals, public policy and Legislature to decide. The judiciary can only
order, and because of the following issues: intervene when there are violations of the
statutes passed by Congress regulating or
ISSUES: prohibiting monopolies.

(1) WON it waived the Manila City gov't's right


to impose taxes and license fees, which is 138 Province of North Cotabato vs. GRP Peace
recognized by law. Panel, (GR No. 183591, Oct. 14, 2008)

(2) WON it has intruded into the LGUs' right to


impose local taxes and license fees, and thus Facts:
contrary to the principle of local autonomy
enshrined in the Constitution. Subject of this case is the Memorandum of
Agreement on the Ancestral Domain (MOA-
(3) WON it violates the equal protection AD) which is scheduled to be signed by the
clause as it allows some gambling acts but also Government of the Republic of the Philippines
prohibits other gaming acts. and the MILF in August 05, 2008. Five cases
bearing the same subject matter were
(4) WON it violates the Cory gov't's policy of consolidated by this court.
being away from monopolistic and crony
economy, and toward free enterprise and The MOA-AD is a result of various agreements
privatization. entered into by and between the government
and the MILF starting in 1996; then in 1997,
HELD: they signed the Agreement on General
Cessation of Hostilities; and the following year,
they signed the General Framework of
(1) No. The fact that PAGCOR, under its
Agreement of Intent on August 27, 1998.
charter, is exempt from paying tax of any kind
However, in 1999 and in the early of 2000, the
is not violative of the principle of local
MILF attacked a number of municipalities in
autonomy. LGUs' have no inherent right to
Central Mindanao. In March 2000, they took
impose taxes. LGUs' power to tax must always
the hall of Kauswagan, Lanao del Norte;
yield to a legislative act which is superior
hence, then Pres. Estrada declared an all-out
having been passed by the state itself which
war-which tolled the peace negotiation. It was
has the inherent power to tax. The charter of
when then Pres. Arroyo assumed office, when
LGUs is subject to control by Congress as they
the negotiation regarding peace in Mindanao
are mere creatures of Congress. Congress,
continued. MILF was hesitant; however, this
therefore, has the power of control over LGUs.
negotiation proceeded when the government
And if Congress can grant the City of Manila
of Malaysia interceded. Formal peace talks
the power to tax certain matters, it can also
resumed and MILF suspended all its military
provide for exemptions or even take back the
actions. The Tripoli Agreement in 2001 lead to
power.
the ceasefire between the parties. After the
death of MILF Chairman Hashim and Iqbal took
(2) No. LGUs' right to impose license fees on over his position, the crafting of MOA-AD in its
"gambling", has long been revoked. As early final form was born.
as 1975, the power of local governments to
regulate gambling thru the grant of "franchise,
MOA-AD Overview
licenses or permits" was withdrawn by P.D. No.
771 and was vested exclusively on the
National Government. Furthermore, LGUs' This is an agreement to be signed by the GRP
have no power to tax instrumentalities of the and the MILF. Used as reference in the birth of
gov't such as PAGCOR which exercises this MOA-AD are the Tripoli Agreement,
governmental functions of regulating gambling organic act of ARMM, IPRA Law, international
activities. laws such as ILO Convention 169, the UN
Charter etc., and the principle of Islam i.e
compact right entrenchment (law of compact,
(3) No. The clause does not preclude
treaty and order). The body is divided into
classification of individuals who may be
concepts and principles, territory, resources,
accorded different treatment under the law as
and governance.
long as the classification is not unreasonable
or arbitrary. A law does not have to operate in
equal force on all persons or things to be Embodied in concepts and principles, is the
conformable to Article III, Section 1 of the definition of Bangsamoro as all indigenous
Constitution. The Constitution does not require peoples of Mindanao and its adjacent islands.

Page 94 of 121
403 CASE DIGESTS | CONSTITUTIONAL LAW REVIEW | JUDGE ESTELA ALMA SINGCO | FIRST SEMESTER | 2020-2021

These people have the right to self- discussed in the comprehensive compact as
governance of their Bangsamoro homeland to well.
which they have exclusive ownership by virtue
of their prior rights of occupation in the land. Issues:
The MOA-AD goes on to describe the
Bangsamoro people as "the ‘First Nation' with 1. WON respondents violate constitutional
defined territory and with a system of and statutory provisions on public consultation
government having entered into treaties of and the right to information when they
amity and commerce with foreign nations." It negotiated and later initialed the MOA-AD; and
then mentions for the first time the
"Bangsamoro Juridical Entity" (BJE) to which it
2. WON the contents of the MOA-AD violated
grants the authority and jurisdiction over the
the Constitution and the laws
Ancestral Domain and Ancestral Lands of the
Bangsamoro.
Ruling:
As defined in the territory of the MOA-AD, the
BJE shall embrace the Mindanao-Sulu-Palawan The SC declared the MOA-AD contrary to law
geographic region, involving the present and the Constitution.
ARMM, parts of which are those which voted in
the inclusion to ARMM in a plebiscite. The The SC ruled that the MOA-AD is a matter of
territory is divided into two categories, “A” public concern, involving as it does the
which will be subject to plebiscite not later sovereignty and territorial integrity of the
than 12 mos. after the signing and “B” which State, which directly affects the lives of the
will be subject to plebiscite 25 years from the public at large.
signing of another separate agreement.
Embodied in the MOA-AD that the BJE shall As enshrined in the Constitution, the right to
have jurisdiction over the internal waters- information guarantees the right of the people
15kms from the coastline of the BJE territory; to demand information, and integrated therein
they shall also have "territorial waters," which is the recognition of the duty of the officialdom
shall stretch beyond the BJE internal waters up to give information even if nobody demands.
to the baselines of the Republic of the The policy of public disclosure establishes a
Philippines (RP) south east and south west of concrete ethical principle for the conduct of
mainland Mindanao; and that within these public affairs in a genuinely open democracy,
territorial waters, the BJE and the government with the people's right to know as the
shall exercise joint jurisdiction, authority and centerpiece. It is a mandate of the State to be
management over all natural resources. There accountable by following such policy. These
will also be sharing of minerals in the territorial provisions are vital to the exercise of the
waters; but no provision on the internal freedom of expression and essential to hold
waters. public officials at all times accountable to the
people.
Included in the resources is the stipulation that
the BJE is free to enter into any economic Also, it was held that such stipulation in the
cooperation and trade relations with foreign Constitution is self-executory with reasonable
countries and shall have the option to establish safeguards —the effectivity of which need not
trade missions in those countries, as well as await the passing of a statute. Hence, it is
environmental cooperation agreements, but essential to keep open a continuing dialogue or
not to include aggression in the GRP. The process of communication between the
external defense of the BJE is to remain the government and the people. It is in the
duty and obligation of the government. The interest of the State that the channels for free
BJE shall have participation in international political discussion be maintained to the end
meetings and events" like those of the ASEAN that the government may perceive and be
and the specialized agencies of the UN. They responsive to the people's will.
are to be entitled to participate in Philippine
official missions and delegations for the The idea of a feedback mechanism was also
negotiation of border agreements or protocols sought for since it is corollary to the twin rights
for environmental protection and equitable to information and disclosure. And feedback
sharing of incomes and revenues involving the means not only the conduct of the plebiscite as
bodies of water adjacent to or between the per the contention of the respondents. Clearly,
islands forming part of the ancestral domain. what the law states is the right of the
The BJE shall also have the right to explore its petitioners to be consulted in the peace
resources and that the sharing between the agenda as corollary to the constitutional right
Central Government and the BJE of total to information and disclosure. As such,
production pertaining to natural resources is to respondent Esperon committed grave abuse of
be 75:25 in favor of the BJE. And they shall discretion for failing to carry out the furtive
have the right to cancel or modify concessions process by which the MOA-AD was designed
and TLAs. and crafted runs contrary to and in excess of
the legal authority, and amounts to a
And lastly in the governance, the MOA-AD whimsical, capricious, oppressive, arbitrary
claims that the relationship between the GRP and despotic exercise thereto. Moreover, he
and MILF is associative i.e. characterized by cannot invoke of executive privilege because
shared authority and responsibility. This he already waived it when he complied with
structure of governance shall be further the Court’s order to the unqualified disclosure
discussed in the Comprehensive Compact, a of the official copies of the final draft of the
stipulation which was highly contested before MOA-AD.
the court. The BJE shall also be given the right
to build, develop and maintain its own In addition, the LGU petitioners has the right
institutions, the details of which shall be to be involved in matters related to such peace
talks as enshrined in the State policy. The
Page 95 of 121
403 CASE DIGESTS | CONSTITUTIONAL LAW REVIEW | JUDGE ESTELA ALMA SINGCO | FIRST SEMESTER | 2020-2021

MOA-AD is one peculiar program that become part of the ARMM. The stipulation of
unequivocally and unilaterally vests ownership the respondents in the MOA-AD that these
of a vast territory to the Bangsamoro people, areas need not participate in the plebiscite is
which could pervasively and drastically result in contrary to the express provision of the
to the diaspora or displacement of a great Constitution. The law states that that "[t]he
number of inhabitants from their total creation of the autonomous region shall be
environment. effective when approved by a majority of the
votes cast by the constituent units in a
With respect to the ICC/IPPs they also have plebiscite called for the purpose, provided that
the right to participate fully at all levels on only provinces, cities, and geographic areas
decisions that would clearly affect their lives, voting favorably in such plebiscite shall be
rights and destinies. The MOA-AD is an included in the autonomous region." Clearly,
instrument recognizing ancestral domain, assuming that the BJE is just an expansion of
hence it should have observed the free and the ARMM, it would still run afoul the wordings
prior informed consent to the ICC/IPPs; but it of the law since those included in its territory
failed to do so. More specially noted by the are areas which voted in its inclusion to the
court is the excess in authority exercised by ARMM and not to the BJE.
the respondent—since they allowed
delineation and recognition of ancestral On the powers vested in the BJE as an entity.
domain claim by mere agreement and The respondents contend that the powers
compromise; such power cannot be found in vested to the BJE in the MOA-AD shall be
IPRA or in any law to the effect. within sub-paragraph 9 of sec 20, art. 10 of
the constitution and that a mere passage of a
With regard to the provisions of the MOA-AD, law is necessary in order to vest in the BJE
there can be no question that they cannot be powers included in the agreement. The Court
all accommodated under the present was not persuaded. SC ruled that such
Constitution and laws. Not only its specific conferment calls for amendment of the
provisions but the very concept underlying Constitution; otherwise new legislation will not
them: concur with the Constitution. Take for instance
the treaty making power vested to the BJE in
On matters of the Constitution. the MOA-AD. The Constitution is clear that only
the President has the sole organ and is the
country’s sole representative with foreign
Association as the type of relationship
nation. Should the BJE be granted with the
governing between the parties. The parties
authority to negotiate with other states, the
manifested that in crafting the MOA-AD, the
former provision must be amended
term association was adapted from the
consequently. Section 22 must also be
international law. In international law,
amended—the provision of the law that
association happens when two states of equal
promotes national unity and development.
power voluntarily establish durable links i.e.
Because clearly, associative arrangement of
the one state, the associate, delegates certain
the MOA-AD does not epitomize national unity
responsibilities to the other, principal, while
but rather, of semblance of unity. The
maintaining its international status as state;
associative ties between the BJE and the
free association is a middle ground between
national government, the act of placing a
integration and independence. The MOA-AD
portion of Philippine territory in a status which,
contains many provisions that are consistent
in international practice, has generally been a
with the international definition of association
preparation for independence, is certainly not
which fairly would deduced that the agreement
conducive to national unity.
vest into the BJE a status of an associated
state, or at any rate, a status closely
approximating it. The court vehemently On matters of domestic statutes.
objects because the principle of association is
not recognized under the present Constitution. - Provisions contrary to the organic act of
ARMM. RA 9054 is a bar to the adoption of the
On the recognition of the BJE entity as a state. definition of Bangsamoro people used in the
The concept implies power beyond what the MOA-AD. Said law specifically distinguishes
Constitution can grant to a local government; between the Bangsamoro people and the
even the ARMM do not have such recognition; Tribal peoples that is contrary with the
and the fact is such concept implies recognition definition of the MOA-AD which includes all
of the associated entity as a state. There is indigenous people of Mindanao.
nothing in the law that contemplate any state
within the jurisdiction other than the Philippine - Provisions contrary to the IPRA law. Also,
State, much less does it provide for a the delineation and recognition of the ancestral
transitory status that aims to prepare any part domain is a clear departure from the
of Philippine territory for independence. The procedure embodied in the IPRA law which
court disagrees with the respondent that the ironically is the term of reference of the MOA-
MOA-AD merely expands the ARMM. BJE is a AD.
state in all but name as it meets the criteria of
a state laid down in the Montevideo On matters of international law.
Convention, namely, a permanent population,
a defined territory, a government, and a The Philippines adopts the generally accepted
capacity to enter into relations with other principle of international law as part of the law
states. As such the MOA-AD clearly runs of the land. In international law, the right to
counter to the national sovereignty and self-determination has long been recognized
territorial integrity of the Republic. which states that people can freely determine
their political status and freely pursue their
On the expansion of the territory of the BJE. economic, social, and cultural development.
The territory included in the BJE includes those There are the internal and external self-
areas who voted in the plebiscite for them to determination—internal, meaning the self-
Page 96 of 121
403 CASE DIGESTS | CONSTITUTIONAL LAW REVIEW | JUDGE ESTELA ALMA SINGCO | FIRST SEMESTER | 2020-2021

pursuit of man and the external which takes specific duty to prevent and suppress rebellion
the form of the assertion of the right to and lawless violence.
unilateral secession. This principle of self-
determination is viewed with respect accorded As such, the President is given the leeway to
to the territorial integrity of existing states. explore, in the course of peace negotiations,
External self-determination is only afforded in solutions that may require changes to the
exceptional cases when there is an actual block Constitution for their implementation. At all
in the meaningful exercise of the right to event, the president may not, of course,
internal self-determination. International law, unilaterally implement the solutions that she
as a general rule, subject only to limited and considers viable; but she may not be
exceptional cases, recognizes that the right of prevented from submitting them as
disposing national territory is essentially an recommendations to Congress, which could
attribute of the sovereignty of every state. then, if it is minded, act upon them pursuant
to the legal procedures for constitutional
On matters relative to indigenous people, amendment and revision.
international law states that indigenous
peoples situated within states do not have a While the President does not possess
general right to independence or secession constituent powers - as those powers may be
from those states under international law, but exercised only by Congress, a Constitutional
they do have rights amounting to what was Convention, or the people through initiative
discussed above as the right to internal self- and referendum - she may submit proposals
determination; have the right to autonomy or for constitutional change to Congress in a
self-government in matters relating to their manner that does not involve the arrogation of
internal and local affairs, as well as ways and constituent powers. Clearly, the principle may
means for financing their autonomous be inferred that the President - in the course
functions; have the right to the lands, of conducting peace negotiations - may validly
territories and resources which they have consider implementing even those policies that
traditionally owned, occupied or otherwise require changes to the Constitution, but she
used or acquired. may not unilaterally implement them without
the intervention of Congress, or act in any way
Clearly, there is nothing in the law that as if the assent of that body were assumed as
required the State to guarantee the indigenous a certainty. The President’s power is limited
people their own police and security force; but only to the preservation and defense of the
rather, it shall be the State, through police Constitution but not changing the same but
officers, that will provide for the protection of simply recommending proposed amendments
the people. With regards to the autonomy of or revisions.
the indigenous people, the law does not
obligate States to grant indigenous peoples - The Court ruled that the suspensive clause is
the near-independent status of a state; since not a suspensive condition but is a term
it would impair the territorial integrity or because it is not a question of whether the
political unity of sovereign and independent necessary changes to the legal framework will
states. take effect; but, when. Hence, the stipulation
is mandatory for the GRP to effect the changes
On the basis of the suspensive clause. to the legal framework –which changes would
include constitutional amendments. Simply
- It was contented by the respondents that put, the suspensive clause is inconsistent with
grave abuse of discretion cannot be had, since the limits of the President's authority to
the provisions assailed as unconstitutional propose constitutional amendments, it being a
shall not take effect until the necessary virtual guarantee that the Constitution and the
changes to the legal framework are effected. laws of the Republic of the Philippines will
certainly be adjusted to conform to all the
The Court is not persuaded. This suspensive "consensus points" found in the MOA-AD.
clause runs contrary to Memorandum of Hence, it must be struck down as
Instructions from the President stating that unconstitutional.
negotiations shall be conducted in accordance
to the territorial integrity of the country—such On the concept underlying the MOA-AD.
was negated by the provision on association
incorporated in the MOA-AD. Apart from this, While the MOA-AD would not amount to an
the suspensive clause was also held invalid international agreement or unilateral
because of the delegated power to the GRP declaration binding on the Philippines under
Peace panel to advance peace talks even if it international law, respondents' act of
will require new legislation or even guaranteeing amendments is, by itself,
constitutional amendments. The legality of the already a constitutional violation that renders
suspensive clause hence hinges on the query the MOA-AD fatally defective. The MOA-AD not
whether the President can exercise such power being a document that can bind the Philippines
as delegated by EO No.3 to the GRP Peace under international law notwithstanding,
Panel. Well settled is the rule that the respondents' almost consummated act of
President cannot delegate a power that she guaranteeing amendments to the legal
herself does not possess. The power of the framework is, by itself, sufficient to constitute
President to conduct peace negotiations is not grave abuse of discretion. The grave abuse lies
explicitly mentioned in the Constitution but is not in the fact that they considered, as a
rather implied from her powers as Chief solution to the Moro Problem, the creation of a
Executive and Commander-in-chief. As Chief state within a state, but in their brazen
Executive, the President has the general willingness to guarantee that Congress and the
responsibility to promote public peace, and as sovereign Filipino people would give their
Commander-in-Chief, she has the more imprimatur to their solution. Upholding such
an act would amount to authorizing a
usurpation of the constituent powers vested
Page 97 of 121
403 CASE DIGESTS | CONSTITUTIONAL LAW REVIEW | JUDGE ESTELA ALMA SINGCO | FIRST SEMESTER | 2020-2021

only in Congress, a Constitutional Convention, alleged violations of the right of suffrage of the
or the people themselves through the process people of ARMM, as well as the failure to
of initiative, for the only way that the adhere to the "elective and representative"
Executive can ensure the outcome of the character of the executive and legislative
amendment process is through an undue departments of the ARMM. Lastly, the
influence or interference with that process. petitioners challenged the grant to the
President of the power to appoint OICs to
139 Abas Kida vs. Senate of the Philippines, undertake the functions of the elective ARMM
659 SCRA 270 officials until the officials elected under the
May 2013 regular elections shall have
FACTS: On August 1, 1989 or two years after assumed office. Corrolarily, they also argue
the effectivity of the 1987 Constitution, that the power of appointment also gave the
Congress acted through Republic Act (RA) No. President the power of control over the ARMM,
6734 entitled "An Act Providing for an Organic in complete violation of Section 16, Article X of
Act for the Autonomous Region in Muslim the Constitution.
Mindanao."The initially assenting provinces
were Lanao del Sur,Maguindanao, Sulu and ISSUE: Does the 1987 Constitution mandate
Tawi-tawi.RA No. 6734 scheduled the first the synchronization of elections?
regular elections for the regional officials of the
ARMM on a date not earlier than 60 days nor Does the passage of RA No. 10153 violate the
later than 90 days after its ratification. provisions of the 1987 Constitution?

Thereafter, R.A. No. 9054 was passed to HELD: Court dismissed the petition and
further enhance the structure of ARMM under affirmed the constitutionality of R.A. 10153 in
R.A. 6734. Along with it is the reset of the toto. The Court agreed with respondent Office
regular elections for the ARMM regional of the Solicitor General (OSG) on its position
officials to the second Monday of September that the Constitution mandates
2001. synchronization, citing Sections 1, 2 and 5,
Article XVIII (Transitory Provisions) of the
RA No. 9333was subsequently passed by 1987 Constitution. While the Constitution does
Congress to reset the ARMM regional elections not expressly state that Congress has to
to the 2ndMonday of August 2005, and on the synchronize national and local elections, the
same date every 3 years thereafter. Unlike RA clear intent towards this objective can be
No. 6734 and RA No. 9054, RA No. 9333 was gleaned from the Transitory Provisions (Article
not ratified in a plebiscite. XVIII) of the Constitution,which show the
extent to which the Constitutional
Pursuant to RA No. 9333, the next ARMM Commission, by deliberately making
regional elections should have been held adjustments to the terms of the incumbent
onAugust 8, 2011. COMELEC had begun officials, sought to attain synchronization of
preparations for these elections and had elections.
accepted certificates of candidacies for the
various regional offices to be elected.But The objective behind setting a common
onJune 30, 2011, RA No. 10153 was enacted, termination date for all elective officials, done
resetting the ARMM elections to May 2013, to among others through the shortening the
coincide with the regular national and local terms of the twelve winning senators with the
elections of the country.With the enactment least number of votes, is to synchronize the
into law of RA No. 10153, the COMELEC holding of all future elections whether national
stopped its preparations for the ARMM or local to once every three years.This
elections. intention finds full support in the discussions
during the Constitutional Commission
Several cases for certiorari, prohibition and deliberations. Furthermore, to achieve
madamus originating from different parties synchronization, Congress necessarily has to
arose as a consequence of the passage of R.A. reconcile the schedule of the ARMMs regular
No. 9333 and R.A. No. 10153 questioning the elections (which should have been held in
validity of said laws. August 2011 based on RA No. 9333) with the
fixed schedule of the national and local
elections (fixed by RA No. 7166 to be held in
OnSeptember 13, 2011, the Court issued a
May 2013).
temporary restraining order enjoining the
implementation of RA No. 10153 and ordering
the incumbent elective officials of ARMM to In Osme v. Commission on Elections, the court
continue to perform their functions should thus explained:
these cases not be decided by the end of their
term onSeptember 30, 2011. It is clear from the aforequoted provisions of
the 1987 Constitution that the terms of office
The petitioners assailing RA No. 9140, RA No. of Senators, Members of the House of
9333 and RA No. 10153 assert that these laws Representatives, the local officials, the
amend RA No. 9054 and thus, have to comply President and the Vice-President have been
with the supermajority vote and plebiscite synchronized to end on the same hour, date
requirements prescribed under Sections 1 and and year noon of June 30, 1992.
3, Article XVII of RA No. 9094 in order to
become effective. It is likewise evident from the wording of the
above-mentioned Sections that the term of
The petitions assailing RA No. 10153 further synchronization is used synonymously as the
maintain that it is unconstitutional for its phrase holding simultaneously since this is the
failure to comply with the three-reading precise intent in terminating their Office
requirement of Section 26(2), Article VI of the Tenure on the same day or occasion.This
Constitution.Also cited as grounds are the common termination date will synchronize
future elections to once every three years
Page 98 of 121
403 CASE DIGESTS | CONSTITUTIONAL LAW REVIEW | JUDGE ESTELA ALMA SINGCO | FIRST SEMESTER | 2020-2021

(Bernas, the Constitution of the Republic of the stay at three (3) years as fixed by the
Philippines, Vol. II, p. 605). Constitution and cannot be extended by
holdover by Congress.
That the election for Senators, Members of
the House of Representatives and the local RA No. 10153, does not in any way amend
officials (under Sec. 2, Art. XVIII) will have to what the organic law of the ARMM(RA No.
be synchronized with the election for President 9054) sets outs in terms of structure of
and Vice President (under Sec. 5, Art. XVIII) is governance.What RA No. 10153 in fact only
likewise evident from the x x xrecords of the does is to"appoint officers-in-charge for the
proceedings in the Constitutional Commission. Office of the Regional Governor, Regional Vice
Governor and Members of the Regional
Although called regional elections, the ARMM Legislative Assembly who shall perform the
elections should be included among the functions pertaining to the said offices until the
elections to be synchronized as it is a "local" officials duly elected in the May 2013 elections
election based on the wording and structure of shall have qualified and assumed office."This
the Constitution. Regional elections in the power is far different from appointing elective
ARMM for the positions of governor, vice- ARMM officials for the abbreviated term ending
governor and regional assembly on the assumption to office of the officials
representatives fall within the classification of elected in the May 2013 elections. It must be
"local" elections, since they pertain to the therefore emphasized that the law must be
elected officials who will serve within the interpreted as an interim measure to
limited region of ARMM. From the perspective synchronize elections and must not be
of the Constitution, autonomous regions are interpreted otherwise.
considered one of the forms of local
governments, as evident from Article Xof the
Constitution entitled "Local
LEGISLATIVE DEPARTMENT
Government."Autonomous regions are
established and discussed under Sections 15 140 Santiago vs. COMELEC, 3/19/97
to 21 of this Article the article wholly devoted
to Local Government. FACTS:
In 1996, Atty. Jesus Delfin filed with COMELEC
Second issue: Congress, in passing RA No. a petition to amend Constitution, to lift term
10153, acted strictly within its constitutional limits of elective officials, by people’s initiative.
mandate. Given an array of choices, it acted Delfin wanted COMELEC to control and
within due constitutional bounds and with supervise said people’s initiative the signature-
marked reasonableness in light of the gathering all over the country. The proposition
necessary adjustments that synchronization is: “Do you approve of lifting the term limits of
demands. Congress, therefore, cannot be all elective government officials, amending for
accused of any evasion of a positive duty or of the purpose Sections 4 ) and 7 of Article VI,
a refusal to perform its duty nor is there reason Section 4 of Article VII, and Section 8 of Article
to accord merit to the petitioners claims of 8 of Article X of the 1987 Philippine
grave abuse of discretion. Constitution?” Said Petition for Initiative will
first be submitted to the people, and after it is
In relation with synchronization, both signed by at least 12% total number of
autonomy and the synchronization of national registered voters in the country, it will be
and local elections are recognized and formally filed with the COMELEC.
established constitutional mandates, with one
being as compelling as the other.If their COMELEC in turn ordered Delfin for publication
compelling force differs at all, the difference is of the petition. Petitioners Sen. Roco et al
in their coverage; synchronization operates on moved for dismissal of the Delfin Petition on
and affects the whole country, while regional the ground that it is not the initiatory petition
autonomy as the term suggests directly carries properly cognizable by the COMELEC.
a narrower regional effect although its national a. Constitutional provision on people’s
effect cannot be discounted. initiative to amend the Constitution can only
be implemented by law to be passed by
Congress. No such law has been passed.b.
In all these, the need for interim measures is
Republic Act No. 6735 provides for 3 systems
dictated by necessity; out-of-the-way
on initiative but failed to provide any subtitle
arrangements and approaches were adopted
on initiative on the Constitution, unlike in the
or used in order to adjust to the goal or
other modes of initiative. This deliberate
objective in sight in a manner that does not do
omission indicates matter of people’s initiative
violence to the Constitution and to reasonably
was left to some future law.c. COMELEC has no
accepted norms.Under these limitations, the
power to provide rules and regulations for the
choice of measures was a question of wisdom
exercise of people’s initiative. Only Congress is
left to congressional discretion.
authorized by the Constitution to pass the
implementing law.d. People’s initiative is
However, the holdover contained in R.A. No. limited to amendments to the Constitution, not
10153, for those who were elected in to revision thereof. Extending or lifting of term
executive and legislative positions in the ARMM limits constitutes a revision.e. Congress nor
during the 2008-2011 term as an option that any government agency has not yet
Congress could have chosen because a appropriated funds for people’s initiative.
holdover violates Section 8, Article X of the
Constitution. In the case of the terms of local ISSUE:
officials, their term has been fixed clearly and Whether or not the people can directly propose
unequivocally, allowing no room for any amendments to the Constitution through the
implementing legislation with respect to the system of initiative under Section 2 of Article
fixed term itself and no vagueness that would XVII of the 1987 Constitution.
allow an interpretation from this Court. Thus,
the term of three years for local officials should
Page 99 of 121
403 CASE DIGESTS | CONSTITUTIONAL LAW REVIEW | JUDGE ESTELA ALMA SINGCO | FIRST SEMESTER | 2020-2021

HELD: 6735 failed to satisfy both requirements in


REPUBLIC ACT NO. 6735 subordinate legislation. The delegation of the
power to the COMELEC is then invalid.
It was intended to include or cover people’s
initiative on amendments to the Constitution COMELEC RESOLUTION NO. 2300
but, as worded, it does not adequately cover
such intiative. Article XVII Section 2 of the Insofar as it prescribes rules and regulations
1987 Constitution providing for amendments on the conduct of initiative on amendments to
to Constitution, is not self-executory. While the Constitution is void. COMELEC cannot
the Constitution has recognized or granted the validly promulgate rules and regulations to
right of the people to directly propose implement the exercise of the right of the
amendments to the Constitution via PI, the people to directly propose amendments to the
people cannot exercise it if Congress, for Constitution through the system of initiative.
whatever reason, does not provide for its It does not have that power under Republic Act
implementation. No. 6735.
Whether the COMELEC can take cognizance of,
FIRST: Contrary to the assertion of COMELEC, or has jurisdiction over, a petition solely
Section 2 of the Act does not suggest an intended to obtain an order: (a) fixing the time
initiative on amendments to the Constitution. and dates for signature gathering; (b)
The inclusion of the word “Constitution” instructing municipal election officers to assist
therein was a delayed afterthought. The word Delfin’s movement and volunteers in
is not relevant to the section which is silent as establishing signature stations; and (c)
to amendments of the Constitution. directing or causing the publication of the
unsigned proposed Petition for Initiative on the
SECOND: Unlike in the case of the other 1987 Constitution.
systems of initiative, the Act does not provide
for the contents of a petition for initiative on DELFIN PETITION
the Constitution. Sec 5(c) does not include the
provisions of the Constitution sought to be COMELEC ACTED WITHOUT JURISDICTION OR
amended, in the case of initiative on the WITH GRAVE ABUSE OF DISCRETION IN
Constitution. ENTERTAINING THE DELFIN PETITION. Even if
it be conceded ex gratia that RA 6735 is a full
THIRD: No subtitle is provided for initiative on compliance with the power of Congress to
the Constitution. This conspicuous silence as implement the right to initiate constitutional
to the latter simply means that the main thrust amendments, or that it has validly vested upon
of the Act is initiative and referendum on the COMELEC the power of subordinate
national and local laws. The argument that the legislation and that COMELEC Resolution No.
initiative on amendments to the Constitution is 2300 is valid, the COMELEC acted without
not accepted to be subsumed under the jurisdiction or with grave abuse of discretion in
subtitle on National Initiative and Referendum entertaining the Delfin Petition.
because it is national in scope. Under Subtitle
II and III, the classification is not based on the The Delfin Petition does not contain signatures
scope of the initiative involved, but on its of the required number of voters. Without the
nature and character. required signatures, the petition cannot be
National initiative – what is proposed to be deemed validly initiated. The COMELEC
enacted is a national law, or a law which only requires jurisdiction over a petition for
Congress can pass. initiative only after its filing. The petition then
Local initiative – what is proposed to be is the initiatory pleading. Nothing before its
adopted or enacted is a law, ordinance or filing is cognizable by the COMELEC, sitting en
resolution which only legislative bodies of the banc.
governments of the autonomous regions,
provinces, cities, municipalities, and Since the Delfin Petition is not the initiatory
barangays can pass. petition under RA6735 and COMELEC
Potestas delegata non delegari potest Resolution No. 2300, it cannot be entertained
or given cognizance of by the COMELEC. The
What has been delegated, cannot be petition was merely entered as UND, meaning
delegated. The recognized exceptions to the undocketed. It was nothing more than a mere
rule are: [1] Delegation of tariff powers to the scrap of paper, which should not have been
President; [2] Delegation of emergency dignified by the Order of 6 December 1996, the
powers to the President; [3] Delegation to the hearing on 12 December 1996, and the order
people at large; [4] Delegation to local directing Delfin and the oppositors to file their
governments; and [5] Delegation to memoranda to file their memoranda or
administrative bodies. oppositions. In so dignifying it, the COMELEC
acted without jurisdiction or with grave abuse
COMELEC of discretion and merely wasted its time,
energy, and resources.
Empowering the COMELEC, an administrative
body exercising quasi judicial functions, to Therefore, Republic Act No. 6735 did not apply
promulgate rules and regulations is a form of to constitutional amendment.
delegation of legislative authority. In every
case of permissible delegation, there must be 141 Jose Jesus M. Disini, Jr. et al. vs. The
a showing that the delegation itself is valid. It Secretary of Justice, GR No. 203335,
is valid only if the law (a) is complete in itself, February 11, 2014
setting forth therein the policy to be executed,
carried out, or implemented by the delegate; FACTS: On August 1, 1989 or two years after
and (b) fixes a standard – the limits of which the effectivity of the 1987 Constitution,
are sufficiently determinate and determinable Congress acted through Republic Act (RA) No.
– to which the delegate must conform in the 6734 entitled "An Act Providing for an Organic
performance of his functions. Republic Act No. Act for the Autonomous Region in Muslim
Page 100 of 121
403 CASE DIGESTS | CONSTITUTIONAL LAW REVIEW | JUDGE ESTELA ALMA SINGCO | FIRST SEMESTER | 2020-2021

Mindanao."The initially assenting provinces in complete violation of Section 16, Article X of


were Lanao del Sur,Maguindanao, Sulu and the Constitution.
Tawi-tawi.RA No. 6734 scheduled the first
regular elections for the regional officials of the ISSUE: Does the 1987 Constitution mandate
ARMM on a date not earlier than 60 days nor the synchronization of elections?
later than 90 days after its ratification.
Does the passage of RA No. 10153 violate the
Thereafter, R.A. No. 9054 was passed to provisions of the 1987 Constitution?
further enhance the structure of ARMM under
R.A. 6734. Along with it is the reset of the HELD: Court dismissed the petition and
regular elections for the ARMM regional affirmed the constitutionality of R.A. 10153 in
officials to the second Monday of September toto. The Court agreed with respondent Office
2001. of the Solicitor General (OSG) on its position
that the Constitution mandates
RA No. 9333was subsequently passed by synchronization, citing Sections 1, 2 and 5,
Congress to reset the ARMM regional elections Article XVIII (Transitory Provisions) of the
to the 2ndMonday of August 2005, and on the 1987 Constitution. While the Constitution does
same date every 3 years thereafter. Unlike RA not expressly state that Congress has to
No. 6734 and RA No. 9054, RA No. 9333 was synchronize national and local elections, the
not ratified in a plebiscite. clear intent towards this objective can be
gleaned from the Transitory Provisions (Article
Pursuant to RA No. 9333, the next ARMM XVIII) of the Constitution,which show the
regional elections should have been held extent to which the Constitutional
onAugust 8, 2011. COMELEC had begun Commission, by deliberately making
preparations for these elections and had adjustments to the terms of the incumbent
accepted certificates of candidacies for the officials, sought to attain synchronization of
various regional offices to be elected.But elections.
onJune 30, 2011, RA No. 10153 was enacted,
resetting the ARMM elections to May 2013, to The objective behind setting a common
coincide with the regular national and local termination date for all elective officials, done
elections of the country.With the enactment among others through the shortening the
into law of RA No. 10153, the COMELEC terms of the twelve winning senators with the
stopped its preparations for the ARMM least number of votes, is to synchronize the
elections. holding of all future elections whether national
or local to once every three years.This
Several cases for certiorari, prohibition and intention finds full support in the discussions
madamus originating from different parties during the Constitutional Commission
arose as a consequence of the passage of R.A. deliberations. Furthermore, to achieve
No. 9333 and R.A. No. 10153 questioning the synchronization, Congress necessarily has to
validity of said laws. reconcile the schedule of the ARMMs regular
elections (which should have been held in
OnSeptember 13, 2011, the Court issued a August 2011 based on RA No. 9333) with the
temporary restraining order enjoining the fixed schedule of the national and local
implementation of RA No. 10153 and ordering elections (fixed by RA No. 7166 to be held in
the incumbent elective officials of ARMM to May 2013).
continue to perform their functions should
these cases not be decided by the end of their In Osme v. Commission on Elections, the court
term onSeptember 30, 2011. thus explained:

The petitioners assailing RA No. 9140, RA No. It is clear from the aforequoted provisions of
9333 and RA No. 10153 assert that these laws the 1987 Constitution that the terms of office
amend RA No. 9054 and thus, have to comply of Senators, Members of the House of
with the supermajority vote and plebiscite Representatives, the local officials, the
requirements prescribed under Sections 1 and President and the Vice-President have been
3, Article XVII of RA No. 9094 in order to synchronized to end on the same hour, date
become effective. and year noon of June 30, 1992.

The petitions assailing RA No. 10153 further It is likewise evident from the wording of the
maintain that it is unconstitutional for its above-mentioned Sections that the term
failure to comply with the three-reading ofsynchronizationis used synonymously as the
requirement of Section 26(2), Article VI of the phraseholding simultaneouslysince this is the
Constitution.Also cited as grounds are the precise intent in terminating their Office
alleged violations of the right of suffrage of the Tenure on the sameday or occasion.This
people of ARMM, as well as the failure to common termination date will synchronize
adhere to the "elective and representative" future elections to once every three years
character of the executive and legislative (Bernas, the Constitution of the Republic of the
departments of the ARMM. Lastly, the Philippines, Vol. II, p. 605).
petitioners challenged the grant to the
President of the power to appoint OICs to That the election for Senators, Members of the
undertake the functions of the elective ARMM House of Representatives and the local officials
officials until the officials elected under the (under Sec. 2, Art. XVIII) will have to be
May 2013 regular elections shall have synchronized with the election for President
assumed office. Corrolarily, they also argue and Vice President (under Sec. 5, Art. XVIII) is
that the power of appointment also gave the likewise evident from the x x xrecords of the
President the power of control over the ARMM, proceedings in the Constitutional Commission.

Page 101 of 121


403 CASE DIGESTS | CONSTITUTIONAL LAW REVIEW | JUDGE ESTELA ALMA SINGCO | FIRST SEMESTER | 2020-2021

Although called regional elections, the ARMM Legislative Assembly who shall perform the
elections should be included among the functions pertaining to the said offices until the
elections to be synchronized as it is a "local" officials duly elected in the May 2013 elections
election based on the wording and structure of shall have qualified and assumed office."This
the Constitution. Regional elections in the power is far different from appointing elective
ARMM for the positions of governor, vice- ARMM officials for the abbreviated term ending
governor and regional assembly on the assumption to office of the officials
representatives fall within the classification of elected in the May 2013 elections. It must be
"local" elections, since they pertain to the therefore emphasized that the law must be
elected officials who will serve within the interpreted as an interim measure to
limited region of ARMM. From the perspective synchronize elections and must not be
of the Constitution, autonomous regions are interpreted otherwise.g
considered one of the forms of local
governments, as evident from Article Xof the These consolidated petitions seek to declare
Constitution entitled "Local several provisions of Republic Act (RA) 10175,
Government."Autonomous regions are the Cybercrime Prevention Act of 2012,
established and discussed under Sections 15 unconstitutional and void.
to 21 of this Article the article wholly devoted
to Local Government. The cybercrime law aims to regulate access to
and use of the cyberspace. The cyberspace is
Second issue: Congress, in passing RA No. a boon to the need of a current generation for
10153, acted strictly within its constitutional greater information and facility of
mandate. Given an array of choices, it acted communication. But all is not well with the
within due constitutional bounds and with system since it could not filter out a number of
marked reasonableness in light of the persons of ill will who would want to use
necessary adjustments that synchronization cyberspace technology for mischiefs and
demands. Congress, therefore, cannot be crimes. One of them can, for instance, avail
accused of any evasion of a positive duty or of himself of the system to unjustly ruin the
a refusal to perform its duty nor is there reason reputation of another or bully the latter by
to accord merit to the petitioners claims of posting defamatory statements against him
grave abuse of discretion. that people can read.

In relation with synchronization, both


autonomy and the synchronization of national
and local elections are recognized and And because linking with the internet opens up
established constitutional mandates, with one a user to communication from others, the ill-
being as compelling as the other.If their motivated can use the cyberspace for
compelling force differs at all, the difference is committing theft by hacking into or
in their coverage; synchronization operates on surreptitiously accessing his bank account or
and affects the whole country, while regional credit card or defrauding him through false
autonomy as the term suggests directly carries representations.
a narrower regional effect although its national
effect cannot be discounted.
The wicked can use the cyberspace, too, for
illicit trafficking in sex or for exposing to
In all these, the need for interim measures is pornography guileless children who have
dictated by necessity; out-of-the-way access to the internet.
arrangements and approaches were adopted
or used in order to adjust to the goal or
For these reasons, the government has a
objective in sight in a manner that does not do
legitimate right to regulate the use of
violence to the Constitution and to reasonably
cyberspace and contain and punish
accepted norms.Under these limitations, the
wrongdoings. The government certainly has
choice of measures was a question of wisdom
the duty and the right to prevent these
left to congressional discretion.
tomfooleries from happening and punish their
perpetrators, hence the Cybercrime
However, the holdover contained in R.A. No. Prevention Act.
10153, for those who were elected in
executive and legislative positions in the ARMM
But petitioners claim that the means adopted
during the 2008-2011 term as an option that
by the cybercrime law for regulating
Congress could have chosen because a
undesirable cyberspace activities violate
holdover violates Section 8, Article X of the
certain of their constitutional rights.
Constitution. In the case of the terms of local
officials, their term has been fixed clearly and
unequivocally, allowing no room for any Pending hearing and adjudication of the issues
implementing legislation with respect to the presented in these cases, on February 5, 2013,
fixed term itself and no vagueness that would the Court extended the original 120-day
allow an interpretation from this Court. Thus, temporary restraining order (TRO) that it
the term of three years for local officials should earlier issued on October 9, 2012, enjoining
stay at three (3) years as fixed by the respondent government agencies from
Constitution and cannot be extended by implementing the cybercrime law until further
holdover by Congress. orders.

RA No. 10153, does not in any way amend ISSUES:


what the organic law of the ARMM(RA No.
9054) sets outs in terms of structure of 1. The petitioners contend that Section 4(a)(1)
governance.What RA No. 10153 in fact only fails to meet the strict scrutiny standard
does is to"appoint officers-in-charge for the required of laws that interfere with the
Office of the Regional Governor, Regional Vice
Governor and Members of the Regional
Page 102 of 121
403 CASE DIGESTS | CONSTITUTIONAL LAW REVIEW | JUDGE ESTELA ALMA SINGCO | FIRST SEMESTER | 2020-2021

fundamental rights of the people. Is Section evil purpose for which one uses the name that
4(a)(1) on Illegal Access unconstitutional? the law condemns.

2. Petitioners claim that Section 4(a)(3) 4. Petitioners mainly contend that Congress
suffers from overbreadth in that, while is seeks invalidly delegated its power when it gave the
to discourage data interference, it intrudes into Cybercrime Investigation and Coordinating
the area of protected speech and expression, Center (CICC) the power to formulate a
creating a chilling and deterrent effect on national cybersecurity plan without any
these guaranteed freedoms. sufficient standards or parameters for it to
follow.
3. Petitioners claim that Section 4(a)(6) or
cyber-squatting violates the equal protection In order to determine whether there is undue
clause in that, not being narrowly tailored, it delegation of legislative power, the Court has
will cause a user using his real name to suffer adopted two tests: the completeness test and
the same fate as those who use aliases or take the sufficient standard test. Under the first
the name of another in satire, parody, or any test, the law must be complete in all its terms
other literary device. and conditions when it leaves the legislature
such that when it reaches the delegate, the
4. Is Section 24 on Cybercrime Investigation only thing he will have to do is to enforce
and Coordinating Center (CICC) valid and it.1avvphi1 The second test mandates
constitutional? Is Section 26(a) on CICC’s adequate guidelines or limitations in the law to
power and functions valid and constitutional? determine the boundaries of the delegate’s
authority and prevent the delegation from
HELD: running riot.103

1. No. The strict scrutiny standard, an Here, the cybercrime law is complete in itself
American constitutional construct, is useful in when it directed the CICC to formulate and
determining the constitutionality of laws that implement a national cybersecurity plan. Also,
tend to target a class of things or persons. contrary to the position of the petitioners, the
According to this standard, a legislative law gave sufficient standards for the CICC to
classification that impermissibly interferes follow when it provided a definition of
with the exercise of fundamental right or cybersecurity.
operates to the peculiar class disadvantage of
a suspect class is presumed unconstitutional. Cybersecurity refers to the collection of tools,
The Court finds nothing in Section 4(a)(1) that policies, risk management approaches,
calls for the application of the strict scrutiny actions, training, best practices, assurance
standard since no fundamental freedom, like and technologies that can be used to protect
speech, is involved in punishing what is cyber environment and organization and user’s
essentially a condemnable act – accessing the assets.104 This definition serves as the
computer system of another without right. It parameters within which CICC should work in
is a universally condemnable act. formulating the cybersecurity plan.

2. Under the overbreadth doctrine, a proper Further, the formulation of the cybersecurity
governmental purpose, constitutionally plan is consistent with the policy of the law to
subject to state regulation, may not be "prevent and combat such [cyber] offenses by
achieved by means that unnecessarily sweep facilitating their detection, investigation, and
its subject broadly, thereby invading the area prosecution at both the domestic and
of protected speech. Section 4(a)(3) does not international levels, and by providing
encroach on these freedoms at all. It simply arrangements for fast and reliable
punishes what essentially is a form of international cooperation."105 This policy is
vandalism, the act of wilfully destroying clearly adopted in the interest of law and
without right the things that belong to others, order, which has been considered as sufficient
in this case their computer data, electronic standard.106 Hence, Sections 24 and 26(a)
document, or electronic data message. Such are likewise valid.
act has no connection to guaranteed freedoms.
Ergo, there is no freedom to destroy other 142 Garcia vs, Drilon, GR No. 179267, June 25,
people’s computer systems and private 2013
documents. All penal laws, like the cybercrime
law, have of course an inherent chilling effect,
an in terrorem effect, or the fear of possible Nature of the Case: Petition for Review of
prosecution that hangs on the heads of citizens Republic Act (R.A.) 9262
who are minded to step beyond the boundaries
of what is proper. But to prevent the State
from legislating criminal laws because they
instil such kind of fear is to render the state
Facts:
powerless in addressing and penalizing socially
harmful conduct.

3. No, the challenge to the constitutionality of


Section 4(a)(6) is baseless. The law is Private respondent Rosalie filed a petition
reasonable in penalizing the act of acquiring before the RTC of Bacolod City a Temporary
the domain name in bad faith to profit, Protection Order against her husband, Jesus,
mislead, destroy reputation, or deprive others pursuant to R.A. 9262, entitled “An Act
who are not ill-motivated of the rightful Defining Violence Against Women and Their
opportunity of registering the same. It is the Children, Providing for Protective Measures for
Victims, Prescribing Penalties Therefor, and for
Other Purposes.” She claimed to be a victim of
physical, emotional, psychological and
Page 103 of 121
403 CASE DIGESTS | CONSTITUTIONAL LAW REVIEW | JUDGE ESTELA ALMA SINGCO | FIRST SEMESTER | 2020-2021

economic violence, being threatened of is required of a valid classification is that it be


deprivation of custody of her children and of reasonable, which means that the
financial support and also a victim of marital classification should be based on substantial
infidelity on the part of petitioner. distinctions which make for real differences;
that it must be germane to the purpose of the
The TPO was granted but the petitioner failed law; not limited to existing conditions only;
to faithfully comply with the conditions set and apply equally to each member of the class.
forth by the said TPO, private-respondent filed Therefore, RA9262 is based on a valid
another application for the issuance of a TPO classification and did not violate the equal
ex parte. The trial court issued a modified TPO protection clause by favouring women over
and extended the same when petitioner failed men as victims of violence and abuse to whom
to comment on why the TPO should not be the Senate extends its protection.
modified. After the given time allowance to
answer, the petitioner no longer submitted the 3. RA 9262 is not violative of the due process
required comment as it would be an “axercise clause of the Constitution. The essence of due
in futility.” process is in the reasonable opportunity to be
heard and submit any evidence one may have
Petitioner filed before the CA a petition for in support of one’s defense. The grant of the
prohibition with prayer for injunction and TRO TPO exparte cannot be impugned as violative
on, questioning the constitutionality of the RA of the right to due process.
9262 for violating the due process and equal
protection clauses, and the validity of the 4. The non-referral of a VAWC case to a
modified TPO for being “an unwanted product mediator is justified. Petitioner’s contention
of an invalid law.” that by not allowing mediation, the law
violated the policy of the State to protect and
The CA issued a TRO on the enforcement of the strengthen the family as a basic autonomous
TPO but however, denied the petition for social institution cannot be sustained. In a
failure to raise the issue of constitutionality in memorandum of the Court, it ruled that the
his pleadings before the trial court and the court shall not refer the case or any issue
petition for prohibition to annul protection therof to a mediator. This is so because
orders issued by the trial court constituted violence is not a subject for compromise.
collateral attack on said law. 5. There is no undue delegation of judicial
power to Barangay officials. Judicial power
Petitioner filed a motion for reconsideration includes the duty of the courts of justice to
but was denied. Thus, this petition is filed. settle actual controversies involving rights
which are legally demandable and enforceable
Issues: WON the CA erred in dismissing the and to determine whether or not there has
petition on the theory that the issue of been a grave abuse of discretion amounting to
constitutionality was not raised at the earliest lack or excess of jurisdiction on any part of any
opportunity and that the petition constitutes a branch of the Government while executive
collateral attack on the validity of the law. power is the power to enforce and administer
the laws. The preliminary investigation
conducted by the prosecutor is an executive,
WON the CA committed serious error in failing
not a judicial, function. The same holds true
to conclude that RA 9262 is discriminatory,
with the issuance of BPO. Assistance by Brgy.
unjust and violative of the equal protection
Officials and other law enforcement agencies
clause.
is consistent with their duty executive
WON the CA committed grave mistake in not
function.
finding that RA 9262 runs counter to the due
process clause of the Constitution

WON the CA erred in not finding that the law The petition for review on certiorari is denied
does violence to the policy of the state to for lack of merit.
protect the family as a basic social institution
143 Pimentel III vs. COMELEC, G. R. No.
WON the CA seriously erredin declaring RA 178413, March 13, 2008
9262 as invalid and unconstitutional because it
allows an undue delegation of judicial power to FACTS:
Brgy. Officials.
2 months after the May 14, 2007 national
Decision: 1. Petitioner contends that the elections, 11 candidates for senatorial posts
RTC has limited authority and jurisdiction, were proclaimed and had taken their oaths
inadequate to tackle the complex issue of except to the 12thand last post where Pimentel
constitutionality. Family Courts have authority and Zubiri were the contenders. A new board
and jurisdiction to consider the of canvassers (Special Provincial Board of
constitutionality of a statute. The question of Canvassers for Maguindanao) was created
constitutionality must be raised at the earliest because Pimentel thru his counsel questioned
possible time so that if not raised in the some irregularities as to the authenticity and
pleadings, it may not be raised in the trial and due execution of Cert. of Canvass. During the
if not raised in the trial court, it may not be proceedings, they were not allowed to ask
considered in appeal. questions and present evidence to prove their
claim. Instead, their questions were noted in
2. RA 9262 does not violate the guaranty of the minutes. Hence, Pimentel a petition for
equal protection of the laws. Equal protection certiorari and mandamus seeking the court to
simply requires that all persons or things issue tro as to enjoin comelec from
similarly situated should be treated alike, both canvassing;to annul such proceedings because
as to rights conferred and responsibilities it is unconstitutional and illegal proceedings
imposed. In Victoriano v. Elizalde Rope and; to allow them to raise their objections and
Workerkers’ Union, the Court ruled that all that present evidence to prove their claims. All
Page 104 of 121
403 CASE DIGESTS | CONSTITUTIONAL LAW REVIEW | JUDGE ESTELA ALMA SINGCO | FIRST SEMESTER | 2020-2021

petitions were denied. Eventually, Zubiri were NON-DELEGATION OF LEGISLATIVE


proclaimed, had taken oath and assumed POWER
office. Pimentel protested. Zubiri filed a 144 Abakada Guro Party List vs. Executive
motion to dismiss contending that Pimentel Secretary, September 1, 2005
should have filed with Senate Electoral
Tribunal (SET) for the annulment of his Facts:
proclamation.
On May 24, 2005, the President signed into law
ISSUES: Republic Act 9337 or the VAT Reform Act.
Before the law took effect on July 1, 2005, the
1. WON the case of Pimentel is an exception to Court issued a TRO enjoining government from
the prohibition on pre-proclamation in cases of implementing the law in response to a slew of
Senators petitions for certiorari and prohibition
questioning the constitutionality of the new
2. WON the comelec has the jurisdiction over law.
the case
The challenged section of R.A. No. 9337 is the
RULING: common proviso in Sections 4, 5 and 6: “That
the President, upon the recommendation of
1. No. The SC did not recognize the pre- the Secretary of Finance, shall, effective
proclamation case of Pimentel, which could January 1, 2006, raise the rate of value-added
have prospered if he met the requirement of tax to 12%, after any of the following
law, because SPBOC-Maguindanao is not conditions has been satisfied:
Congress nor COMELEC en banc acting as the
NBC, specifically charged by Section 30 of (i) Value-added tax collection as a percentage
Republic Act No. 7166, as amended by of Gross Domestic Product (GDP) of the
Republic Act No. 9369, with the duty to previous year exceeds two and four-fifth
determine the authenticity and due execution percent (2 4/5%);
of the certificates of canvass submitted to it.
or (ii) National government deficit as a
In elections for President, Vice-President, percentage of GDP of the previous year
Senators and Members of the House of exceeds one and one-half percent (1½%)”
Representatives, the general rule still is that
pre-proclamation cases on matters relating to Petitioners allege that the grant of stand-by
the preparation, transmission, receipt, custody authority to the President to increase the VAT
and appreciation of election returns or rate is an abdication by Congress of its
certificates of canvass are still prohibited. As exclusive power to tax because such
with other general rules, there are recognized delegation is not covered by Section 28 (2),
exceptions to the prohibition, namely: (1) Article VI Consti. They argue that VAT is a tax
correction of manifest errors; (2) questions levied on the sale or exchange of goods and
affecting the composition or proceedings of the services which can’t be included within the
board of canvassers; and (3) determination of purview of tariffs under the exemption
the authenticity and due execution of delegation since this refers to customs duties,
certificates of canvass as provided in Section tolls or tribute payable upon merchandise to
30 of Republic Act No. 7166, as amended by the government and usually imposed on
Republic Act No. 9369.And, in this case, the imported/exported goods.
exception applies only to Congress or the
COMELEC en banc acting as the NBC, and not Petitioners further alleged that delegating to
to local boards of canvassers who must still be the President the legislative power to tax is
deemed covered by the prohibition on pre- contrary to republicanism. They insist that
proclamation controversies. accountability, responsibility and transparency
should dictate the actions of Congress and
2. No. As provided in Article VI, Section 17 of they should not pass to the President the
the 1987 Constitution SET shall be the sole decision to impose taxes. They also argue that
judge of all contests relating to the election, the law also effectively nullified the President’s
returns, and qualifications of their respective power of control, which includes the authority
members. Thus, once a winning candidate has to set aside and nullify the acts of her
been proclaimed, taken his oath, and assumed subordinates like the Secretary of Finance, by
office as a senator, COMELEC’s jurisdiction mandating the fixing of the tax rate by the
over election contests relating to his election, President upon the recommendation of the
returns, and qualifications ends, and the SET’s Secretary of Justice.
own jurisdiction begins. The word "sole" in
Article VI, Section 17 of the 1987 Constitution Issue:
underscores the exclusivity of the electoral
tribunals' jurisdiction over election contests Whether or not the RA 9337's stand-by
relating to their respective members. It is authority to the Executive to increase the VAT
therefore crystal clear that this Court has no rate, especially on account of the
jurisdiction to entertain a petition for certiorari recommendatory power granted to the
and mandamus on matters which may be Secretary of Finance, constitutes undue
threshed out in an election contest. It is the delegation of legislative power?
SET which has exclusive jurisdiction to act on
the complaint of Pimentel involving, as it does,
Ruling:
a contest relating to the election of Zubiri, now
a member of the Senate.
The powers which Congress is prohibited from
delegating are those which are strictly, or
inherently and exclusively, legislative. Purely
legislative power which can never be delegated
Page 105 of 121
403 CASE DIGESTS | CONSTITUTIONAL LAW REVIEW | JUDGE ESTELA ALMA SINGCO | FIRST SEMESTER | 2020-2021

is the authority to make a complete law- policy is determined and implemented,


complete as to the time when it shall take considering that he possesses all the facilities
effect and as to whom it shall be applicable, to gather data and information and has a much
and to determine the expediency of its broader perspective to properly evaluate
enactment. It is the nature of the power and them. His function is to gather and collate
not the liability of its use or the manner of its statistical data and other pertinent information
exercise which determines the validity of its and verify if any of the two conditions laid out
delegation. by Congress is present.

The exceptions are: Congress does not abdicate its functions or


unduly delegate power when it describes what
(a) delegation of tariff powers to President job must be done, who must do it, and what is
under Constitution the scope of his authority; in our complex
economy that is frequently the only way in
(b) delegation of emergency powers to which the legislative process can go forward.
President under Constitution
There is no undue delegation of legislative
(c) delegation to the people at large power but only of the discretion as to the
execution of a law. This is constitutionally
permissible. Congress did not delegate the
(d) delegation to local governments
power to tax but the mere implementation of
the law.
(e) delegation to administrative bodies
145 Gerochi vs. DOE, GR. No. 159796, July 17,
For the delegation to be valid, it must be 2007
complete and it must fix a standard. A
sufficient standard is one which defines
On June 8, 2001 Congress enacted RA 9136 or
legislative policy, marks its limits, maps out its
the Electric Power Industry Act of 2001.
boundaries and specifies the public agency to
Petitioners Romeo P. Gerochi and company
apply it.
assail the validity of Section 34 of the EPIRA
Law for being an undue delegation of the
In this case, it is not a delegation of legislative power of taxation. Section 34 provides for the
power BUT a delegation of ascertainment of imposition of a “Universal Charge” to all
facts upon which enforcement and electricity end users after a period of (1) one
administration of the increased rate under the year after the effectively of the EPIRA Law. The
law is contingent. The legislature has made the universal charge to be collected would serve as
operation of the 12% rate effective January 1, payment for government debts, missionary
2006, contingent upon a specified fact or electrification, equalization of taxes and
condition. It leaves the entire operation or royalties applied to renewable energy and
non-operation of the 12% rate upon factual imported energy, environmental charge and
matters outside of the control of the executive. for a charge to account for all forms of cross
No discretion would be exercised by the subsidies for a period not exceeding three
President. Highlighting the absence of years. The universal charge shall be collected
discretion is the fact that the word SHALL is by the ERC on a monthly basis from all end
used in the common proviso. The use of the users and will then be managed by the PSALM
word SHALL connotes a mandatory order. Its Corp. through the creation of a special trust
use in a statute denotes an imperative fund.
obligation and is inconsistent with the idea of
discretion.
ISSUE:
Thus, it is the ministerial duty of the President
Whether or not there is an undue delegation of
to immediately impose the 12% rate upon the
the power to tax on the part of the ERC
existence of any of the conditions specified by
Congress. This is a duty, which cannot be
evaded by the President. It is a clear directive HELD:
to impose the 12% VAT rate when the
specified conditions are present. No, the universal charge as provided for in
section 34 is not a tax but an exaction of the
Congress just granted the Secretary of Finance regulatory power (police power) of the state.
the authority to ascertain the existence of a The universal charge under section 34 is
fact--- whether by December 31, 2005, the incidental to the regulatory duties of the ERC,
VAT collection as a percentage of GDP of the hence the provision assailed is not for
previous year exceeds 2 4/5 % or the national generation of revenue and therefore it cannot
government deficit as a percentage of GDP of be considered as tax, but an execution of the
the previous year exceeds one and 1½%. If states police power thru regulation.
either of these two instances has occurred, the
Secretary of Finance, by legislative mandate, Moreover, the amount collected is not made
must submit such information to the President. certain by the ERC, but by the legislative
parameters provided for in the law (RA 9136)
In making his recommendation to the itself, it therefore cannot be understood as a
President on the existence of either of the two rule solely coming from the ERC. The ERC in
conditions, the Secretary of Finance is not this case is only a specialized administrative
acting as the alter ego of the President or even agency which is tasked of executing a
her subordinate. He is acting as the agent of subordinate legislation issued by congress;
the legislative department, to determine and which before execution must pass both the
declare the event upon which its expressed will completeness test and the sufficiency of
is to take effect. The Secretary of Finance standard test. The court in appreciating
becomes the means or tool by which legislative Section 34 of RA 9136 in its entirety finds the
Page 106 of 121
403 CASE DIGESTS | CONSTITUTIONAL LAW REVIEW | JUDGE ESTELA ALMA SINGCO | FIRST SEMESTER | 2020-2021

said law and the assailed portions free from No. SC held that there was a valid delegation
any constitutional defect and thus deemed of powers.
complete and sufficient in form.
The authority to issue the said regulation is
146 Echegaray vs. Secretary of Justice G.R. clearly provided in Section 4(a) of Executive
No. 132601 January 19, 1999 Order No. 797. … “The governing Board of the
Administration (POEA), as hereunder provided
FACTS: shall promulgate the necessary rules and
On January 4, 1999, the SC issued a TRO regulations to govern the exercise of the
staying the execution of petitioner Leo adjudicatory functions of the Administration
Echegaray scheduled on that same day. The (POEA).”
public respondent Justice Secretary assailed
the issuance of the TRO arguing that the action It is true that legislative discretion as to the
of the SC not only violated the rule on finality substantive contents of the law cannot be
of judgment but also encroached on the power delegated. What can be delegated is the
of the executive to grant reprieve. discretion to determine how the law may be
enforced, not what the law shall be. The
ISSUE: ascertainment of the latter subject is a
Whether or not the court abused its discretion prerogative of the legislature. This prerogative
in granting a Temporary Restraining Order cannot be abdicated or surrendered by the
(TRO) on the execution of Echegaray despite legislature to the delegate.
the fact that the finality of judgment has
already been rendered… that by granting the
TRO, the Honorable Court has in effect granted The reasons given above for the delegation of
reprieve which is an executive function. legislative powers in general are particularly
applicable to administrative bodies. With the
HELD: proliferation of specialized activities and their
No. Respondents cited sec 19, art VII. The attendant peculiar problems, the national
provision is simply the source of power of the legislature has found it more and more
President to grant reprieves, commutations, necessary to entrust to administrative
and pardons and remit fines and forfeitures agencies the authority to issue rules to carry
after conviction by final judgment. The out the general provisions of the statute. This
provision, however, cannot be interpreted as is called the “power of subordinate legislation.”
denying the power of courts to control the With this power, administrative bodies may
enforcement of their decisions after their implement the broad policies laid down in a
finality. statute by “filling in’ the details which the
Congress may not have the opportunity or
The powers of the Executive, the Legislative competence to provide. This is effected by
and the Judiciary to save the life of a death their promulgation of what are known as
convict do not exclude each other for the supplementary regulations, such as the
simple reason that there is no higher right than implementing rules issued by the Department
the right to life. For the public respondents of Labor on the new Labor Code. These
therefore to contend that only the Executive regulations have the force and effect of law.
can protect the right to life of an accused after
his final conviction is to violate the principle of There are two accepted tests to determine
co-equal and coordinate powers of the three whether or not there is a valid delegation of
branches of our government. legislative power:

147 EASTERN SHIPPING LINES V. POEA, 166 1. Completeness test – the law must be
SCRA 533 complete in all its terms and conditions when
it leaves the legislature such that when it
FACTS: reaches the delegate the only thing he will
A Chief Officer of a ship was killed in an have to do is enforce it.
accident in Japan. The widow filed a complaint 2. Sufficient standard test – there must be
for charges against the Eastern Shipping Lines adequate guidelines or stations in the law to
with POEA, based on a Memorandum Circular map out the boundaries of the delegate’s
No. 2, issued by the POEA which stipulated authority and prevent the delegation from
death benefits and burial for the family of running riot.
overseas workers. ESL questioned the validity Both tests are intended to prevent a total
of the memorandum circular as violative of the transference of legislative authority to the
principle of non-delegation of legislative delegate, who is not allowed to step into the
power. It contends that no authority had been shoes of the legislature and exercise a power
given the POEA to promulgate the said essentially legislative
regulation; and even with such authorization,
the regulation represents an exercise of
legislative discretion which, under the 148 Secretary of Finance, et al. vs. La Suerte
principle, is not subject to delegation. Cigar, GR No. 166498, June 11, 2009
Nevertheless, POEA assumed jurisdiction and
decided the case. FACTS:
To implement RA 8240, the BIR issued
ISSUE: Revenue Regulations No. 1-97 which provided
Whether or not the Issuance of Memorandum that new brands, or those registered after
Circular No. 2 is a violation of non-delegation January 1, 1997, shall be initially assessed at
of powers. their suggested retail prices. Three months
after a new brand is launched in the market, a
RULING: survey shall be conducted to determine its
actual net retail price which shall be the basis
in determining its specific tax classification.
Page 107 of 121
403 CASE DIGESTS | CONSTITUTIONAL LAW REVIEW | JUDGE ESTELA ALMA SINGCO | FIRST SEMESTER | 2020-2021

implementation of Revenue Regulations No. 9-


In February 1999, respondents introduced into 2003. This was patent error on the part of the
the market Astro and Memphis cigarettes and BIR for being contrary to the plain text and
their variants with suggested net retail prices legislative intent of RA 8240.
below P5.00 per pack and a temporary excise
tax pegged at P1.00 per pack. On May 15, It is clear that the afore-quoted portions of
1999, respondents requested the BIR to Revenue Regulations No. 1-97, as amended by
conduct a survey to determine the final tax Section 2 of Revenue Regulations 9-2003, and
classification of said brands of cigarettes. Revenue Memorandum Order No. 6-2003
unjustifiably emasculate the operation of
On February 17, 2003, BIR issued the assailed Section 145 of the NIRC because they
Revenue Regulations No. 9-2003, Section 2 of authorize the Commissioner of Internal
which amended Revenue Regulations No. 1- Revenue to update the tax classification of new
97, by providing for a periodic review every brands every two years or earlier subject only
two years or earlier of the current net retail to its issuance of the appropriate Revenue
prices of new brands of cigars and cigarettes Regulations, when nowhere in Section 145 is
and their variants to establish and update their such authority granted to the Bureau. Unless
tax classification. Respondents filed a case for expressly granted to the BIR, the power to
injunction with the trial court assailing the reclassify cigarette brands remains a
validity of Revenue Regulations No. 9-2003 prerogative of the legislature which
and praying for the issuance of a temporary cannot be usurped by the former.
restraining order and/or writ of preliminary
injunction to enjoin the implementation of said
regulation insofar as it authorizes the BIR to
update the tax classification of cigarettes 149 Review Center Assos. of the Philippines
registered after January 1, 1997. The vs. Ermita, GR No. 180046, April 2, 2009
complaint was later amended to include
Revenue Regulations No. 22-2003 FACTS.
(implementing the revised tax classification). Due to a cheating incident which occurred
Respondents asserted that Section 145 of the during the 2006 nursing board exam wherein
NIRC does not give the BIR the power to a handwritten copy of the questions and
reclassify cigarettes introduced into the answers were passed around by examinees of
market after January 1, 1997, hence, the RA Gapuz Reviewing center during the
reclassification thereof by the BIR constitutes examination period then PGMA was prompted
usurpation of legislative powers. to issue EO 566 which authorized the CHED to
supervise the establishment and operation of
Petitioners, on the other hand, maintained that all review centers and similar entities in the
the assailed revenue regulations constitute a Philippines. The IRR was also revised pursuant
valid exercise of subordinate legislation having to this change. The petitioners come to this
been issued pursuant to the powers of the court assailing the validity of the EO and the
Commissioner of Internal Revenue and the revised IRR. The SC held that the EO and the
Secretary of Finance. revised IRR were void. Review centers are not
institutions of higher learning, to include them
ISSUE: in the scope of CHED would be to amend the
WON there is usurpation of legislative powers law. The president does not have the power to
amend laws nor was it granted by the congress
RULING: to amend the law through EO 566
The reclassification of Astro and Memphis
pursuant to Revenue Regulations Nos. 9-2003 ISSUE:
and 22-2003 constitutes the prohibited Whether EO 566 is an unconstitutional
reclassification contemplated in British exercise by the Executive as it usurp the
American Tobacco v. Camacho. It will be legislative power.
recalled that these brands were already
classified by the BIR based on their current net RULING:
retail prices in 1999 through a market survey. Section 20, Title I of Book III of EO 292 speaks
Consequently, their upward reclassification in of other powers vested in the President under
2003 by the BIR through another market the law. The exercise of the President’s
survey is a prohibited reclassification. residual powers under this provision requires
legislation, as the provision clearly states that
In order to implement RA 8240 following its the exercise of the President’s other powers
effectivity on January 1, 1997, the BIR issued and functions has to be "provided for under
Revenue Regulations No. 1-97, dated the law." There is no law granting the
December 13, 1996, which mandates a one- President the power to amend the functions of
time classification only. Upon their launch, new the CHED. The President may not amend RA
brands shall be initially taxed based on their 7722 through an Executive Order without a
suggested net retail price. Thereafter, a survey prior legislation granting her such power.
shall be conducted within three (3) months to The President may not amend RA 7722
determine their current net retail prices and, through an Executive Order without a prior
thus, fix their official tax classifications. legislation granting her such power.
However, the BIR made a turnaround by
issuing Revenue Regulations No. 9-2003, As head of the Executive Department, the
dated February 17, 2003, which partly President is the Chief Executive. He represents
amended Revenue Regulations No. 1-97, by the government as a whole and sees to it that
authorizing the BIR to periodically reclassify all laws are enforced by the officials and
new brands (i.e., every two years or earlier) employees of his department. He has control
based on their current net retail prices. over the executive department, bureaus and
Thereafter, the BIR issued Revenue offices. This means that he has the authority
Memorandum Order No. 6-2003, dated March to assume directly the functions of the
11, 2003, prescribing the guidelines on the executive department, bureau and office, or
Page 108 of 121
403 CASE DIGESTS | CONSTITUTIONAL LAW REVIEW | JUDGE ESTELA ALMA SINGCO | FIRST SEMESTER | 2020-2021

interfere with the discretion of its officials. in the 2007 elections. Bantay Republic Act
Corollary to the power of control, the President (BA-RA 7941) and the Urban Poor for Legal
also has the duty of supervising the Reforms (UP-LR) filed with the Comelec an
enforcement of laws for the maintenance of Urgent Petition seeking to disqualify the
general peace and public order. Thus, he is nominees of certain party-list organizations.
granted administrative power over bureaus Meanwhile petitioner Rosales, in G.R. No.
and offices under his control to enable him to 177314, addressed 2 letters to the Director of
discharge his duties effectively. the Comelec’s Law Department requesting a
Administrative power is concerned with the list of that groups’ nominees. Evidently
work of applying policies and enforcing orders unbeknownst then to Ms. Rosales, et al., was
as determined by proper governmental the issuance of Comelec en banc Resolution
organs. It enables the President to fix a 07-0724 under date April 3, 2007 virtually
uniform standard of administrative efficiency declaring the nominees’ names confidential
and check the official conduct of his agents. To and in net effect denying petitioner Rosales’
this end, he can issue administrative orders, basic disclosure request. According to
rules and regulations. COMELEC, there is nothing in R.A. 7941 that
requires the Comelec to disclose the names of
An administrative order is an ordinance issued nominees, and that party list elections must
by the President which relates to specific not be personality oriented according to
aspects in the administrative operation of Chairman Abalos.
government. It must be in harmony with the ISSUE:
law and should be for the sole purpose of Whether respondent Comelec is mandated by
implementing the law and carrying out the the Constitution to disclose to the public the
legislative policy. Just like AO 308 in Ople v. names of said nominees.
Torres, EO 566 in this case is not supported by
any enabling law. RULING:
Section 7, Article III of the Constitution, viz:
Sec.7. The right of the people to
150 SEMA VS. COMELEC, G. R. No. 177597, information on matters of public
July 16, 2008 concern shall be recognized. Access to
official records, and to documents,
FACTS: and papers pertaining to official acts,
The Autonomous Region in Muslim Mindanao transactions, or decisions, as well to
(ARMM) was created by RA 9054. Section 19, government research data used as
Article VI of RA 9054 allows ARMM’s basis for policy development, shall be
legislature, the ARMM National Assembly, to afforded the citizen, subject to such
create provinces. Thus it enacted Muslim limitations as may be provided by law.
Mindanao Autonomy Act No. 201 (MMA Act
201) creating the province of Shariff Section 28, Article II of the Constitution
Kabunsuan composed of eight municipalities in reading:
the first district of Maguindanao. Comelec and Sec. 28. Subject to reasonable
Didagen Dilanganen (Dilanganen) now conditions prescribed by law, the
questions the constitutionality of Section 19, State adopts and implements a policy
Article VI of RA 9054. of full public disclosure of all its
transactions involving public interest.
ISSUE:
Whether Section 19, Article VI of RA 9054, As may be noted, no national security or like
delegating to the ARMM National Assembly the concerns is involved in the disclosure of the
power to create provinces is constitutional. names of the nominees of the partylist groups
(NO) in question. Doubtless, the Comelec
committed grave abuse of discretion in
RULING: refusing the legitimate demands of the
Section 19, Article VI of RA 9054 is petitioners for a list of the nominees of the
unconstitutional. Section 19, Article VI of RA party-list groups subject of their respective
9054, insofar as it grants to the ARMM petitions. Mandamus, therefore, lies.
Regional Assembly the power to create
provinces, is void for being contrary to Section The last sentence of Section 7 of R.A. 7941
5 of Article VI and Section 20 of Article X of the reading: "[T]he names of the party-list
Constitution. Only Congress can create nominees shall not be shown on the certified
provinces and cities because the creation of list" is certainly not a justifying card for the
provinces and cities necessarily includes the Comelec to deny the requested disclosure. To
creation of legislative r only Congress can us, the prohibition imposed on the Comelec
exercise under Section 5, Article VI of the under said Section 7 is limited in scope and
Constitution. The ARMM Regional Assembly duration, meaning that it extends only to the
cannot create a province without a legislative certified list which the same provision requires
district because the Constitution mandates to be posted in the polling places on election
that every province shall have a legislative day. To stretch the coverage of the prohibition
district. to the absolute is to read into the law
something that is not intended. As it were,
there is absolutely nothing in R.A. No. 7941
that prohibits the Comelec from disclosing or
PARTY-LIST SYSTEM
even publishing through mediums other than
151 BA-RA 7941 vs. COMELEC, G.R. No. the "Certified List" the names of the party-list
1777271, May 4, 2007 nominees. The Comelec obviously misread the
limited nondisclosure aspect of the provision
FACTS: as an absolute bar to public disclosure before
A number of organized groups filed the the May 2007 elections. The interpretation
necessary manifestations and subsequently thus given by the Comelec virtually tacks an
were accredited by the Comelec to participate unconstitutional dimension on the last
Page 109 of 121
403 CASE DIGESTS | CONSTITUTIONAL LAW REVIEW | JUDGE ESTELA ALMA SINGCO | FIRST SEMESTER | 2020-2021

sentence of Section 7 of R.A. No. 7941. xxx issuance of a writ of mandamus to compel
Comelec has a constitutional duty to disclose respondent Commission: a) to convene as the
and release the names of the nominees of the National Board of Canvassers for the Party-List
party-list groups. System; b) to declare them as entitled to one
(1) additional seat each; c) to immediately
proclaim their respective second nominees; d)
152 Veterans Federation Party vs. COMELEC, to declare other similarly situated party-list
342 SCRA 244, October 6, 2000; organizations as entitled to one (1) additional
seat each; and e) to immediately proclaim
FACTS: similarly situated parties' second nominees as
Petitioner assailed public respondent COMELEC duly elected representatives to the House of
resolutions ordering the proclamation of 38 Representatives.
additional party-list representatives to
complete the 52 seats in the House of ISSUE:
Representatives as provided by Sec 5, Art VI Whether or not respondent COMELEC EN
of the 1987 Constitution and RA 7941. BANC, as the National Board of Canvassers for
the Party-List System could be compelled by
On the other hand, Public Respondent, the Honorable Court to Mechanically apply the
together with the respondent parties, avers simplified formula
that the filling up of the twenty percent
membership of party-list representatives in RULING:
the House of Representatives, as provided Under the Constitution, this Court has original
under the Constitution, was mandatory, jurisdiction over petitions for certiorari,
wherein the twenty (20%) percent prohibition and mandamus.[20] We have
congressional seats for party-list consistently ruled that where the duty of the
representatives is filled up at all times. respondent Commission is ministerial,
mandamus lies to compel its performance. A
ISSUE: purely ministerial act, as distinguished from a
Whether or not the twenty percent allocation discretionary act, is one which an officer or
for party-list lawmakers is mandatory. tribunal performs in a given state of facts, in a
prescribed manner, in obedience to the
RULING: mandate of legal authority, without regard to
Section VI 5(2) of Article of the Constitution is or the exercise of his own judgment upon the
not mandatory. It merely provides a ceiling for propriety or impropriety of the act done.
the party-list seats in the House of
Representatives. The Supreme Court ruled The case at bar is one of mandamus over
that the Constitution and RA 7941 mandate at which this Court has jurisdiction for it is
least 4 inviolable parameters: respondent Commission's ministerial duty to
(1) the 20% allocation: the combined number apply the formula as decided by this Court
of all party-list congressmen shall not exceed after interpreting the existing law on party-list
20% of the total membership of the House of representation. It is given that this Court has
Representatives; (2) the 2% threshold: only the ultimate authority to interpret laws and the
those parties garnering a minimum of 2% of Constitution. Respondent Commission has no
the total votes cast for the party list system discretion to refuse enforcement of any
are qualified to a have a seat in the House; (3) decision of this Court under any guise or guile.
the three seat limit: each qualified party,
regardless of the number of votes it actually In any event, it is the averments in the
obtained, is entitled to a maximum of three complaint, and not the nomenclature given by
seats, i.e., one qualifying and two additional; the parties, that determine the nature of the
and (4) proportional representation: the action.[24] Though captioned as a Petition for
additional seats which a qualified party is Mandamus, the same may be treated as a
entitled to shall be computed ―in proportion petition for certiorari and mandamus
to their total number of votes”. considering that it alleges that the respondent
Commission acted contrary to prevailing
153 Partido Ng Manggagawa vs. COMELEC, jurisprudence, hence, with grave
March 15, 2006 abuse of discretion and without
jurisdiction. In previous rulings, we have
FACTS: treated differently labeled actions as special
The petition at bar involves the formula for civil actions for certiorari under Rule 65 for
computing the additional seats due, if any, for reasons such as "justice, equity and fairplay"
winners in party-list elections. Comelec issued and "novelty of the issue presented and its far-
Resolution No. 6835,1adopting the simplified reaching effects." The petition at bar involves
formula of "one additional seat per additional the rightful representation in the House of
two percent of the total party-list votes. Representatives of the marginalized groups by
"Section 11(b) and Section 12 of R.A. 7941 the party-list winners and their constitutional
(Party-List System Act) provide that "the claim merits more than a disposition based on
parties, organizations, and coalitions receiving thin technicality.
at least two percent (2%) of the total votes
cast for the party-list system shall be entitled 154 BANAT vs. COMELEC, G.R. No. 179271,
to one seat each, provided that those April 21, 2009
garnering more than two percent (2%) of the
votes shall be entitled to additional seats in FACTS:
proportion to their total number of votes xxx. In July and August 2007, the Comelec, sitting
For failure of the respondent Commission to as the National Board of Canvassers, made a
resolve the substantive issues raised by partial proclamation of the winners in the
petitioners and to cause there-tabulation of party-list elections which was held in May
the party-list votes despite the lapse of time, 2007. In proclaiming the winners and
petitioners PM and BUTIL filed the instant apportioning their seats, the Comelec
petition on August 18, 2004. They seek the considered the following rules: (1) In the lower
Page 110 of 121
403 CASE DIGESTS | CONSTITUTIONAL LAW REVIEW | JUDGE ESTELA ALMA SINGCO | FIRST SEMESTER | 2020-2021

house, 80% shall comprise the seats for


legislative districts, while the remaining 20% However, we cannot allow the
shall come from party-list representatives continued existence of a provision in
(Sec. 5, Article VI, 1987 Constitution); (2) the law which will systematically
Pursuant to Sec. 11b of R.A. 7941 or the Party- prevent the constitutionally allocated
List System Act, a party-list which garners at 20% party-list representatives from
least 2% of the total votes cast in the party- being filled. The three-seat cap, as a
list elections shall be entitled to one seat; (3) limitation to the number of seats that
If a party-list garners at least 4%, then it is a qualified party-list organization may
entitled to 2 seats; if it garners at least 6%, occupy, remains a valid statutory
then it is entitled to 3 seats – this is pursuant device that prevents any party from
to the 2-4-6 rule or the Panganiban Formula dominating the party-list elections.
from the case of Veterans Federation Party vs Seats for party-list representatives
Comelec; (4) In no way shall a party be given shall thus be allocated in accordance
more than three seats even if it garners more with the procedure used in Table 3
than 6% of the votes cast for the party-list above.
election (3 seat cap rule, same case).
The Barangay Association for National However, by a vote of 8-7, the Court
Advancement and Transparency (BANAT), a decided to continue the ruling in
party-list candidate, questioned the Veterans disallowing major political
proclamation as well as the formula being parties from participating in the party-
used. BANAT averred that the 2% threshold is list elections, directly or indirectly.
invalid; Sec. 11 of RA 7941 is void because its
provision that a party-list, to qualify for a 2. 2% threshold in relation to the
congressional seat, must garner at least 2% of distribution of additional seats as
the votes cast in the party-list election, is not found in the second clause of Section
supported by the Constitution. Further, the 2% 11(b) of R.A. No. 7941 is declared
rule creates a mathematical impossibility to unconstitutional. The two percent
meet the 20% party-list seat prescribed by the threshold presents an unwarranted
Constitution. BANAT also questioned if the obstacle to the full implementation of
20% rule is a mere ceiling or is it mandatory. Section 5(2), Article VI of the
If it is mandatory, then with the 2% qualifying Constitution and prevents the
vote, there would be instances when it would attainment of ―the broadest possible
be impossible to fill the prescribed 20% share representation of party, sectoral or
of party-lists in the lower house. group interests in the House of
Representatives.
On the other hand, BAYAN MUNA, another
party-list candidate, questions the validity of
In determining the allocation of seats
the 3 seat rule (Section 11a of RA 7941). It
for party-list representatives under
also raised the issue of whether or not major
Section 11 of R.A. No. 7941, the
political parties are allowed to participate in
following procedure shall be
the party-list elections or is the said elections
observed:
limited to sectoral parties.

ISSUES: 1) The parties, organizations,


and coalitions shall be ranked
from the highest to the lowest
1. Whether or not the 20% allocation for
based on the number of votes
party-list representatives is
they garnered during the
mandatory. (NO)
elections.
2. Whether or not the 2% threshold to
2) The parties, organizations,
qualify for a seat is valid. (NO)
and coalitions receiving at
3. Whether or not the Constitution
least two percent (2%) of the
prohibit the major political parties
total votes cast for the party-
from participating in the party-list
list system shall be entitled to
elections?
one guaranteed seat each.
3) Those garnering sufficient
RULINGS:
number of votes, according to
the ranking in paragraph 1,
1. Neither the Constitution nor R.A. No. shall be entitled to additional
7941 mandates the filling-up of the seats in proportion to their
entire 20% allocation of party-list total number of votes until all
representatives found in the the additional seats are
Constitution. The Constitution, in allocated.
paragraph 1, Section 5 of Article VI, 4) Each party, organization, or
left the determination of the number coalition shall be entitled to
of the members of the House of not more than three (3)
Representatives to Congress: ―The seats.
House of Representatives shall be
composed of not more than two
In computing the additional seats, the
hundred and fifty members, unless
guaranteed seats shall no longer be
otherwise fixed by law, x x x. The 20%
included because they have already
allocation of party-list representatives
been allocated, at one seat each, to
is merely a ceiling; party-list
every two-percenter. Thus, the
representatives cannot be more than
remaining available seats for
20% of the members of the House of
allocation as ―additional seats‖ are
Representatives.
the maximum seats reserved under
the Party List System less the
guaranteed seats. Fractional seats are
Page 111 of 121
403 CASE DIGESTS | CONSTITUTIONAL LAW REVIEW | JUDGE ESTELA ALMA SINGCO | FIRST SEMESTER | 2020-2021

disregarded in the absence of a organizations for electoral or political


provision in R.A. No. 7941 allowing for purposes. There should not be a
a rounding off of fractional seats. problem if, for example, the Liberal
Party participates in the party-list
In declaring the two percent threshold election through the Kabataang
unconstitutional, we do not limit our Liberal ng Pilipinas (KALIPI), its
allocation of additional seats in Table sectoral youth wing. The other major
3 below to the two-percenters. The political parties can thus organize, or
percentage of votes garnered by each affiliate with, their chosen sector or
party-list candidate is arrived at by sectors. To further illustrate, the
dividing the number of votes garnered Nacionalista Party can establish a
by each party by 15,950,900, the total fisherfolk wing to participate in the
number of votes cast for party-list party-list election, and this fisherfolk
candidates. There are two steps in the wing can field its fisherfolk nominees.
second round of seat allocation. First, Kabalikat ng Malayang Pilipino
the percentage is multiplied by the (KAMPI) can do the same for the
remaining available seats, 38, which urban poor.
is the difference between the 55
maximum seats reserved under the 155 ATONG PAGLAUM, INC. vs. COMELEC, GR
Party-List System and the 17 No. 203646, April 2, 2013
guaranteed seats of the two-
percenters. The whole integer of the
product of the percentage and of the The Comelec disqualified petitioners from
remaining available seats corresponds participating in the May 2013 party-list
to a party‘s share in the remaining elections either by the denial of their new
available seats. Second, we assign petitions for registration or by cancellation of
one party-list seat to each of the their existing registrations.
parties next in rank until all available
seats are completely distributed. We Is the Comelec correct?
distributed all of the remaining 38
seats in the second round of seat No. In determining who may participate in the
allocation. Finally, we apply the three- party-list elections, the COMELEC shall adhere
seat cap to determine the number of to the following parameters:
seats each qualified party-list 1. Three different groups may participate
candidate is entitled. in the party-list system: (1) national
parties or organizations, (2) regional
parties or organizations, and (3) sectoral
3. The Constitutional Commission parties or organizations.
adopted a multi-party system that
allowed all political parties to 2. National parties or organizations and
participate in the party-list elections. regional parties or organizations do not
need to organize along sectoral lines and
Neither the Constitution nor R.A. No. do not need to represent any
7941 prohibits major political parties "marginalized and underrepresented"
from participating in the party-list sector.
system. On the contrary, the framers
of the Constitution clearly intended 3. Political parties can participate in
the major political parties to party-list elections provided they register
participate in party-list elections under the party-list system and do not
through their sectoral wings. In fact, field candidates in legislative district
the members of the Constitutional elections. A political party, whether major
Commission voted down, 19-22, any or not, that fields candidates in legislative
permanent sectoral seats, and in the district elections can participate in party-
alternative the reservation of the list elections only through its sectoral wing
party-list system to the sectoral that can separately register under the
groups. In defining a “party” that party-list system. The sectoral wing is by
participates in party-list elections as itself an independent sectoral party, and is
either “a political party or a sectoral linked to a political party through a
party, “R.A. No. 7941 also clearly coalition.
intended that major political parties
will participate in the party-list 4. Sectoral parties or organizations may
elections. Excluding the major political either be "marginalized and
parties in party-list elections is underrepresented" or lacking in "well-
manifestly against the Constitution, defined political constituencies." It is
the intent of the Constitutional enough that their principal advocacy
Commission, and R.A. No. 7941. This pertains to the special interest and
Court cannot engage in socio-political concerns of their sector. The sectors that
engineering and judicially legislate the are "marginalized and underrepresented"
exclusion of major political parties include labor, peasant, fisherfolk, urban
from the party- list elections in patent poor, indigenous cultural communities,
violation of the Constitution and the handicapped, veterans, and overseas
law. workers. The sectors that lack "well-
defined political constituencies" include
Read together, R.A. No. 7941 and the professionals, the elderly, women, and the
deliberations of the Constitutional youth.
Commission state that major political
parties are allowed to establish, or 5. A majority of the members of sectoral
form coalitions with, sectoral parties or organizations that represent the
Page 112 of 121
403 CASE DIGESTS | CONSTITUTIONAL LAW REVIEW | JUDGE ESTELA ALMA SINGCO | FIRST SEMESTER | 2020-2021

"marginalized and underrepresented" order to be valid, neither should such be


must belong to the "marginalized and needed for an additional district in a province,
underrepresented" sector they represent. considering moreover that a province is
Similarly, a majority of the members of entitled to an initial seat by the mere fact of its
sectoral parties or organizations that lack creation and regardless of its population.
"well-defined political constituencies"
must belong to the sector they represent.
The nominees of sectoral parties or 157 Aldaba, et al. vs. COMELEC, GR No.
organizations that represent the 188078, January 25, 2010
"marginalized and underrepresented," or
that represent those who lack "well-
defined political constituencies," either RA 9591 was passed creating a legislative
must belong to their respective sectors, or district for the city of Malolos, Bulacan.
must have a track record of advocacy for According to petitioners, the law violates the
their respective sectors. The nominees of minimum population requirement under the
national and regional parties or Constitution for the creation of a legislative
organizations must be bona-fide members district in a city.
of such parties or organizations.

5. National, regional, and sectoral


parties or organizations shall not be
disqualified if some of their nominees Petitioners are correct. RA 9591 is
are disqualified, provided that they unconstitutional for being violative of Section
have at least one nominee who 5 (3), Article VI of the 1987 Constitution and
remains qualified. Section 3 of the Ordinance appended to the
6. 1987 Constitution. It requires that for a city to
156 Aquino vs. COMELEC, GR No. 189793, have a legislative district, the city must have a
April 7, 2010 population of at least 250,000.

Petitioners contend that the reapportionment


introduced by RA 9716, runs afoul of the Sec. 5(3). Each legislative district shall
explicit constitutional standard that requires a comprise, as far as practicable, contiguous,
minimum population of 250,000 for the compact, and adjacent territory. Each city
creation of a legislative district. The petitioners with a population of at least two hundred
claim that the reconfiguration of the first and fifty thousand, or each province, shall have
second districts of Camarines Sur is at least one representative.
unconstitutional, because the proposed first
district will end up with a population of less
than 250,000 or only 176,383. They anchor on
Section 5(3), Article VI of the Constitution, A city that has attained a population of
which provides: 250,000 is entitled to a legislative district only
in the "immediately following election." In
Sec. 5(3). Each legislative district shall short, a city must first attain the 250,000
comprise, as far as practicable, contiguous, population, and thereafter, in the immediately
compact, and adjacent territory. Each city following election, such city shall have a
with a population of at least two hundred district representative. There is no showing in
fifty thousand, or each province, shall have the present case that the City of Malolos has
at least one representative. attained or will attain a population of 250,000,
whether actual or projected, before the 10 May
Petition is denied. There is no specific provision 2010 elections.
in the Constitution that fixes a 250,000
minimum population that must compose a
legislative district. The 2nd sentence of Sec. Clearly, there is no official record that the
5(3) draws a plain and clear distinction population of the City of Malolos will be at least
between the entitlement of a city to a district 250,000, actual or projected, prior to the 10
on one hand, and the entitlement of a province May 2010 elections, the immediately following
to a district on the other. For while a province election after the supposed attainment of such
is entitled to at least a representative, with population. Thus, the City of Malolos is not
nothing mentioned about population, a city qualified to have a legislative district of its own
must first meet a population minimum of under Section 5(3), Article VI of the 1987
250,000 in order to be similarly entitled. Sec. Constitution and Section 3 of the Ordinance
5(3) of the Constitution requires a 250,000 appended to the 1987 Constitution.
minimum population only for a city to be
entitled to a representative, but not so for a 158 Ang Ladlad LGBT Party v. COMELEC, GR
province. No. 190582, April 8, 2010

While Section 5(3) requires a city to have a Petitioner is an organization composed of men
minimum population of 250,000 to be entitled and women who identify themselves as
to a representative, it does not have to lesbians, gays, bisexuals, or trans-gendered
increase its population by another 250,000 to individuals (LGBTs). Comelec refused to
be entitled to an additional district (Mariano v. accredit petitioner as a party-list organization
Comelec). There is no reason why the Mariano for allegedly advocating immoral doctrines.
case, which involves the creation of an
additional district within a city, should not be The Comelec erred. Governmental reliance on
applied to additional districts in provinces. religious justification is inconsistent with this
Indeed, if an additional legislative district policy of neutrality." Thus, it was grave
created within a city is not required to violation of the non-establishment clause for
represent a population of at least 250,000 in the Comelec to utilize the Bible and the Koran
Page 113 of 121
403 CASE DIGESTS | CONSTITUTIONAL LAW REVIEW | JUDGE ESTELA ALMA SINGCO | FIRST SEMESTER | 2020-2021

to justify the exclusion of petitioner. the people" are multifarious. The accused-
Government must act for secular purposes and appellant asserts that the duty to legislate
in ways that have primarily secular effects. ranks highest in the hierarchy of government.
The accused-appellant is only one of 250
The Comelec posits that the majority of the members of the House of Representatives, not
Philippine population considers homosexual to mention the 24 members of the Senate,
conduct as immoral and unacceptable, and this charged with the duties of legislation.
constitutes sufficient reason to disqualify the Congress continues to function well in the
petitioner. However, the Philippine electorate physical absence of one or a few of its
has expressed no such belief. No law exists to members.
criminalize homosexual behavior or
expressions or parties about homosexual
behavior. The Philippines has not seen fit to Moreover, the members of Congress cannot
criminalize homosexual conduct. Evidently, compel absent members to attend sessions if
therefore, these "generally accepted public the reason for the absence is a legitimate one.
morals" have not been convincingly The confinement of a Congressman charged
transplanted into the realm of law. with a crime punishable by imprisonment of
more than six months is not merely authorized
by law, it has constitutional foundations.
In refusing petitioner’s accreditation, there
should have been a finding by the Comelec 160 Trillanes IV vs. Pimentel, June 27, 2008
that the group's members have committed or
are committing immoral acts. Mere allegation A group of more than 300 heavily armed
of violation of laws is not proof, and a mere soldiers led by junior officers of the AFP
blanket invocation of public morals cannot stormed into the Oakwood Premier
replace the institution of civil or criminal Apartments in Makati City and publicly
proceedings and a judicial determination of demanded the resignation of the President and
liability or culpability. Moral disapproval, key national officials. Petitioner Trillanes was
without more, is not a sufficient governmental charged, along with his comrades, with coup
interest to justify exclusion of homosexuals d'etat. Close to four years later, petitioner won
from participation in the party-list system. a seat in the Senate.
Comelec’s blanket justifications give rise to the
inevitable conclusion that it targets Petitioner points out that former Rep. Jalosjos
homosexuals themselves as a class, not (in People v. Jalosjos) was already convicted,
because of any particular morally albeit his conviction was pending appeal, when
reprehensible act. It is this selective targeting he filed a motion similar to petitioner's
that implicates our equal protection clause. Omnibus Motion, whereas he (petitioner) is a
mere detention prisoner. He asserts that he
LEGISLATIVE PERKS continues to enjoy civil and political rights
since the presumption of innocence is still in
159 PP vs. Jalosjos, 324 SCRA 689 his favor.

Jalosjos is confined at the national penitentiary Can petitioner be allowed to attend senate
while his conviction for statutory rape on two sessions?
counts and acts of lasciviousness on six counts
is pending appeal. No. Election to Congress is not a reasonable
classification in criminal law enforcement as
Can he be allowed to fully discharge the duties the functions and duties of the office are not
of a Congressman, including attendance at substantial distinctions which lift one from the
legislative sessions and committee meetings class of prisoners interrupted in their freedom
despite his having been convicted in the first and restricted in liberty of movement.
instance of a non-bailable offense?
It is impractical to draw a line between
No. Election to the position of Congressman is convicted prisoners and pre-trial detainees for
not a reasonable classification in criminal law the purpose of maintaining jail
enforcement. The functions and duties of the
office are not substantial distinctions which lift security; and while pre-trial detainees do not
him from the class of prisoners interrupted in forfeit their constitutional rights upon
their freedom and restricted in liberty of confinement, the fact of their detention makes
movement. Lawful arrest and confinement are their rights more limited than those of the
germane to the purposes of the law and apply public.
to all those belonging to the same class.
All prisoners whether under preventive
Allowing accused-appellant to attend detention or serving final sentence cannot
congressional sessions and committee practice their profession nor engage
meetings for five (5) days or more in a week
will virtually make him a free man with all the
in any business or occupation, or hold office,
privileges appurtenant to his position. Such an
elective or appointive, while in detention. This
aberrant situation not only elevates accused-
is a necessary consequence of arrest and
appellant's status to that of a special class, it
detention (People v. Maceda). These inherent
also would be a mockery of the purposes of the
limitations, however, must be taken into
correction system.
account only to the extent that confinement
restrains the power of locomotion or actual
The performance of legitimate and even physical movement. It bears noting that in
essential duties by public officers has never Jalosjos, which was decided en banc one
been an excuse to free a person validly in month after Maceda, the Court recognized that
prison. The duties imposed by the "mandate of

Page 114 of 121


403 CASE DIGESTS | CONSTITUTIONAL LAW REVIEW | JUDGE ESTELA ALMA SINGCO | FIRST SEMESTER | 2020-2021

the accused could somehow accomplish Section 13, Article VI of the Constitution,
legislative results. which reads:

Sec. 13. No Senator or Member of the


161 Trillanes vs. Marigomen, G.R. No. 179817, House of Representatives may hold any
March 14, 2018 other office or employment in the
Government, or any subdivision, agency,
or instrumentality thereof, including
Petitioner, as senator, is conducting an government-owned or controlled
investigation, in aid of legislation, for the corporations or their subsidiaries, during
alleged overpricing of the Makati City Hall his term without forfeiting his seat.
Parking Building. Petitioner, during media Neither shall he be appointed to any office
interviews at the Senate, particularly during which may have been created or the
gaps and breaks in the plenary hearings as emoluments thereof increased during the
well as committee hearings, expressed his term for which he was elected.
opinion that based on his office's review of the
documents, private respondent appears to be Is the PNRC a government office or a GOCC for
a "front" or "nominee" or is acting as a purposes of the prohibition in Section 13,
"dummy" of the actual and beneficial owner of Article VI of the Constitution?
the estate, VP Binay. Thus, private respondent
filed a Complaint for Damages against No. The PNRC is a privately owned, privately
petitioner for the latter's alleged defamatory funded, and privately run charitable
statements before the media, specifically his organization. The PNRC is not a GOCC.
repeated accusations that private respondent
is a mere "dummy" of VP Binay. The PNRC is a non-profit, donor-funded,
voluntary, humanitarian organization. The
Are petitioner's statements in media interview Republic of the Philippines, adhering to the
covered by the parliamentary "speech or Geneva Conventions, established the PNRC as
debate" privilege? a voluntary organization. Thus, the PNRC must
not only be, but must also be seen to be,
No. Petitioner's remarks fall outside the autonomous, neutral and independent in order
privilege of speech or debate under Section 11, to conduct its activities in accordance with the
Article VI of the 1987 Constitution. The Fundamental Principles. It is imperative that
statements were clearly not part of any speech the PNRC must be autonomous, neutral, and
delivered in the Senate or any of its independent in relation to the State.
committees. They were also not spoken in the
course of any debate in said fora. It cannot To ensure and maintain its autonomy,
likewise be successfully contended that they neutrality, and independence, the PNRC
were made in the official discharge or cannot be owned or controlled by the
performance of petitioner's duties as a government. Indeed, the Philippine
Senator, as the remarks were not part of or government does not own the PNRC. The PNRC
integral to the legislative process. does not have government assets and does not
receive any appropriation from the Philippine
Parliamentary non-accountability cannot be Congress. The PNRC is financed primarily by
invoked when the lawmaker's speech or contributions from private individuals and
utterance is made outside sessions, hearings private entities obtained through solicitation
or debates in Congress, extraneous to the "due campaigns organized by its Board of
functioning of the legislative process." To Governors.
participate in or respond to media interviews
is not an official function of any lawmaker; it is The government does not control the PNRC.
not demanded by his sworn duty nor is it a Under the PNRC Charter, as amended, only six
component of the process of enacting laws. of the thirty members of the PNRC Board of
Indeed, a lawmaker may well be able to Governors are appointed by the President of
discharge his duties and legislate without the Philippines.
having to communicate with the press. A
lawmaker's participation in media interviews is Under Section 16, Article VII of the
not a legislative act, but is "political in nature," Constitution, the President appoints all officials
outside the ambit of the immunity conferred and employees in the Executive branch whose
under the Speech or Debate Clause in the 1987 appointments are vested in the President by
Constitution. Contrary to petitioner's stance, the Constitution or by law. The President also
therefore, he cannot invoke parliamentary appoints those whose appointments are not
immunity to cause the dismissal of private otherwise provided by law. Under this Section
respondent's Complaint. The privilege arises 16, the law may also authorize the heads of
not because the statement is made by a departments, agencies, commissions, or
lawmaker, but because it is uttered in boards to appoint officers lower in rank than
furtherance of legislation. such heads of departments, agencies,
commissions or boards.

162 Liban v. Gordon, G.R. No. 175352, July 15, The President does not appoint the Chairman
2009 of the PNRC. Neither does the head of any
department, agency, commission or board
Petitioners seek to declare the forfeiture of the appoint the PNRC Chairman. Thus, the PNRC
Senate seat of Sen. Gordon, by contending Chairman is not an official or employee of the
that respondent’s acceptance of the Executive branch since his appointment does
chairmanship of the Phil. National Red Cross not fall under Section 16, Article VII of the
(PNRC) Board of Governors, he has ceased to Constitution. Certainly, the PNRC Chairman is
be a member of the Senate, as provided under not an official or employee of the Judiciary or
Legislature. This leads us to the obvious
Page 115 of 121
403 CASE DIGESTS | CONSTITUTIONAL LAW REVIEW | JUDGE ESTELA ALMA SINGCO | FIRST SEMESTER | 2020-2021

conclusion that the PNRC Chairman is not an unconstitutionality, which was not even
official or employee of the Philippine originally a party to this case, was being
Government. Not being a government official compelled, as a consequence of the Decision,
or employee, the PNRC Chairman, as such, to suddenly reorganize and incorporate under
does not hold a government office or the Corporation Code, after more than sixty
employment. Under Section 17, Article VII of (60) years of existence in this country.
the Constitution, the President exercises
control over all government offices in the The purpose of the constitutional provision
Executive branch. If an office is legally not prohibiting Congress from creating private
under the control of the President, then such corporations was to prevent the granting of
office is not part of the Executive branch. special privileges to certain individuals,
families, or groups, which were denied to other
However, RA 95 or the PNRC Charter (enacted groups. Based on the above discussion, it can
on March 1947) is unconstitutional as it be seen that the PNRC Charter does not come
violates the proscription under the 1935 within the spirit of this constitutional provision,
Constitution against the creation of private as it does not grant special privileges to a
corporations by special law, to wit: particular individual, family, or group, but
creates an entity that strives to serve the
Art. XIV, Sec. 7. The Congress shall not, common good.
except by general law, provide for the
formation, organization, or regulation of Furthermore, a strict and mechanical
private corporations, unless such interpretation of Article XII, Section 16 of the
corporations are owned or controlled by 1987 Constitution will hinder the State in
the Government or any subdivision or adopting measures that will serve the public
instrumentality thereof. good or national interest. It should be noted
that a special law, R.A. No. 9520, the
The elements of government ownership and Philippine Cooperative Code of 2008, and not
control are clearly lacking in the PNRC. Thus, the general corporation code, vests corporate
although the PNRC is created by a special power and capacities upon cooperatives which
charter, it cannot be considered a are private corporations, in order to implement
government-owned or controlled corporation the State's avowed policy.
in the absence of the essential elements of
ownership and control by the government. In Based on the above, the sui generis status of
creating the PNRC as a corporate entity, the PNRC is now sufficiently established.
Congress was in fact creating a private Although it is neither a subdivision, agency, or
corporation. However, the constitutional instrumentality of the government, nor a
prohibition against the creation of private GOCC or a subsidiary thereof, as succinctly
corporations by special charters provides no explained in the Decision of July 15, 2009, so
exception even for non-profit or charitable much so that respondent, under the Decision,
corporations. Consequently, the PNRC was correctly allowed to hold his position as
Charter, insofar as it creates the PNRC as a Chairman thereof concurrently while he served
private corporation and grants it corporate as a Senator, such a conclusion does not ipso
powers, is void for being unconstitutional. facto imply that the PNRC is a "private
Liban v. Gordon, G.R. No. 175352, corporation" within the contemplation of the
163 January 18, 2011 provision of the Constitution, that must be
organized under the Corporation Code.
This is a resolution on the Motion for
Clarification and/or for Reconsideration by
respondent on the decision earlier rendered by 164 Santiago vs. Guingona (298 SCRA 756)
the Court on 2009. Respondent raises the Principle: The term majority simply means
following grounds: (1) as the issue of the greater number or more than half. Who
constitutionality of Republic Act (R.A.) No. 95 shall sit as officers is the sole prerogative of
was not raised by the parties, the Court went the Senate. (Note: splitting of term between
beyond the case in deciding such issue; and Senate President Drilon and another Senator).
(2) as the Court decided that Petitioners did When the Constitution provides that the
not have standing to file the instant Petition, Senate President shall be elected by the
the pronouncement of the Court on the validity majority it does not delineate who comprises
of R.A. No. 95 should be considered obiter. the majority or the minority. The defeated
senator (s) in the election for the Senate
As correctly pointed out in respondent's presidency are not necessarily the minority.
Motion, the issue of constitutionality of R.A.
No. 95 was not raised by the parties, and was FACTS: During the first regular session of the
not among the issues defined in the body of eleventh Congress, Sen. Fernan was declared
the Decision; thus, it was not the very lis mota the duly elected President of the Senate. The
of the case. A court should not pass upon a following were likewise elected: Senator Ople
constitutional question and decide a law to be as president pro tempore, and Sen. Franklin M.
unconstitutional or invalid, unless such Drilon as majority leader.
question is raised by the parties.
Senator Tatad thereafter manifested that, with
This Court should not have declared void the agreement of Senator Santiago, allegedly
certain sections of R.A. No. 95, as amended by the only other member of the minority, he was
P.D. Nos. 1264 and 1643, the PNRC Charter. assuming the position of minority leader. He
Instead, the Court should have exercised explained that those who had voted for
judicial restraint on this matter, especially Senator Fernan comprised the "majority,"
since there was some other ground upon which while only those who had voted for him, the
the Court could have based its judgment. losing nominee, belonged to the "minority."
Furthermore, the PNRC, the entity most
adversely affected by this declaration of
Page 116 of 121
403 CASE DIGESTS | CONSTITUTIONAL LAW REVIEW | JUDGE ESTELA ALMA SINGCO | FIRST SEMESTER | 2020-2021

During the discussion on who should constitute


the Senate "minority," Sen. Flavier FACTS: A petition was filed challenging the
manifested that the senators belonging to the validity of RA 8240, which amends certain
Lakas-NUCD-UMDP Party — numbering seven provisions of the NIRC. Petitioners, who are
(7) and, thus, also a minority — had chosen members of the House of Representatives,
Senator Guingona as the minority leader. charged that there is violation of the rules of
No consensus on the matter was arrived at. the House which petitioners claim are
constitutionally-mandated so that their
On July 30, 1998, the majority leader informed violation is tantamount to a violation of the
the body chat he was in receipt of a letter Constitution.
signed by the seven Lakas-NUCD-UMDP
senators, stating that they had elected Senator The law originated in the House of
Guingona as the minority leader. By virtue Representatives. The Senate approved it with
thereof, the Senate President formally certain amendments. A bicameral conference
recognized Senator Guingona as the committee was formed to reconcile the
minority leader of the Senate. disagreeing provisions of the House and
Senate versions of the bill. The bicameral
The following day, Senators Santiago and committee submitted its report to the House.
Tatad filed before this Court the subject During the interpellations, Rep. Arroyo made
petition for quo warranto, alleging in the main an interruption and moved to adjourn for lack
that Senator Guingona had been usurping, of quorum. But after a roll call, the Chair
unlawfully holding and exercising the position declared the presence of a quorum. The
of Senate minority leader, a position that, interpellation then proceeded. After Rep.
according to them, rightfully belonged to Arroyo’s interpellation of the sponsor of the
Senator Tatad. committee report, Majority Leader Albano
moved for the approval and ratification of the
ISSUE: In recognizing Respondent Guingona conference committee report. The Chair called
as the Senate minority leader, did the Senate out for objections to the motion. Then the
or its officials, particularly Senate President Chair declared: “There being none, approved.”
Fernan, violate the Constitution or the laws? At the same time the Chair was saying this,
Rep. Arroyo was asking, “What is that…Mr.
HELD: No. The interpretation proposed by Speaker?” The Chair and Rep. Arroyo were
petitioners finds no clear support from the talking simultaneously. Thus, although Rep.
Constitution, the laws, the Rules of the Senate Arroyo subsequently objected to the Majority
or even from practices of the Upper House. The Leader’s motion, the approval of the
term “majority,” when referring to a certain conference committee report had by then
number out of a total or aggregate, it simply already been declared by the Chair.
means the number greater than half or
more than half of any total. In effect, while On the same day, the bill was signed by the
the Constitution mandates that the President Speaker of the House of Representatives and
of the Senate must be elected by a number the President of the Senate and certified by the
constituting more than one half of all the respective secretaries of both Houses of
members thereof, it does not provide that Congress. The enrolled bill was signed into law
the members who will not vote for him by President Ramos.
shall ipso facto constitute the minority,
who could thereby elect the minority leader. ISSUE: WON RA 8240 is null and void because
No law or regulation states that the it was passed in violation of the rules of the
defeated candidate shall automatically House
become the minority leader.
HELD: NO. Rules of each House of
While the Constitution is explicit in the manner Congress are hardly permanent in
of electing a Senate President and a House character. They are subject to revocation,
Speaker, it is, however, dead silent on the modification or waiver at the pleasure of
manner of selecting the other officers in both the body adopting them as they are
chambers of Congress. All that the Charter primarily procedural. Courts ordinarily have
says under Art. VI, Sec. 16(1) is that “each no concern with their observance. They may
House shall choose such other officers as it be waived or disregarded by the
may deem necessary.” The method of legislative body. Consequently, mere
choosing who will be such other officers is failure to conform to them does not have
merely a derivative of the exercise of the the effect of nullifying the act taken if the
prerogative conferred by the said requisite number of members has agreed
constitutional provision. Therefore, such to a particular measure. But this is subject
method must be prescribed by the Senate to qualification. Where the construction to be
itself, not by the Court. given to a rule affects person other than
members of the legislative body, the question
presented is necessarily judicial in character.
Even its validity is open to question in a case
RULES OF PROCEEDINGS
where private rights are involved.
165 Arroyo vs. De Venecia, 277 SCRA 268
In this case, no rights of private individuals are
Principle: Courts cannot inquire into the involved but only those of a member who,
allegations that in enacting a law, a House of instead of seeking redress in the House, chose
Congress failed to comply with its own rules in to transfer the dispute to the Court.
the absence of showing that there was
violation of a constitutional provision or private The matter complained of concerns a matter of
rights. Parliamentary rules are mere internal procedure of the House with which the
procedures which may be waived or Court should not be concerned. The claim is
disregarded by the legislative body. not that there was no quorum but only that
Rep. Arroyo was effectively prevented from
Page 117 of 121
403 CASE DIGESTS | CONSTITUTIONAL LAW REVIEW | JUDGE ESTELA ALMA SINGCO | FIRST SEMESTER | 2020-2021

questioning the presence of a quorum. Rep. imprints of permanence and


Arroyo’s earlier motion to adjourn for lack of obligatoriness during their effectivity. In
quorum had already been defeated, as the roll fact, they 'are subject to revocation,
call established the existence of a quorum. The modification or waiver at the pleasure of the
question of quorum cannot be raised body adopting them.' Being merely matters of
repeatedly especially when the quorum is procedure, their observance are of no concern
obviously present for the purpose of delaying to the courts, for said rules may be waived or
the business of the House. disregarded by the legislative body at will,
upon the concurrence of a majority of the
166 Baguilat vs. Alvarez, G.R. No. 227757, House of Representatives." Hence, as a
July 25, 2017 general rule, “this Court has no authority to
interfere and unilaterally intrude into that
FACTS: Petitioner Baguilat filed a petition for exclusive realm, without running afoul of
mandamus praying that respondents Alvarez Constitutional principles that it is bound to
et. al be compelled to recognize: Rep. Baguilat protect and uphold x x x. Constitutional
as the Minority Leader of the 17th Congress of respect and a becoming regard for the
the House of Representatives and petitioners sovereign acts of a coequal branch prevents
as the legitimate members of the Minority. the Court from prying into the internal
Petitioners hoped that as a "long-standing workings of the House of Representatives."
tradition" of the House – where the candidate
who garnered the second second-highest
number of votes for Speakership automatically
DISCIPLINING MEMBERS
becomes the Minority Leader. However,
despite numerous follow-ups from 167 Osmeña vs Pendatun, G.R. No. L-17144
respondents, Rep. Baguilat was never
recognized as such. Principle: The House of Representatives is the
judge of what constitutes disorderly behavior.
Representative Abayon manifested before the The courts will not assume jurisdiction in any
Plenary that on those who did not vote for case which will amount to an interference by
Speaker Alvarez convened and elected Rep. the judicial department with the legislature.
Suarez as the Minority Leader. Thereafter,
Majority Leader Fariñas moved for the FACTS: Congressman Sergio Osmeña Jr.,
recognition of Rep. Suarez as the Minority herein petitioner, delivered his privilege
Leader. speech before the House making serious
imputations of bribery against the President of
ISSUE: Whether or not respondents may be the Philippines. Because of this, a Resolution
compelled via a writ of mandamus to recognize was issued authorizing the creation of special
Rep. Baguilat as the Minority Leader of the House Committee to investigate the truth of
House of Representatives and petitioners as the charges made against the President, to
the only legitimate members of the House summon petitioner to substantiate his
Minority. charges, and in case petitioner fails to do so,
to require petitioner to show cause why he
should not be punished by the House.
HELD: NO. The petition is without merit.
Petitioner then resorted to the Court seeking
Under Section 16 of Article VI, the Speaker of for the annulment of said resolution on the
the House of Representatives shall be elected ground that it infringes his constitutional
by a majority vote of its entire membership. absolute parliamentary immunity for speeches
Said provision also states that the House of delivered in the House. Meanwhile, the Special
Representatives may decide to have Committee continued with its proceeding, and
officers other than the Speaker, and that after giving petitioner a chance to defend
the method and manner as to how these himself, found the latter guilty of seriously
officers are chosen is something within disorderly behavior. A House resolution was
its sole control. In the case of Defensor- issued and petitioner was suspended from
Santiago v. Guingona, which involved a office for 15 months.
dispute on the rightful Senate Minority Leader
during the 11th Congress (1998-2001), the Thereafter, respondents filed their answer
Supreme Court observed that "while the challenging the jurisdiction of this Court to
Constitution is explicit on the manner of entertain the petition, and defended the power
electing x x x (a Speaker of the House of of Congress to discipline its members with
Representative,] it is, however, dead silent on suspension.
the manner of selecting the other officers [of
the Lower House). All that the Charter says is ISSUE: Whether the House Resolution
that each House shall choose such other violated petitioner’s constitutionally granted
officers as it may deem necessary.' [As such], parliamentary immunity for speeches
the method of choosing who will be such other
officers is merely a derivative of the exercise HELD: NO. Section 15, Article VI of our
of the prerogative conferred by the Constitution provides that “for any speech or
aforequoted constitutional provision. debate” in Congress, the Senators or Members
Therefore, such method must be prescribed by of the House of Representative “shall not be
the House of Representatives itself, not by the questioned in any other place.” Our
Court." Constitution enshrines parliamentary
immunity which is a fundamental privilege
Corollary thereto, Section 16 (3), Article VI of cherished in every legislative assembly of the
the Constitution vests in the House of democratic world. t guarantees the legislator
Representatives the sole authority to, complete freedom of expression without fear
inter alia, "determine the rules of its of being made responsible in criminal or civil
proceedings." These "legislative rules, actions before the courts or any other forum
unlike statutory laws, do not have the outside of the Congressional Hall. But it does
Page 118 of 121
403 CASE DIGESTS | CONSTITUTIONAL LAW REVIEW | JUDGE ESTELA ALMA SINGCO | FIRST SEMESTER | 2020-2021

not protect him from responsibility before committee meetings will virtually make him a
the legislative body itself whenever his free man.
words and conduct are considered by the
latter disorderly or unbecoming a It was never the intention of the framers of the
member thereof. constitution to shield a member of congress
from the consequences of his wrongdoings. A
On the question whether delivery of speeches member of Congress could only invoke the
attacking the Chief Executive constitutes immunity from arrests for relatively minor
disorderly conduct for which Osmeña may be offenses, punishable at most by correctional
discipline, We believe, however, that the penalties.
House is the judge of what constitutes
disorderly behavior, not only because the
Constitution has conferred jurisdiction upon it,
but also because the matter depends mainly 169 Paredes vs. Sandiganbayan, G.R. No.
on factual circumstances of which the House 89989
knows best but which cannot be depicted in FACTS: Atty. Paredes, provincial attorney of
black and white for presentation to, and Agusan del Sur was granted land through free
adjudication by the Courts. patent. Later, a former Mayor of Agusan filed
a criminal complaint charging Atty. Paredes
168 People vs. Jalosjos, 324 SCRA 689 with having violated Section 3 (a) of the Anti-
Graft and Corrupt Practices Act (RA 3019)
FACTS: The accused-appellant, Romeo G. because he allegedly used his office as
Jalosjos is a full-fledged member of Congress Provincial Attorney to influence, persuade, and
who is now confined at the national induce Armando Luison, Land Inspector of the
penitentiary while his conviction for statutory District Land Office in Agusan del Sur, to
rape on two counts and acts of lasciviousness favorable indorse his free patent application.
on six counts is pending appeal. The accused-
appellant filed this motion asking that he be In the meantime, Paredes was elected
allowed to fully discharge the duties of a Governor of Agusan del Sur and the free
Congressman, including attendance at patent was reverted back to public domain.
legislative sessions and committee meetings Information was filed and warrant of arrest
despite his having been convicted in the first was issued against Paredes. He refused to post
instance of a non-bailable offense. bail “in protest against the injustice to him as
Governor”. A petition for habeas corpus was
Jalosjos’ primary argument is the "mandate filed by the wife of Paredes, alleging that the
of sovereign will." He states that the warrant of her husband’s arrest was void
sovereign electorate of the First District of because the PI was void and that the crime
Zamboanga del Norte chose him as their charged in the information against him had
representative in Congress. Having been re- already prescribed.
elected by his constituents, he has the duty to
perform the functions of a Congressman. He ISSUE: WON the arrest was void because the
calls this a covenant with his constituents PI was conducted without notice to the
made possible by the intervention of the State. accused
He adds that it cannot be defeated by
insuperable procedural restraints arising from HELD: No, it was not void. The absence of PI
pending criminal cases. Jalosjos also invoked does not affect the court’s jurisdiction over the
the doctrine of condonation citing Aguinaldo case nor impair the validity of the Information
v. Santos. or otherwise render it defective. The remedy is
to demand that PI be conducted before
ISSUE: WON accused-appellant should be entering his plea, and the court should
allowed to discharge mandate as member of suspend trial and order the fiscal to conduct
the House of Representatives PI. Habeas corpus would not lie after the
Warrant of Commitment was issued by the
Court on the basis of the Information filed
HELD: NO! Doctrine of condonation does not against the accused. A writ of habeas corpus
apply to criminal cases. The Aguinaldo case will not issue where the person alleged to be
involves the administrative removal of a public restrained of his liberty is in custody of an
officer for acts done prior to his present term officer under a process issued by the court
of office. It does not apply to imprisonment which has jurisdiction to do so.
arising from the enforcement of criminal law.
Moreover, in the same way that preventive NOTE: There is no Constitutional Law related
suspension is not removal, confinement topic discussed in this case. But Judge Cinco
pending appeal is not removal. He remains a notes: Suspension imposed by Congress
congressman unless expelled by Congress or, to a colleague is distinct from suspension
otherwise, disqualified. spoken in Section 13 of RA 3019 which is
not a penalty but a preliminary preventive
His election as congressman did not thereby measure, prescinding from the fact that
amount to a condonation of his offense; the latter is not being imposed for
neither does it entitle him, pending appeal of misbehavior as a member of Congress.
his case, to be free from confinement and to
be allowed to attend sessions of congress, for
the people elected him with full awareness of
the limitations on his freedom of action and
movement. They did so with the knowledge
that he could achieve only such legislative
results which he could accomplish within the
ELECTORAL TRIBUNALS
confines of prison. To allow accused-appellant
to attend congressional sessions and 170 Vera vs. Avelino, G.R. No. L-543

Page 119 of 121


403 CASE DIGESTS | CONSTITUTIONAL LAW REVIEW | JUDGE ESTELA ALMA SINGCO | FIRST SEMESTER | 2020-2021

Principle: The members of the Senate pending before it at the time of the
validly suspended the oath-taking of the 3 proclamation.
senators elect. This does not fall within
the powers of the electoral tribunal. The FACTS: At the core of these contentious
latter has jurisdiction only over electoral consolidated petitions are: (1) the Joint
contests in which contestant seeks not only to Resolution of COMELEC disqualifying
oust the intruder, but also have himself Jocelyn D. Sy Limkaichong (Limkaichong)
inducted into office. from running as a congressional candidate
for the First District of Negros Oriental; (2) the
FACTS: The Commission on Elections COMELEC En Banc Resolution affirming her
submitted last May 1946 to the President and disqualification; and (3) the COMELEC En
the Congress a report regarding the national Banc Resolution resolving that all
elections held in 1946. It stated that by reason pending incidents relating to her
of certain specified acts of terrorism and qualifications should now be determined
violence in certain provinces, namely by the House of Representatives Electoral
Pampanga, Nueva Ecija, Bulacan and Tarlac, Tribunal (HRET)
the voting in said region did not reflect the
accurate feedback of the local electorate. Limkaichong ran as a representative in the 1st
District of Negros Oriental. Two petitions for
During the session on May 25, 1946, a her disqualification were filed by Napoleon
pendatum resolution was approved Camero and Renald Vilando on the grounds of
referring to the report ordering that the her lacking of citizenship which would
three senators, namely: Jose O. Vera, invalidate her candidacy. COMELEC
Ramon Diokno and Jose E. Romero – who had consolidated such complaints but the cases
been included among the 16 candidates for remained pending when the elections were
senator receiving the highest number of votes conducted. Limkaichong thereafter emerged
and as proclaimed by the Commissions on as the victor from opponent Paras. Paras filed
Elections – shall not be sworn, nor seated, to COMELEC for the suspension of
as members of the chamber, pending the Limkaichong’s proclamation as winner.
termination of the protest filed against
their election. Thereafter, COMELEC En Banc issued
Resolution 8062 not suspending the
Petitioners then immediately instituted an proclamation of winning candidates with
action against their colleagues who instituted pending disqualification cases which shall be
the resolution, praying for its annulment and without prejudice to the continuation of the
allowing them to occupy their seats and to hearing and resolution of the involved cases.
exercise their senatorial duties. Respondents LIMKAICHONG filed a motion for
assert the validity of the pendatum resolution. reconsideration and to lift the suspension.
Later, PBOC proclaimed her as First District
ISSUE: Whether administration of oath and Representative. Numerous cases followed suit
the sitting of Jose O. Vera, Ramon Diokno and reaching the Supreme Court in a petition for
Jose Romero should be deferred pending certiorari.The Court granted it and
hearing and decision on the protests lodged dismissed all petitions, including Biraogo’s
against their elections. petition.

HELD: No. The Supreme Court refused to


intervene, under the concept of separation of ISSUE: Upon proclamation, would it have
powers, holding that the case was not a been the HRET and not COMELEC whom have
“contest”, and affirmed that it is the inherent had the jurisdiction over the disqualification
right of the legislature to determine who case?
shall be admitted to its membership.
Following the powers assigned by the HELD: Yes. It should be the HRET’s
Constitution, the question raised was jurisdiction. The court has invariable held that
political in nature and therefore not under once a winning candidate has been
the juridical review of the courts. proclaimed, taken his oath, and assumed
office as a Member of the House of
The members of the Senate validly suspended Representatives, the COMELEC’s jurisdiction
the oath-taking of the 3 senators elect. This over election contests relating to his election,
does not fall within the powers of the electoral returns, and qualifications ends, and the
tribunal. The latter has jurisdiction only over HRET’s own jurisdiction begins.
electoral contests in which contestant seeks
not only to oust the intruder, but also have 172 Vilando v. House of
himself inducted into office. Representatives Electoral Tribunal, G.R.
Nos. 192147 & 192149, [August 23,
2011], 671 PHIL 524-540)
171 LIMKAICHONG vs. COMELEC; Biraogo vs.
Nograles; Paras vs. Nograles & Villando FACTS: Limkaichong ran as a representative in
vs. COMELEC, G.R. Nos. 178831-32 April the 1st District of Negros Oriental. Because of
1, 2009 this, her opponent, Paras and some other
concerned citizens filed disqualification cases
Principle: Once a winning candidate has been against Limkaichong. They alleged that
proclaimed, taken his oath, and assumed Limkaichong was not a natural born citizen of
office as member of the House of the Philippines because when she was born,
Representatives, COMELEC’s jurisdiction over her father was still a Chinese and that her
the election contests relating to his election, mother, lost her Filipino citizenship by virtue
returns and qualifications, ends and the of her marriage to Limkaichong’s father.
HRET’s own jurisdiction begins. The
proclamation of a winning candidate divests During the pendency of the case against
the COMELEC of its jurisdiction over matters Limkaichong before the COMELEC, election
Page 120 of 121
403 CASE DIGESTS | CONSTITUTIONAL LAW REVIEW | JUDGE ESTELA ALMA SINGCO | FIRST SEMESTER | 2020-2021

day came and votes were cast. Results came would operate as a collateral attack on the
in and Limkaichong won over her rival Paras. citizenship of the father which is not
permissible. (Aug. 23, 2011).
COMELEC after due hearing declared
Limkaichong as disqualified. On the following Accordingly, after the proclamation of the
days however, notwithstanding their winning candidates in the congressional
proclamation disqualifying Limkaichong, the elections, the remedy of those who may assail
COMELEC issued a proclamation announcing one’s eligibility or ineligibility, qualification or
Limkaichong as the winner of the recently disqualification is to file before the HRET a
conducted elections. This is in compliance with petition for an election protest, or a petition for
Resolution No. 8062 adopting the policy- quo warranto, within the period provided by
guidelines of not suspending the proclamation the HRET Rules.
of winning candidates with pending
disqualification cases which shall be without
prejudice to the continuation of the hearing
and resolution of the involved cases. Paras
countered the proclamation and she filed a
petition before the COMELEC. Limkaichong
assailed Paras’ petition arguing that since she
is now the proclaimed winner, it should be the
HRET which has the jurisdiction over the
matter and not the COMELEC. COMELEC
agreed with Limkaichong.

ISSUE: WON the proclamation done by


COMELEC is valid – YES
WON the HRET already acquired jurisdiction
over the case – YES

HELD:
(1) YES. The proclamation of Limkaichong was
valid. Limkaichong timely filed with the
COMELEC En Banc her motion for
reconsideration as well as for the lifting of the
incorporated directive suspending her
proclamation. The filing of the motion for
reconsideration effectively suspended the
execution of the COMELEC’s Joint Resolution.
Since the execution of the Joint Resolution was
suspended, there was no impediment to the
valid proclamation of Limkaichong as the
winner pursuant to Section 2, Rule 19 of the
COMELEC Rules of Procedure

(2) YES. The HRET must exercise jurisdiction


after Limkaichong’s proclamation. The SC has
invariably held that once a winning candidate
has been proclaimed, taken his oath, and
assumed office as a Member of the House of
Representatives the COMELEC’s jurisdiction
over election contests relating to his election,
returns, and qualifications ends, and the
HRET’s own jurisdiction begins. It follows then
that the proclamation of a winning candidate
divests the COMELEC of its jurisdiction over
matters pending before it at the time of the
proclamation. The party questioning his
qualification should now present his case in a
proper proceeding before the HRET, the
constitutionally mandated tribunal to hear and
decide a case involving a Member of the House
of Representatives with respect to the latter’s
election, returns and qualifications. The use of
the word “sole” in Section 17, Article VI of the
Constitution and in Section 250 of the OEC
underscores the exclusivity of the Electoral
Tribunals’ jurisdiction over election contests
relating to its members

Lifted from J5’s notes: Clearly under law and


jurisprudence, it is the State thru its
representatives designated by statute, that
may question the illegally or invalidly procured
certificate of naturalization in the appropriate
denaturalization proceedings. HRET no matter
how complete and exclusive, does not carry
with it authority to delve into the legality of the
judgment of naturalization in the pursuit of
disqualifying Limkaichong. To rule otherwise
Page 121 of 121

You might also like